You are on page 1of 580

P a g e 1 | 580

1. Twice the speed of a boat downstream is equal to thrice the speed upstream.
The ratio of its speed in still water to the speed of current is

A. 1 : 5
B. 1 : 3

C. 5 : 1
D. 2 : 3
Answer & Explanation :
Answer: Option C
Explanation :

Let the boat speed in still water be b.


Let the stream speed be x.
2(b+ x) = 3(b-x)
5x=b
b/x=5/1

2. A person has a chemical of Rs. 25 per litre. In what ratio should water be
mixed with that chemical so that after selling the mixture at Rs. 20/litre he may
get a profit of 25%?
A. 13 : 16

B. 12 : 15
C. 9 : 16

D. 19 : 22
Answer & Explanation :

Answer: Option C
Explanation :
This can be solved using alligation.

What is required at the end of mixing is a price of 20/1.25 = 16.


So the alligation would look like this –

Water/0 Mixture/16 Chemical/25

P a g e 2 | 580
Hence the ratio would be (25 – 16) : 16 = 9 : 16
Hence required ratio of Water : Chemical is 9:16.

3. The difference between the simple interest and compound interest on a


certain sum of money for 2 years at 15% p. a. is Rs. 45. Find the sum.
A. Rs. 2700
B. Rs. 2500
C. Rs. 2000
D. None of these

Answer & Explanation :


Answer: Option C
Explanation :
Since we know that the interest rate is 0.15, and knowing that the difference
between two years of compound interest is nothing but interest on interest, we
can find the first year’s interest as –
45/0.15 = 300.

Now if the interest is 300 at the end of one year, then the principal is 300 / 0.15
= 2,000

4. How many terms are there in an A.P. whose first and fifth terms are -14 and
2, respectively, and the sum of terms is 40?
A. 15
B. 10
C. 5
D. 20
Answer & Explanation :

Answer: Option B
Explanation :

Now the common difference of this AP is 16/4 = 4.


The sum of an AP is n/2 {2a + (n – 1)d}
P a g e 3 | 580
Substituting we get, 40 = n/2 {2×-14 + (n – 1)4}
The best way to solve this is by plugging options. Put in n = 10 and get the RHS
as 40.

5. In a class, 50 students play cricket, 20 students play football and 10 play


both cricket and football. How many play at least one of these two games?
A. 10
B. 80
C. 50

D. 60
Answer & Explanation :
Answer: Option D
Explanation :
The required answer is 50 + 20 – 10 = 60.

6. A bottle is full of Dettol. One-third of it is taken out and then an equal


amount of water is poured into the bottle to fill it. This operation is done four
times. Find the final ratio of dettol and water in the bottle.
A. 13 : 55

B. 20 : 74
C. 16 : 65
D. 10 : 48
Answer & Explanation :

Answer: Option C
Explanation :
As in denominator we have to take 1/3 four times so, we start by assuming 81
ml of dettol in the bottle. After the first iteration you will be left with
2/3 × 81 = 54 ml. After the second iteration you will be left with

2/3 × 54 = 36 ml. After the third iteration you will be left with
2/3 × 36 = 24 ml. After the fourth iteration you will be left with
P a g e 4 | 580
2/3 × 24 = 16 ml. So the required ratio will be 16 : (81 – 16) = 16 : 65

7. In a survey of defaulted payments of electrical bills of a residential complex


of 125 houses, it is found that 50 houses defaulted on their payment of
electrical bills in January, 60 in February and 40 in March. Houses can default in
consecutive months only. 20 defaulted in January and February. 10 defaulted in
February and March. How many houses defaulted in all the 3 months?
A. 3
B. 5

C. 7
D. 9
Answer & Explanation :
Answer: Option B
Explanation :

We use formula for intersection of three sets, keeping in mind that Jan ∩ Mar
does not exist, since they are not consecutive months.

Let x be the number of people defaulting in all 3 months.


We get the equation as : 125 = 50 + 60 + 40 – 20 – 10 + x. Solving we get x
= 5.

8. A person standing on the bank of a river observes that the angle of elevation
of the top of a tree on the opposite bank of the river is 60° and when he retires
40 metres away from the tree the angle of elevation becomes 30°. The breadth
of the river is
A. 40 m
B. 20 m

C. 30 m
D. 60 m
Answer & Explanation :
Answer: Option B

Explanation :

P a g e 5 | 580
Let the breadth of the river be x, Using tangent rule we get,
So x = 20

9. India plays two matches each with West Indies and Australia. In any match
the probabilities of India getting points 0, 1, 2 are 0.45, 0.05 and 0.50
respectively. Assuming that outcomes are independent, the probability of India
getting at least 7 points is
A. 0.8750
B. 0.0624

C. 0.0875
D. 0.0250
Answer & Explanation :
Answer: Option C
Explanation :

Getting 7 points is possible in 2 cases.


Case 1: India wins all 4 matches.

Probability: (.5)4 = .0625.


Case 2: India wins any of the 3 matches and draws the remaining match. This
can happen in total 4 ways. Probability: 4 x (.50)3 x (.05) = .025.

So, required probability: .0625 + .025 = .0875

10. Three persons work independently on a problem. If the respective


probabilities that they will solve it are 1/3, 1/4 and 1/5, then the probability
that none can solve it is
A. 1/5
B. 1/3

C. 2/5
D. None of these
Answer & Explanation :
Answer: Option C

P a g e 6 | 580
Explanation :
Joint probability of all not being able to solve it is

2/3*3/4*4/5=2/5
11. The total cost of 8 buckets and 5 mugs is Rs. 92 and the total cost of 5
buckets and 8 mugs is Rs. 77. Find the cost of 2 mugs and 3 buckets.
A. Rs. 35
B. Rs. 70
C. Rs. 30
D. Rs. 38

Answer & Explanation :


Answer: Option A
Explanation :
CP of 1 bucket = Rs. X
CP of 1 mug = Rs. Y
∴ 8x + 5y = 92………….. (i)

5x + 8y = 77 …………….(ii)
By equation (i) × 5 – equation (ii) × 8.
40x + 25y – 40x – 64y
= 460 – 616 ⇒ − 39y = - 156⇒ y = 4
From equation (i),
8x + 20 = 92 ⇒8x = 92 – 20 = 72 ⇒ x = 9
∴ CP of 2 mugs and 3 buckets

= 2 × 4 + 3 × 9 = 8 + 27 = Rs. 35

12. If 4x/3 + 2P = 12 for what value of P, x = 6?


A. 6
B. 4
C. 2

P a g e 7 | 580
D. 1
Answer & Explanation :

Answer: Option C
Explanation :

When x = 6, (4 * 6)/3 + 2P = 12
⇒ 8 + 2P = 12
⇒ 2P = 12 – 8 = 4

⇒P=2

13. What number must be added to the expression 16a2 – 12a to make it a
perfect square?

A. 9/4
B. 11/2
C. 13/2
D. 16
Answer & Explanation :
Answer: Option A
Explanation :

a2 - 2ab + b2 = (a-b)2
∴ 16a2 – 12a = (4a)2 - 2*4a*3/2
Hence, on adding (3/2)2 = 9/4, expression will be a perfect square.

14. The straight line 2x + 3y = 12 passes through:


A. 1st, 2nd and 3rd quadrant

B. 1st, 2nd and 4th quadrant


C. 2nd, 3rd and 4th quadrant
D. 1st, 3rd and 4th quadrant
Answer & Explanation :

P a g e 8 | 580
Answer: Option B
Explanation :

The usual way to solve these type of questions is to put x = 0 once and find y
coordinate. This would represent the point where the line cuts the Y axis.

Similarly put y = 0 once and find x coordinate. This would represent the point
where the line cuts the X axis. Then join these points and you will get the graph
of the line.
So when we put x = 0 we get y = 4.
When we put y = 0 we get x = 6.

So when we join these points we see that we get a line in 1st quadrant, which
when extended both sides would go to 4th and 2nd quadrants. So option B.

15. In ΔABC, ∠A + ∠B = 65°, ∠B + ∠C = 140°, then find ∠B.


A. 40°
B. 25°

C. 35°
D. 20°
Answer & Explanation :
Answer: Option B

Explanation :
∠A + ∠B = 65°

∴ ∠C = 180° - 65° = 115°


∠B + ∠C = 140°

∴ ∠B = 140° - 115° = 25°

16. The average age of a man and his son is 28 years. The ratio of their ages is
3 :1 respectively. What is the man's age?
A. 30 years

B. 38 years
C. 44 years
D. 42 years
P a g e 9 | 580
E. None of these
Answer & Explanation :

Answer: Option D
Explanation :

Total sum of man's age & his son's age =28 × 2 = 56 Now, the Ratio of their
ages is 3 : 1.Therefore, Man's age = (3/4) × 56 = 42
So, the correct answer is option D.

17. A cyclic quadrilateral ABCD is such that AB = BC, AD = DC, AC is


perpendicular to BD and ∠CAD = θ, then find the ∠ABC.
A. θ
B. θ/2
C. 2θ
D. 3θ
Answer & Explanation :

Answer : Option C
Explanation :
∠B + ∠D = 180°

∠A + ∠C = 180°
∠BAC = ∠BCA ∠DAC = ∠DCA

∴∠DAB = ∠DCB = 90°


∠DAC = θ

∴∠ADE = 90° - θ = ∠CDE

∴ ∠ABC = 180° – 2(90° - θ) = 2θ

18. If tan θ + cot θ = 2, then the value of tan2θ + cot2θ is


A. 2

B. 1
C. √2
P a g e 10 | 580
D. 0
Answer & Explanation :

Answer: Option A
Explanation :

tan θ + cot θ = 2 , On squaring both sides,


(tan θ + cot θ)2 = 4, ⇒ tan2θ + cot2θ + 2 tan θ cot θ = 4,
⇒ tan2θ + cot2θ = 4 – 2 = 2 , [tan θ . cot θ = 1]

19. How many integers are there between 300 and 600 that are divisible by 9?
A. 33
B. 31
C. 28
D. 25

E. None of these
Answer & Explanation :
Answer: Option A.
Explanation :
The sequence is 306,… 594

594=306+(n-1)9⇒288=(n-1)9⇒n=33

20. What will be the ratio of petrol and kerosene in the final solution formed by
mixing petrol and kerosene that are present in three identical vessels in the
ratio 4:1,5:2 and 6 :1 respectively?

A. 166 : 22
B. 83 : 22
C. 83 : 44

D. 78 : 55
E. None of these
Answer & Explanation :
P a g e 11 | 580
Answer: Option B
Explanation :

Three identical vessels in the ratio 4:1,5:2 and 6 :1 respectively.


Petrol : kerosene

(4 : 1 = 5)7
(5 : 2 = 7)5
(6 : 1 = 7)5
28 : 7 = 35
25 : 10 = 35

30 : 5 = 35
83 : 22

21. The respective ratio of radii of two right circular cylinders (A & B) is 2 : 5.
The respective ratio of the heights of cylinders A and B is 3 : 1. What is the
respective ratio of volumes of cylinders A and B?
A. 12 : 25

B. 9 : 25
C. 9 : 20
D. 3 : 5

E. 12 : 35
Answer & Explanation :

Answer: Option A
Explanation :

R1/R2/=2/5 H1/H2=3/1 V1/V2= (R1/R2)2 × ( H1/H2)= 12/25

22. Raja gives 30% of his salary to his mother, 40% of the remaining salary he
invests in an insurance scheme and PPF in the respective ratio of 4 : 3 and the
remaining he keeps in his bank account. If the difference between the amount
he gives to his mother and that he invests in insurance scheme is Rs. 8400,
how much is Raja’s salary?

P a g e 12 | 580
A. Rs. 60,000
B. Rs. 62,000

C. Rs. 64,000
D. Rs. 65,000

E. Rs. 54,000
Answer & Explanation :
Answer: Option A
Explanation :
Let Raja salary= R

Salary given to mother = 0.3 R


Money left=0.7 R
Now, Money invested in Insurance scheme= 0.7*0.4*4/7 R= 0.16 R
Difference of money in bank and with mother= 0.14R
Now, 0.14 R=8400, Hence R= 60,0000

23. A, B, C and D are four consecutive odd numbers and their average is 42.
What is the product of B and D?
A. 1860
B. 1890

C. 1845
D. 1677
E. None of these
Answer & Explanation :

Answer: Option C
Explanation :
As there are As diff. Is same so average should lie between B and C so B is 41 &
C is 43 so D must be 45 as we have to find the product of B and D so it would
be 1845.

P a g e 13 | 580
24. A basket contains 3 blue, 5 black and 3 red balls. If 2 balls are drawn at
random, what is the probability that one is black and one is red?

A. 2/11
B. 8/11

C. 9/11
D. 3/11
E. None of these
Answer & Explanation :
Answer: Option D

Explanation :
Selecting 1 black ball out of 5 = 5C1 ways.Selecting on red ball out of 3 = 3C1
ways
The required probability = ( 5C1 × 3C1)/ 11C2 = 3/11

25. A man buys a land and gives for it 20 times the annual rent Find the rate of
interest he gets for his money.

A. 10%
B. 24%
C. 45%

D. 18%
E. 5%

Answer & Explanation :


Answer: Option B

Explanation :
Let annual rent is 1 Rs. so buys the land at 20 Rs. So by investing Rs.20 he is
getting Rs.1 as interest. so on Rs.100 he gets Rs.5 . so rate%=5%.Hence
option E is the answer.
26. 7 cannibals of XYZ island, decide to throw a party. As you may be aware,
cannibals are guys who eat human beings. The senior among them – Father
Cannibal decides that any 6 of them will eat up one cannibal, then out of the
remaining six – five of them will eat up one cannibal and so on till one is left.
P a g e 14 | 580
What is the time until one cannibal is left, if it takes one cannibal 3 hours to eat
up one cannibal independently?

A. 7 hrs 11 min
B. 6 hrs 12 min

C. 7 hrs 21 min
D. 18 hrs 16 min
Answer & Explanation :
Answer: Option C
Explanation :

At the beginning 6 cannibals will eat one, so time required will be 180/6 = 30
min.
Then out of the remaining six – five will devour one, so time required will be
180/5 = 36 min.
Thus the time until one cannibal is left will be = (180/6 + 180/5 + 180/4 +
180/3 + 180/2 + 180/1) min
= (30 + 36 + 45 + 60 + 90 + 180) min

= 441 min
= 7 hrs 21 min.
Hence option 3.

27. Three articles are purchased for Rs. 1050, each with a different cost. The
first article was sold at a loss of 20%, the second at 1/3rd gain and the third at
60% gain. Later he found that their SPs were same. What was his net
gain/loss?
A. 14.28% gain
B. 13% loss

C. 12% loss
D. 11.11% gain
Answer & Explanation :
Answer: Option A

P a g e 15 | 580
Explanation :
Let us assume that their CPs are x, y & z respectively.

According to the given condition 0.8x = 1.33y = 1.6z


⇒ (80/100)x = 400y/(3 × 100) = (160/100)z

⇒x:y=5:3&y:z=6:5
Thus x : y : z = 10 : 6 : 5

Hence CPs of the articles are x = (10/21) × 1050 = 500,


y = (6/21) × 1050 = 300 &

z = (5/21) × 1050 = 250.


SP of the article with CP Rs. x is 0.8x = 0.8 × 500 = 400.
Since SPs are same, the total SP will be 400 × 3 = 1200.
Hence the gain % = (SP – CP)/CP × 100 = (1200 – 1050)/1050 × 100 =
14.28%.

28. In a game of tennis, A gives B 21 points and gives C 25 points. B gives C 10


points. How many points make the game?
A. 50
B. 45

C. 35
D. 30
Answer & Explanation :
Answer: Option C
Explanation :
When B scored p -10 then C scored p - 25.
When B scores 1 then C scores (p-25)/(p-10)
So when B scores p points then C will score (p-25)/(p-10) × p
As per question (p-25)/(p-10) × p = p -10 . Solving this we get p = 35

A B C
p points (p-21) points (p-25) points
P a g e 16 | 580
p points (p-10) points

29. What is the value of a if x3 + 3x2 + ax + b leaves the same remainder


when divided by (x – 2) and (x + 1)?

A. 18
B. 3
C. -6
D. Cannot be determined
Answer & Explanation :

Answer: Option C
Explanation :
Suppose the remainder is R.
Substituting x = 2 and x = –1, we get R = 8 + 12 + 2a + b = –1 + 3 – a + b
⇒ 3a = –18
⇒ a = –6

30. What is the range of values of k if (1 + 2k)x2 – 10x + k – 2 = 0 has real


roots?

A. –3 ≤ k ≤ 4.5
B. –1.5 ≤ k ≤ 9
C. k ≥ 4.5, k ≤ –3
D. k ≥ 3, k ≤ –9
Answer & Explanation :
Answer: Option A
Explanation :
Since the given expression has real roots, we know that (–10)2 – 4(1 + 2k)(k –
2) ≥ 0

100 – 8k2 + 12k + 8 ≥ 0


8k2 – 12k – 108 ≤ 0

P a g e 17 | 580
2k2 – 3k – 27 ≤ 0
–3 ≤ k ≤ 9/2

31. The sum of the first n terms of an AP is Sn = 4n2 – 2n. Three terms of this
series, T2, Tm and T32 are consecutive terms in GP. Find m
A. 7
B. 10
C. 16
D. 5

Answer & Explanation :


Answer: Option A
Explanation :
From the given information, S1 = 4 × 12 – 2 × 1 = 2 => T1 = 2. Now, S2 = 4
× 22 – 2 × 2 = 12.
Since S2 = T1 + T2 and T1 = 2, we get T2 = 10. So, the 1st term of the AP is 2
and the common difference is 8.

From this, we get T32 = 2 + 31 × 8 = 250.


Since T2 , T3 and T32 are consecutive terms in GP, we know that Tm / T2 =
T32 / Tm

⇒ (Tm )2 = T2 × T32 = 2500


⇒ Tm = 50.

32. A trader sells two bullocks for Rs. 8,400 each, neither losing nor gaining in
total. If he sold one of the bullocks at a gain of 20%, the other is sold at a loss
of
A. 20 %
B. 18%

C. 14%
D. 21 %
E. None of these

P a g e 18 | 580
Answer & Explanation :
Answer: Option A

Explanation :
A trader sells two bullocks for Rs. 8,400 each, neither losing nor gaining in total.

As he sold one bullocks at a gain of 20%, it means 120% of C.P = 8400


We get C.P of one bullock = 7000
So gain on one bullock = Rs 1400
Other bullocks is sold at lose and there is neither losing nor gaining in total
So loss on 2nd bullock = 1400/7000×100 = 20%

33. Two trains 130 and 110 meters long are going in the same direction. The
faster train takes one minute to pass the other completely. If they are moving
in opposite directions, they pass each other completely in 3 seconds. Find the
speed of the faster train.
A. 38 m/sec

B. 46 m/sec
C. 42 m/sec

D. 50 m/sec
Answer & Explanation :
Answer: Option C

Explanation :
Total Distance to be travelled by both the trains

= 130 + 110 = 240m


Let ‘F ’and ‘S’ be the speeds of fast and slow trains in m/sec. 240=60(F-S),
240= (F+S)
In the same direction , F – S = 4 m/sec ……(1)
In the opposite direction , F + S =80 m/sec……(2)

Solving them we get F = 42 m/sec.

P a g e 19 | 580
34. A motor boat can travel at 10 km/h in still water. It travelled 91 km
downstream in a river and then returned, taking altogether 20 hours. Find the
rate of flow of the river.
A. 6 km/hr

B. 5 km/hr
C. 8 km/hr
D. 3 km/hr
Answer & Explanation :
Answer: Option D

Explanation :
Given, speed of the Boat in still water (B)=10 km/hr
Let S be the speed of flow of river, then
91/(10+S) + 91/(10-S) = 20, Then going by options
91/13 - 91/7 = 20
So, S = 3 km/hr.

35. The total tractor population in a state is 2,94,000, out of which 1,50,000 are
made by Mahindra & Mahindra. Out of every 1,000 Mahindra tractors, 98 are
red in colour, but only 5.3% of the total tractor population is red. Find the
percentage of non-Mahindra tractors that are red.
A. 0.5025%
B. 0.5130%
C. 0.6125%
D. 0.6140%
Answer & Explanation :
Answer: Option C

Explanation :
Total tractor population = 2,94,000

Mahindra & Mahindra = 1,50,000


So, Non Mahindra trucks = 1,44,000
P a g e 20 | 580
Since out of every 1000 Mahindra tractors, 98 are red, out of 1,50,000
Mahindra tractors 14,700 are red.

5.3% of 2,94,000 = 15,582 are red tractors in all.


So non Mahindra tractors which are red

=15,582 – 14,700 = 882


Hence percentage of non Mahindra tractors that are red = 882/144,000 * 100 =
0.6125%

36. 7% of the total quantity of wheat is lost in grinding when a country has to
import 12 million tonnes, but when only 51⁄5 is lost, it can import 3 million
tonnes. Find the quantity of wheat grown in the country.
A. 500 million tonnes
B. 400 million tonnes
C. 600 million tonnes

D. 700 million tones


Answer & Explanation :

Answer: Option A
Explanation :
Difference in % of wheat lost = 7 – 26/5 = 9/5%

Difference in import = 12 – 3 = 9 million As 9/5% % of total qty of wheat = 9


million
⇒ 9x/500 = 9

⇒ x = 500 million

37. A man who can swim 48 m/min in still water swims 200 m against the
current and 200 m with the current. If the difference between those 2 times is
10 minutes, find the speed of the current.

A. 30 m/min
B. 29 m/min
C. 31 m/min

P a g e 21 | 580
D. 32 m/min
Answer & Explanation :

Answer: Option A
Explanation :

Try option & get answer as fourth option:


(200/(48 - 32)) - (200/ (48 + 32)) (12(1/2)min - 2(1/2)min)
= 10 min

38. A and B run a 5 km race on a round course of 400 m. if their speed be in


the ratio 5 : 4, how often does the winner pass the other on circular track?
A. 4 times
B. 3 times
C. 5 times
D. 2 times
Answer & Explanation :
Answer: Option D

Explanation :
Total no. of round will be 5000/400 = 12.5
No. of rounds A completes to finish the race will be 12.5 and by the time B can
complete only 10 rounds.
(As ratio of speed of A & B is 5 : 4. So they meet for the first time after A has
finished 5 rounds and B has finished 4 rounds.)
So difference in no. of round will be = 21⁄2

The winner meets the other 2 times because the winner meets the other after
every 2000 meters.

39. Mira’s expenditure and saving are in the ratio 3:2. Her income increases by
10%. Her expenditure also increases by 12%. By how much % do her saving
increase?
A. 7%

P a g e 22 | 580
B. 9%
C. 10%

D. 13%
Answer & Explanation :

Answer: Option A
Explanation :
Let total income be = 5
Increased income will be = 5.5
Increased expenditure will be = 3.36

Increased saving will be = 5.5 – 3.36 = 2.14


Percentage increase will be = 14/2 * 100 = 7%

40. Two vessels contain spirit of 0.5 and 0.75 concentrations. If two liters from
the first vessel and three liters from the second vessel are mixed, then what will
be the ratio of the spirit and the water in the resultant solution?
A. 13 : 7

B. 15 : 17
C. 7 : 17
D. 17 : 15

Answer & Explanation :


Answer: Option A

Explanation :
Take 12, 12 liters in both mixtures which gives us

First
Spirit - 6
Water - 6

Second
Spirit - 9
P a g e 23 | 580
Water - 3

Taking 2 liters from 1st mixture and 3 liters from 2nd mixture which gives us
First

Spirit - 1
Water - 1

Second
Spirit - 9/4

Water - 3/4
So total of sprit and water in new mixture will be =
Spirit Water
1 + (9/4) 1 + (3/4)
13/4 7/4
Which will give 1st option as the answer.

41. Two small circular parks of diameters 16 m, 12 m are to be replaced by a


bigger circular park. What would be the radius of this new park, if the new park
occupies the same space as the two small parks?

A. 10
B. 20

C. 15
D. 25

Answer & Explanation :


Answer: Option A
Explanation :

(Some of areas of 2 smaller parks = New bigger one


π(8)² + π(6)² = πr²
π[64+36] = πr²
P a g e 24 | 580
π[100 = πr²
r= 10

42. The length of a rectangular field is double its width, inside the field there is
a square-shaped pond 8 m long. If the area of the pond is 1/8 of the field, what
is the length of the field?
A. 32 m
B. 16 m
C. 64 m

D. 20 m
Answer & Explanation :
Answer: Option A
Explanation :
Area of Pond = 64
Area of field = 64 × 8 = 512
Area of field ⇒ x . 2x = 512

⇒ 2x2 = 512 ⇒ x2 = 256 ⇒ x = 16


Length = 2 × 16 = 32. So A option is answer.

43. A garden is 24 m long and 14 m wide. There is a path 1 m wide outside the
garden along its sides. If the oath is to be constructed with square marble tiles
20 cm × 20 cm, find the number of tiles required to cover the path.
A. 1800
B. 2000
C. 200
D. 2150
Answer & Explanation :
Answer: Option B

Explanation :
Area of field = 16 × 26 = 416 m2

P a g e 25 | 580
Area of inner field = 24 × 14 = 336 m2
Remaining area = 416 – 336 = 80 m2

Area in cm = 80 × 100 × 100 = 800,000 cm2


Area of each marble = 20 × 20 = 400

No. of tiles = 800000/400 = 2000

44. From the top of a cliff 25 m high, the angle of elevation of a tower is found
to be equal to the angle of depression of the foot of the tower. Find the height
of the tower.

A. 40 m
B. 50 m
C. 48 m
D. 52 m
Answer & Explanation :
Answer: Option B
Explanation :

ZP is parallel to XY, so angle ZPY = angle ZYX,


And angle PZY = angle QZP

Hence angle QZP = ZYX = PZY


As ZXY is 90, so the only possible value that satisfy the above angles is 45

Then XY = 25 m, PZ = 25 and PQ = 25
As x = 450, total height of tower will be

= 25 + 25 = 50 m

45. A bath tub can be filled by a cold water pipe in 20 minutes and by a hot
water pipe in 30 minutes. A person leaves the bathroom after turning on both
pipes simultaneously and returns at the moment when the bath tub should be
full. Finding however, that the waste pipe has been open, he now closes it. In 3
minutes more the bath tub is full. In what time would the waste pipe empty it?

P a g e 26 | 580
A. 38 min
B. 45 min

C. 42 min
D. 48 min

Answer & Explanation :


Answer: Option D
Explanation :
Total work = 1/20 + 1/30
(3+2)/60 = 5/60 = 1/12

For both of them needed = 12 min.


But due to leakage it take = 15 min.
So we can say for filling pipe it takes 3 minutes
work for the waste pipe will be = 12 min.
3 min. of both filling pipe = 12 min. of waste pipe
12 min. of filling pipe will be = 48 min. of waste pipe i.e. our answer is 4th
option

46. A mixture of 45 liters of spirit and water contains 20% of water in it. How
much water must be added to make the water 25% in the new mixture?

A. 5 litres
B. 3 litres

C. 4 litres
D. 6 litres

Answer & Explanation :


Answer: Option B
Explanation :

In Mixture Spirit Water


36 9

P a g e 27 | 580
On adding water in the mixture it becomes 25 % of mixture, so spirit will be 75
% of the mixture.

75% of Total = 36
⇒ Total = 36/0.75 = 48 liter. Hence 3 liter water is needed.

47. In a year 28th February is Tuesday; if the leap year is excluded, then 28th
March will be a

A. Sunday
B. Tuesday
C. Monday
D. Saturday
Answer & Explanation :
Answer: Option B
Explanation :
Simple counting of days.

28th Feb = Tuesday


28th March = Tuesday

48. Ram’s age was square of a number last year and it will be cube of a number
next year. How long must he wait before his age is again the cube of a number
A. 39 years
B. 38 years
C. 10 years
D. 64 years

Answer & Explanation :


Answer: Option B
Explanation :

First square then cube, this no will be after25, 27 years respectively. Next cube
will be at 64 so he has to wait for 38 years

P a g e 28 | 580
49. Two small circular parks of diameters 16 m, 12 m are to be replaced by a
bigger circular park. What would be the radius of this new park, if the new park
occupies the same space as the two small parks?

A. 10
B. 20
C. 15
D. 25
Answer & Explanation :

Answer: Option A
Explanation :
Some of areas of 2 smaller parks
= New bigger one
π(8)² + π(6)² = πr²
π[64+36] = πr²
π[100 = πr²

r= 10

50. A bottle contains 3/4 of milk and the rest water. How much of the mixture
must be taken away and replaced by equal quantity of water so that the
mixture has half milk and half water?
A. 25%
B. 33.33%

C. 45%
D. 50%
Answer & Explanation :

Answer: Option B
Explanation :

Question can be solved by just putting value in Exp. Take total as 12 liter
Quantity of milk & water will be 9, 3
P a g e 29 | 580
Take out 1/3rd of mixture out now we have quantity of milk & water will be 6, 2
Now replace with water we get quantity of mixture will be 6, 6 liters. This is the
required answer (2nd option) because we have replaced mixture with 4 liters of
water.

51. Two vessels contain spirit of 0.5 and 0.75 concentrations. If two liters from
the first vessel and three liters from the second vessel are mixed, then what will
be the ratio of the spirit and the water in the resultant solution?
A. 13 : 7

B. 15 : 17
C. 7 : 17
D. 17 : 15
Answer & Explanation :
Answer: Option A

Explanation :
Take 12, 12 liters in both mixtures which gives us

First
Spirit - 6
Water - 6

Second
Spirit - 9
Water - 3

Taking 2 liters from 1st mixture and 3 liters from 2nd mixture which gives us
First

Spirit - 1
Water - 1

P a g e 30 | 580
Second
Spirit - 9/4

Water - 3/4
So total of sprit and water in new mixture will be =

Spirit Water
1 + (9/4) 1 + (3/4)
13/4 7/4
Which will give 1st option as the answer.

52. A and B run a 5 km race on a round course of 400 m. if their speed be in


the ratio 5 : 4, how often does the winner pass the other on circular track?
A. 4 times
B. 3 times
C. 5 times
D. 2 times
Answer & Explanation :

Answer: Option D
Explanation :
Total no. of round will be 5000/400 = 12.5 No. of rounds A completes to finish
the race will be 12.5 and by the time B can complete only 10 rounds. (As ratio
of speed of A & B is 5 : 4. So they meet for the first time after A has finished 5
rounds and B has finished 4 rounds.) So difference in no. of round will be =
21⁄2.
The winner meets the other 2 times because the winner meets the other after
every 2000 meters.
53. A function f(x) is defined as f(x) = f(x - 2) - x(x + 2) for all the integer
values of x and f(1) + f(4) = 0. What is the value of f(1) + f(2) + f(3) + f(4) +
f(5) + f(6)?
A. 0
B. 89
C. -89
P a g e 31 | 580
D. None of these
Answer & Explanation :

Answer: Option C
Explanation :

Let S = f(1) + f(2) + f(3) + f(4) + f(5) + f(6)


As f(1) + f(4) = 0, therefore S = f(2) + f(3) + f(5) + f(6) ------ (1)
f(2) = f(0) - 8
f(3) = f(1) - 15
f(4) = f(2) - 24 = f(0) - 32

f(5) = f(3) - 35 = f(1) - 50


f(6) = f(4) - 48 = f(0) - 80
Put the above values in equation (1), we get
S = f(0) - 8 + f(1) - 15 + f(1) - 50 + f(0) - 80
S = 2(f(0) + f(1)) - 153 ------ (2)
As we already know f(1) + f(4) = 0 ⇒f(1) + f(0) - 32 = 0 ⇒f(1) + f(0) = 32

Putting this value in equation 2, we get S = 2(32) - 153 = -89


So, Ans is option C.

54. In ΔABC, the internal bisectors of ∠ABC and ∠ACB met at I and ∠BAC = 50°.
The measure of ∠BIC is
A. 105°
B. 115°
C. 125°
D. 130°
Answer & Explanation :
Answer: Option B

Explanation :
I is the In-centre

P a g e 32 | 580
So, ∠BIC = 90° + 1/2 ∠BAC

= 90° + 1/2 × 50 = 115°

55. The difference between [3/5] of [2/3] a number and [2/5] of[1/4] of the
same number is 288. What is the number?
A. 960
B. 850

C. 895
D. 955
Answer & Explanation :
Answer: Option A
Explanation :
Let the no. Be [3/5] x [{2/3}x]-[2/5] x [1/4]x,then = 288 Solving it we will get
960

56. A, B, C and D are four consecutive odd numbers and their average is 42.
What is the product of B and D?
A. 1860
B. 1890
C. 1845
D. 1677
E. None of these
Answer & Explanation :
Answer: Option C

Explanation :
As diff. Is same so average should lie between B and C so B is 41 & C is 43 so D
must be 45 as we have to find the product of B and D so it would be 1845

P a g e 33 | 580
57. The cost price of item A, Item A was sold at a profit of 10% and Item B was
sold at a loss of 20%. If the respective ratio of selling price of items A and B is
11:12, what is the cost price of item B?
A. Rs.450/-

B. Rs.420/-
C. Rs.400/-
D. Rs.350/-
E. Rs.480/-
Answer & Explanation :

Answer: Option A
Explanation :
The cost price of item B is Rs. 150/- more than the Let us assume cost price of
A= X
So that Cost price of B= X+150.

SP of A= X*1.1
SP of B=(X+150)*0.8

Given that
SPA: SPB
11:12

So that 1.1X/(X+150)*0.8= 11/12


X=300

CP of B= 300+150=450
Answer is 450.

58. A sum of Rs 731 is divided among A, B and C such that 'A' receive 25%
more than 'B' and 'B' receive 25% less than 'C'. What is C's share in the
amount?
A. Rs. 172

B. Rs. 200
C. Rs. 262
P a g e 34 | 580
D. Rs. 258
E. None of these

Answer & Explanation :


Answer: Option E

Explanation :
A + B + C = 731 ..... (i)
A = 1.25B, gives A = 1.25 * 0.75 C = 0.9375 C....(ii)
B = 0.75C....(iii)
Using (ii) and (iii) in (i) we get

0.9375C + 0.75C + C = 731, gives C = 272.


So option E is the answer.

59. In how many different ways can letters of the word "PRAISE" be arranged?
A. 720
B. 610
C. 360

D. 210
E. None of these
Answer & Explanation :

Answer: Option A
Explanation :
As total number of alphabets in PRAISE are 6, so total no. of ways is 6!=720 So
option A is the answer

60. If the numerator of a fraction is increased by 150% and the denominator of


the fraction is increased by 300%, the resultant fraction is [5/18]. What is
original fraction?
A. 4/9

B. 4/5

P a g e 35 | 580
C. 8/9
D. 8/11

E. None of these
Answer & Explanation :

Answer: Option A
Explanation :
Let the fraction be [n/d] & after that it becomes [2.5n/4d]=5/18 . So we get the
result as [4/9]

61. A car covers the first 30 km of its journey in 45 minutes and the remaining
25 km in 30 minutes. What is the average speed of the car?
A. 60
B. 64
C. 49
D. 48
E. None of these

Answer & Explanation :


Answer: Option E
Explanation :

Total distance/Total time =(30+25)/[(3/4)+(1/2)]


Average Speed=44kmph

62. Four examiners can examine a certain number of answer papers in 10 days
by working for 5 hours a day. For how many hours a day would 2 examiners
have to work in order to examine twice the number of answer papers in 20
days?

A. 8
B. 7.5

C. 10
D. 8.5
P a g e 36 | 580
E. None of The above
Answer & Explanation :

Answer: Option C
Explanation :

Examiners Days Hours/Day Papers


4 10 5 x
2 20 ? 2x
So, we required number of hours = so optionC is the answer
Using chain rule,

(4×10×5)/X= (2×20×Y)/2X
X=10 hrs/day
63. If 4x - 3y = 19 and x + 2y = 13, what are the respective values of x and y?
A. 5, 4
B. 6, 5
C. 7, 3
D. 8, 7

Answer & Explanation :


Answer: Option C
Explanation :

4x-3y=19, x+2y=13 Option 3rd satisfies all two equations

64. Due to global recession starting in January, Ram's monthly salary of Rs


8,000 was cut by 10%. The monthly expenses, which were Rs 6,000, increased
at the rate of 5% per month. From which month will he have no savings if the
recession lasted for a year?
A. May

B. April
C. March

D. June

P a g e 37 | 580
Answer & Explanation :
Answer: Option B

Explanation :
Month Salary Expenses
Saving
January 7200 6000+ 5% of 6000 = 6300
7200-6300-900
February 7200 6300+5% of 6300= 6615
7200-6615=585

March 7200 6615+5% of 6615= 6945.75


7200-6945.75=254.25
April 7200 6945.75+ 5% of 6945.75 = 7293.0375
Expenses are more than salary
From April onwards, he will have no savings.

65. A hemispherical bowl is 176 cm round the brim. Supposing it to be half full,
how many persons may be served from it in hemispherical glasses with
diameter of 4 cm at the top?
A. 1372
B. 1272
C. 1172
D. 1472
Answer & Explanation :
Answer: Option A
Explanation :
Circumference of bowl = 176 = 2 π r ; r = 28

Quantity of liquid = ½ * (2/3 * π * 283) = 3 * 283 π.


Volume of a glass is = 2/3 *π * 23 = 16π/3.
Number of glasses = (3 * 283π / (16π/3)
= 1372

P a g e 38 | 580
66. A and B working together can complete a piece of work in 12 days. B and C
working together can complete the same work in 16 days. A worked at it for 5
days and B worked at it for 7 days. C finished the remaining work in 13 days.
How many days would C alone take to complete it?
A. 32
B. 24
C. 10
D. 40

Answer & Explanation :


Answer: Option B
Explanation :
1/A + 1/B = 1/12 and 1/B + 1/C = 1/16
= 5/A + 7/B + 13/C = (1/A + 1/B)5 + (1/B + 1/C )2 + 11/C = 1. Substituting
from the above equations, we get
C = 24.

67. Sixteen men can complete a work in twelve days. Twenty-four children can
complete the same work in eighteen days. Twelve men and eight children
started working and after eight days three more children joined them. How
many days will they now take to complete the remaining work?
A. 6
B. 4
C. 2
D. None of these
Answer & Explanation :

Answer: Option B
Explanation :
Let the total work = 36
A man does 36/(12 * 16) = 3/16 work per day.

P a g e 39 | 580
A child does 36/(18 * 24) = 1/12 work per day.
12 men & 8 children do 12(3/16) + 8 (1/12) = 9/4 + 2/3

Now for 8 days, 8(9/4 + 2/3) = 70 / 3


The remaining work = 36 – (70/3) = 38/3

12 men and 11 children will complete the remaining work in x days.


x[(12 * 9/4) + (11 * 1/12)] = 38/3 * x = 4.

68. Two men A and B working together complete a piece of work which it would
have taken them 30 and 40 days respectively to complete if they worked
separately. If they received a payment of ` 21,000, B's share is
A. 8000
B. 12000
C. 9000
D. None of these
Answer & Explanation :
Answer: Option C

Explanation :
Let the amount of work be 120 units.
A would have done 4 units a day.

B would have done 3 units a day.


So the ratio when they work together = 4 : 3

From 21000, B' share will 3/7 Rs 21000 = 9,000.

69. If Rs 10 be allowed as true discount on a bill of Rs 110 due at the end of a


certain time, then the discount allowed on the sum due at the end of double the
time is

A. 22
B. 21.81

C. 20

P a g e 40 | 580
D. 18.33
Answer & Explanation :

Answer: Option C
Explanation :

Discount of Rs.10 on Rs.110 discount = 10%.


Bill= Rs.100
After the end of double the time, Bill = 200
So, 10 % of 200 = 20.

70. A man purchased a cycle for Rs 3,000 and sold it the same day for Rs
3,600, allowing the buyer a credit of 2 years. If the rate of interest be 10% per
annum, then the man has a gain of
A. 7.5%
B. 5%
C. 20%
D. None of these

Answer & Explanation :


Answer: Option D
Explanation :

The rate of interest is 10%.


For 2 years, It will become 20% of 3600 = 720.

So he gain 720 + 600 = 1320 on 3000.


So a man has a gain of 44%.

71. The angle of elevation on the top of a tower from two horizontal points at
distance of 'a' and 'b' metres from the tower are 'α' and '90 -α ' respectively.
The height of the tower will be (where a > b)
A. ab metres

B. √ ab metres

P a g e 41 | 580
C. √ a/b metres
D. √ b/a metres

Answer & Explanation :


Answer: Option B

Explanation :

72. A person observes the angle of elevation of a building as 30°. The person
proceeds towards the building with a speed of 25(√ 3 -1)m/hour. After 2 hours,
he observes the angle of elevation as 45°. The height of the building (in m) is
A. 50 ( √ 3 +1)
B. 50

C. 100
D. 50 ( √ 3 -1)
Answer & Explanation :
Answer: Option B
Explanation :

73. A, B and C started a business by investing Rs. 12,800/-. Rs.16,800/- and


Rs. 9,600/- respectively. If after 8 months B received Rs. 13,125/- as his share
of profit, what amount did C get as his share of profit?
A. Rs. 7,800/-

B. Rs. 7,150/-
C. Rs. 7,750/-
P a g e 42 | 580
D. Rs. 8,250/-
E. Rs. 7,500/-

Answer & Explanation :


Answer: Option E

Explanation :
Here the profit will be distributed in the ratio of their investments
i.e. 12800:16800:9600 → 16:21:12
Let 'x' be the total profit
we know that profit received by B = 13125/-

Therefore,
21 / (16+21+12) * x = 13125
Share received by C = (12 / 49) * 30625 = 7500/-
Hence, the answer is E.

74. 456 ÷ 24 * 38 - 958 + 364 = ?


A. 112

B. 154
C. 128

D. 136
E. 118
Answer & Explanation :
Answer: Option C
Explanation :

Applying BODMAS
? = 456 ÷ 24 * 38 - 958 + 364
= 19 * 38 - 958 + 364

= 722 - 958 + 364


= 1086 - 958

P a g e 43 | 580
= 128
Hence, the answer is option C.

75. (34.5 × 14 × 42) ÷ 2.8 =?

A. 7150
B. 7365
C. 7245
D. 7575
E. 7335

Answer & Explanation :


Answer: Option C
Explanation :
? = (34.5 × 14 × 42) ÷ 2.8
= 20286 / 2.8
= 7245
Hence, the answer is option C.

76. -676.76 + 1237.87 + 897.34 - ? = 1294.25


A. 168.6

B. 164.2
C. 156.4
D. 172.2
E. 158.6

Answer & Explanation :


Answer: Option B
Explanation :

Applying BODMAS
? = -676.76 + 1237.87 + 897.34 - 1294.25

P a g e 44 | 580
= 164.2
Hence, the answer is option B.

77. Pure milk costs Rs. 16/- per liter. After adding water the milkman sells the
mixture @ Rs. 15/- per liter and thereby makes a profit of 25% in what
respective ratio does he mix milk with water?
A. 3 : 1
B. 4 : 3
C. 3 : 2

D. 5 : 3
E. 4 : 1
Answer & Explanation :
Answer: Option A
Explanation :
If SP of mixture is 15/- per liter and profit is 25%, then
CP of mixture = (100/125) * 15 = 12/- liter

As 16/- is the cost of pure 1000 ml, therefore, for 12/-, the quantity of milk =
(12/16) * 1000 = 750 ml
So for 12/- per liter, the milk is 750 ml, then water will be 250 ml.

Thus, required ratio = 750:250 = 3:1


Hence answer is option A.

78. What will come in place of question mark (?) in the given question?

36 38.8 42.8 ? 54.4 62


A. 46.2
B. 46.6

C. 48.2
D. 48
E. 49
P a g e 45 | 580
Answer & Explanation :
Answer: Option B

Explanation :
Here the given pattern is as follows:

36 38.8 42.8 ? 54.4 62


+2.8 +4.0 +5.2 +6.4 +7.6
+1.2 +1.2 +1.2 +1.2
So ? = 42.8 + 5.2 = 48
Hence answer is option D.

79. A merchant bought some goods worth Rs. 6,000/- and sold half of them at
12% profit. At what profit percent should he sell the remaining goods to make
an overall profit of 18%?
A. 24%
B. 28%
C. 18%

D. 20%
E. 26%
Answer & Explanation :

Answer: Option A
Explanation :

Let CPT = 6000/-


Therefore, cost of half of goods CP1= 3000/- and of other half goods = CP2=
3000/-
So, SP1=3000 × 1.12 = 3360/- and SPT= 6000 × 1.18 = 7080/-
Thus, SP2= 7080 – 3360 = 3720/-

Profit % = [(3720 - 3000) / (3000)] * 100 = 24%


Hence answer is option A

P a g e 46 | 580
80. A firm of readymade garments makes both men's and women's shirts. Its
profit average is 6% of sales. Its profits in men's shirts average 8% of sales and
women's shirts comprise 60% of output. The average profit per sales rupee in
women's shirts is

A. 0.0466
B. 0.0666
C. 0.0166
D. 0.0380
Answer & Explanation :

Answer: Option A
Explanation :
Let sales be Rs. 100 totally.
Of this Rs. 40 is sales of men's shirts.
Margin on that is 40 x 8% = Rs. 3.2

Now total margin is 100 x 6% = Rs. 6.


So margin from women's shirts is 6 – 3.2 = 2.8

This is from a sale of Rs. 60.


So profit per rupee sale is 2.8/60 = 0.047.

81. A student is to answer 10 out of 12 questions in an examination such that


he must choose at least 4 from the first five questions. The number of choices
available to him is
A. 140
B. 280
C. 196
D. 346

Answer & Explanation :


Answer: Option A

Explanation :
4 questions can be chosen from the first 5 in 5 ways.
P a g e 47 | 580
Remaining 6 questions can be selected from 8 questions in 8C6 ways.
So required answer is 8C6 x 5 = 28 x 5 = 140.

82. In how many ways can a student choose a programme of 5 courses if 9


courses are available and 2 courses are compulsory for every student?
A. 45 ways
B. 55 ways
C. 35 ways
D. 65 ways

Answer & Explanation :


Answer: Option C
Explanation :
7C3 = 35 as 2 cases are fixed.

83. Two cm rain has fallen on a sq. km of land. Assuming that 50% of the rain
drops could have been collected and contained in a pool having a 100 m × 10 m
base by what level would the water level in the pool has increased?
A. 15 m
B. 10 m

C. 20 m
D. 25 m

Answer & Explanation :


Answer: Option B

Explanation :
Volume of water collected = volume of pool
⇒ 1/2×2/100 × 1000 × 1000

= 100 × 10 × h

So h = 10 m is the answer.

P a g e 48 | 580
84. In a race of 600 metres, A can beat B by 60 metres and in a race of 500
metres, B can beat C by 25 metres. By how many metres will A beat C in a 400
metres race?
A. 50 m

B. 64 m
C. 54 m
D. 58 m
Answer & Explanation :
Answer: Option D

Explanation :
In 600 A beat B by 60 meter, so in a 400 m race the gap will be 40 only.
B beat C by 25 in 500; in 360 meter run he can beat C by 18 meter
So the total is 58 meter.

85. Two cards are drawn from a pack full of cards, in succession, with
replacement. What is the probability that both are of different colours?

A. 1/2
B. 5/52
C. 2/13

D. 1/13
Answer & Explanation :

Answer: Option A
Explanation :

Probability for the first card = 52/52.


Probability for the second card will be = 26/52.
For both cards is =52/52 × 26/52 = ½.

Thus the first option is the answer.

P a g e 49 | 580
86. Two cards are drawn from a pack full of cards, in succession, with
replacement. What is the probability that both are of different colours?

A. 1/2
B. 5/52

C. 2/13
D. 1/13
Answer & Explanation :
Answer: Option A
Explanation :

Probability for the first card = 52/52.


Probability for the second card will be = 26/52.
For both cards is =52/52 × 26/52 = ½.
Thus the first option is the answer.

87. In a coaching institute, every studentis selected in atleast one of the exam
i.e. banking exam, and staff selection exam . 40 students are selected in the
banking exam, 30 students are selected in the staff selection exam and 20
students are selected in both the examinations. How many students are there in
the institute?

A. 45
B. 55
C. 50
D. 40
Answer & Explanation :
Answer: Option C
Explanation :

Banking exam = 40,


Staff exam = 30,

Both exam = 20
So banking only is 20, staff exam only is 10, and both is 20, so total is 50
P a g e 50 | 580
88. In two alloys, Copper and Zinc are related in the ratio of 4 : 1 and 1 : 3. 10
kg of 1st alloys, 16 kg of 2nd alloy and some pure copper are melted together.
An alloy was obtained in which the ratio of copper to zinc was 3 : 2. Find the
weight of the new alloy.
A. 35 kg
B. 45 kg
C. 40 kg
D. 50 kg

Answer & Explanation :


Answer: Option A
Explanation :
Copper Zinc
Ist alloy 8 2

2nd alloy 4 12
3rd alloy? =9 0

Final Sol 21 (7 x 3) 14 (7 x 2)
Answer is 35

89. An instruments manufactured by a company consists of two parts A and B.


In manufacturing part A, 9 out of 100 are likely to be defective and in
manufacturing part B, 5 out of 100 are likely to be defective. Calculate the
probability that the instrument will not be defective.
A. 0.91
B. 0.86
C. 0.95

D. 0.83
Answer & Explanation :
Answer: Option B
Explanation :

P a g e 51 | 580
Probability of Non-defective part in A is = 91/100
Probability of Non-defective part in B is = 95/100

Overall non-defective part will be possible only when both components are non-
defective.

Hence required probability is 0.91 x 0.95 = 0.86


90. A cricketer played 80 innings and scored an average of 99 runs. His score in
the last inning was zero run. To have an average of 100 at the end, his score in
the last innings should have been
A. 60 runs

B. 80 runs
C. 10 runs
D. 1 run
Answer & Explanation :
Answer: Option B

Explanation :
Let x be the score in the last inning to make the average of 100

Therefore, when average is 99, total score of 80 innings = 80 × 99 = 7920


And when the average is 100, total score of 80 innings = 80 × 100 = 8000
Therefore, x = 8000 – 7920 = 80

91. A man spends an average of Rs. 1,694.70 per month for the first 7 months
and Rs.1,810.50 per month for the next 5 months. His monthly salary if he
saves Rs. 3,084.60 during the whole year is
A. Rs. 1,000
B. Rs. 2,000
C. Rs. 2,400

D. Rs. 3,000
Answer & Explanation :

Answer: Option B
Explanation :
P a g e 52 | 580
Monthly salary = (1694.70 x 7 + 1810.50 x 5 +3084.60 / 12)
= (24000 / 12)

=2000/-

92. A and B undertake to do a piece of work for Rs. 2,200. A alone can do it in 8
days, while B can do it in 6 days. With the help of C, they complete it in 3 days.
Find C’s share.
A. Rs. 150
B. Rs. 275

C. Rs. 245
D. Rs. 175
Answer & Explanation :
Answer: Option B
Explanation :
Let 1 be the total work
Therefore, C's one Day work = (1 / 3) - ( (1 / 8) + (1 / 6) )

=1/24
Ratio of share of A, B and C
i.e A : B : C = (1/8) : (1/6) : (1/24)

⇒3:4:1
So, C's share = (1/8) x 2200 = 275 /-

93. By selling an article at 80% of its marked price, a trader makes a loss of
10%. What will be the profit percentage if he sells it at 95% of its marked
price?
A. 5.9
B. 12.5
C. 6.9
D. 4.5

P a g e 53 | 580
Answer & Explanation :
Answer: Option C

Explanation :
Let us assume the MP to be 900/-

Therefore, SP = 80% of MP = 0.80 × 900 = 720/


As loss is 10%, so CP = 720/90 × 100 = 800/-
Now, if SP = 95% of MP = 0.95 × 900 = 855/-
Profit% = (855-800)/800 × 100 = 55/8 = 6.875 ≈ 6.9

94. By selling an umbrella for Rs. 30, a shopkeeper gains 20%. During a
clearance sale, the shopkeeper allows a discount of 10% of the marked price.
His gain during the sale season is
A. 8
B. 9
C. 7
D. 7.5

Answer & Explanation :


Answer: Option A
Explanation :

Given that SP of umbrella = 30/- and profit% = 20.


Therefore, CP of umbrella = 30/120 × 100 = 25/- SP of umbrella after 10%
discount = 90/100 × 30 = 27/- Thus, profit% = (27 - 25)/25 × 100 = 8

95. From each of two given numbers, half the smaller number is subtracted.
After such subtraction, the larger number is 4 times as large as the smaller
number. What is the ratio of the numbers?

A. 4 : 1
B. 4 : 5

C. 5 : 2
D. 1 : 4
P a g e 54 | 580
Answer & Explanation :
Answer: Option C

Explanation :
Let x and y be the two numbers, with x > y

Now are given that x - y/2 = 4[y - y/2]


Solving the above equation, we get x/y = 5/2

96. Men, women and children are employed to do a work in the proportion of 3
: 2 : 1 and their wages per person are in the proportion of 5 : 3 : 2. When 90
men are employed, total daily wages of all amounts to Rs. 10,350. Find the
daily wage of a man.
A. Rs. 115
B. Rs. 75
C. Rs. 45

D. Rs. 57.50
Answer & Explanation :

Answer: Option B
Explanation :
Let the daily wages be 5k, 3k and 2k of men, women and children respectively.

We are given that there are 90 men and total amount of wages is 10350/-
Thus, men, women and children are 90, 60 and 30 respectively.

Hence, total wages = 90 x 5k + 60 x 3k + 30 x 2k = 10350


⇒ k = Rs.15

Thus, daily wages of each man = 5k = Rs.15 x 5 = Rs. 75

97. The population of a town is 3,11,250. The ratio between women and men is
43 : 40. If there are 24% literate among men and 8% literate among women,
the total number of literate persons in the town is
A. 56,800

B. 99,600
P a g e 55 | 580
C. 41,800
D. 48,900

Answer & Explanation :


Answer: Option D

Explanation :
Total number of literate people is 24 % of men and 8 % of women.
Therefore, No of literate persons = (24/100 × 40/83 × 311250) + (8/100 ×
43/83 × 311250) = 48900

98. In an examination, 52% of the candidates failed in English and 43% failed
in Mathematics. If 17% failed in both the subjects, then the percentage of
candidates, who passed in both the subjects, was
A. 15
B. 22

C. 23
D. 21

Answer & Explanation :


Answer: Option B
Explanation :

% of students failed in Mathematics only = 43% – 17% = 26%


% of students failed in English only = 52% – 17% = 35%. So % of students
passed in both = 100 – (26 + 35 + 17) = 100 – 78 = 22%

Alternate Solution:
% of students failed in Mathematics or English or both = 52% + 43% – 17% =
78%

So % of students passed in both = 100% – 78% = 22%. Hence, option B is


correct.
99. A is thrice as efficient as B and takes 10 days less to do a piece of work
than B takes to do the same work. In how many days, B alone can finish the
whole work?
P a g e 56 | 580
A. 15 days
B. 10 days

C. 9 days
D. 8 days

E. 7 days
Answer & Explanation :
Answer: Option A
Explanation :
Efficiency A:B = 3:1

Therefore, no. of days A:B = 1:3


Given, 3x – x = 10
Solving, A = 5 days and B= 15 days.

100. The compound interest on a certain sum for 2 years at 10% per annum is
Rs. 525. The simple interest on the same sum for double the time at half the
rate percent per annum is ________

A. Rs. 400
B. Rs. 500
C. Rs. 600

D. Rs. 800
E. Rs 550

Answer & Explanation :


Answer: Option B

Explanation :
Let 'P' be the principle
P(1.1)2 – P = 525

P = 2500
S.I = (2500*4*5)/100 = 500

P a g e 57 | 580
101. The cost price of item B is Rs. 150/- more than the cost price of item A,
Item A was sold at a profit of 10% and Item B was sold at a loss of 20%. If the
respective ratio of selling price of items A and B is 11:12, what is the cost price
of item B?

A. Rs. 450/-
B. Rs. 420/-
C. Rs. 400/-
D. Rs. 350/-
E. Rs. 480/-

Answer & Explanation :


Answer: Option A
Explanation :
Let us assume cost price of A= X
So that Cost price of B= X+150.

SP of A= X*1.1
SP of B=(X+150)*0.8

Given that
SPA: SPB
11:12

So that 1.1X/(X+150)*0.8= 11/12


X=300

CP of B= 300+150=450
Answer is 450.

102. A vessel contains a mixture of milk and water in the respective ratio of 10:
3. Twenty-six litres of this mixture was taken out and replaced with 8 litres of
water. If the resultant respective ratio of milk and water in the mixture was 5 :
2, what was the initial quantity of mixture in the vessel? (in litres)
A. 143
B. 182

P a g e 58 | 580
C. 169
D. 156

E. 130
Answer & Explanation :

Answer: Option A
Explanation :
26 L mixture is taken out.
Quantity of Milk is taken out= 26*(10/13)=20
Quantity of Water is taken out=26*(3/13)=6

As we know that,
(10X-20)/(3X-6+8)=5/2
X=10,
Initial quantity of mixture in the vessel is= 13*X=13*10=130L.

103. There are 6 consecutive odd numbers. The difference between the square
of the average of the first three numbers and the square of the average of the
last three numbers is 288. What is the last odd number?
A. 31
B. 27

C. 29
D. 25

E. 33
Answer & Explanation :

Answer: Option C
Explanation :
Let the 6 consecutive odd no.'s are:

X, X+2, X+4,X+6, X+8, X+10


Avg. of 1st three no’s is X+2.
Avg. of Last three no’s is X+8.
P a g e 59 | 580
Given that (X+8)2-(X+2)2=288
X=19

Last odd no. is X+10= 29.


Answer is 29.

104. In a bag there are 6 red balls and 9 green balls. Two balls are drawn at
random, what is the probability that at least one of the balls drawn is red?
A. 29/35
B. 7/15

C. 23/35
D. 2/5
E. 19/35
Answer & Explanation :
Answer: Option C
Explanation :
Probability of at least one of the balls drawn is red= 1- (9/15)*(8/14) =23/35.

Answer is 23/35.

DIRECTIONS for question 7 to 9: Study the following information and


answer the question.
What approximate value will come in place of the question mark (?) in the given
question? (You are not expected to calculate exact value)

105. ?% of 750.11 × 34.90 +6.995 = 3000


A. 15
B. 16

C. 11
D. 6

E. 19

P a g e 60 | 580
Answer & Explanation :
Answer: Option C

Explanation :
X% of 750 *34+7=3000

11.40. (approx)
Answer is 11.40

106. 815.002 +29.98 – 53.998 + 3.012= ?


A. 800

B. 880
C. 840
D. 900
E. 750
Answer & Explanation :
Answer: Option A
Explanation :

815+30-54+32=?
800. (approx)
Answer is 800.

107. 40.1% of 360.2 + 58.98% of ? = 150


A. 10
B. 20

C. 30
D. 40
E. 50

Answer & Explanation :


Answer: Option A

P a g e 61 | 580
Explanation :
40*360+59%of X=150

144+59%of X=150
X=10(approx)

108. A is thrice as efficient as B. A started working and after 4 days he was


replaced by B. B then worked for 15 days and left. If A and B together finished
75% of the total work, in how many days B alone can finish the whole work?
A. 27

B. 45
C. 24
D. 36
E. 42
Answer & Explanation :
Answer: Option D
Explanation :

A = 3B, Given:
4A + 15B = ¾ W => 27B = ¾ W
=> B = 36 days.

109. A toothpaste manufacturer is giving two different offers on its 100 g tube.
The first one is giving 30% extra at the same price, and the second one is
giving 30% off on the marked price. By what percentage is the first offer
costlier than the second one?
A. 4.2%
B. 2.4%
C. 9.89%

D. 0%
Answer & Explanation :

Answer: Option C
Explanation :
P a g e 62 | 580
Suppose the price of the toothpaste is Rs. 10. In the first offer, the price per
gram would be Rs. 10/130 while in the second offer, the price per gram would
be Rs. 7/100.
So the first offer is costlier than the second offer by 10/130 – 7/100 = Rs.
9/1300. Hence the first offer is costlier than the second one by 9/1300 ÷ 7/100
× 100 = 9.89%.

110. A covered wooden box has the inner measures as 115 cm, 75 cm and 35
cm and the thickness of wood is 2.5 cm. Find the volume of the wood.

A. 82125 cu. Cm
B. 81775 cu. Cm

C. 81000 cu. Cm
D. None of the above
Answer & Explanation :
Answer: Option A
Explanation :

Required volume = Outer volume of the box - Inner volume = (120 × 80 × 40)
– (115 × 75 × 35) = 82125 cu. cm.

111. From the top of a light house 60 meters high with its base at the sea level,
the angle of depression of a boat is 15°. The distance of the boat from the foot
of the light house is (in m)
A. (√3-1)/(√3+1) × 60

B. (√3+1)/(√3-1) × 60
C. (√3+1)/(√3-1)
D. (√3-1)/(√6-1)

Answer & Explanation :


Answer: Option B
Explanation :

P a g e 63 | 580
112. The area of the triangle formed by the tangents from the point (4,3) to the
circle x2 + y2 = 9 and the line joining their points of contact is:

A. 25/192 square units


B. 192/25 square units
C. 385/25 square units
D. 185/25 square units
Answer & Explanation :

Answer: Option B
Explanation :

P a g e 64 | 580
113. The Hussain Sagar Express travels from Mumbai to Hyderabad. Along the
way the train stops at 18 different places. So, in total, there are 20 stops
including stop at Mumbai and Hyderabad. How many different tickets can be
given out by the railways?
A. 190 different tickets
B. 380 different tickets
C. 145 different tickets

D. 120 different tickets


Answer & Explanation :
Answer: Option B
Explanation :
As there are 20 stations in total, so number of tickets = 20P2 = 380.

6. Sea water contains 5% salt by weight. How many kilograms of fresh water
must be added to 40 kg of sea water so that the salt content of the solution
becomes 2%?

A. 60 kg
B. 50 kg

P a g e 65 | 580
C. 40 kg
D. 55 kg

Answer & Explanation :


Answer: Option A

Explanation :
5% salt by weight means 2 kgs out of 40 kgs is salt.
Let x be the total amount of mixture
Therefore, 2% of x = 2 → x = 100 kg
So the amount of fresh water to be added in the mixture is 100 – 40 = 60 kg

114. A spherical ball 6 cm in diameter is melted and recast into 3 smaller


spherical balls. The radius of the two of the there are 1.5 cm and 2 cm
respectively. The radius of the third is
A. 3.5cm
B. 3.0cm
C. 2.0cm

D. 2.5cm
Answer & Explanation :

Answer: Option D
Explanation :

P a g e 66 | 580
115. 18 litres of pure water was added to a vessel containing 80 litres of pure
milk. 49 litres of the resultant mixture was then sold and some more quantity of
pure milk and pure water was added to the vessel in the respective ratio of 2 :
1. If the resultant respective ratio of milk and water in the vessel was 4 : 1,
what was the quantity of pure milk added in the vessel? (in litres)
A. 4
B. 8
C. 10
D. 12

E. 2
Answer & Explanation :
Answer: Option E
Explanation :
80(M) + 18(W) = 98

49 liters sold => 49 is left


40(M) + 9(W)

Let x be the quantity of pure milk added


Given, (40 + 2x)/(9 + x) = 4/1
Solving, x = 2

116. The respective ratio of radii of two right circular cylinders (A and B) is 4 :
5. The respective ratio of volume of cylinders A and B is 12:25. What is the
respective ratio of the heights of cylinders A and B?
A. 2 : 3
B. 3 : 5
C. 5 : 8

D. 4 : 5
E. 3 : 4
Answer & Explanation :
Answer: Option E

P a g e 67 | 580
Explanation :
Volume = πr2h

π*4*4*hA / π*5*5*hB = 12/25


Solving, hA/ hB = 3:4

117. Dhruva gave 35% of her monthly salary to her mother. From the
remaining salary, she paid 18% towards rent and 42% she kept aside for her
monthly expenses. The remaining amount she kept in bank account. The sum of
the amount she kept in bank and that she gave to her mother was Rs. 43,920.
What was her monthly salary?
A. Rs. 80,000
B. Rs. 75,000
C. Rs. 64,000
D. Rs. 76,000

E. Rs. 72,000
Answer & Explanation :

Answer: Option E
Explanation :
Let ‘x’ be the monthly salary, then

(65/100 × 40/100)x + 35/100x = 43920


Solving, X= 72000

DIRECTIONS for the questions 4-6: What approximate value will come
in place of question mark (?) in the given question?( You are not
expected to calculate the exact value.)

118. 26.003 - (154.001/6.995) = ?


A. 4
B. 18
C. 9

P a g e 68 | 580
D. 10
E. 14

Answer & Explanation :


Answer: Option A

Explanation :
26 – (154/7) = 4

119. 17.995/3.01 + 104.001/12.999 = ?


A. 11

B. 20
C. 23
D. 14
E. 17
Answer & Explanation :
Answer: Option D
Explanation :

18/3 + 104/13 = 14

120. 3/5 of 4/7 of 7/9 of 425 = ?

A. 121
B. 110
C. 118
D. 113

E. 124
Answer & Explanation :
Answer: Option D

Explanation :
3/5*4/7*7/9*425 = 113

P a g e 69 | 580
121. A number is such that when it is multiplied by ‘8’, it gives another number
which is as much more than 153 as the original number itself is less than 153.
What is 25% of the original number?

A. 8
B. 7.5
C. 10
D. 8.5
E. 6.5

Answer & Explanation :


Answer: Option D
Explanation :
Let the no. be X
Given 8X-153=153-X
Hence X=34
25% of X=8.5

122. A and B can complete a piece of work in 80 days and 120 days
respectively. They started working together but A left after 20 days. After
another 12 days C joined B and they completed the work in 28 more days. In
how many days can C alone complete the work?
A. 110 days
B. 112 days
C. 114 days
D. 120 days
Answer & Explanation :

Answer: Option B
Explanation :

E. None of these
So efficiency of A and B are 3 units and 2 units respectively.
P a g e 70 | 580
As they worked for 20 days together after that A left so total unit contribution in
20 days= (3+2) units * 20 days = 100 units

Remaining units = 240 – 100 = 140 units


After another 12 days C joined B and they completed the work in 28

more days, so total units contribution of B in 40 days = 2*40 = 80 units


Remaining units i:e = 140-80 = 60 units
Now 60 units is done by C in 28 days
So to do 240 units C require = 28/60 *240 = 112 days

123. A man sets out on cycle from Delhi to Faridabad, and at the same time
another man starts from Faridabad on cycle for Delhi. After passing each other
they complete their journeys in 2 6 7 and 5 3 5 hours respectively. At what rate
does the second man cycles if the first man cycles at 14 kmph?
A. 10 kmph

B. 5 kmph
C. 7 kmph

D. 8 kmph
E. None of these
Answer & Explanation :

Answer: Option E
Explanation :
Speed1/speed2 =√time2/√time1
14 km/h/speed2 = √535/√267

We get Speed2 =√97.81 = 9.88 km/h

124. In an examination, a student scores 4 marks for every correct answer and
loses 1 mark for every wrong answer. A student attempted all the 200
questions and scored in all 200 marks. The number ofquestions, he answered
correctly was:
A. 82

P a g e 71 | 580
B. 80
C. 68

D. 60
E. None of these

Answer & Explanation :


Answer: Option B
Explanation :
Total questions = 200
Attempted questions = 200

The student scores 4 marks for every correct answer and loses 1 mark for every
wrong answer and he scored 200 marks.
Maximum marks = 200×4 = 800
Scored marks = 200
So marks deducted = 800-200 = 600
For each wrong answer 5 marks are deducted.
So total wrong answers = 600/5 = 120

Correct answers = 80
125. There are 6 consecutive odd numbers. The difference between the square
of the average of the first three numbers and the square of the average of the
last three numbers is 288. What is the last odd number?
A. 31

B. 27
C. 29

D. 25
E. 33
Answer & Explanation :

Answer: Option C
Explanation :
Let the 6 consecutive odd no.’s are:
P a g e 72 | 580
X, X+2, X+4, X+6, X+8, X+10
Avg. of 1st three no’s is X+2.

Avg. of Last three no’s is X+8.


Given that (X+8)2-(X+2)2=288

X=19
Last Odd no. is X+10= 29.
Answer is 29.

126. In a bag there are 6 red balls and 9 green balls. Two balls are drawn at
random, what is the probability that at least one of the balls drawn is red?
A. 29/35
B. 7/15
C. 23/35
D. 2/5
E. 19/35
Answer & Explanation :

Answer: Option C
Explanation :
Probability of atleast one of the balls drawn is red= 1- (9/15) x (8/14)=23/35.

Answer is 23/35.

127. A started a business with an investment of Rs. 28,000. After 5 months


from the start of the business, B and C joined with Rs. 24,000and Rs. 32,000
respectively and withdrew Rs. 8000 from the business. If the difference
between A’s share and B’s share in the annual profit is Rs. 2,400, what was the
annual profit received?

A. Rs. 15,600
B. Rs. 14,400

C. Rs. 14,040
D. Rs. 15,360
P a g e 73 | 580
E. Rs. 13,440
Answer & Explanation :

Answer: Option B
Explanation :

Equivalent Contribution of A= 28000 x5+20000 x 7= 280000


Equivalent Contribution of B= 24000x 7= 168000
Equivalent Contribution of C= 32000 x 7= 224000
Let total profit be X.
Given that,

280000X/672000 – 168000X/672000=2400
112000/672000 x X=2400
or X=2400 x 672/112
X=14400

128. At present, Ami’s age is twice Dio’s age and Cami is two years older than
Ami. Two years ago, the respective ratio between Dio’s age at that time and
Cami’s age at that time was 4 : 9. What will be Ami’s age four years hence?
A. 40 years
B. 30 years

C. 42 years
D. 36 years

E. 48 years
Answer & Explanation :

Answer: Option A
Explanation :
D; A = 2D; C = A+2 = 2D+2

Given, D-2/(2D+2)-2 = 4/9


Solving, D = 18 Years and A = 36+4 = 40 years.

P a g e 74 | 580
129. A and B can complete a piece of work in 80 days and 120 days
respectively. They started working together but A left after 20 days. After
another 12 days C joined B and they completed the work in 28 more days. In
how many days can C alone complete the work?

A. 110 days
B. 112 days
C. 114 days
D. 120 days
E. None of these

Answer & Explanation :


Answer: Option B
Explanation :
So efficiency of A and B are 3 units and 2 units respectively.
As they worked for 20 days together after that A left so total unit contribution in
20 days= (3+2) units * 20 days = 100 units
Remaining units = 240 – 100 = 140 units

After another 12 days C joined B and they completed the work in 28


more days, so total units contribution of B in 40 days = 2*40 = 80 units
Remaining units i:e = 140-80 = 60 units

Now 60 units is done by C in 28 days


So to do 240 units C require = 28/60 *240 = 112 days
130. In a mathematics exam, a student scored 30% in the first paper out of a
total of 180. How much should he score in the second paper (out of 150) if he is
to get at least 50% marks overall?
A. 75%
B. 80%

C. 74%
D. 84%
Answer & Explanation :
Answer: Option C

P a g e 75 | 580
Explanation :
(30/100) x 180 + (x/100) x 150 = (50/100) x 330 ⇒ x = 74

131. If 15 men or 24 women or 36 boys can do a piece of work in 12 days,


working 8 hours a day, how many men must be associated with 12 women and
6 boys to do another piece of work 21⁄4 times as great in 30 days working 6
hours a day?

A. 4
B. 8
C. 6
D. 10
Answer & Explanation :
Answer: Option B
Explanation :
Total man days = 15 x 12 x 8

= (x + (15/2) + (15/6) x 6 x (4/9) ⇒ x = 8

132. Two cogged wheels of which one has 16 cogs and the other 27, work into
each other. If the latter turns 80 times in three-quarters of a minute, how often
does the other turn in 8 seconds?
A. 18
B. 30
C. 24
D. 36
Answer & Explanation :
Answer: Option C
Explanation :

No of times the 16 cogs wheel size move in 8 seconds is (27 * 80)/45 * 8/16

P a g e 76 | 580
133. A milkman buys milk contained in 10 vessels of equal size. If he sells his
milk at Rs. 5 a litre, he loses Rs. 200. If he sells it at Rs. 6 a litre, he would gain
Rs. 150 on the whole. Find the number of litres contained in each vessel.
A. 20 litres

B. 30 litres
C. 25 litres
D. 35 litres
Answer & Explanation :
Answer: Option D

Explanation :
Let the Milk purchased be x
Let the Total Cost of Purchasing be x litre milk be y
When x Litre milk is sold for Rs. 5 the loss is Rs. 200
the equation can be written as:
y - 5x = 200
When x Litre milk is sold for Rs. 6 the profit is Rs. 150

the equation can be written as:


6x - y = 150
By solving the two equations:

y - 5x = 200
-y + 6x = 150

we get x = 350Litres. For 10 vessels we can have 35 litres

134. 16 January 1997 was a Thursday. What day of the week was 4 January
2000?
A. Tuesday

B. Wedsday
C. Thursday

D. Friday

P a g e 77 | 580
Answer & Explanation :
Answer: Option A

Explanation :
16th Jan 1997 – Thursday

16th Jan 1998 – Friday


16th Jan 1999 – Saturday
4th Jan 2000 – Tuesday

135. Out of a group of swans, 7/2 times the square root of the number are
playing on the shore of a pond. The remaining two are inside the pond. What is
the total number of swans?
A. 10
B. 14
C. 12
D. 16
Answer & Explanation :

Answer: Option D
Explanation :
7√x / 2 + 2 = x

Check by option.

136. A wooden box of dimensions 8 m × 7 m × 6m is to carry rectangular


boxes of dimension: 8 cm × 7 cm × 6 cm. The maximum number of boxes that
can be carried in the wooden box is
A. 98,00,000
B. 10,00,000

C. 75,00,000
D. 12,00,000

Answer & Explanation :

P a g e 78 | 580
Answer: Option B
Explanation :

(800 x 700 x 600 / 8 x 7 x 6) = 10,00,000

137. The horizontal distance between two towers is 60 m. The angular elevation
of the top of the taller tower as seen from the top of the shorter one is 30°. If
the height of the taller tower is 150 m, the height of the shorter one is
A. 116 m
B. 216 m

C. 200 m
D. None of these
Answer & Explanation :
Answer: Option A
Explanation :
tan30 = (150 - x /60) ⇒(1/√3) = (150 - x/60)

On Solving, we get x = 116

138. An aeroplane travels distances of 2500 km, 1200 km and 500 km at


speeds of 500 km/hr, 400 km/hr and 250 km/hr respectively. The average
speed is
A. 420 km/hr
B. 405 km/hr

C. 410 km/hr
D. 575 km/hr

Answer & Explanation :


Answer: Option A
Explanation :
Total distance = 2500 + 1200 + 500 = 4200 km
Total time taken = 2500/500 + 1200/400 + 500/250 = 10

P a g e 79 | 580
Avg speed = Total distance/Total Time Taken = 4200/10 = 420 km/hr
139. A and B are partners in a business. They invest in the ratio 5:6, at the end
of 8 months A withdraws. If they receive profits in the ratio of 5 : 9, find how
long B's investment was used?

A. 12 months
B. 10 months
C. 15 months
D. 14 months
E. 18 months

Answer & Explanation :


Answer: Option A
Explanation :
Ratio of profit is always distributed in the ratio of their investment and time.
5unit × 8 months:6 units × B’s months = 5:9
So B’s investment time = 12 months

140. There are 3 red balls, 4 blue balls and 5 white balls. 2 balls are chosen
randomly. Find probability that 1 is red and the other is white.
A. 5/22

B. 5/23
C. 7/22

D. 4/9
E. None of these

Answer & Explanation :


Answer: Option E
Explanation :

There are 3 red balls, 4 blue balls and 5 white balls. 2 balls are chosen
randomly.

probability that 1 is red and the other is white = 3/12× 5/11 = 5/44

P a g e 80 | 580
141. According to a new plan rolled out by H1SP Bank, the rate of simple
interest on a sum of money is 8% p.a. for the first two years, 10% p.a. for the
next three years and 6% p.a. for the period beyond the first five years. Simple
interest accrued on a sum for a period of eight years is Rs. 12,800. Find the
sum.
A. Rs. 24,000
B. Rs. 16,000
C. Rs. 15,000

D. Rs. 13,500
E. None of these
Answer & Explanation :
Answer: Option E
Explanation :

Let the sum of money be x.


So interest in first two years = 8×2 = 16% of x

Interest in next three years = 10×3 =30% of x


Interest in next five years = 6×3 = 18% of x
Total interest = 64% of x = Rs 12,800

We get x = Rs 20,000

142. Three Science classes A, B and C take a Life Science test. The average
score of students of class A is83. The average score of students class B is 76.
The average score of class C is 85. The average score of class A and 8 is 79 and
average score of class B and C is 81. Then the average score. Of classes A, B
and C is

A. 80
B. 80.5
C. 81
D. 81.5
E. None of these
P a g e 81 | 580
Answer & Explanation :
Answer: Option D

Explanation :
A:B:C = 3:4:5

Sum of A+B+C = 3*83+4*76+5*85 = 978


Average score = 978/12 = 81.5

143. A hemispherical bowl of internal diameter 54 cm contains a liquid. The


liquid is to be filled in cylindrical bottles of radius 3 cm and height 9 cm. How
many bottles are required to empty the bowl?
A. 221
B. 343
C. 81
D. 243
E. None of these
Answer & Explanation :

Answer: Option E
Explanation :
Area of a hemispherical bowl = 2/3 ×π ×r3

Area of a cylinder = πr2h


Area of a cylinder = n × Area of a hemispherical bowl

2/3 ×π ×273 = n ×π×32×9


we get n = 162

DIRECTIONS for the questions 6 & 10: What approximate value will
come in place of question mark (?) in the given question?( You are not
expected to calculate the exact value.)

144. 26 .00 - (154.001/6.995) = ?

P a g e 82 | 580
A. 4
B. 18

C. 9
D. 10

E. 14
Answer & Explanation :
Answer: Option A
Explanation :
26 – (154/7) = 4

145. 17.995/3.01 + 104.001/12.999 = ?


A. 11
B. 20
C. 23
D. 14
E. 17

Answer & Explanation :


Answer: Option D
Explanation :

18/3 + 104/13 = 14

146. 3/5 of 4/7 of 7/9 of 425 = ?


A. 121

B. 110
C. 118
D. 113

E. 124
Answer & Explanation :

P a g e 83 | 580
Answer: Option D
Explanation :

3/5*4/7*7/9*425 = 113

147. 124.001 14.001 ÷ 3.4999 + 22 = ?


A. 500
B. 450
C. 425
D. 475

E. 550
Answer & Explanation :
Answer: Option A
Explanation :
124*(14/3.5)+ 4 = 500

148. 18.0009 ÷ √ (369 + ?) = 2040.05

A. 302
B. 298
C. 322

D. 319
E. 311
Answer & Explanation :
Answer: Option E

Explanation :
18/√36 * (369 + ?)= 2040.05
3(369+?)=2040

?=311

P a g e 84 | 580
149. A number x when divided by 289 leaves 18 as a remainder. The same
number when divided by 17 leaves y as a remainder. The value of y is

A. 3
B. 1

C. 5
D. 2
Answer & Explanation :
Answer: Option B
Explanation :

Here, the first divisor (289) is a multiple of second divisor (17)


∴ Required remainder = Remainder obtained on dividing 18 by 17 = 1. Hence
the answer is option B

150. An equation of the form ax + by + c = 0 where a ≠ 0, b ≠ 0, c = 0


represents a straight line which passes through

A. (0, 0)
B. (3, 2)
C. (2, 4)
D. None of these
Answer & Explanation :
Answer: Option A
Explanation :
Ax+by+c = 0
When c = 0

ax+by = 0
by = -ax ⇒ y = - ax/b
when x = 0, y = 0 i.e., this line passes through the origin (0,0).

P a g e 85 | 580
151. The numerator of a fraction is 4 less than its denominator. If the
numerator is decreased by 2 and the denominator is increased by 1, then the
denominator becomes eight times the numerator. Find the fraction.
A. -4/8

B. 2/7
C. 3/8
D. 3/7
Answer & Explanation :
Answer: Option D

Explanation :
Original fraction = (x - 4)/x
In case II,
8(x - 4 - 2) = x + 1
⇒ 8x - 48 = x + 1
⇒ 7x = 49 ⇒ x = 7

∴Original fraction
= (7 - 4)/7 = 3/7

152. Determine the probability that a digit chosen at random from the digits 1,
2, 3, ....., 9 will be a multiple of 3.
A. 1/5
B. 1/3
C. 2/5

D. 3/5
E. 4/7
Answer & Explanation :

Answer: Option B
Explanation :

P a g e 86 | 580
Total possible outcomes = 9. Favorable outcomes = 3. (i.e. 3, 6 OR 9 one out of
three)

Probability =3/9 = 1/3

153. What is the probability that a coin will turn up heads at least once in six
tosses of a coin?
A. 11/53
B. 41/53
C. 63/64

D. 1/64
E. None of these
Answer & Explanation :
Answer: Option C
Explanation :
Reqd. Probability = 1 - Probability of Not Getting Even One Head = 1 - (1/64)
=63/64

154. The simple interest on Rs. 4000 in 3 years at the rate of x% per annum
equals the simple interest on Rs. 5000 at the rate of 12% per annum in 2 years.
The value of x is
A. 8%
B. 9%
C. 10%
D. 6%
Answer & Explanation :
Answer: Option C

155. Even after reducing the marked price of a transistor by Rs. 32, a
shopkeeper makes a profit of 15%. If the cost price be Rs. 320, what
percentage of profit would he have made if he had sold the transistor at the
marked price?
P a g e 87 | 580
A. 25%
B. 20%

C. 10%
D. 15%

E. None of these
Answer & Explanation :
Answer: Option A
Explanation :
Let x be the marked price,

So x - 32 = 320 × 1.15 ⇒ x = 400.


So required value is
400 = 320 (1 + profit/100),
So profit is 25%

156. The sum of five numbers is 260. The average of first two numbers is 30
and average of the last two numbers is 70. What is third number?
A. 33
B. 60
C. 75
D. Can't determined
E. None of these
Answer & Explanation :

Answer: Option B
Explanation :

(a + b + c + d + e)/5 = 260 ...........(1)


(a + b)/2 = 30 , Therefore a + b = 60
(d + e)/2 = 70 , Therefore d + e = 140

Using the values in (1) , We get

P a g e 88 | 580
c= 60
As sum of first two is 60 & sum of last two is 140 so third no. is 260 - 60 - 140
= 60.

157. 20 boys and 32 girls form a group for social work. During their
membership drive same no. of boys and girls joined the group. How many
members does the group have now, if the ratio of boys to girls is 3:4
respectively?
A. 75

B. 86
C. 68
D. 82
E. None of these
Answer & Explanation :

Answer: Option E
Explanation :

Let x be the new boys as well as girls , Therefore


Solving this we get x = 16
So total will be 36 + 48 = 84

158. A copper wire is bent in the form of an equilateral triangle and has area
121√3cm2 . If the same wire is bent into the form of a circle. The area (in cm2)
enclosed by the wire is (take π - 22/7)
A. 364.5
B. 693.5
C. 346.5
D. 639.5

E. None of these
Answer & Explanation :
Answer: Option C

P a g e 89 | 580
159. What will be the ratio of petrol and kerosene in the final solution formed by
mixing petrol and kerosene that are present in three identical vessels in the
ratio 4:1,5:2 and 6 :1 respectively?
A. 166 : 22

B. 83 : 22
C. 83 : 44
D. 78 : 55
E. None of these
Answer & Explanation :

Answer: Option B
Explanation :
Three identical vessels in the ratio 4:1, 5:2 and 6:1 respectively. Petrol :
kerosene (4 : 1 = 5)7 (5 : 2 = 7)5 (6 : 1 = 7)5 28 : 7 =35 25 : 10 =35 30 : 5
=35 83 : 22

160. Mrs. Sharma invests 15% of her monthly salary, i.e., Rs. 4428 in Mutual
Funds. Later she invests 18% of her monthly salary on Pension Policies also she
invests another 9% of her salary on Insurance Policies. What is the total
monthly amount invested by Mrs. Sharma?
A. Rs. 113356.8
B. Rs. 12398.4
C. Rs. 56678.4
D. Can't determined
E. None of these
Answer & Explanation :
Answer: Option B

Explanation :
15% of monthly salary = Rs 4428
So monthly salary = Rs 29500
Total money invested = 42% of 29500 = Rs 12398.4

P a g e 90 | 580
161. At present, Aanshi is five years younger to Binny. Binny’s age twenty-
years hence will be equal to twice of Aanshi’s age five years ago. What will be
Binny’s age eight year hence?

A. 42 years
B. 35 years
C. 30 years
D. 48 years
E. None of these

Answer & Explanation :


Answer: Option D
Explanation :
Let age of Aanshi be A, Bunny be B.
According to equation, A= B-5
Also, B+20 = 2(B-10)
Solving these we get, B= 40. So after 8 years, his age will be 48 years.

162. In a bag, there are 8 red balls and 7 green balls. Three balls are picked at
random. What is the probability that two balls are red and one ball is green in
colour?
A. 28/65
B. 22/65
C. 37/65

D. 3/13
E. 1/13
Answer & Explanation :

Answer: Option A
Explanation :

Probability that two balls are red and one ball is green in colour = 8*7*7*3! /
15*14*13*2! = 28/65
P a g e 91 | 580
163. A vessel contains 120 litres of mixture of milk and water in the respective
ratio of 11 : 4. Forty-five litres of this mixture was taken out and replaced with
5 litres of water. What is the percentage of water in the resultant mixture?

A. 1.35
B. 31.25
C. 25
D. 20
E. 15

Answer & Explanation :


Answer: Option B
Explanation :
Total volume 120 litres
Milk Water
88 32
if 45 litres of mixture is removed 5/8 of the mixture is left

Milk Water
55 20
5 litres water is added

Milk Water
55 25

% of water in the mixture= 25*100/ 80 = 31.25%

164. What would be the compound interest accrued on an amount of Rs. 8400
at the rate 12.5% per annum at the end 3 yr? (Rounded, off to two digits after
decimal)

A. Rs. 420.62
B. Rs. 2584.16

C. Rs. 3560.16

P a g e 92 | 580
D. Rs. 3820.14
E. None of these

Answer & Explanation :


Answer: Option C

165. Twice the speed of a boat downstream is equal to thrice the speed
upstream. The ratio of its speed in still water to the speed of current is
A. 1 : 5
B. 1 : 3

C. 5 : 1
D. 2 : 3
Answer & Explanation :
Answer: Option C
Explanation :
Let the boat speed in still water be b.
Let the stream speed be x.

2(b+ x) = 3(b-x)
5x=b
b/x=5/1

166. How many terms are there in an A.P. whose first and fifth terms are -14
and 2, respectively, and the sum of terms is 40?
A. 15

B. 10
C. 5
D. 20

Answer & Explanation :


Answer: Option B
Explanation :
P a g e 93 | 580
Now the common difference of this AP is 16/4 = 4.
The sum of an AP is n/2 {2a + (n – 1)d}

Substituting we get, 40 = n/2 {2×-14 + (n – 1)4}


The best way to solve this is by plugging options. Put in n = 10 and get the RHS
as 40.

167. A bottle is full of Dettol. One-third of it is taken out and then an equal
amount of water is poured into the bottle to fill it. This operation is done four
times. Find the final ratio of dettol and water in the bottle.

A. 13 : 55
B. 20 : 74
C. 16 : 65
D. 10 : 48
Answer & Explanation :

Answer: Option C
Explanation :

As in denominator we have to take 1/3 four times so, we start by assuming 81


ml of dettol in the bottle. After the first iteration you will be left with 2/3 × 81 =
54 ml. After the second iteration you will be left with 2/3 × 54 = 36 ml. After
the third iteration you will be left with 2/3 × 36 = 24 ml. After the fourth
iteration you will be left with 2/3 × 24 = 16 ml. So the required ratio will be 16
: (81 – 16) = 16 : 65
168. 7 cannibals of XYZ island, decide to throw a party. As you may be aware,
cannibals are guys who eat human beings. The senior among them – Father
Cannibal decides that any 6 of them will eat up one cannibal, then out of the
remaining six – five of them will eat up one cannibal and so on till one is left.
What is the time until one cannibal is left, if it takes one cannibal 3 hours to eat
up one cannibal independently?
A. 7 hrs 11 min
B. 6 hrs 12 min
C. 7 hrs 21 min

D. 18 hrs 16 min

P a g e 94 | 580
Answer & Explanation :
Answer: Option C

Explanation :
At the beginning 6 cannibals will eat one, so time required will be 180/6 = 30
min.
Then out of the remaining six – five will devour one, so time required will be
180/5 = 36 min.
Thus the time until one cannibal is left will be = (180/6 + 180/5 + 180/4 +
180/3 + 180/2 + 180/1) min

= (30 + 36 + 45 + 60 + 90 + 180) min


= 441 min
= 7 hrs 21 min

169. Three articles are purchased for Rs. 1050, each with a different cost. The
first article was sold at a loss of 20%, the second at 1/3rd gain and the third at
60% gain. Later he found that their SPs were same. What was his net
gain/loss?
A. 14.28% gain
B. 13% loss

C. 12% loss
D. 11.11% gain
Answer & Explanation :
Answer: Option A
Explanation :
Let us assume that their CPs are x, y & z respectively.
According to the given condition 0.8x = 1.33y = 1.6z

⇒ (80/100)x = 400y/(3 × 100) = (160/100)z


⇒x:y=5:3&y:z=6:5

Thus x : y : z = 10 : 6 : 5
Hence CPs of the articles are x = (10/21) × 1050 = 500,

P a g e 95 | 580
y = (6/21) × 1050 = 300 &
z = (5/21) × 1050 = 250.

SP of the article with CP Rs. x is 0.8x = 0.8 × 500 = 400.


Since SPs are same, the total SP will be 400 × 3 = 1200.

Hence the gain % = (SP – CP)/CP × 100 = (1200 – 1050)/1050 × 100 =


14.28%.

170. In a game of tennis, A gives B 21 points and gives C 25 points. B gives C


10 points. How many points make the game?

A. 50 points
B. 45 points
C. 35 points
D. 30 points
Answer & Explanation :
Answer: Option C

171. A square, S1, circumscribes the circum circle of an equilateral triangle of


side 10 cm. A square, S2, is inscribed in the in circle of the triangle. What is the
ratio of the area of S1 to the area of S2?

A. 4:1
B. 32:1
C. 8:1
D. 2:1

Answer & Explanation :


Answer: Option C
Explanation :

The height of the equilateral triangle is 5√3 cm.


Since the height is also the median, we know that the circum-radius is 2/3 ×
5√3 = 103/3 and the in-radius is 1/3 × 5√3 = 5√3/3.

P a g e 96 | 580
The diameter of the circumcircle is the side of square S1. So the area of S1 is (2
× 103/3)2 = 1200/9.

The diameter of the in-circle is the diagonal of square S2. So the area of S2 is
½ × (2 × 5√3/3)2 = 300/18.

Thus the ratio of areas S1 : S2 is 1200/9 : 300/18 = 8 : 1.

172. Three casks of equal capacities contain three liquids A, B & C in the ratio 1
: 2 : 3, 3 : 4 : 5 & 5 : 6 : 7 respectively. The mixtures from these casks are
taken in the ratio 1 : 2 : 3 and poured into a 4th cask with the same capacity as
that of the three casks and the cask is completely filled. What is the ratio of the
liquids A, B and C in the resulting mixture?
A. 25:36:47
B. 16:21:26
C. 3:4:5
D. 1:2:3
Answer & Explanation :

Answer: Option C
Explanation :
⇒(1 + 2 + 3) = 6, (3 + 4 + 5) = 12 & (5 + 6 + 7) = 18. Common multiple of
(6, 12, 18) = 36. So let us fix the capacities of the four casks as 36 litres each.
Liquid A Liquid B Liquid C
Cask 1 (36 liters) 6 12 18

Cask 2 (36 liters) 9 12 15


Cask 3 (36 liters) 10 12 14

Since the mixtures are taken in the ratio 1:2:3, 6litres, 12 litres and 18 litres
mixture are drawn from the three casks respectively.
Liquid A Liquid B Liquid C
Cask 1 (6 liters) 1 2 3
Cask 2 (12 liters) 3 4 5

Cask 3 (18 liters) 5 6 7


Cask 3 (36 liters) 9 12 15
P a g e 97 | 580
Hence the ratio of the liquids in the resulting mixture is 9 : 12 : 15 = 3 : 4 : 5
173. A 30% loss on cost price is what percent loss on selling price?

A. 30%
B. 20%

C. 15%
D. None of these
Answer & Explanation :
Answer: Option D
Explanation :

Let CP = 100 ; SP=70


Loss= 30/70 × 100 = 42.85%

174. A, B and C hire a taxi for Rs. 2400 for one day. A, B and C used the car for
6 hours, 8 hours and 10 hours respectively. How much did C pay?
A. Rs. 800
B. Rs. 1000

C. Rs. 600
D. Rs. 1200
Answer & Explanation :

Answer: Option B
Explanation :
Let total fair be = 2400 ;
Therefore c share =10/24 × 2400 = 1000

175. The ratio of investments of A and B is 8 : 7 and the ratio of their yearend
profits is 20 : 21. If B invested for 12 months, then find the period of
investment of A:
A. 6 months

B. 8 months

P a g e 98 | 580
C. 10 months
D. 12 months

Answer & Explanation :


Answer: Option C

Explanation :
Let A invest for x months ; A = 8x months,
B = 7 × 12 = 84 months
8x/84 = 20/21
⇒ x = 10

176. What percent is 2 minutes 24 seconds of an hour?


A. 6%
B. 2%
C. 4%
D. 8%

Answer & Explanation :


Answer: Option C

Explanation :
%=144/60×60 = 4%

177. Evaluate: 3 cos 80° cosec 10° + 2 cos 59° cosec 31°
A. 1

B. 3
C. 2

D. 5
Answer & Explanation :
Answer: Option D

Explanation :

P a g e 99 | 580
3 cos 80°. Cosec 10° + 2 cos 59° . cosec 31°
= 3 cos (90° - 10°). Cosec 10° + 2 cos (90° - 31°).Cosec 31°

=3sin10°.Cosec10° +2sin31°.cosec31°
=3+2=5

178. The total cost of 8 buckets and 5 mugs is Rs. 92 and the total cost of 5
buckets and 8 mugs is Rs. 77. Find the cost of 2 mugs and 3 buckets.
A. Rs. 35
B. Rs. 70

C. Rs. 30
D. Rs. 38
Answer & Explanation :
Answer: Option A
Explanation :
CP of 1 bucket = Rs. X
CP of 1 mug = Rs. Y

∴ 8x + 5y = 92....... (i)
5x + 8y = 77........(ii)
By equation (i) × 5 – equation (ii) × 8.
40x + 25y – 40x – 64y
= 460 – 616 ⇒ − 39y = - 156⇒ y = 4

From equation (i),

8x + 20 = 92 ⇒8x = 92 – 20 = 72 ⇒ x = 9
∴ CP of 2 mugs and 3 buckets

= 2 × 4 + 3 × 9 = 8 + 27 = Rs. 35

179. If 4x/3 + 2P = 12 for what value of P, x = 6?


A. 6

P a g e 100 | 580
B. 4
C. 2

D. 1
Answer & Explanation :

Answer: Option C
Explanation :
When x = 6, (4 * 6)/3 + 2P = 12
⇒ 8 + 2P = 12

⇒ 2P = 12 – 8 = 4
⇒P=2

179. The straight line 2x + 3y = 12 passes through:


A. 1st, 2nd and 3rd quadrant
B. 1st, 2nd and 4th quadrant
C. 2nd, 3rd and 4th quadrant
D. 1st, 3rd and 4th quadrant
Answer & Explanation :
Answer: Option B
Explanation :
The usual way to solve these type of questions is to put x = 0 once and find y
coordinate. This would represent the point where the line cuts the Y axis.
Similarly put y = 0 once and find x coordinate. This would represent the point
where the line cuts the X axis. Then join these points and you will get the graph
of the line.
So when we put x = 0 we get y = 4.
When we put y = 0 we get x = 6.

So when we join these points we see that we get a line in 1st quadrant, which
when extended both sides would go to 4th and 2nd quadrants. So option B.

P a g e 101 | 580
180. In ΔABC, ∠A + ∠B = 65°, ∠B + ∠C = 140°, then find ∠B.

A. 40°

B. 25°
C. 35°
D. 20°
Answer & Explanation :
Answer: Option B

Explanation :
∠A + ∠B = 65°
∴ ∠C = 180° - 65° = 115°
∠B + ∠C = 140°

∴ ∠B = 140° - 115° = 25°


181. The retail price of a water geyser is Rs. 1,265. If the manufacturer gains
10 %, the wholesale dealer gains 15 % and the retailer gains 25 %, then the
cost of the product is:
A. 800
B. 900
C. 700
D. 600
Answer & Explanation :
Answer: Option A
Explanation :

C.P = 1265*100*100*100/110/115/125
C.P = 800

182. What percent of selling price would be 34 % of cost price if gross profit is
26 % of the selling price?

A. 17.16
B. 74

P a g e 102 | 580
C. 25.16
D. 88.40

Answer & Explanation :


Answer: Option C

Explanation :
X% of SP = 34% of CP
Also, P = 26% of SP
SP - CP = 0.26(SP)
CP = 0.74(SP)

Now, (34/100)×74
X = 25.16

183. The tax on a commodity is diminished by 10 % and its consumption


increased by 10 %. The effect on the revenue derived from it changes by K %.
Find the value of K.
A. 1.

B. -2
C. -1
D. 2

Answer & Explanation :


Answer: Option C

Explanation :
Directly using the formula, when a value is increased by R% and then
decreased by R%, then net there is ( R∧2)/100 decrease. Putting R = 10, we
get 1% decrease.

184. Ratio of Ashok's age to Pradeep's age is 4 : 3. Ashok will be 26 years old
after 6 years. How old is Pradeep now?
A. 18

B. 21
P a g e 103 | 580
C. 15
D. 24

Answer & Explanation :


Answer: Option C

Explanation :
Given A/p= 4/3 Also A = 26 after 6 years, so his present age = 20years,
Substituting we get P = 15 years.

185. The incomes of Chanda and Kim are in the ratio 9 : 4 and their
expenditures are in the ratio 7 : 3. If each saves Rs. 2,000, then Chanda's
expenditure is
A. 60000
B. 80000
C. 70000

D. None of these
Answer & Explanation :

Answer: Option C
Explanation :
Let the incomes of Chanda and Kim be 9x and expenditures be 7y and 3y
respectively. Since = Income – Expenditure, we get 9x – 7y = 2000 and 4x –
3y = 2000. Solving, we get, x = 8000 and y = 10000. So Chanda’s expenditure
= 7y = 7 × 10000 = Rs. 70,000.

186. A student purchased a computer system and a colour printer. If he sold


the computer system at 10 % loss and the colour printer at 20 % gain, he
would not lose anything. But if he sells the computer system at 5 % gain and
the colour printer at 15 % loss, he would lose Rs. 800 in the bargain. How much
did he pay for the colour printer?
A. 8000
B. 16000
C. 9000

P a g e 104 | 580
D. 533 4
Answer & Explanation :

Answer: Option B
Explanation :

Let C and P be the cost price of Computer and Printer respectively.


So CP = C + P, Case I,
SP = 0.9C + 1.2P.
Since he did not lose anything C + P = 0.9C + 1.2P C = 2P.
Case II, SP = 1.05C + 0.85P

Since there was the loss of Rs. 800


Rs.800 = C + P – 1.05C – 0.85P
80000 = 15P – 5C
Using equation from Case I, we get P = Rs.16000.

187. X and Y entered into partnership with Rs. 700 and Rs. 600 respectively.
After 3 months X withdrew 2/7 of his stock but after 3 months, he puts back
3/5 of what he had withdrawn. The profit at the end of the year is Rs. 726. How
much of this should X receive?
A. 336

B. 366
C. 633

D. 663
Answer & Explanation :

Answer: Option B
Explanation :
X’s profit : Y’s profit

= 700 × 3 + 500 × 3 + 620 × 6 : 600 × 12


= 2,100 + 1,500 + 3,720 : 7,200

= 7,320 : 7,200

P a g e 105 | 580
= 61 : 60
X’s share in the profit = 61/(60+61) × 726 = 366

188. A man sitting in a train travelling at the rate of 50 km/hr observes that it
takes 9 sec for a goods train travelling in the opposite direction to pass him. If
the goods train is 187.5 m long, find its speed.
A. 25
B. 45
C. 35

D. 36
Answer & Explanation :
Answer: Option A
Explanation :
Let required speed be x.
So,187.5/{ (x+50)*5/18} =9

189. A runs 5/3 times as fast as B. If A gives B a start of 80m, how far must the
winning post be, so that A and B might reach it at the same time?
A. 200

B. 300
C. 270

D. 160
Answer & Explanation :

Answer: Option A
Explanation :
Ratio of speeds of A : B = 5 : 3, So If A runs 5, B runs 3. So difference in
distance = 2. So if difference is 2, winning post is 5m. Hence if difference is
2.5*80, winning post is Hence 1st option.

P a g e 106 | 580
190. A team of workers was employed by a contractor who undertook to finish
360 pieces of an article in a certain number of days. Making four more pieces
per day than was planned, they could complete the job a day ahead of
schedule. How many days did they take to complete the job?

A. 8
B. 9
C. 10
D. 12
Answer & Explanation :

Answer: Option B
Explanation :
Days taken in the general scenario = 360/N;
Days taken when 4 articles are prepared extra per day = 360/N + 4;
The difference in the day is one, therefore;

360/n - 360/n+4 =1
N2 + 4N – 1440 = 0;

N = 36, i.e. number of item prepared in general scenario is 36, and where 4
articles prepared extra is 40. Therefore no of days taken to complete the job =
360/40 = 9.

191. What is the equation of the line parallel to the line 2x + 3y = 12?
A. 2.4x + 1.8y = 16
B. 3.4x + 5.1y = 8
C. 4.2x + 3.3y = 18
D. 13.2x + 4.8y = 24
Answer & Explanation :
Answer: Option B

Explanation :
Slope of the given line is (-2/3). So, slope of the line parallel to it should be the
same.

P a g e 107 | 580
As only in option 2, (-3.4/5.1) = -2/3 while the other equations do not have
same slope.

Thus option 2 is the answer

192. What is the difference between cost price and the marked price of an
article, sold at a loss of 11% for Rs. 2136 with a discount of 20%?
A. Rs. 178
B. Rs. 267
C. Rs. 240

D. Rs. 270
Answer & Explanation :
Answer: Option D
Explanation :
M.P. = 2136 /0.8 = Rs 2670 & C.P= 2136/0.89 = Rs 2400.
So required difference = 2670 – 2400 = Rs 270.

193. A gives B a start of 200m and still wins a km race by 200m. B gives a start
of 100m and still wins by 100m in another km race with C. How much start
should A give C in a km race in order to beat him by 250m?

A. 270
B. 480
C. 520
D. 230

Answer & Explanation :


Answer: Option A
Explanation :

Ratio of distance travelled by A and B ⇒ A : B = 1000 : 600, while that of B and


C ⇒ B : C = 1000 : 800

Therefore, A : C = 1000 : 480.

P a g e 108 | 580
Since A beats him by 250m therefore C should be 750m away from the starting
point.

So, A should give C a start of (750-480) = 270m in a km race in order to beat


him

194. The simple interest accumulated in 4 years, with interest, R% p.a, on a


particular sum is half of its principal. Find R.
A. 20%
B. 6%

C. 12.5%
D. 8.33%
Answer & Explanation :
Answer: Option C
Explanation :

S.I. = P*R*T/100 ⇒ P/2 = P*R*4/100 ⇒ R = 12.5%

195. If two dices are rolled simultaneously, then what is the probability of
getting the sum of the numbers a prime number?
A. 14/36

B. 5/17
C. 15/29
D. 5/12

Answer & Explanation :


Answer: Option D

Explanation :
Prime no.s = 2, 3, 5, 7 & 11. The no of ways to get :
2 as the sum is 1 i.e. 1 +1
3 as the sum are 2 i.e. 1 + 2, 2 + 1
5 as the sum are 4 i.e. 1 + 4, 4 + 1, 2 + 3, 3 + 2

P a g e 109 | 580
7 as the sum are 6 i.e. 1 + 6, 6 + 1, 2 + 5, 5 + 2, 3 + 4, 4 + 3
11 as the sum are 2 i.e. 5 + 6, 6 + 5

So total no of ways are 1 + 2 + 4 + 6 + 2 = 15


So the probability is 15 / 36 = 5 / 12 (where 36 is total number of possible
cases in case of two dices)

196. A multiplex owner increases the price of a ticket by 25%, because of which
the number of viewers decrease by 25%. What is the percentage change in the
revenue?

A. 6.25% increase
B. 9.375% increase
C. 6.25% decrease
D. 9.375% decrease
Answer & Explanation :

Answer: Option C
Explanation :

Using the formula a + b + ab/100, we get 25 - 25 - (625/100) i.e. -6.25.


So there is decrease of 6.25% in revenue.

197. A and B start a business by investing 5 lks and 4 lks. B remains in the
business for a complete year. How long did A remain in the business, if they
received equal profit at the end of the year?
A. 9.6 months
B. 2.4 months
C. 8.4 months
D. 6.9 months

Answer & Explanation :


Answer: Option A

Explanation :

P a g e 110 | 580
Let N be the time period of A. Since the profits are equal, we get 4 x 12 = 5 x
N,

On solving, we get N = 0.8 year or 9.6 months

198. What is the remainder when addition of all the 2-digit multiples of 9 is
divided by 11?
A. 0
B. 1
C. 2

D. 3
Answer & Explanation :
Answer: Option C
Explanation :
Two digit multiples of 9 are 18, 27, ......, 99 which are 10 in number.
By A.P. formula we can find the addition of 2 digit multiples of 9.
Sum = [10/2(18+99)] = 585. So 585/11, we get remainder as 2.

199. 60% of the students learn German and 50% of the students learn French.
If student in the class learns at least one subject out of the given two, then
what percent of the students do not learn both the subjects?
A. 10
B. 12
C. 80

D. 90
Answer & Explanation :
Answer: Option D

Explanation :
8. Total percentage of both german and english = 50+60 = 110%. So 10%
learn both
Therefore, 100-10 = 90% do not learn both.
P a g e 111 | 580
200. In the inequality, x2 - 14x + 50 < 5, find the range of x.

A. 5 > x > 9
B. 5 < x < 9

C. -5 < x < 9
D. -5 > x > -9
Answer & Explanation :
Answer: Option B
Explanation :

x2 - 14x + 50 < 5 ⇒ (x-9)(x-5)<0


Now, here 2 cases arises
Case 1: (x-9)<0 & (x-5)>0
So we get range to be 5 < x < 9
Case 2: (x-9)>0 & (x-5)<0
Which gives x > 9 and x < 5 which cannot occur simultaneously, hence
rejected.
201. Percent profit earned when an article is sold for Rs. 546/- is double the
percent profit earned when the same article is sold for Rs. 483/-. If the marked
price of the article is 40% above the cost price, what is the marked price of the
article?
A. Rs. 588/-
B. Rs. 608/-

C. Rs. 616/-
D. Rs. 596/-

E. Rs. 586/-
Answer & Explanation :
Answer: Option A
Explanation :
Let profit be P and C.P.= x

P a g e 112 | 580
Now, x+2P=546
x+P=483

subtracting both,
P=63

x=483-63=420
M.P.= 1.4*420=588

202. A vessel contains 180 litres of mixture of milk and water in the respective
ratio of 13 : 5. Fifty-four litres of this mixture was taken out and replaced with 6
litres of water, what is the approximate percentage of water in the resultant
mixture?
A. 41
B. 31
C. 24

D. 9
E. 17

Answer & Explanation :


Answer: Option B
Explanation :

Milk: water = 13:5


Volume of solution=180 l

Solution taken out= 54 l


Volume of solution left= 180-54=126 l

In 126 l solution,
Milk= 126*13/18= 91 l
Water=126*5/18=35 l

As 6 l water is added
Water= 35+6= 41 l

Total solution volume= 126+6= 132 l

P a g e 113 | 580
Percentage of water= 41/132*100= 31%

203. The average age of all the 100 employees in an office is 29 years, where
2/5 employees are ladies and the ratio of average age of men to women is 5 :
7. The average age of female employees is:
A. 18 years
B. 35 years
C. 25 years
D. 30 years

E. None of these
Answer & Explanation :
Answer: Option C
Explanation :
Total no. of employees =100
No. of females= 2/5 of 100= 40
No. of males = 60.

Avg. of employee= 29
Sum of ages of all the employees= 29×100= 2900
ratio of average age of men to women is 5 : 7

Let the avr of men = 5X and avg of women =7X


5X ×60+ 7X×40 =2900

X=5
Average age of female employees= 5 ×7=35

204. A man sets out on cycle from Delhi to Faridabad, and at the same time
another man starts from Faridabad on cycle for Delhi. After passing each other
they complete their journeys in 2 6 7 and 5 3 5 hours respectively. At what rate
does the second man cycles if the first man cycles at 14 kmph?

A. 10 kmph
B. 5 kmph
P a g e 114 | 580
C. 7 kmph
D. 8 kmph

E. None of these
Answer & Explanation :

Answer: Option E
Explanation :
Speed1/speed2 = √time2/√time1
14 km/h/speed2 = √535/√267
We get Speed2 =√97.81 = 9.88 km/h

DIRECTIONS for question 5 to 7: What approximate value will come in


place of question mark (?) in the given question? (You are not expected
to calculate the exact value)

205. A number is such that when it is multiplied by ‘8’, it gives another number
which is as much more than 153 as the original number itself is less than 153.
What is 25% of the original number?
A. 8
B. 10

C. 8.5
D. 6.5
E. 9
Answer & Explanation :

Answer: Option D
Explanation :
Let the no. be X

Given 8X-153=153-X
Hence X=34

25% of X=8.5

P a g e 115 | 580
206. The HCF and LCM of two num¬bers are 12 and 924 respectively. Then the
number of such pairs is
A. 0

B. 1
C. 2
D. 3
E. 4
Answer & Explanation :

Answer: Option A
Explanation :
Let the numbers be 12x and 12y where x and y are prime to each other.
∴ LCM = 12xy

∴ 12xy = 924

⇒ xy = 77
∴ Possible pairs = (1, 77) and (7,11)

207. By walking at 3/4 of his usual speed, a man reaches his office 20 minutes
later than his usual time. The usual time taken by him to reach his office is
A. 75 minutes
B. 60 minutes
C. 40 minutes

D. 30 minutes
E. 20 minutes

Answer & Explanation :


Answer: Option B
Explanation :
4/3 of usual time = Usual time + 20 minutes
1/3 of usual time
P a g e 116 | 580
= 20 minutes
Usual time = 20 × 3 = 60 minutes

208. A man sells two pipes at Rs. 12 each. He gains 20% on one and loses 20%
on the other. In the whole transaction, there is
A. neither loss nor gain
B. Profit of Rs. 1
C. Loss of Rs. 1
D. profit of Rs. 2

E. None of these
Answer & Explanation :
Answer: Option B
Explanation :
S.P of each pipe = Rs 12
One is sell at 20% i:e 120%of C.P =12
C.P of 1st pipe= Rs 10

Other is sold at loss of 20% i:e 80% of C.P of 2nd pipe = Rs 12


So C.P of 2nd pipe = Rs 15
Total C.P = Rs 25

Total S.P = Rs 24
Net profit = Rs 1

209. C is 20% more efficient than A. A and B together can finish a piece of work
in 16 days. B and C together can do it in 15 days. In how many days A alone
can finish the same piece of work?
A. 42

B. 48
C. 54

D. 36

P a g e 117 | 580
E. 45
Answer & Explanation :

Answer: Option B
Explanation :

Let total of 240 units of work to be done.


Now as given A+ B can do 240/16= 15 units/day
Also B+C can do 240/15= 16 units/day
And given, C= 1.2 A., Substituting and solving the equations we get, A= 5
units/day, ie.e A will complete work in 240/5= 48 days

210. A started a business with an investment of Rs. 16,000. After 6 months


from the start of the business, B and C joined with Rs. 12,000 and Rs. 18,000
respectively and A invested an additional amount of Rs. 4000. If the difference
between A’s share and B’s share in the annual profit is Rs. 6000, what was the
annual profit received?
A. Rs. 17,600

B. Rs. 13,200
C. Rs. 14,300
D. Rs. 16,500

E. Rs. 11,000
Answer & Explanation :
Answer: Option D
Explanation :
Amount invested by A= 16000 for first 6months, and 20000 for next 6 months
Amount invested by B= 12000 for 6 months
Amount invested by C= 18000 for 6 months

Now by compound partnership, IA:IB:IC = PA : PB : PC


16000*6 + 20000*6 : 12000*6 : 18000*6 = 6:2:3

Given 6x – 2x= 4x = 6000; x=1500


so total profit = 11x = 16500.
P a g e 118 | 580
211. Type A, 12 kg of rice worth Rs. 40/kg is mixed with Type B, rice worth Rs.
24/kg. What should be the quantity of Type B rice, if the mixture is sold at Rs.
45/kg with 25% profit added in it?
A. 18

B. 48
C. 4
D. Can’t Say
Answer & Explanation :
Answer: Option C

Explanation :
C.P. of the mixture = 45/1.25 = Rs. 36/kg..
So, (40-36)/(36-24) = (N)/(12) ⇒ N = 4 Kg.

212. A person spends 1/7th of his salary on travel,1/3rd of the remaining on


food, he then spends 1/4th of the remaining on rent. Finally he puts 1/6th of
the remaining as a monthly savings, after which he has 25000 left. What is his
salary (in Rs.)?
A. 70 K
B. 14 K

C. 84 K
D. 26 K
Answer & Explanation :

Answer: Option A
Explanation :

Let M be the total salary


Therefore, as per question, M × (6/7) × (2/3) × (3/4) × (5/6) = Rs 25000
M = Rs 70000

213. Point C(x,y) divides the distance AB with point A(8,12) and point B(16, 18)
in a ratio of 3:5, with AC being shorter than BC. What are the co-ordinates of C?

P a g e 119 | 580
A. (12,15)
B. (14.5, 12.5)

C. (11,14.25)
D. (8.3, 45)

Answer & Explanation :


Answer: Option C
Explanation :
x co-ordinate of C = [(5×8) + (3×16)] / (3+5) = 11
y co-ordinate of C = [(5×12) + (3×18)] / (3+5) = 14.25

Co-ordinates of C is ( 11, 14.25 )

214. How many terms of the sequence -12, -8, -4,…so on, to make a sum of
120?
A. 11
B. 12
C. 10

D. 13
Answer & Explanation :
Answer: Option B

Explanation :
The series is in A.P. where a = -12 and d = 4.
So 120 = (n/2)[2 × (-12) + (n -1) × 4].
On solving this, we get n = 12

215. A TV set listed at Rs 3200 is sold to a retailer at a successive discount of


25% and 15%. The retailer desires a profit of 20%, after allowing a discount of
10% to the customer. At what price should he list the TV set (in Rs.)?
A. 2720

B. 2448

P a g e 120 | 580
C. 2448
D. 2133

Answer & Explanation :


Answer: Option A

Explanation :
The retailers C.P. = 3200 × 0.75 × 0.85 = Rs 2040.
His expected S.P. = 2040 × 1.2 = Rs 2448.
But S.P. is 90% of the L.P., as there is a discount of 10%.
So L.P. = 2448/0.9 = Rs 2720

216. What is the average of the first 100 odd natural numbers?
A. 100.5
B. 125
C. 50
D. 100
Answer & Explanation :

Answer: Option D
Explanation :
As we know, sum of first n odd numbers is n2

Therefore, sum of 1st 100 odd numbers is 1002 and average will be 1002/100
= 100.

217. A, B and C invested capitals in the ratio 2 : 3 : 5, the timing of their


investment being in the ratio 9 : 5 : 6. The profit if any will be distributed in the
ratio
A. 1:2:3

B. 11:8:11
C. 7:2;1

D. 6:5:10

P a g e 121 | 580
Answer & Explanation :
Answer: Option D

Explanation :
Ratio of capitals = 2 : 3 : 5 and that of time = 9 :5: 6

Therefore, ratio of profits = 2 × 9 : 3 × 5 : 5 × 6 = 18 : 15 : 30 or 6 : 5 : 10

218. It takes 5 sec. for a clock to strike at 5’o clock. If the striking intervals are
uniform how much time will it take to strike 9’o clock (in sec.)?
A. 9

B. 10
C. 11
D. 12
Answer & Explanation :
Answer: Option B
Explanation :
There are 4 intervals in 5 strokes.

Time taken to strike 1 stroke will be 5/4 sec.


At 9, there will be 9 strokes and 8 intervals between two strokes.
Thus time required = 5/4 × 8 = 10 sec

219. From the top of the Tower which is 240m high, if the angle of depression
of a point on the ground is 30°, then the distance of the point from the foot of
the Tower is <

A. 40√3
B. 80√3
C. 120√3

D. 240√3
Answer & Explanation :

Answer: Option D

P a g e 122 | 580
Explanation :

220. Three cubes of edges 6 cms, 8 cms and 10 cms are meted without loss of
metal into a single cube. The edge of the new cube will be:
A. 8 cms
B. 12 cms

C. 14 cms
D. 16 cms
Answer & Explanation :
Answer: Option B

221. If 378 coins consist of rupee, 50 paise and 25 paise coins, whose values
are proportional to 13 :11 : 7, the number of 50 paise coins will be :
A. 128
B. 132
C. 133
D. 136
Answer & Explanation :
Answer: Option B
Explanation :
If values are proportional to 13 : 11 : 7, then the number of coins will be
proportional to 13/1 : 11/0.50 : 7/0.25 ⇒ 13 : 22 : 28. Now from this the
number of coins of 50 paise will be 378 × 22/63 = 132.

P a g e 123 | 580
222. A person travels 48 kms at 12 kms/hour and further 48 kms at
16km.s/hour. His average speed for the whole
A. 14 km/hour

B. 13(4/7)km/hour
C. 12(5/7)km/hour
D. 13(5/7)km/hour
Answer & Explanation :
Answer: Option D

Explanation :
Avg. Speed = (Total Distance / Total Time)
TD = 48 + 48= 96,
T1 = 48/12 = 4hrs
T2 = 48/16 = 3hrs
T1 + T2 = 4 + 3 = 7
Avg Speed = 96/7 = 13(5/7) km/hr

4. Simplify (0.001344 / 0.3 x 0.7) = ?


A. 0.0064

B. 0.064
C. 0.64
D. 6.4
Answer & Explanation :

Answer: Option A
Explanation :
(0.001344 / 0.3 x 0.7) = 0.0064

223. The difference of two numbers is 11 and one fifth of their sum is 9. The
numbers are :
P a g e 124 | 580
A. 31, 20
B. 30, 19

C. 29, 18
D. 28, 17

Answer & Explanation :


Answer: Option D
Explanation :
x − y = 11, x + y = 5 × 9 x − y = 11, x + y = 45, y = 17, x = 28

224. How many numbers between 1 and 100 are divisible by 7 ?


A. 9
B. 11
C. 17
D. 14
Answer & Explanation :
Answer: Option D

Explanation :
No. of divisible by 7 7, 14 --------- 98, n = a + (N - 1)d
98 = 7 + (N - 1) 7, 98 = 7 + 7N - 7

98/7= N = 14

225. What is the number which when multiplied by 13 is increased by 180?


A. 13

B. 15
C. 23
D. 35

Answer & Explanation :


Answer: Option B

P a g e 125 | 580
Explanation :
13 × 15 = 195

226. In 24 minutes, the hour hand of a clock moves through an angle of:

A. 60°
B. 24°
C. 12°
D. 5°
Answer & Explanation :

Answer: Option C
Explanation :
12 hour = 360°, 1 hr. = 360/12 = 30° 60 min = 30°, 1 min 30/60 = .5° 24
min. = 1/2 ×24 = 12°

227. √0.0081 is equal to :


A. 0.09

B. 0.9
C. ±0.08
D. 0.81

Answer & Explanation :


Answer: Option A
Explanation :
√0.0081 = √0.0081/10000 = √81/10000 = 9/100 =0.09

228. A reduction of 20% in the price of mangoes enables a person to purchase


12 more for Rs. 15. What was the price of 16 mangoes before reduction of price
?
A. Rs. 6

B. Rs. 5

P a g e 126 | 580
C. Rs.7
D. Rs. 9

Answer & Explanation :


Answer: Option B

Explanation :
Price x Consumption = Expenditure
Consumption = Exp. Price
(15 / 8x) - (15 / x) = 12
x = (15 x 2) / (12 x 8)

For 16 Mangoes = [(15 x 2) / (12 x 8)] x 16 = 5


229. Dharma invested Rs. P for 3 years in scheme A which offered 12% p.a.
simple interest. She also invested Rs. P + 400 in scheme B which offered 10%
compound interest (compounded annually), for 2 years. If the amount received
from scheme A was less than that received from scheme B, by Rs. 304, what is
the value of P?
A. Rs. 1400

B. Rs. 1000
C. Rs. 1500
D. Rs. 900

E. Rs. 1200
Answer & Explanation :
Answer: Option E
Explanation :

As given, P + 3*12*P/100 + 304 = (P+400)(1+11/100)2


P(0.15)= 180;
P= 1200

230. Shiva gives 20% of her monthly salary to his mother, 50% of the
remaining salary he invests in an insurance scheme and PPF in the respective
ratio of 5 : 3 and the remaining he keeps in his bank account. If the sum of the

P a g e 127 | 580
amount he gives to his mother and that he invests in PPF is Rs. 12,600, how
much is Shiva’s monthly salary?

A. Rs. 36,000
B. Rs. 64,000

C. Rs. 42,000
D. Rs. 40,000
E. None of these
Answer & Explanation :
Answer: Option A

Explanation :
Let the total amount be x.
0.2x = given to mother.
0.25x= invested in insurance
0.15x= invested in ppf
0.4x= Bank account
Given, 0.2x+0.15x = 0.35x = 12600; x=36000

231. The respective ratio of radii of two right circular cylinders (A & B) is 4 : 7.
The respective ratio of the heights of cylinders A and B is 2 : 1. What is the
respective ratio of volumes of cylinders A and B?
A. 25 :42

B. 23 : 42
C. 32 : 49

D. 30 : 49
E. 36 : 49
Answer & Explanation :

Answer: Option C

232. If 4x/3 + 2P = 12 for what value of P, x = 6?

P a g e 128 | 580
A. 6
B. 4

C. 2
D. 1

Answer & Explanation :


Answer: Option C
Explanation :
When x = 6, (4 * 6)/3 + 2P = 12
⇒ 8 + 2P = 12

⇒ 2P = 12 – 8 = 4

⇒P=2

233. In ΔABC, ∠A + ∠B = 65°, ∠B + ∠C = 140°, then find ∠B.

A. 40°
B. 25°
C. 35°
D. 20°

Answer & Explanation :


Answer: Option B
Explanation :
∠A + ∠B = 65°

∴ ∠C = 180° - 65° = 115°

∠B + ∠C = 140°
∴ ∠B = 140° - 115° = 25°

234. There are two motor cycles (A & B) of equal cost price. Motorcycle A was
sold at a profit of 14% and motorcycle B was sold for Rs. 4,290/- more than its
cost price. The net profit earned after selling both the motor cycles (A & B) is
20%. What is the cost price of each motorcycle?

P a g e 129 | 580
A. Rs. 16,500/-
B. Rs. 16,000/-

C. Rs. 15,500/-
D. Rs. 71,500/-

E. Rs. 17,000/-
Answer & Explanation :
Answer: Option A
Explanation :
Let the cost price of each motorcycle be Rs. ‘A’. So SP of A = 1.14A and SP of B
= A + 4290. Total CP = 2A. As net profit is given to be 20% on both the
motorcycles, so we can form the equation as (2.14A + 4290 - 2A)/2A = 20%.
Solving it further, we get (0.14A + 4290)5 = 2A. Solving this equation, we get
value of A as 16,500. Hence answer is option A

235. Raman invested Rs. P for 2 years in scheme A which offered 20% p.a.
compound interest (compounded annually). He lent the interest earned from
scheme A to Shubh, at the rate of 7.5% p.a. simple. If at the end of 2 years,
Shubh gave Rs. 3036 to Raman and thereby repaid the whole amount(actual
loan + interest), what is the value of P?
A. Rs. 6000
B. Rs. 5800

C. Rs. 6800
D. Rs. 5400

E. Rs. 6400
Answer & Explanation :
Answer: Option A

Explanation :
P*(1.2)2 = 1.44P
Interest = P – 1.44P = 0.44P
0.44P + (0.44P * 7.5 * 2)/100 = 3036
Solving, we get P = 6000.
P a g e 130 | 580
236. Three persons work independently on a problem. If the respective
probabilities that they will solve it are 1/3, 1/4 and 1/5, then the probability
that none can solve it is

A. 1/5
B. 1/3
C. 2/5
D. None of these
Answer & Explanation :

Answer: Option C
Explanation :
Joint probability of all not being able to solve it is

237. A person standing on the bank of a river observes that the angle of
elevation of the top of a tree on the opposite bank of the river is 60° and when
he retires 40 metres away from the tree the angle of elevation becomes 30°.
The breadth of the river is
A. 40 m
B. 20 m

C. 30 m
D. 60 m
Answer & Explanation :
Answer: Option B
238. If A's income is 50% less than that of B's, then B's income is what per cent
more than that of A?
A. 125

B. 100
C. 75
D. 50
Answer & Explanation :

P a g e 131 | 580
Answer: Option B
Explanation :

Required percentage

239. 1.14 expressed as a per cent of 1.9 is


A. 6%

B. 10%
C. 60%
D. 90%
Answer & Explanation :
Answer: Option C

Explanation :
Required percentage

240. Two natural numbers are in the ratio 3 : 5 and their product is 2160. The
smaller of the numbers is
A. 36

B. 24
C. 18
D. 12

Answer & Explanation :


Answer: Option A
Explanation :

P a g e 132 | 580
241. If 60% of A = ¾ of B, then A : B is
A. 9 : 20

B. 20 : 9
C. 4 : 5
D. 5 : 4
Answer & Explanation :
Answer: Option D
Explanation :

242. Two successive price increases of 10% and 10% of an article are
equivalent to a single price increase of

A. 19%
B. 20%

C. 21%
D. 22%
Answer & Explanation :

Answer: Option C
P a g e 133 | 580
Explanation :
Single equivalent percentage increase in price

243. If then x is equal to


A. 5/12
B. 12/5
C. 5/7
D. 7/5
Answer & Explanation :
Answer: Option B
Explanation :

244. An equilateral triangle of side 6 cm has its corners cut off to form a regular
hexagon. Area (in cm2) of this regular hexagon will be

A.

P a g e 134 | 580
B.

C.

D.
Answer & Explanation :

Answer: Option C
Explanation :

side of regular hexagon =1/3 *6 = 2 cm

Area of the hexagon = 3 √3/ 2 a2


= 3 √3/2 *2*2 =6 √3 sq.cm

245. A and B together can do a work in 10 days. B and C together can do the
same work in 6 days. A and C together can do the work in 12 days. Then A, B
and C to¬gether can do the work in
A. 28 days
B. 14 days
C. 5 5/7 days
D. 8 2/7 days

Answer & Explanation :


Answer: Option C
Explanation :
(A+B)' 1 day's work = 1/10
(B+C) 1 day's work = 1/6

P a g e 135 | 580
(C+ A) 1 day's work = 1/12
Adding all three

2(A+B+C)' 1 day's work


=1/10 + 1/6 + 1/12 = 6+10+5 / 60

=21/60 = 7/20
=>(A+B+C)' 1 day's work = 7/40
=>All three together will complete the work in 40/7 = 5 5/7days.

246. A does half as much work as B in three- fourth of the time. If together
they take 18 days to complete a work, how much time shall B take to do it
alone?
A. 30 days
B. 35 days
C. 40 days
D. 45 days
Answer & Explanation :

Answer: Option A
Explanation :

P a g e 136 | 580
247. If a wire is bent into the shape of a square, then the area of the square so
formed is 81cm2. When the wire is rebent into a semicircular shape, then the
area, (in cm2) of the semicircle will be (Take π = 22/7)
A. 22
B. 44

C. 77
D. 154

Answer & Explanation :


Answer: Option C
Explanation :

P a g e 137 | 580
248. In an examination hall, there are four rows of chairs. Each row has 8
chairs, one behind the other. There are two classes sitting for the examination
with 16 students in each class. It is desired that in each row all students belong
to the same class should sit and that no two adjacent rows are allotted to the
same class. In how many ways can these 32 students be seated?
A. 2 x 16! x 16!

B. 2 x 16! x 15!
C. 2 x 15! x 15!

P a g e 138 | 580
D. 2 x 14! x 16!
Answer & Explanation :

Answer: Option A
Explanation :

Students can be adjusted in 16! x 16! ways.


Rows can be adjusted in 2 ways.
So the final answer will be the 1st option.

249. There are three events A, B and C, only one of which must happen. The
odds are 8 to 3 against A, 5 to 2 against B. Find the odds against C.
A. 43 : 34
B. 43 : 77
C. 34 : 43
D. 77 : 43
Answer & Explanation :
Answer: Option A

Explanation :
P(A) = 3/11 & P(B) = 2/7 . So probability of happening of C = 1 – ( 3/11+ 2/7)
= 34/77 . So the odds against C are 43:34.

250. 1,496 cm3 of a metal is used to cast a pipe of length 28 cm. If the internal
radius of the pipe is 8 cm, the outer radius of the pipe is
A. 7 cm

B. 10 cm
C. 9 cm
D. 12 cm

Answer & Explanation :


Answer: Option C

P a g e 139 | 580
251. A man is standing on the 8 m long shadow of a 6 m long pole. If the length
of the man’s shadow is 2.4 m, then the height of the man is

A. 1.4 m
B. 1.8 m

C. 1.6 m
D. 2.0 m
Answer & Explanation :
Answer: Option B
Explanation :

8 m shadow. Length of pole is = 6m.


2.4 m shadow will have = 6/8 x 2.4 = 1.8 m height of the man as answer.

252. In a class, there are 20 boys whose average age is decreased by 2


months, when one boy aged 18 years is replaced by a new boy. The age of the
new boy is
A. 14 years 8 months

B. 16 years 4 months
C. 15 years
D. 17 years 10 months

Answer & Explanation :


Answer: Option A

Explanation :
Decrease of 20 x 2 = 40 months means that incoming boy has age 18 years –
40 months
= 14 years and 8 months.

253. Two taps can separately fill a cistern in 10 minutes and 15 minutes,
respectively and when the waste pipe is open, they can together fill it in 18
minutes. The waste pipe can empty the full cistern in
A. 7 minutes

P a g e 140 | 580
B. 13 minutes
C. 9 minutes

D. 23 minutes
Answer & Explanation :

Answer: Option C
Explanation :
1/6 - 1/x = 1/18
Try option in place of x and get answer as third option.

254. A boatman goes 2 km against the current of the stream in 1 hr and goes 1
km along the current in 10 min. How long will he take to go 5 km in stationary
water?
A. 1 hour
B. 3/2 hours
C. 1 hour 15 minutes
D. 40 minutes

Answer & Explanation :


Answer: Option C
Explanation :

In 1 hr he goes 2 km against the stream and with the current he will go 6 km in


1 hr.

In still water he will go 4 km in 1 hr. So it will take 1.15 hr for 5 km.

255. A person travels 285 km in 6 hours in two stages. In the first part of the
journey, he travels by bus at the speed of 40 km per hour. In the second part of
the journey, he travels by train at the speed of 55 km per hour. How much
distance did he travel by train?
A. 205 km
B. 165 km
C. 145 km

P a g e 141 | 580
D. 185 km
Answer & Explanation :

Answer: Option B
Explanation :

Just go by options. In 3 hours he will travel 165 by train and By bus 120 km in
next the 3 hours.
So 2nd option.

256. If there are six periods in each working day of a school, in how many ways
can one arrange 5 subjects such that each subject is allowed at least one
period?
A. 3500
B. 1800
C. 3600

D. 1750
Answer & Explanation :

Answer: Option B
Explanation :
Let us take a case where we have decided which specific subject has a double
class.
Now we have 6 periods and we have filled all of them up – but with one subject
being repeated.
These can be arranged in 6!/2! = 360 ways. Now any of the 5 subjects can be
repeated.
So total no. of possibilities is – 360 x 5 = 1800.

257. A monument has 50 cylindrical pillars each of diameter 50 cm and height 4


m. What will be the labour charges for getting these pillars cleaned at the rate
of 50 paise per sq. m? (Use π = 3.14)
A. Rs. 237
B. Rs. 157
P a g e 142 | 580
C. Rs. 257
D. Rs. 353

Answer & Explanation :


Answer: Option B

Explanation :
Direct formula putting
= 2 x π x r x h x 50 ÷ ( 2 x 100)
= 50 x 2 x 3.14 x 25 x 4 ÷ (2 x 100)
258. A mixture contains wine and water in the ratio 3 : 2 and another mixture
contains them in the ratio 4: 5. How many liters of the latter must be mixed
with 3litres of the former so that the resultant mixture may contain equal
quantities of wine and water?
A. 1 2/3 litre
B. 2/5 litre

C. 3 3/4 litre
D. 4 1/2 litre

E. None of these
Answer & Explanation :
Answer: Option E

Explanation :
By applying allegation and mixture

So we got ratio of two mixture = 5:9

It means 5 unit = 3 liters

P a g e 143 | 580
We get 9 unit = 3/5 ×9 = 27/5 liters

259. A trader sells two bullocks for Rs. 8,400 each, neither losing nor gaining in
total. If he sold one of the bullocks at a gain of 20%, the other is sold at a loss
of
A. 20%
B. 18 2/9%
C. 14 2/7%
D. 21%

E. None of these
Answer & Explanation :
Answer: Option A
Explanation :
A trader sells two bullocks for Rs. 8,400 each, neither losing nor gaining in total.
As he sold one bullocks at a gain of 20%, it means 120% of C.P = 8400
We get C.P of one bullocks = 7000

So gains on one bullocks = Rs 1400


Other bullocks is sold at lose and there is neither losing nor gaining in total
So loss on 2nd bullocks = 1400/7000×100 =20%

260. Two trains, A and B, start from stations X and Y towards each other, they
take 4 hours 48 minutes and 3 hours 20 minutes to reach Y and X respectively
after they meet if train A is moving at 45 km/hr., then the speed of the train B
is
A. 60 km/hr
B. 64.8 km/hr

C. 54 km/hr
D. 37.5 km/hr

E. None of these
Answer & Explanation :
P a g e 144 | 580
Answer: Option C
Explanation :

Speed1 : Speed2 = √Time2 : √Time1


45km/h :speed2 = √10/3 : √24/5

We get speed2 = 54km/hr

261. Out of his total income, Mr. Kapoor spends 20% on house rent and 70% of
the rest on house hold expenses. If he saves Rs 1,800 what is his total income
(in rupees)?

A. Rs 7,800
B. Rs 7,000
C. Rs 8,000
D. Rs 7,500
E. None of these
Answer & Explanation :
Answer: Option E

Explanation :
Let the total income of Mr. Kapoor be 100 units.
As Mr. Kapoor spends 20% on house rent and 70% of the rest on house hold
expenses.
So he spends 76% of his income.

It means 24 unit income = Rs 1,800


Total income = 1,800/24 ×100 = Rs. 7500

262. A can do a piece of work in 8 days which B can destroy in 3 days. A has
worked for 6 days, during the last 2 days of which B has been destroying. How
many days must A now work alone to complete the work?
A. 7 days

B. 7 2/3 days
C. 7 1/3 days
P a g e 145 | 580
D. 8 days
E. None of these

Answer & Explanation :


Answer: Option C

Explanation :
A can do a piece of work in 8days which B can destroy in 3 days.
So let the total work be 24 units.
So unit contributed by A and B in one day is 3 units and -8 units respectively .
A has worked for 6 days, during the last 2 days of which B has been destroying.

So total units completed in 6 days = 6×3 + 2× (-8) = 2 units.


So remaining 22 units is done by A alone with efficiency of 3 units per day.
So number of days required to complete the work = 22/3 days

263. A garrison of 750 men has provisions for 20 weeks. If at the end of 4
weeks, they are re-inforced by 450 men, how long will the provision last?
A. 8 weeks

B. 12 weeks
C. 14 weeks
D. 15 weeks

E. 10 weeks
Answer & Explanation :
Answer: Option E
Explanation :

750 * 20 = 750 * 4 + 1200 * W => W = 10 weeks

264. Bill, Simon, and John are brothers, given Simon is the eldest. Bill is as
many years younger than one brother as he is older than the other. Simon is 7
years younger than twice the age of John. John is 5 years older than half the
age of Bill. What is the sum of the ages of Bill, Simon and John?

P a g e 146 | 580
A. 12
B. 24

C. 48
D. Can’t say

Answer & Explanation :


Answer: Option C
Explanation :
S – B = B – J J = B/2 + 5. S = 2J – 7.
S = B + 10 – 7 = B + 3.

J = + 5. 2S = B + 10. + 5 + B + 3 = 2B. = 8. B = 16, S = 19, J = 13. So B + S


+ J = 16 + 19 + 13 =48.

265. The cost price of item B is Rs. 150/- more than the cost price of item A,
Item A was sold at a profit of 10% and Item B was sold at a loss of 20%. If the
respective ratio of selling price of items A and B is 11:12, what is the cost price
of item B?

A. Rs. 450/-
B. Rs. 420/-
C. Rs. 400/-

D. Rs. 350/-
E. Rs. 480/-
Answer & Explanation :
Answer: Option A

Explanation :
Let us assume cost price of A= X
So that Cost price of B= X+150.

SP of A= X*1.1
SP of B=(X+150)*0.8

Given that

P a g e 147 | 580
SPA: SPB
11:12

So that 1.1X/(X+150)*0.8= 11/12


X=300

CP of B= 300+150=450
Answer is 450.

266. The number of ways in which 7 boys and 8 girls can be seated in a row so
that they are alternate

A. 203121800
B. 29030400
C. 3628800
D. 203212800
E. 3628800
Answer & Explanation :
Answer: Option D

Explanation :
Reqd. number of ways = 7! × 8! = 203212800.

267. There are 13 married couples, 5 single men and 7 single women in a party.
Every man shakes hand with every woman once, but no one shakes hand with
his wife. How many handshakes took place in the party?
A. 247

B. 347
C. 360
D. 191

E. 100
Answer & Explanation :

Answer: Option B

P a g e 148 | 580
Explanation :
Any single man will have = 13 + 7 = 20 options.

Total number of handshakes by single men = 20 × 5= 100. Any married man


will have 12 + 7 = 19 options. Total number of handshakes by married men =
19 × 13 = 247. Total number = 247 + 100 = 347.
268. The HCF and LCM of two numbers are 12 and 924 respectively. Then the
number of such pairs is
A. 0
B. 1

C. 2
D. 3
Answer & Explanation :
Answer: Option C
Explanation :

Let the numbers be 12x and 12y where x and y are prime to each other.
∴ LCM = 12xy

∴ 12xy = 924
=> xy = 77

∴ Possible pairs = (1,77) and (7,11)

269. What is the least number which, when divided by 5, 6, 7, 8 gives the
remainder 3 but is divisible by 9?
A. 1463

B. 1573
C. 1683
D. 1793

Answer & Explanation :


Answer: Option C

Explanation :

P a g e 149 | 580
LCM of 5, 6, 7, 8 = 35 × 24 = 840
∴ Required number = 840 k + 3 which is exactly divisible by 9.

For k = 2, it is divisible by 9.
∴ Required number = 840k + 3

= 840 × 2 + 3 = 1683

270. Three numbers are in the ratio 3:4: 5. The sum of the largest and the
smallest equals the sum of the second and 52. The smallest number is

A. 20
B. 27
C. 39
D. 52
Answer & Explanation :

Answer: Option C
Explanation :
Let the numbers be 3x, 4x and 5x.
∴ 5x + 3x = 4x + 52
=> 4x = 52 => x =13

∴ The smallest number = 3x = 3 × 13 = 39

271. If the radius of a circle is increased by 50%, its area is increased by


A. 125%

B. 100%
C. 75%

D. 50%
Answer & Explanation :
Answer: Option A
Explanation :

P a g e 150 | 580
Percentage increase in area

272. A and B working separately can do a piece of work in 9 and 12 days


respectively. If they work for a day alternately with A beginning, the work would
be completed in
A. 10 2/3 days
B. 10 1/2 days
C. 10 1/4 days

D. 10 1/3 days
Answer & Explanation :
Answer: Option C
Explanation :
Part of work done by A and B in first two days

Part of work done in first 10 days = 35/36


Remaining work = 1 – 35/36 = 1/36

Now it is the turn of A.


∴ Time taken by A = 1/36 × 9 = ¼
∴ Total time = 10 + ¼ = 10 ¼ days

273. In a family, the average age of a father and a mother is 35 years. The
average age of the father, mother and their only son is 27 years. What is the
age of the son?
A. 12 years
B. 11 years

C. 10.5 years
D. 10 years

P a g e 151 | 580
Answer & Explanation :
Answer: Option B

Explanation :
Father + mother = 2 × 35 = 70 years

Father + mother + son = 27 × 3 = 81 years


Son’s age = 81 – 70 = 11 years

274. If 5 men or 7 women can earn Rs. 5,250 per day, how much would 7 men
and 13 women earn per day?

A. Rs. 11,600
B. Rs. 11.700
C. Rs. 16,100
D. Rs. 17,100
Answer & Explanation :
Answer: Option D
Explanation :

5 men = 7 women

7 men
Mark Price = 140

7 men + 13 women

Now,
∵ 7 women = Rs. 5250

P a g e 152 | 580
275. If A and B together can complete a piece of work in 15 days and B alone in
20 days, in how many days can A alone complete the work?
A. 60

B. 45
C. 40
D. 30
Answer & Explanation :
Answer: Option A

Explanation :

(A + B)’s 1 day’s work =

B’s 1 day’s work =

A’s 1 day’s work =


A alone will do the work in 60 days.

276. By walking at 3/4 of his usual speed, a man reaches his office 20 minutes
later than his usual time. The usual time taken by him to reach his office is
A. 75 minutes
B. 60 minutes

C. 40 minutes
D. 30 minutes
Answer & Explanation :
Answer: Option B
Explanation :

of usual time = Usual time + 20 minutes

P a g e 153 | 580
of usual time
= 20 minutes

Usual time = 20 × 3 = 60 minutes

277. 4.41*0.16/2.1*1.6* 0.21 is simplified to


A. 1
B. 0.1
C. 0.01
D. 10

Answer & Explanation :


Answer: Option A
Explanation :

278. The list price of an article is Rs. 160 and a customer buys it for Rs. 122.40
after two successive discounts. If the first discount is 10%, then second
discount is
A. 12%

B. 10%
C. 14%
D. 15%
Answer & Explanation :

Answer: Option D
Explanation :
SP after a discount of 10%

= (160*90) / 100 = Rs. 144


Second discount

= 144 – 122.40 = Rs. 21.6

P a g e 154 | 580
If the second discount be x% then

279. In a school, 10% of number of girls is equal to 1/20 of number of boys.


Ratio between the 'number of boys to number of girls is
A. 1:2

B. 2:1
C. 1:4
D. 4:1
Answer & Explanation :
Answer: Option B
Explanation :
LIf boys = x and girls = y, then

280. If a, b, c, d, e are five consecutive odd numbers, their average is


A. 5(a + 4)

B. abde/5
C. 5(a + b + c + d + e)
D. a + 4
Answer & Explanation :
Answer: Option D

P a g e 155 | 580
Explanation :
b=a+2

c=b+2=a+4
d=c+2=a+6

e=d+2=a+8
Therefore, Required average = (a + a + 2 + a + 4 + a + 6 + a + 8)/5
=a+4

281. The average of 20 numbers is 15 and the average of first five is 12. The
average of the rest is
A. 16
B. 15
C. 14
D. 13
Answer & Explanation :
Answer: Option A

Explanation :
If the average of remaining numbers be x, then
20 × 15 = 5 × 12 + 15x

⇒ 300 = 60 + 15x
⇒ 15x = 300 – 60 = 240

=> x = 240/15 = 16

282. A tradesman sold an article at a loss of 20%. If the selling price had been
increased by Rs. 100, there would have been a gain of 5%. The cost price of the
article (in Rs.) was
A. 100

B. 200
C. 400

P a g e 156 | 580
D. 500
Answer & Explanation :

Answer: Option C
Explanation :

C. P. of article = Rs. X
∴ First SP = 80x/100 = Rs. 4x/5
Case II

⇒ 5x = 2000 ⇒ x = 2000/5 = Rs. 400

283. The price of an article is first decreased by20% and then increased by
30%. If the resulting price is Rs. 416, the original price of the article is
A. Rs. 350
B. Rs. 405
C. Rs. 400
D. Rs. 450

Answer & Explanation :


Answer: Option C
Explanation :
If the original price of article be Rs. X, then

284. A man performs 2/15 of the total journey by train 9/20 by bus and the
remaining 10 km on foot. His total journey in km is
P a g e 157 | 580
A. 15.6
B. 24

C. 16.4
D. 12.8

Answer & Explanation :


Answer: Option B
Explanation :
If the total journey be x km, then
2x/15+9x/20+10=x

=>x-2x/15-9x/20=10
=> 60x-8x-27x/60=10
=> 25x/60=10
=> X=60*10/25=24km

285. By walking at 3/4 of his usual speed, a man reaches his office 20 minutes
later than usual. His usual time is

A. 30 min
B. 75 min
C. 90 min.

D. 60 min.
Answer & Explanation :
Answer: Option D
Explanation :

New speed = 3/4 × usual speed


∴ New time = 4/3 × usual time.

∴ 1/3 × usual time = 20 minutes


⇒ Usual time = 3 × 20

= 60 minutes

P a g e 158 | 580
286. If the compound interest on a certain sum for two years at 12% per
annum is Rs. 2544 the simple interest on it at the same rate for 2 years will be
A. Rs. 2400

B. Rs. 2500
C. Rs. 2480
D. Rs. 2440
Answer & Explanation :
Answer: Option A

Explanation :

287. The mean daily profit made by a shopkeeper in a month of 30 days was
Rs. 350. If the mean profit for the first fifteen days was Rs. 275, then the mean
profit for the last 15 days would be
A. Rs. 200
B. Rs. 350
C. Rs. 275
D. Rs. 425

Answer & Explanation :


P a g e 159 | 580
Answer: Option D
Explanation :

Average would be : 350 = (275 + x)/2


On solving, x = 425.

288. There were 35 students in a hostel. If the number of students increases by


7, the expenses of the mess increase by Rs. 42 per day while the average
expenditure per head diminishes by Re 1. Find the original expenditure of the
mess.

A. Rs. 480
B. Rs. 520
C. Rs. 420
D. Rs. 460
Answer & Explanation :

Answer: Option C
Explanation :

Let d be the average daily expenditure


Original expenditure = 35 × d
New expenditure = 35 × d + 42

New average expenditure will be :


(35 × d + 42)/42 = d - 1

On solving, we get d = 12
Therefore original expenditure = 35 × 12 = 420

289. The ratio between the number of passengers travelling by I and II class
between the two railway stations is 1 : 50, whereas the ratio of I and II class
fares between the same stations is 3 : 1. If on a particular day Rs. 1,325 were
collected from the passengers travelling between these stations, then what was
the amount collected from the II class passengers?
A. Rs. 750

P a g e 160 | 580
B. Rs. 1000
C. Rs. 850

D. Rs. 1250
Answer & Explanation :

Answer: Option D
Explanation :
Let x be the number of passengers and y be the fare taken from passengers.
3xy + 50xy = 1325 => xy = 25
Amount collected from II class passengers = 25 × 50 = Rs. 1250.

290. A boat travels upstream from B to A and downstream from A to B in 3


hours. If the speed of the boat in still water is 9 km/hour and the speed of the
current is 3 km/hour, the distance between A and B is
A. 4 km
B. 8 km
C. 6 km

D. 12 km
Answer & Explanation :
Answer: Option D

Explanation :
Let d be the distance between A and B

So, d/12 + d/6 = 3 d = 12 km

291. A man while returning from his factory, travels 2/3 of the distance by bus,
¾ of the rest partly by car and partly by foot. If he travels 2 km on foot, find
the distance covered by him.

A. 24 km
B. 22 km

C. 28 km

P a g e 161 | 580
D. 26 km
Answer & Explanation :

Answer: Option A
Explanation :

Therefore D = 24 km

292. The fuel indicator in a car shows 1/5th of the fuel tank as full. When 22
more liters of fuel are poured in to the tank, the indicator rests at the 3/4of the
full mark. Find the capacity of the tank.

A. 25 litres
B. 35 litres
C. 30 litres
D. 40 litres
Answer & Explanation :
Answer: Option D
Explanation :

x/5 + 22 = 3x/4 ⇒ x = 40 litres

293. A pump can be operated both for filling a tank and for emptying it. The
capacity of the tank is 2400 m3. The emptying capacity of the pump is 10m3
per minute higher than its filling capacity. Consequently, the pump needs 8
minutes less to empty the tank than to fill it. Find the filling capacity of the
pump.
A. 45 m3/min

B. 40 m3/min
C. 50 m3/min
D. 55 m3/min
Answer & Explanation :
Answer: Option C

Explanation :

P a g e 162 | 580
(2400/x) - (2400/(x + 10)) = 8, Solving this we get x
= 50m3/min

Or by options (2400/50) - (2400/60) = 48 - 40 = 8 minutes

294. A sum of money is accumulating at compound interest at a certain rate of


interest. It simple interest instead of compound were reckoned, the interest for
the first two years would be diminished by Rs. 20 and that for the first three
years, by Rs 61. Find the sum
A. Rs. 7000

B. Rs. 8000
C. Rs. 7500
D. Rs. 6500
Answer & Explanation :
Answer: Option B

295. In a kilometer race, A can give B a 100 m start and C a 150 m start. How
many meters start can B give to C?
A. 50
B. 50/9

C. 8500/9
D. 500/9
E. None of these
Answer & Explanation :

Answer: Option D
Explanation :
A can give B a 100 m start and C a 150m. Start means when A runs 1000m, B
runs 900m and C runs 850m. When B runs 1000m, C will run 1000 x (850/900)
m (i.e. 8500/9 m) Thus, B can give C a start of - 1000 - (8500/9), i.e. 500/9 m.

P a g e 163 | 580
296. The average age of all the student of a class is 18 years. The average age
of boys of the class is 20 years and that of the girls is 15 years. If the number
of girls in the class is 20, then find the number of boys in the class.
A. 15

B. 45
C. 30
D. 50
Answer & Explanation :
Answer: Option C

Explanation :
Let Boys in class = B
Girls in class = 20
Now, (20B+15*20)/(B+20) = 18
⇒ B = 30
297. 0 6 24 60 120 210 ?

A. 343
B. 280
C. 335
D. 295
E. 336
Answer & Explanation :
Answer: Option E
Explanation :
0 6 24 60 120 210 ?

+6 +18 +36 +60 +90 +126


+12 +18 +24 +30 +36
Hence number is 210+114 = 324

298. 32 49 83 151 287 559 ?


P a g e 164 | 580
A. 1118
B. 979

C. 1103
D. 1120

E. 1110
Answer & Explanation :
Answer: Option C
Explanation :
32 49 83 151 287 559 ?

+17 +34 +68 +136 +272 +544


+17 +34 +68 +136 +272
So answer is 559 + 544 = 1103.

299. 462 552 650 756 870 992 ?


A. 1040
B. 1122

C. 1132
D. 1050
E. 1124

Answer & Explanation :


Answer: Option B
Explanation :
462 552 650 756 870 992 ?

+90 +98 +106 +114 +122 +130


+8 +8 +8 +8 +8
So next number is 992 + 130 = 1122.

P a g e 165 | 580
300. The average age of a woman and her daughter is 42 years. The ratio of
their ages is 2 : 1 respectively. What is the daughter’s age?

A. 28 years
B. 48 years

C. 52 years
D. 31 years
E. 25 years
Answer & Explanation :
Answer: Option A

Explanation :
Let the age of mother be M and that of her daughter be D
Therefore, M+D/2 =42 and M/D=2/1
Solving the above equations we get D = 28 yrs

301. The price of sugar is increased by 25%.Find by how much percent the
consumption of sugar be decreased so as not to increase the expenditure?

A. 25%
B. 40%
C. 20%

D. 30%
E. None of these

Answer & Explanation :


Answer: Option C

Explanation :
Using the formula to calculate % decrease as [R/(100+R)]x100 where R =
percentage increase in price, we get

Required % decrease in consumption = 25/125 × 100 = 20%

P a g e 166 | 580
302. A car travels a distance of 45 km at the speed of 15 km/hr. It covers the
next 50 km of its journey at the speed of 25km/hr and the last 25 km of its
journey at the speed of 15 km/hr. What is the average speed of the car?
A. 40 km/hr

B. 24 km/hr
C. 15 km/hr
D. 18 km/hr
E. 20km/hr
Answer & Explanation :

Answer: Option D
Explanation :
We know, Average speed = Total distance travelled / Total time taken
Average = 45+50+25/3+2+25/15= 18kmph.

303. A car travels a distance of 170 km in 2 hours partly at a speed of 100


km/h and partly at 50 km/h. The distance travelled at a speed of 50 km/h is

A. 50 km
B. 40 km
C. 30 km

D. 60 km
E. 45 km

Answer & Explanation :


Answer: Option C

Explanation :
Suppose he covers x km at 100 kmph
he covers 170-x at 50 kmph

Solving this equation, we get x = 140.

he covers 30km at 50 kmph.

P a g e 167 | 580
304. Even after reducing the marked price of a transistor by Rs. 32, a
shopkeeper makes a profit of 15%. If the cost price be Rs. 320, what
percentage of profit would he have made if he had sold the transistor at the
marked price?
A. 25%
B. 20%
C. 10%
D. 15%

E. None of these
Answer & Explanation :
Answer: Option A
Explanation :
Let x be the marked price,

So x - 32 = 320 × 1.15 ⇒ x = 400.

So required value is
400 = 320 (1 + profit/100),
So profit is 25%

305. The sum of five numbers is 260. The average of first two numbers is 30
and average of the last two numbers is 70. What is third number?
A. 33

B. 60
C. 75

D. Can't determined
E. None of these
Answer & Explanation :
Answer: Option E
Explanation :

P a g e 168 | 580
(a + b + c + d + e)/5 = 260 ...........(1)
(a + b)/2 = 30 , Therefore a + b = 60

(d + e)/2 = 70 , Therefore d + e = 140


Using the values in (1) , We get

c= 60
As sum of first two is 60 & sum of last two is 140 so third no. is 260 - 60 - 140
= 60.

306. 20 boys and 32 girls form a group for social work. During their
membership drive same no. of boys and girls joined the group. How many
members does the group have now, if the ratio of boys to girls is 3:4
respectively?
A. 75
B. 86

C. 68
D. 82

E. 84
Answer & Explanation :
Answer: Option E

Explanation :
Let x be the new boys as well as girls , Therefore
Solving this we get x = 16
So total will be 36 + 48 = 84

307. The simple interest on Rs. 4000 in 3 years at the rate of x% per annum
equals the simple interest on Rs. 5000 at the rate of 12% per annum in 2 years.
The value of x is

A. 8%
B. 9%
C. 10%
D. 6%

P a g e 169 | 580
Answer & Explanation :
Answer: Option C

308. If (4x - 3)/x + (4y - 3)/y + (4z - 3)/z = 0, then the value of 1/x + 1/y +
1/z is
A. 4
B. 6
C. 9
D. 3

Answer & Explanation :


Answer: Option A
Explanation :
(4x - 3)/x + (4y - 3)/y + (4z - 3)/z = 0
=> 4x/x - 3/x + 4y/y - 3/y + 4z/z - 3/z = 0
=> 3/x + 3/y + 3/z = 4 + 4 + 4 = 12
=> 1/x + 1/y + 1/z = 12/3 = 4

309. Find the simplest value of 2√50 + √18 - √72 (given √2 = 1.414)
A. 10.312

B. 8.484
C. 4.242
D. 9.898
Answer & Explanation :

Answer: Option D
Explanation :

P a g e 170 | 580
310. A number x when divided by 289 leaves 18 as a remainder. The same
number when divided by 17 leaves y as a remainder. The value of y is
A. 3
B. 1
C. 5

D. 2
Answer & Explanation :
Answer: Option B
Explanation :
Here, the first divisor (289) is a multiple of second divisor (17)
∴ Required remainder = Remainder obtained on dividing 18 by 17 = 1. Hence
the answer is option 2.

311. An equation of the form ax + by + c = 0 where a ≠ 0, b ≠ 0, c = 0


represents a straight line which passes through

A. (0,0)
B. (3,2)
C. (2,4)

D. None of these
Answer & Explanation :

Answer: Option A
Explanation :
P a g e 171 | 580
Ax+by+c = 0
When c = 0

ax+by = 0
by = -ax ⇒ y = - ax/b

when x = 0, y = 0 i.e., this line passes through the origin (0,0).

312. The numerator of a fraction is 4 less than its denominator. If the


numerator is decreased by 2 and the denominator is increased by 1, then the
denominator becomes eight times the numerator. Find the fraction.
A. -4/8
B. 2/7
C. 3/8
D. 3/7
Answer & Explanation :
Answer: Option D

Explanation :
Original fraction = (x - 4)/x
In case II,
8(x - 4 - 2) = x + 1
⇒ 8x - 48 = x + 1
⇒ 7x = 49 ⇒ x = 7
∴Original fraction

= (7 - 4)/7 = 3/7
313. The simple interest on Rs. 4000 in 3 years at the rate of x% per annum
equals the simple interest on Rs. 5000 at the rate of 12% per annum in 2 years.
The value of x is
A. 8%
B. 9%
C. 10%

P a g e 172 | 580
D. 6%
Answer & Explanation :

Answer: Option C
Explanation :

S.I = Principal*Time*Rate/100
Therefore 4000*3**/100 = 5000*2*12/100 => * 5*2*12/4*3
=10% per annum

314. If x2 – 3x + 1 = 0, then the value of x2 + x + 1/x + 1/x2 is

A. 6
B. 8
C. 10
D. 2
Answer & Explanation :
Answer: Option C
Explanation :

x2 - 3x + 1 = 0
=> X2 + 1 - 3x
Dividing both sides by x, => X + 1/3 - 3

Therefore x2 + x + 1/x + 1/x2


= (x2 + 1/x2) + (x + 1/x)
= (x + 1/x)2 -2+ (x + 1/x)
= 9-2 + 3 = 10

315. If (4x - 3)/x + (4y - 3)/y + (4z - 3)/z = 0, then the value of 1/x + 1/y +
1/z is

A. 4
B. 6

P a g e 173 | 580
C. 9
D. 3

Answer & Explanation :


Answer: Option A

Explanation :
Z(4x - 3)/x + (4y - 3)/y + (4z - 3)/z = 0
=> 4x/x - 3/x + 4y/y - 3/y + 4z/z - 3/z = 0
=> 3/x + 3/y + 3/z = 4 + 4 + 4 = 12
=> 1/x + 1/y + 1/z = 12/3 = 4

316. Find the simplest value of 2√50 + √18 - √72 (given √2 = 1.414)
A. 10.312
B. 8.484
C. 4.242
D. 9.898
Answer & Explanation :

Answer: Option D
Explanation :

317. A number x when divided by 289 leaves 18 as a remainder. The same


number when divided by 17 leaves y as a remainder. The value of y is

P a g e 174 | 580
A. 3
B. 1

C. 5
D. 2

Answer & Explanation :


Answer: Option B
Explanation :
Here, the first divisor (289) is a multiple of second divisor (17)
∴ Required remainder = Remainder obtained on dividing 18 by 17 = 1. Hence
the answer is option 2.

318. An equation of the form ax + by + c = 0 where a ≠ 0, b ≠ 0, c = 0


represents a straight line which passes through
A. (0, 0)
B. (3, 2)

C. (2, 4)
D. None of these
Answer & Explanation :
Answer: Option A
Explanation :
Ax+by+c = 0
When c = 0
ax+by = 0
by = -ax ⇒ y = - ax/b

when x = 0, y = 0 i.e., this line passes through the origin (0,0).

319. The numerator of a fraction is 4 less than its denominator. If the


numerator is decreased by 2 and the denominator is increased by 1, then the
denominator becomes eight times the numerator. Find the fraction.

P a g e 175 | 580
A. -4/8
B. 2/7

C. 3/8
D. 3/7

Answer & Explanation :


Answer: Option D
Explanation :
Original fraction = (x - 4)/x
In case II,

8(x - 4 - 2) = x + 1
⇒ 8x - 48 = x + 1
⇒ 7x = 49 ⇒ x = 7
∴Original fraction

= (7 - 4)/7 = 3/7

320. In a triangle ABC, ∠A = 90°, ∠C = 55°, AD bar ⊥ BC bar. What is the value
of ∠BAD?

A. 45
B. 55
C. 35
D. 60
Answer & Explanation :

Answer: Option B
Explanation :

P a g e 176 | 580
321. Eight litres are drawn from a cask full of wine and the cask is filled with
water. This operation is performed three more times. The ratio of the quantity
of wine now left in case to that of water is 16:65. How much wine did the cask
hold originally?
A. 18 litres
B. 24 litres
C. 32 litres
D. 42 litres
E. None of these

Answer & Explanation :


Answer: Option B
Explanation :

Ratio of the quantity of wine now left in case to that of water is 16:65
So out of total 81 units 16 units is wine.
Wine left : total solution = 16:81
As 8 litres are drawn off and replaced by water and is repeated 3 times more
This means (2/3)4

So after one removal ratio of wine and water = 2:3


It means (3u-2u)1 unit = 8 litres

3 unit = 8*3 = 24 litres

P a g e 177 | 580
322. A copper wire is bent in the form of an equilateral triangle and has area
121√3cm2 . If the same wire is bent into the form of a circle. The area (in cm2)
enclosed by the wire is (take - 22/7)
A. 364.5

B. 693.5
C. 346.5
D. 639.5
E. None of these
Answer & Explanation :

Answer: Option C

323. What will be the ratio of petrol and kerosene in the final solution formed by
mixing petrol and kerosene that are present in three identical vessels in the
ratio 4:1,5:2 and 6 :1 respectively?

A. 166 : 22
B. 83 : 22

C. 83 : 44
D. 78 : 55
Answer & Explanation :

Answer: Option B
Explanation :
E. None of these
Three identical vessels in the ratio 4:1,5:2 and 6 :1 respectively.

Petrol : kerosene
(4:1 = 5)7
(5:2 = 7)5

(6:1 = 7)5
28:7 =35

25:10=35

P a g e 178 | 580
30:5 =35
83:22

324. Mrs. Sharma invests 15% of her monthly salary, i.e., Rs. 4428 in Mutual
Funds. Later she invests 18% of her monthly salary on Pension Policies also she
invests another 9% of her salary on Insurance Policies. What is the total
monthly amount invested by Mrs. Sharma?
A. Rs. 113356.8
B. Rs. 12398.4

C. Rs. 56678.4
D. Can't determined
E. None of these
Answer & Explanation :
Answer: Option B

Explanation :
15% of monthly salary = Rs 4428

So monthly salary = Rs 29500


Total money invested = 42% of 29500 = Rs 12398.4

325. If 30 men working 7 hours a day can do a piece of work in 18 days, in how
many days will 21 men working 8 hours a day do the same work ?
A. 20 1/2 days
B. 22 1/2 days

C. 24 1/2 days
D. 25 days
E. None of these

Answer & Explanation :


Answer: Option B

Explanation :

P a g e 179 | 580
D = 18 7/8*30/21 = 22 1/2 days

326. lf 3 cannon firing 4 rounds in 6 minutes kill 250 men in half an hour, how
many cannon, firing 3 rounds in 5 minutes will kill 600 men in an hour ?

A. 4 cannon
B. 5 cannon
C. 6 cannon
D. 8 cannon
E. 9 cannon

Answer & Explanation :


Answer: Option E
Explanation :
In first case round fired in one min = 4/6 and in second round = 3/5 Therefore
new cannons required = 3 5/2*600/250*1/2 = 9

327. Nine students of a class contribute a certain sum. Seven of them give Rs.
5 each. The remaining two give Rs. 5 and Rs. 9 more than the average
contribution of all the 9 students respectively. The average contribution of the
class of 9 students is -

A. Rs. 10
B. Rs. 14
C. Rs. 7
D. Rs. 12
Answer & Explanation :
Answer: Option C
Explanation :

Let the average contribution of class = x (7 5) + (x + 5) + (x + 9) = 9x 35 +


2x + 14 = 9x 49
= 7x x = 7.

P a g e 180 | 580
328. One-fourth of my marks in English is equal to one third of my marks in
Hindi, The total number of marks secured by me in both the subjects is 140.
The marks secured by me in English are ...
A. 60

B. 80
C. 75
D. None of these
Answer & Explanation :
Answer: Option B

Explanation :
Let marks in English = x & Hindi = y.
x/4 = y/3 Also x + y = 140. Solving the 2 equations we get x = 80.

329. Percent profit earned when an article is sold for Rs. 546/- is double the
percent profit earned when the same article is sold for Rs. 483/-. If the marked
price of the article is 40% above the cost price, what is the marked price of the
article?
A. Rs. 588/-
B. Rs. 608/-

C. Rs. 616/-
D. Rs. 596/-
E. Rs. 586/-
Answer & Explanation :
Answer: Option A
Explanation :
Let profit be P and C.P.= x

Now, x+2P=546
x+P=483

subtracting both,
P=63
P a g e 181 | 580
x=483-63=420
M.P.= 1.4*420=588

330. C is 40% efficient less than A. A and B together can finish a piece of work
in 10 days. B and C together can do it in 15 days. In how many days A alone
can finish the same piece of work?
A. 18
B. 12
C. 14

D. 20
E. 15
Answer & Explanation :
Answer: Option B
Explanation :
Let work is completed by A in A days
work is completed by B in B days

work is completed by C in C days


Given that, C= A/0.6
Also 1/A+1/B=1/10

1/B+1/C==1/15
Solving Equations A=12

DIRECTIONS for questions 331 to 335: Refer to the graph and answer the given
question:

P a g e 182 | 580
331. What is the difference between the total number of scarves sold by store M
in 2003 and 2004 together and total number of scarves sold by store N in 2005
and 2006 together?
A. 160
B. 100
C. 140

D. 150
E. 120
Answer & Explanation :
Answer: Option E
Explanation :

Total Number of scarves sold store M in 2003 and 2004= 190+320= 510
Total Number of scarves sold store M in 2005 and 2006=260+370= 630
Difference between the total Number of scarves sold store M in 2003 & 2004
and
Total Number of scarves sold store N in 2005 and 2006 is = 630 – 510 = 120.
P a g e 183 | 580
332. Number of scarves sold by store M decreased by what percent from 2004
to 2005?
A. 40 5/8

B. 45 3/8
C. 42 3/8
D. 30 3/8
E. 35 5/8
Answer & Explanation :

Answer: Option A
Explanation :
Percentage decrease in the number of scarves sold by score M =
320-190/320 * 100 = 130/320 * 100 = 40 5/8%

333. If the respective ratio between total number of scarves sold by stores M
and N together in 2002 and that in 2009 is 15 : 11, what is the total number of
scarves sold by stores M and N together in 2009?
A. 430
B. 450

C. 420
D. 460

E. 440
Answer & Explanation :

Answer: Option E
Explanation :
Ratio between total number scarves sold by stores M & N together in 2002 and
that in 2009 is 15:11.
Total number scarves sold by stores M & N together in 2002 = 240+360 = 600

According to question,= 600/x = 15/11 Or, x = 440

P a g e 184 | 580
Total number scarves sold by stores M & N together in 2009 = 440.

334. If the total number of scarves sold by stores M and N together in 2008 is
10% more than that in 2006, what is the total number of scarves sold by stores
M and N together in 2008?
A. 638
B. 406
C. 414
D. 396

E. 408
Answer & Explanation :
Answer: Option A
Explanation :
Total number of scarves sold by stores M & N in 2006 = 210+370 = 580
Total number of scarves sold by stores M & N in 2008 = 580×110/100 = 638

335. What is the average number of scarves sold by store N in 2003, 2004 and
2005?
A. 260

B. 270
C. 290

D. 250
E. 290

Answer & Explanation :


Answer: Option B
Explanation :

Average number of scarves sold store N in 2003, 2004 & 2005 = 300+250+260
/3 = 270

P a g e 185 | 580
336. What is the central angle corresponding to number of bags available in
store T? (In degrees)
A. 91.2
B. 95.6

C. 93.6
D. 94.2
E. 92.5
Answer & Explanation :
Answer: Option C
Explanation :
Central angle corresponding to number of bags available in store T =
26/100*360= 93.6 Deg

337. What is the difference between the average number of bags available in
stores P and R together and the average number of bags available in stores S
and T together?

P a g e 186 | 580
A. 12
B. 22

C. 15
D. 18

E. 20
Answer & Explanation :
Answer: Option C
Explanation :
Average percentage of bags available in stores P and R together = 35%/2 =
17.5%
Average percentage of bags available in stores S and T together = 40%/2 =
20%
Difference between Average Number of bags available in stores P and R
together and

Average Number of bags available in stores S and T together = (20% - 17.5%)


of 600 = 15

338. The respective ratio between number of bags available in store P in August
and that available in the same store in July was 5: 4. How many bags were
available in store P in August as compared to July?
A. 150
B. 90
C. 24
D. 60
E. 45
Answer & Explanation :

Answer: Option A
Explanation :
Number of bags available in P store in July = 20% of 600 = 120.
Now, since we have

P a g e 187 | 580
Number of bags available in store P in August : Number of bags available in P
store in July = 5:4

=> P : 120 = 5 : 4
Thus, number of bags available in Store P (In August) = (5/4)x120 = 150

Total Number of Percentage of Number of


Universities faculty Assistant Associate
members professors professors
J 250 60 75
K 180 75 24
L 150 80 16
M 100 63 21

339. What is the difference between the total number of Associate Professors in
Universities J and M together and the total number of Professors in the same
universities together?
A. 54
B. 55
C. 68
D. 58
E. 53
Answer & Explanation :
Answer: Option B
Explanation :
Total number of Associate professor in universities J and M together = 75+21=
96
Total number of professor in universities J and M together = 25+16= 41
Difference between the Total number of Associate professor in universities J and
M together and Total number of professor in universities J and M together= 96-
41= 55

P a g e 188 | 580
340. In University M, 8/21 of the Assistant Professors are males and in
University L, 3/5 of the Assistant Professors are males. What is the respective
ratio between male assistant Professors in University M and that in University L?
A. 2:5

B. 1:3
C. 3:5
D. 2:7
Answer & Explanation :
Answer: Option B

Explanation :
341. A Chartered Accountant applies for a job in two firms X & Y. The
probability of his being selected in firm X is 0.7, and being rejected at Y is 0.5
and the probability of rejection at least one of his applications is 0.6. What is
the probability that he will be selected in one of the firms?
A. 0.8
B. 0.2

C. 0.4
D. 0.7
Answer & Explanation :

Answer: Option A
Explanation :
P(X) = 0.7, P(Y) = 0.5 and P(X &union; Y) = 0.4. Therefore, P(X &intersection;
Y) = P(X) + P(Y) – P(X &intersection; Y) = 0.7 + 0.5 – 0.4 = 0.8

342. Two small circular parks of diameters 16 m, 12 m are to be replaced by a


bigger circular park. What would be the radius of this new park, if the new park
has to occupy the same space as the two small parks?
A. 15m
B. 10m
C. 20m
D. 25m
P a g e 189 | 580
Answer & Explanation :
Answer: Option B

343. A conical vessel of base radius 2 cm and height 3 cm is filled with


kerosene. This liquid leaks through a hole in the bottom and collects in a
cylindrical jar of radius 2 cm. The kerosene level in the jar is
A. π cm
B. 1.5 cm
C. 1 cm

D. 3 cm
Answer & Explanation :
Answer: Option C
Explanation :
Let the level of kerosene in the cylinder = H.
volume of cone = volume of the cylinder
=> 1/3π(2)2(3) = π(2)2H => H = 1 cm

344. The angle of elevation of an aeroplane from a point on the ground is 45°.
After 15 seconds flight, the elevation changes to 30°. If the aeroplane is flying
at a height of 3000 m, the speed of the plane in km per hour is
A. 208.34
B. 306.72
C. 402.56

D. 527
Answer & Explanation :
Answer: Option D

Explanation :

P a g e 190 | 580
Let the plane at the start is at point C and after 15 seconds it reaches point D
after covering x km as shown in diagram.

In Δ ABC, tan 45° = BC/AB => 1 = BC/AB => AB = BC = 3,000.


In %Delta; ADE, tan 30°= ED/(AB+BE) => 3000/(3000+BE) => BE = 2196 m .
Therefore x = 2.196 km.
So aeroplane covers 2.196 km in 15 seconds, so speed of aeroplane = (2.196 x
60 x 60)/15 = 527kmph.

345. At the first stop on his route, a driver unloaded 2/5 of the packages in his
van. After he unloaded another three packages at his next stop, 1/2 of the
original number of packages remained. How many packages were in the van
before the first delivery?
A. 25
B. 10

C. 30
D. 36
Answer & Explanation :
Answer: Option C
Explanation :

Let the packages = P. Therefore, after first delivery he unloaded 2/5 of P and
then he unloaded 3
more packages. So total packages unloaded = 2/5 of P + 3. According to the
given condition

2/5 of P + 3 = 1/2 of P => P = 30

P a g e 191 | 580
346. If S is 150 percent of T, then T is what percent of S + T?
A. 40%

B. 100/3%
C. 75%

D. 80%
Answer & Explanation :
Answer: Option A
Explanation :
S = 1.5 T. Now T/(S + T) x 100 = T/2.5T x 100 = 40%

347. An investor earns 3% return on 1/4th of his capital, 5% on 2/3rd and 11%
on the remainder. What is the average rate of return he earns on his total
capital?
A. 10%
B. 5%
C. 5.5%

D. 10.5%
Answer & Explanation :
Answer: Option B

Explanation :
Let the capital be C.

Therefore, 3% of 1/4 C + 5% of 2/3 C + 11% of (C - 1/4C - 2/3C) = R % of C


=> R = 5%

348. Mixture of milk and water has been kept in two separate containers. Ratio
of milk to water in one of the containers is 5 : 1 and that in the other container
is 7 : 2. In what ratio should the mixtures of these two containers be added
together so that the quantity of milk in the new mixture may become 80%?
A. 3 : 2
B. 2 : 3

P a g e 192 | 580
C. 4 : 5
D. None of these

Answer & Explanation :


Answer: Option D

Explanation :
Let x litres of mixture be taken from container having 5 : 1 ratio and y litres be
taken from 7 : 2 mixture container.
Therefore, according to the given condition,

So they should be mixed in ratio 2 : 3

349. Rahul started a business with a capital of Rs. 8,000. After six months,
Sanjay joined him with an investment of some capital. If at the end of the year
each of them gets an equal amount as profit, how much did Sanjay invest in the
business?
A. Rs. 17,500/-
B. Rs. 18,000/-

C. Rs. 16,000/-
D. Rs. 16,500/-
Answer & Explanation :
Answer: Option C
Explanation :
Rahul’s investment
= Capital × Time period = 8000 × 12. Sanjay’s Investment = C × 6.
As both of them share equal profit, so Rahul’s investment = Sanjay’s
investment

=> 8000 × 12 = C × 6 => C = Rs. 16,000.

P a g e 193 | 580
350. Sita buys a fridge at 15/16 of its original value & sells it for 10 % more
than its value. Then gain % is

A. 15.55
B. 11.67

C. 16.67
D. None of these
Answer & Explanation :
Answer: Option D
Explanation :

Let the original price be O. Cost price = of O and selling price = 1.1 of O.
Gain% = 11/10 of O-15/16 of O / 15/16 of O * 100 = 17.33%
DIRECTIONS for the questions 3 to 4: In the following series, one of the
terms given is wrong. Find that term and mark that as your answer.

351. 2 3 10 39 178 885


A. 855

B. 178
C. 10
D. 304

E. 39
Answer & Explanation :

Answer: Option B
Explanation :

The logic is ×1+1, ×2+4, ×3+9, ×4+16, ×5+25,….


So following the logic we get 178 is wrong instead it should be 172. …..

352. 139 150 149 174 223 304


A. 149
B. 150
P a g e 194 | 580
C. 174
D. 304

E. 223
Answer & Explanation :

Answer: Option B

353. The figure given below, MNP is an equilateral triangle and LMNO is a
square with side 6 cm. What is the area of the pentagon PMLON

A. 72√3 cm2
B. 36√3 cm2
C. 6( 3 + √3 ) cm2

D. 9( 4 + √3 ) cm2
E. 48√3 cm2
Answer & Explanation :
Answer: Option D

354. What is the total cost of construction of a 1m wide path outside the
rectangular plot along all the four sides, given the following 2 condtitions :
I. Perimeter of the plot in 260 m and cost of construction of the path is Rs. 850
per m2

II. Area of the plot is 4125 m2 and cost of construction of the path is Rs. 850
per 2

P a g e 195 | 580
A. The data in statement II alone are sufficient the question the question, while
the data in statement I alone are not sufficient to answer the question.

B. The data in both the statements I & II are insufficient to answer the
question.

C. The data either in statement I alone or in II statement alone are sufficient to


answer the question.
D. The data in both the statements I & II together are necessary to answer the
question.
E. The data in Statement I alone are sufficient to the question, while the data in
statement II alone are not sufficient to answer the question.
Answer & Explanation :

Answer: Option D
Explanation :
By using both the statements together ,
The perimeter of plot ,2( L + B ) = 260m Area of plot, L × B = 4125 m2,
Solving these two equations :

we can find that the length of the plot is 75m and the breadth of the plot is
55m.
Now we can easily find the area of the path all around the rectangular plot.

Also the cost of construction is same as Rs 850/m2. Hence we can find the total
cost of construction of the path.

356. In the stream running at 3 kmph, a motorboat goes 12 km upstream and


returns back to the starting point in 128 minutes. What is the speed of the boat
in still water? (in kmph)
A. 20

B. 12
C. 15
D. 25
E. 10
Answer & Explanation :

P a g e 196 | 580
Answer: Option B
Explanation :

Let speed of boat in still water = x km/hr


Speed of stream = 3 km/hr

Speed downstream = ( x + 3 ) km/hr,


Speed upstream = ( x - 3 ) km/hr
Therefore according to question,
12/( x - 3 ) + 12/( x + 3 ) = 128/60
Solving ,we get x = 12 km/hr.

357. The respective ratio between ages of P and Q is 11: 7. The respective ratio
between P’s age 2 years ago and Q’s age 2 years hence is 7:5. Find the
respective ratio between P’s age 7 years hence and Q’s age 7 years ago?
A. 17:9
B. 15:7
C. 19:13

D. 13:5
E. 17:7
Answer & Explanation :

Answer: Option E
Explanation :

Let the present ages of P and Q be 11x,7x respectively. According to the


question => (11x - 2)/(7x + 2) = 7/5 => x = 4

Ratio of P’s age 7 years hence & Q’s age 7 years ago = (11 × 4 + 7)/(7 × 4 - 7)
= 51/21 = 17/7

358. A jar was containing 60 liters of mixture of milk and water in the ratio 7:5.
From this jar 12 litres of mixtures was taken out and 8 litres of pure milk was
added. What is the respective ratio of milk and water in the mixture after the
final operation?

P a g e 197 | 580
A. 9:5
B. 4:3

C. 9:7
D. 6:5

E. 9:3
Answer & Explanation :
Answer: Option A
Explanation :
Let n be the mixture quantity of milk = 7/12 × 60 Ltr. = 35 Ltr.

Quantity of water = 5/12 × 60 Ltr. = 25 Ltr.


12 Ltr. of mixture removed contains milk = 7/12 × 12 Ltr. = 7 Ltr and water =
5/12 × 12 Ltr = 5 Ltr
After adding 8Ltr of pure milk,
Net milk in the mixture = 35 - 7 + 8 = 36 Ltr.
Net water in the mixture = 25 - 5 = 20 Ltr.
So the required ratio of milk and water now = 36/20 = 9:5

359. A project manager estimated that he would complete the project in time if
he hires 42 people for 38 days. At the end of 30 days he realized that only
3/5th of the work is complete. How many more men does he need to hire to
complete the work in time?
A. 71
B. 47

C. 63
D. 60
E. 75

Answer & Explanation :


Answer: Option C

Explanation :

P a g e 198 | 580
42 people in 30 days did 3/5th of the work
Let x people are required to complete the remaining 2/5th of the work in 8 days

Applying chain rule we get x = 42 × 30/8 × (2×5)/(3×5) = 105 people


Additional no. of people required = ( 105 - 42 ) = 63.

360. A basket contains 3 blue, 5 black and 3 red balls. If two balls are drawn at
random, what is the probability that none of them is blue?
A. 22/55
B. 3/55
C. 28/55

D. 9/11
E. None of these
Answer & Explanation :
Answer: Option C
Explanation :
Here n(S) = 11C2
Now, two balls can be drawn from 5(black)+3(red) balls = 8 balls in 8C2 ways

∴ n(E) = 8C2
Now, P(E) =
8C2/11C = 28/55
Other method:
Required probability
= 1 - (27/55) = 28/55

361. A basket contains 3 blue, 5 black and 3 red balls. If 2 balls are drawn at
random, what is the probability that one is black and one is red?
A. 2/11
B. 8/11

C. 9/11
D. 3/11
P a g e 199 | 580
E. None of these
Answer & Explanation :

Answer: Option D
Explanation :

Selecting 1 black ball out of 5 = 5C1 ways


Selecting on red ball out of 3 = 3C1 ways
The required probability = (5C1 × 3C1)/11C2 = 3/11

362. What should come in place of question mark (?) in the following
number/alphabetic series?
24 ? 109 134 150 159
A. 71
B. 65
C. 86
D. 53
E. None of these

Answer & Explanation :


Answer: Option E

363. A boat takes 8 hours to cover a distance while travelling upstream,


whereas while travelling downstream it takes 6 hours. If the speed of the
current is 4 kmph, what is the speed of the boat in still water?
A. 12 kmph

B. 28 kmph
C. 16 kmph
D. Cannot be determined

E. None of these
Answer & Explanation :

Answer: Option B

P a g e 200 | 580
Explanation :
No matter if the boat goes upstream or downstream, Distance covered is same.
So we can simply equate the two distances.
We have (x+4) × 6 = (x-4) × 8

Where x = speed of the boat in still water


Now, 6x + 24 = 8x - 32 or x = 28
Hence, the required speed = 28 km/hour

364. A man buys a land and gives for it 20 times the annual rent Find the rate
of interest he gets for his money.
A. 10%
B. 24%
C. 45%
D. 18%
E. 5%
Answer & Explanation :

Answer: Option E
Explanation :
Let annual rent is 1 Rs. so buys the land at 20 Rs. So by investing Rs.20 he is
getting Rs.1 as interest. so on Rs.100 he gets Rs.5 . so rate%=5%.Hence
option E is the answer.

365. A man while returning from his factory, travels 2/3 of the distance by bus,
3/4 of the rest by car and remaining by foot. If he travels 2 km on foot, find the
distance covered by him.
A. 24km

B. 22km
C. 28km
D. 26km
E. None of these

P a g e 201 | 580
Answer & Explanation :
Answer: Option A

Explanation :
Let total distance be D kms

2/3D + 1/3*3/4D + 2 = D
Therefore D = 24 km
Or going by options
Bus = (2/3) * 24 = 16,
Remaining = 24 - 16 = 8, car = (3/4) * 8 = 6

Total distance = 16 + 6+ 2 = 24km

366. Mira’s expenditure and saving are in the ratio 3:2. Her income increases by
10%. Her expenditure also increases by 12%. By how much % do her saving
increase?
A. 7%
B. 9%

C. 10%
D. 13%
E. None of these

Answer & Explanation :


Answer: Option A

Explanation :
Let total income be = 5

Increased income will be = 5.5


Increased expenditure will be = 3.36
Increased saving will be = 5.5 - 3.36 = 2.14

Percentage increase will be = (14/2) * 100 = 7%<

P a g e 202 | 580
367. A thief goes away in a car at a speed of 60 kmph. A cop starts chasing the
thief after 15 minutes in a car at a speed of 80 kmph. When will the cop
overtake the thief from the start?
A. 30 minutes

B. 36 minutes
C. 40 minutes
D. 45 minutes
E. 50 minutes
Answer & Explanation :

Answer: Option D
Explanation :
In 15 minutes thief will be 15 Km away . There will be a gap of 15 Km between
thief and police . This gap will be covered with a relative speed of 80 -60=20
Km/h .So thief will be overtaken in 15/20 hours= 45 minutes.

368. A shopkeeper sells sugar at a profit of 20% and uses a weight which is 25
% less. What is total % gain?
A. 45%
B. 22.5%

C. 55%
D. 60%
E. None of these
Answer & Explanation :
Answer: Option D
Explanation :
Let cost of 1 gram of sugar is 1 Rs.

So cost price of 1000 gram is 1000Rs.


As per question, Selling price of 1000 gram is 1200 Rs.

But he uses weight 25% less i.e. 750 gram for a Kg.
So SP of 750 gram is 1200 Rs. while its CP is 750 Rs.
P a g e 203 | 580
Hence profit %=450/750 =3/5=60%

369. In an examination it is required to get 296 of the total maximum


aggregate marks to pass. A student gets 259 marks and is declared failed. The
difference of marks obtained by the student and that required to pass is 5%.
What are the maximum aggregate marks a student can get?
A. 690
B. 780
C. 740

D. 749
E. None of these
Answer & Explanation :
Answer: Option C
Explanation :

Let the aggregate marks be x


Therefore, 296 - 259 = 37 will be 5% of the x

Thus, (5/100) * x = 37
=> x = 740
370. Shyama invested Rs. P for 2 years in scheme A which offered 11% p.a.
simple interest. She also invested Rs. 600 + P in scheme B which offered 20%
compound interest (compounded annually), for 2 years. If the amount received
from scheme A was less than that received from scheme B, by Rs. 1216, what
is the value of P?
A. Rs. 1,500
B. Rs. 1,400
C. Rs. 2,000

D. Rs. 1,600
E. Rs. 1,800
Answer & Explanation :
Answer: Option D

P a g e 204 | 580
Explanation :
Money Invested=P at 11% Simple interest.

P+600 At 20% C.I.


Given that,

1.2P+1216=1.44(P+600)
Or 0.22P= 352
Hence P=1600

371. A vessel contains 180 litres of mixture of milk and water in the respective
ratio of 13 : 5. Fifty-four litres of this mixture was taken out and replaced with 6
litres of water, what is the approximate percentage of water in the resultant
mixture?
A. 41
B. 31

C. 24
D. 9

E. 17
Answer & Explanation :
Answer: Option B

Explanation :
Milk: water = 13:5

Volume of solution=180 l
Solution taken out= 54 l

Volume of solution left= 180-54=126 l


In 126 l solution,
Milk= 126*13/18= 91 l

Water=126*5/18=35 l
As 6 l water is added

Water= 35+6= 41 l

P a g e 205 | 580
Total solution volume= 126+6= 132 l
Percentage of water= 41/132*100= 31 %

372. A started a business with an investment of Rs. 28,000. After 5 months


from the start of the business, B and C joined with Rs. 24,000and Rs. 32,000
respectively and withdrew Rs. 8000 from the business. If the difference
between A’s share and B’s share in the annual profit is Rs. 2,400, what was the
annual profit received?
A. Rs. 15,600

B. Rs. 14,400
C. Rs. 14,040
D. Rs. 15,360
E. Rs. 13,440
Answer & Explanation :

Answer: Option B
Explanation :

Equivalent Contribution of A= 28000*5+20000*7= 280000


Equivalent Contribution of B= 24000*7= 168000
Equivalent Contribution of C= 32000*7= 224000

Let total profit be X.


Given that,
280000X/672000 – 168000X/672000=2400
112000/672000*X=2400

or X=2400*672/112
X=14400

373. The sum of two numbers is equal to 15 and their arithmetic mean is 25 per
cent greater than their geometric mean. Find the numbers.

A. 5 & 10
B. 3 & 12
P a g e 206 | 580
C. 1 & 14
D. 6 & 9

Answer & Explanation :


Answer: Option B

Explanation :
AM of 2 numbers is a+b/2 GM of 2 numbers is √ab When sum of 2 numbers is
15, their AM is 7.5. AM = 1.25 (GM)
GM = 7.5/1.25 = 6. Hence 36 = ab. So product of 2 numbers is 36. Try by
options B is the correct answer.

374. The product of the digits of a two-digit number is twice as large as the sum
of its digits. If we subtract 27 from the required number, we get a number
consisting of the same digits written in the reverse order. Find the number.
A. 36

B. 27
C. 63

D. None of these
Answer & Explanation :
Answer: Option C

Explanation :
Go by options. 3rd option is the answer because 63 = product of digits = 6 3 =
18. Sum of digits = 6 + 3 = 9.
Hence product of digits is twice as the sum of the digits. Also 63 – 27 = 36. So
digits are reversed.

375. Dhruva gave 35% of her monthly salary to her mother. From the
remaining salary, she paid 18% towards rent and 42% she kept aside for her
monthly expenses. The remaining amount she kept in bank account. The sum of
the amount she kept in bank and that she gave to her mother was Rs. 43,920.
What was her monthly salary?
A. Rs. 80,000

P a g e 207 | 580
B. Rs. 75,000
C. Rs. 64,000

D. Rs. 76,000
E. Rs. 72,000

Answer & Explanation :


Answer: Option E
Explanation :
Let ‘x’ be the monthly salary, then
(65/100 × 40/100)x + 35/100x = 43920

Solving, X= 72000

376. 18 litres of pure water was added to a vessel containing 80 litres of pure
milk. 49 litres of the resultant mixture was then sold and some more quantity of
pure milk and pure water was added to the vessel in the respective ratio of 2 :
1. If the resultant respective ratio of milk and water in the vessel was 4 : 1,
what was the quantity of pure milk added in the vessel? (in litres)

A. 4
B. 8
C. 10

D. 12
E. 2
Answer & Explanation :
Answer: Option E

Explanation :
80(M) + 18(W) = 98
49 liters sold => 49 is left

40(M) + 9(W)
Let x be the quantity of pure milk added

Given, (40 + 2x)/(9 + x) = 4/1

P a g e 208 | 580
Solving, x = 2

377. A pipe can fill a cistern in 12 minutes and another fill it in 15 minutes, but
a third pipe can empty it in 6 minutes. The first two are kept open for 6 minutes
in the beginning and then the third pipe is also opened, in what time is the
cistern emptied?
A. 54 min
B. 56 min
C. 58 min

D. 60 min
E. 57 min
Answer & Explanation :
Answer: Option A
Explanation :

First two pipe’s one minute work = 1/12 + 1/15 = 5+4/60 = 9/60 First two
pipe’s 6 minutes work = 9/60 6 = 9/10. Now one minutes work of three pipes =
1/12 + 1/15 - 1/6 = -1/60 -ve sign shows that if all pipes are opened together
they will empty the full cistern in 60 minutes. Now we have 9/10 of cistern filled
with water. Time taken to empty 9/10 of water in cistern = 9/10 60 = 54
minutes

378. A and B promise to do a work for Rs. 75. A alone can do it in 20 days and
B in 30 days, with the help of C they are able to finish it in 8 days. How will A, B
and C respectively distributes the wages?
A. Rs. 20, Rs. 30, Rs. 25
B. Rs. 25, Rs. 20, Rs. 30

C. Rs. 30, Rs. 25, Rs. 20


D. Rs. 30, Rs. 20, Rs. 25
E. None of these
Answer & Explanation :
Answer: Option D

P a g e 209 | 580
Explanation :
Everybody will get the wages according to their labor

A’s one day work = 1/20 , A’s 8 days work = 8/20 = 2/5 B’s one day work =
1/30 B’s 8 days work = 8/30 = 4/15 Remaining work = 1- 2/5 - 4/15 = 1/3 C
did the 1/3 of work in 8 days wages are divided in the ratio of 2/5:4/15:1/3 =
6:4:5 A’s share = 6/15 75= 30 Rs., B’s share = 4/15 75 = 20 Rs. ,C’s share =
5/15 75 = 25 Rs

379. One ball is drawn at random from a box containing 3 red balls, 2 white
balls and 4 blue balls, what is the probability that the ball is a red ball?
A. 1/4

B. 1/3
C. 1/5
D. 2/5
E. 2/3
Answer & Explanation :

Answer: Option C
Explanation :
Total possible outcomes = 9. (i.e. One out of 9 balls) [Favourable outcomes

i.e. One out of 3 Red balls] = 3. Reqd Probability = 3/9 = 1/3

DIRECTIONS for the questions 3 to 4: In the following series, one of the terms
given is wrong. Find that term and mark that as your answer

380. 40 20 24 30 60 150
A. 24

B. 60
C. 20
D. 150
E. 30

P a g e 210 | 580
Answer & Explanation :
Answer: Option A

Explanation :
The logic is ×0.5 , ×1 , ×1.5, ×2 ,×2.5 …..

So following the logic we get 24 is the wrong number and instead it should be
20. Hence 1st option.

381. 29 33 41 50 77 98 157
A. 33

B. 77
C. 50
D. 98
E. 41
Answer & Explanation :
Answer: Option C
Explanation :

The logic is, +4, +8, +9, 27, +16, +64,….


So following the logic we get that 98 is wrong . So Ans is (D)

382. What is the percent profit earned by selling the article after giving 10%
discount? I. Had there been no discount offered, the profit earned would have
been 30%. II. Selling price of the article after giving discount is Rs. 3510/-
A. The data in statement II alone are sufficient to answer the question, while
the data in statement I alone are not sufficient to answer the question.
B. The data either in statement I alone or in statement II alone are sufficient to
answer the question.

C. The data in both the statement I & II together are not sufficient to answer
the question.
D. The data in both the statements I & II together are necessary to answer the
question.

P a g e 211 | 580
E. The data in statement I alone are sufficient to answer the question, while the
data in statement II alone not sufficient to answer the question.

Answer & Explanation :


Answer: Option E

Explanation :
From Statement I, Let CP = Rs.100 then we get MRP = 130 /- If 10% discount
is given then SP = Rs. 117
So, P% = 17%. hence I statement is sufficient to answer the question.
From Statement II we cannot determine the cost price in rupees. Hence we
cannot find the profit %.

383. The distance between point A and B is 722 km. At 8 am a car starts from
point A (towards point B) at 46 kmph and at 10 am another car starts points B (
towards point A) at 38 kmph. At what time will they meet?
A. 6.30 pm
B. 5.30 pm

C. 4.30 pm
D. 5 pm
E. 6 pm

Answer & Explanation :


Answer: Option B
Explanation :
Distance covered by car A from 8 am to 10 am = (46 × 2) km = 92 km
At 10 am the remaining distance between the two cars = 722 km - 92km = 630
km.
Using, Distance = (Relative speed) × time

630 = (46 + 38) × time taken to cross


630/84 = t
t = 71⁄2hr the two cars will meet at 71⁄2 hr from 10am i.e. 5:30 pm.

P a g e 212 | 580
384. A & B started a business together by investing Rs. 36000/- and Rs.
42000/- respectively. Both of them invested for one year whereas after 6
months from the start C joined them by investing a certain amount. If they
earned an annual profit of Rs. 39,200/- out of which C’s share is Rs. 7350/-.
What is the investment of C?
A. Rs. 24000/-
B. Rs. 36000/-
C. Rs. 27000/-
D. Rs. 33000/-

E. Rs. 30000/-
Answer & Explanation :
Answer: Option B
Explanation :
Let the investment of C = Rs x

Ratio of profits of A & B & C


= 36000 × 12 : 42000 × 12 : x × 6

= 72000 : 84000 : x
=> ( Profit of C / Total Profit ) = ( x / 72000 + 84000 + x ) = 7350/39200
On solving, x = 36000.

385. A rectangular plot, 42m long and 32m wide, has two concrete crossroads
(of same width) running in the middle of the plot (one parallel to length and the
other parallel to breadth). The rest of the plot is used as a lawn. If the area of
the lawn is 1064 sq. m., what is the width of the roads?
A. 3m
B. 4.5m

C. 5m
D. 3.5m
E. 4m
Answer & Explanation :

P a g e 213 | 580
Answer: Option E
Explanation :

386. A jar was containing 60 liters of mixture of milk and water in the ratio 7:5.
From this jar 12 litres of mixtures was taken out and 8 litres of pure milk was
added. What is the respective ratio of milk and water in the mixture after the
final operation?

A. 9:5
B. 4:3

C. 9:7
D. 6:5
E. 9:3
Answer & Explanation :
Answer: Option A
Explanation :
Let n be the mixture quantity of milk = 7/12 × 60 Ltr = 35 Ltr
Quantity of water = 5/12 × 60 Ltr. = 25 Ltr

12 Ltr. of mixture removed contains milk = 7/12 × 12 Ltr. = 7 Ltr and water =
5/12 × 12 Ltr = 5 Ltr
After adding 8 Ltr of pure milk,
Net milk in the mixture = 35 - 7 + 8 = 36 Ltr.
Net water in the mixture = 25 - 5 = 20 Ltr.

So the required ratio of milk and water now = 36/20 = 9/5

P a g e 214 | 580
DIRECTIONS for the question 4: In the following series, one of the terms given
is wrong. Find that term and mark that as your answer.

387. 13 16 24 39 73 98

A. 16
B. 73
C. 39
D. 96
E. 24

Answer & Explanation :


Answer: Option B
Explanation :
The first set of differences are
+3,+8,+15,+24,+35,+48
And again the differences are
+5,+7,+9,+11,+13…….which are in AP

So following the logic we get that 73 is wrong. So Ans is B.

388. Equal sums of money were invested in scheme A and scheme B for two
years. Scheme A offers simple interest and scheme B offers compound interest
(compounded annually) and the rate of interest (p.c.p.a) for both the schemes
are same. The interest accrued from Scheme A after two years is Rs. 2560/-
and from scheme B is Rs. 2688/-. Had the rate of interest (p.c.p.a) of scheme A
been 6% more, what would have been the interest accrued from Scheme A
after two years?
A. Rs. 3972/-

B. Rs. 4124/-
C. Rs. 4266/-
D. Rs. 4096/-
E. Rs. 4154/-

P a g e 215 | 580
Answer & Explanation :
Answer: Option D

Explanation :
The SI for 2 years=Rs.2560/-

Since simple interest for each year is same.


SI per annum is Rs.1280
CI for 2 years is Rs.2688/-
Since the rate of interest is same
So CI for 1st year is Rs.1280

CI for 2nd year is (Rs. 1280 + Rs.128)


Hence Rate of interest per annum is = 128/1280×100% = 10%
If the rate of interest is increased by 6% then SI for 2 years = 16%/10%×2560
= Rs. 16×256 = Rs.4096.

389. A & B started a business together by investing Rs. 36000/- and Rs.
42000/- respectively. Both of them invested for one year whereas after 6
months from the start C joined them by investing a certain amount. If they
earned an annual profit of Rs. 39,200/- out of which C’s share is Rs. 7350/-.
What is the investment of C?

A. Rs. 24000/-
B. Rs. 36000/-
C. Rs. 27000/-
D. Rs. 33000/-
E. Rs. 30000/-
Answer & Explanation :
Answer: Option B

Explanation :
Let the investment of C=Rs x

Ratio of profits of A &B &C


=36000×12 : 42000×12 : x×6
P a g e 216 | 580
=72000:84000: x

Solving, x = 36000. B option.

390. What are the marks obtained by Harish in Maths ?

I. Average marks obtained by Harish in Maths and economies are 83.


II. Average marks obtained by Harish in Maths and statistics are 92.
A. The data in statement I alone are sufficient to answer the question, while the
data in statement II alone not sufficient to answer the question.
B. The data in both the statements I & II together are necessary to answer the
question.
C. The data either in statement I alone or in statement II are sufficient to
answer the question.
D. The data in both the statements I & II together are insufficient to answer the
question.
E. The data in statement II alone are sufficient to answer the question, while
the data in statement I alone are not sufficient to answer the question.

Answer & Explanation :


Answer: Option D
Explanation :

We cannot find the marks in maths alone even by using both the statements
together as average marks will not tell us exact marks in Maths.
Data is insufficient. Hence D option.

391. By what percent is Rajesh’s salary more/less than Mangesh’s ?


I. Salary of Rajesh is Rs. 12000/-.
II. Salary of Rajesh and Mangesh together is Rs 28000/-

A. The data in statement I alone are sufficient to answer the question, while the
data in statement II alone not sufficient to answer the question.

P a g e 217 | 580
B. The data in both the statements I & II together are necessary to answer the
question.

C. The data either in statement I alone or in statement II are sufficient to


answer the question.

D. The data in both the statements I & II together are insufficient to answer the
question.
E. The data in statement II alone are sufficient to answer the question, while
the data in statement I alone are not sufficient to answer the question.
Answer & Explanation :

Answer: Option B
Explanation :
By using both the statements together we can find the salary of Mangesh .
Hence we can find the percentage change between the salaries of Rajesh and
Mangesh. B option.

392. What is the strength of MBA institute where students study only HR,
Marketing and Finance ?
I. Number of students studying HR, Marketing and Financial are in the ratio of 2
: 3: 5 respectively.
II. Number of students studying Marketing is more than those studying HR by
800.
A. The data in statement I alone are sufficient to answer the question, while the
data in statement II alone are not sufficient to answer the question.
B. The data either in statement I alone or in statement II alone are sufficient to
answer the question.
C. The data in both the statements I & II together are necessary to answer the
question.
D. The data in statement II alone are sufficient to answer the question, while
the data in statement I alone not sufficient to answer the question.
E. The data in both the statements I & II together are insufficient to answer the
question.

Answer & Explanation :


Answer: Option C
P a g e 218 | 580
Explanation :
Using both statements together , Let no. of students of HR, marketing and
Finance be 2x,3x,5x and also 3x-2x=800
x=800

So total students =2x+3x+5x=10×800=8000. Hence C.

393. A shopkeeper purchased 96 identical shirts @ Rs. 220/- each. He spent Rs.
3800/- on transport and packing and fixed a marked price of Rs. 450/- each
shirt. However, he decided to give discount of 20% on the marked price of each
shirt. If he could sell all the shirts, what is the approximate percent profit
earned by him?

A. 44
B. 39
C. 33
D. 31
E. 30

Answer & Explanation :


Answer: Option D
Explanation :

Cost of 96 shirts=Rs96×220=Rs.21120
Cost of transportation and packaging =Rs3800
Total Cost price (CP)
=Rs 3800+21120 = Rs24920
Marked Price (MP) of 1 shirt =Rs.450
Discount=20% of Rs450
Net Selling Price (SP) of 1 shirt

=80% of Rs450 = Rs360


Total SP of 96 shirts =Rs 360×96=Rs.34560

Profit Percentage = (SP - CP)/CP * 100%


(34560 - 24920)/24920 * 100% = 38.68% = 39%. Hence, option B.
P a g e 219 | 580
394. A school room is to be built to accommodate 70 children, so as to allow 2.2
m2 of floor and 11 m3 of space for each child. If the room be 14 metres long,
what must be its breadth and height?
A. 12 & 5.5 metres

B. 13 & 6 metres
C. 11 & 5 metres
D. 11 & 4 metres
Answer & Explanation :
Answer: Option C

Explanation :
22 × 70 = 154 m2 for 70 students.
11 × 70 = 770 m3 for 70 students.
Matching through options answer is 3rd option.

395. Two pipes P and Q can fill a cistern in 3 and 6 minutes respectively, while
an empty pipe R can empty the cistern in 4 minutes. All the three pipes are
opened together and after 2 minutes pipe R is closed. Find when the tank will
be full
A. 3 minutes

B. 6 minutes
C. 5 minutes
D. 8 minutes
Answer & Explanation :
Answer: Option A
Explanation :
1/3 + 1/6 – 1/4 = 3/12.

Work done = 3 × 2 = 6.
Remaining work = 12 – 6 = 6

Hence in addition to 2 minutes, 1 minute more will be needed. So answer is 1st


option.

P a g e 220 | 580
396. There is a leak in the bottom of a cistern. Before the leak, it could be filled
in 4 1/2 hours. It now takes 1/2 hour longer. If the cistern is full, in how much
time would the leakage empty the full cistern?

A. 23 hours
B. 35 hours
C. 52 hours
D. 45 hours
Answer & Explanation :

Answer: Option D
Explanation :
2/9 – 1/x = 1/5.
x = 45 hours.
So answer is 4th option.

397. Two filling pipes A and B can fill a tank in 30 and 20 hours respectively.
Pipe B alone is kept open for half the time and both pipes are kept open for the
remaining time. In how many hours, will the tank be completely full?
A. 25 hours

B. 40 hours
C. 15 hours
D. 28 hours
Answer & Explanation :

Answer: Option C
Explanation :
1/30 + 1/20 = 5/60.

Let tank will be completely full in x hours.


x/2 × 1/20 + x/2 × 1/12 = 1.

So x = 15.

P a g e 221 | 580
Hence, 3rd option.

398. A man covers a certain distance on a toy train. If the train moved 4 km/hr
faster, it would take 30 minutes less. If it moved 2 km/hr slower, it would have
taken 20 minutes more. Find the distance
A. 60 km
B. 45 km
C. 30 km
D. 20 km

Answer & Explanation :


Answer: Option A
Explanation :
Distance = Speed × Time.
Distance = (Speed+4) (Time-30/60).
Distance= (Speed-2) (Time+20/60).
Solving all equations we get Time = 3 hours,

Speed = 20 km/h.
So, Distance=20 x 3 = 60km
So answer is 1st option.

399. The average speed of a train is 20% less on the return journey than on the
onward journey. The train halts for half an hour at the destination station before
starting on the return journey. If the total time taken for the to and fro journey
is 23 hours, covering a distance of 1000 km, the speed of the train on the
return journey is
A. 60 km/hr

B. 40 km/hr
C. 50 km/hr
D. 55 km/hr
Answer & Explanation :

P a g e 222 | 580
Answer: Option B
Explanation :

Let speed of onward journey = x km/h.


Equation will be = 500/x +500/0.8x = 22.5.

By solving we can get the answer as 40 km/h which is 2nd option.

400. Two trains move from station A and station B towards each other at the
speed of 50 km/hr and 60 km/hr. At the meeting point, the driver of the second
train felt that the train has covered 120 km more. What is the distance between
A and B?
A. 1320 km
B. 1100 km
C. 1200 km
D. 960 km

Answer & Explanation :


Answer: Option A

Explanation :
Do this question through options we get the answer as 1st option.

401. Rohit took a loan of ` 20,000 to purchase an LCD TV set from a finance
company. He promised to make the payment after three years. The company
charges compound interest at the rate of 10% per annum for the same. But,
suddenly the company announces the rate of interest as 15% per annum for the
last one year of the loan period. What extra amount does Rohit have to pay due
to this announcement of the new rate of interest?
A. 1320 km

B. 1100 km
C. 1200 km
D. 960 km
Answer & Explanation :
Answer: Option D
P a g e 223 | 580
Explanation :
20000(1+10/100)3,

20000(1+10/100)2(1+15/100) = 27830.
So taking the difference = 1210.

So answer is 4th option.

402. A tree was planted three years ago. The rate of its growth is 30% per
annum. If at present the height of the tree is 670 cm, what was the height of
the tree when it was planted?

A. 305 cm
B. 500 cm
C. 405 cm
D. 625 cm
Answer & Explanation :
Answer: Option A
Explanation :

Let 3 years ago the height of tree = x cm.


After 3 years = x(130/100)3 = 670.
⇒ x=305

So answer is 1st option.


403. A sum was put at simple interest at a certain rate for 2 years. Had it been
put at 3% higher rate, it would have fetched Rs 300 more. The sum is
A. 5300
B. 5500

C. 5000
D. None of these
Answer & Explanation :

Answer: Option C
Explanation :

P a g e 224 | 580
Increase of 3% fetched Rs.300 more.
It is for 2 years.

For 1 year Increase of 3% will fetch Rs.150.


So 1 % will fetch Rs.50

100% = 5000.

404. The difference between the simple interest received from two different
sources on Rs 3 lakhs for 2 years is rs 1,500. The difference between their rates
of interest is

A. 0.20%
B. 0.3%
C. 0.25%
D. 0.4%
Answer & Explanation :
Answer: Option C
Explanation :

Let x and y be the interest rates.


x% of 3 lakhs – y% of 3 lakhs = 1500
(x– y) % of 3 lakhs = 1500

300000(x – y)/100 = 1500


(x – y) = 0.5% for 2 years For 1 year = 0.25%

405. A square tin sheet of side 12 cm is converted into a box with open top in
the following steps: The sheet is placed horizontally. Then, equal-sized squares,
each of side x cm, are cut from the four corners of the sheet. Finally, the four
resulting sides are bent vertically upwards in the shape of a box. If x is an
integer, then what value of x maximizes the volume of the box?
A. 1
B. 4
C. 3

P a g e 225 | 580
D. 2
Answer & Explanation :

Answer: Option D
Explanation :

Volume of the box = l × b × h = (12 – 2x) (12 – 2x) (x)

Putting x = 1, 2, 3, 4, we get the maximum value of the above equation at


x = 2. So maximum v = 128. i.e. 4th option

406. A spherical ball of lead, 3 cm in diameter is melted and recast into three
spherical balls. The diameter of two of these is 1.5 cm and 2 cm respectively.
The diameter of the third ball is
A. 3 cm

B. 2.66 cm
C. 2.5 cm

D. 5 cm
Answer & Explanation :
Answer: Option C
Explanation :
The sum of the volumes of the new balls will equal the volume of the original
ball. So, 1.53 = 0.753 + 13 + r3. Solving this equation gives r = 1.25
So, d = 2.5 cm.

P a g e 226 | 580
407. A cylinder is filled to 4/5th of its volume. It is then tilted so that the level
of water coincides with one edge of its bottom and top edge of the opposite
side. In the process, 30 cc of the water is spilled. What is the volume of the
cylinder?

A. 75 cc
B. 100 cc
C. 96 cc
D. Data Insufficient
Answer & Explanation :

Answer: Option B
Explanation :
When tilted, the water level will form the diagonal of the tank and the volume of
the water will he half that of the cylinder. Suppose the volume of the cylinder is
V. Since 30 ml = 30 cc has spilled out, 4V/5 = 1/2 V + 30.
Solving this equation gives V = 100 cc.

408. A and B invest Rs 3 lakhs and Rs 4 lakhs in a business. A receives Rs


1,000 per month out of the profit as a remuneration for running the business
and the rest of profit is divided in proportion to the investments. If in a year 'A'
totally receives X 39,000, what does B receive?
A. 63,000

B. 46,000
C. 36,000

D. 26,000
Answer & Explanation :
Answer: Option C

Explanation :
A and B invest in the ratio 3 : 4.
A receives 1000 per month so he will get 12000 for a year.
If A receives 39000 in total then 39000 – 12000 = 27000 will be his share.
If 3x = 27000, then 4x = 36000.
P a g e 227 | 580
409. In a partnership, A invests 1/6 of the capital for 1/6 of the time, B invests
1/3 of the capital for 1/3 of the time and C, the rest of the capital for whole
time. Find A's share of the total profit of Rs 2,300.

A. 110
B. 10
C. 100
D. 101
Answer & Explanation :

Answer: Option C
Explanation :
Let the total capital is 6 and time is also 6 years.
A invests 1 for 1 year.
B invests 2 for 2 years
Then, C invests 6 – (1 + 2) = 3 for 6 years.
The ratio of investment is A : B : C = 1 : 4 : 18

If total profit is Rs 2300 then A's share


= 1/23 x 2300 = Rs 100

410. A, B and C enter into a partnership by investing in the ratio of 3 : 2 : 4.


After one year, B invests another Rs 2,70,000 and C, at the end of 2 years
invests Rs 2,70,000. At the end of three years, profits are shared in the ratio of
3 : 4 : 5. Find the initial investment of A, B and C.
A. Rs 2,70,000; Rs 80,000 and Rs 3,60,000
B. Rs 1,70,000; Rs 1,80,000 and Rs 3,60,000
C. Rs 2,70,000; Rs 1,80,000 and Rs 3,60,000

D. Rs 2,70,000; Rs 1,80,000 and Rs 3,00,000


Answer & Explanation :

Answer: Option C
Explanation :
P a g e 228 | 580
1st year, A = 3x, B = 2x and C = 4x
2nd year, A = 3x, B = 2x + 270000 and C = 4x

3rd year, A = 3x, B = 2x + 270000 and


C = 4x + 270000

Total investment by A, B and C is


A = 3x + 3x + 3x = 9x
B = 2x + 2x + 2x + 270000 + 270000
= 6x + 540000
C = 4x + 4x + 4x + 270000 = 12x + 270000

After three years the ratio is 3 : 4 : 5


x = 90000.
Thus initial investment
= (3 * 90000): (2 * 90000): (4 * 90000)
` 270000, ` 180000, ` 360000

411. Two vessels contain mixtures of milk and water in the ratio of 8 : 1 and 1 :
5 respectively. The contents of both of these are mixed in a specific ratio into a
third vessel. How much mixture must be drawn from the second vessel to fill
the third vessel (capacity 26 gallons) completely in order that the resulting
mixture may be half milk and half water?
A. 10 gallons
B. 12 gallons
C. 14 gallons

D. 13 gallons
Answer & Explanation :
Answer: Option C

Explanation :
The ratio of milk to total volume of the mixture in the 3 vessels is 8/9, 1/6 and
1/2 respectively. We can consider these values as 16/18, 3/18 and 9/18
respectively. By alligation, we get the ratio in which the two mixtures are mixed

P a g e 229 | 580
as 6:7 respectively. Since the total quantity is 26, the quantity from the 2nd
vessel is (7/13) * 26 = 14 gallons.

412. Meera purchased 23 bracelets at the rate of Rs 160 per bracelet. At what
rate per bracelet should she sell the bracelets so that profit earned is 15% ?

A. Rs. 184/-
B. Rs. 186/-
C. Rs. 192/-
D. Rs. 198/-
E. None of these

Answer & Explanation :


Answer: Option A
Explanation :
Since the CP of each bracelet is Rs 160, to make a profit of 15%, the SP should
be 160 × 1.15 = Rs 184.

413. Kajal spends 55% of her monthly income on grocery, clothes and
education in the ratio of 4 : 2 : 5 respectively. If the amount spent on clothes is
Rs. 5,540/-, what is Kajal's monthly income ?
A. Rs. 55,400/-

B. Rs. 54,500/-
C. Rs. 55,450/-
D. Rs. 55,650/-
E. None of these
Answer & Explanation :
Answer: Option A
Explanation :

Let the amount spent on grocery, clothes and education is x. We have 2x/11 =
5540. So, x = Rs. 30470. This is 55% of the total income. Hence total income =
(30470 X 100)/55 = Rs 55400.

P a g e 230 | 580
414. 35 percent of a number is two times 75 percent of another number. What
is the ratio between the first and the second number respectively?

A. 35 : 6
B. 31 : 7

C. 23 : 7
D. 32 : 9
E. None of these
Answer & Explanation :
Answer: Option E

Explanation :
Let two numbers be a and b. Now, 35% of a = 2 times 75% of b. solving we get
a : b = 30 : 7 Therefore, the correct answer is option E (none of these).

415. Area of a rectangular field is 3584 m2 and the length and the breadth are
in the ratio 7 : 2 respectively. What is the perimeter of the rectangle ?
A. 246m

B. 292m
C. 286m
D. 288m

E. None of these
Answer & Explanation :

Answer: Option D

416. Last year there were 610 boys in a school. The number decreased by 20
percent this year. How many girls are there in the school if the number of girls
is 175 percent of the total number of boys in the school this year ?

A. 854
B. 848

C. 798
D. 782
P a g e 231 | 580
E. None of these
Answer & Explanation :

Answer: Option A
Explanation :

The number of boys this year = 0.8 × 610 = 488. Since the girls are 175% of
the boys, So, the number of girls = 175% of 488 = (175 × 488)/100 = 854

417. Aryan got 350 marks and Vidya scored 76 percent marks in the same test.
If Vidya scored 296 marks more than Aryan, what were the maximum marks of
the test ?
A. 650
B. 900
C. 850
D. 950

E. None of these
Answer & Explanation :

Answer: Option C
Explanation :
Aryan scored =350, Vidya scored 296 more than Aryan i.e. 350 + 296 = 646.
Also, Vidya scored 76% of maximum marks. So, 76% of maximum marks =
646 => maximum marks = 850

418. A student was awarded certain marks in an examination. However, after


re-evaluation, his marks were reduced by 40% of the marks that were originally
awarded to him so that the new score now became 96. How many marks did
the student lose after re-evaluation?

A. 58
B. 68
C. 63
D. 56
E. 64
P a g e 232 | 580
Answer & Explanation :
Answer: Option E

Explanation :
Since the student has lost 40% marks, So, he has 60% of the original marks
say x. => 0.6 x = 96 => x = 160. Hence, marks lost by him is 160 - 96 = 64.

419. Average of five numbers is 61. If the average of first and third number is
69 and the average of second and fourth number is 69, what is the fifth number
?

A. 31
B. 29
C. 25
D. 35
E. None of these

Answer & Explanation :


Answer: Option B

Explanation :
Since the average of all five numbers is 61, so the total of all five numbers is 61
× 5 = 305. Total of Ist and IIIrd numbers = 138 and total of IInd and IVth
numbers is 138. So the fifth number is 305 - 138 -138 = 29.

420. The respective ratio between the present ages of father, mother and
daughter is 7 : 6 : 2. The difference between mother's and the daughter's age is
24 years. What is the father's age at present?
A. 43 years
B. 42 years

C. 39 years
D. 38 years
E. None of these
Answer & Explanation :

P a g e 233 | 580
Answer: Option B
Explanation :

The ratio between the present ages of father, mother and daughter is 7 : 6 : 2.
Let their ages be 7x, 6x and 2x respectively. Now, we have, 6x - 2x = 24 => x
= 6. Hence, age of father is 6 × 7 = 42 years.

421. Average weight of 19 men is 74 kgs. and the average weight of 38 women
is 63 kgs. What is the average weight (rounded off to the nearest integer) of all
the men and the women together

A. 59kgs
B. 65kgs
C. 69kgs
D. 67kgs
E. 71kgs

Answer & Explanation :


Answer: Option D

Explanation :
The average weight of all men and women = (19x74+38x63)/57 = 66.6 =
67kgs

422. A basket contains 3 blue, 5 black and 3 red balls. If 3 balls are drawn at
random what is the probability that all are black?
A. 2/11
B. 1/11
C. 3/11
D. 8/33
E. None of these

Answer & Explanation :


Answer: Option A
Explanation :
Ways of selecting 3 black balls out of 5 - 5C3

P a g e 234 | 580
Total ways of selecting 3 balls - 11C3The required probability =
5C3 / 11C3 = 10/165 = 2/33

DIRECTIONS for questions 423-424: What should come in place of


question mark (?) in the following number/alphabetic series?

423. 17 9 10 ? 35 90
A. 21
B. 27.5

C. 19
D. 16.5
E. None of these
Answer & Explanation :
Answer: Option D
Explanation :
17×0.5+0.5=9

9×1+1=10
10×1.5+1.5=16.5
16.5×2+2=35

35×2.5+2.5=90
Hence, the question mark(?) should be replaced by 16.5

424. 3 20 78 334 1696 ?

A. 8410
B. 9836
C. 10206

D. 1150
E. None of these

P a g e 235 | 580
Answer & Explanation :
Answer: Option C

Explanation :
3×2+14=20

20×3+18=78
78×4+22=334
334×5+26=1696
1696×6+30=10206
Hence, the question mark should be replaced by 10206

425. The magnitude of the area of a circle is seven times that of its
circumference. What is the circumference (in units) of the circle?
A. 616
B. 132
C. 88
D. Can't be determined

E. None of these
Answer & Explanation :
Answer: Option C

Explanation :
We have πr2 = 7 x 2πr
∴ r = 14

∴ circumference = 2πr = 2 x (22/7) * 14 = 88

426. 15.002 × ? × 25.0210 = 7113.018


A. 19
B. 26

C. 11

P a g e 236 | 580
D. 31
E. 35

Answer & Explanation :


Answer: Option A

Explanation :
15.002 × ? × 25.0210 = 7113.018

427. A mixture of 45 liters of spirit and water contains 20% of water in it. How
much water must be added to make the water 25% in the new mixture?
A. 5 liters
B. 3 liters
C. 4 liters
D. 6 liters
E. None of these
Answer & Explanation :
Answer: Option B
Explanation :
Quantity of water in the mixture = (20/100) x 45 = 9 L
Let x litres of water must be added to make the water 25 % in the solution
Therefore, [(9+x)/(45+x)] x 100 = 25

⇒ 36 + 4x = 45 + x
⇒ 3x = 9

⇒ x = 3 litres

P a g e 237 | 580
428. The sum of four consecutive integers is 102. Find the product of the
extremes:

A. 548
B. 684

C. 648
D. 578
E. None of these
Answer & Explanation :
Answer: Option C

Explanation :
a,a+1,a+2,a+3 , let these be the numbers.
Sum=102⇒4a+6=102⇒4a=96⇒a=24
Hence extremes=24 , 27
24X27=648

429. A bottle contains 3/4 of milk and the rest water. How much of the mixture
must be taken away and replaced by equal quantity of water so that the
mixture has half milk and half water?
A. 25%
B. 331⁄2%
C. 45%
D. 50%
E. None of these
Answer & Explanation :

Answer: Option B
Explanation :
Let the total quantity of mixture = 1 liter.

Now supposing x liters of mixture is withdrawn which contains 3/4x milk and
rest water.

P a g e 238 | 580
So, according to the question 3/4 -3/4x =1/2.
Solving x = 1/3 which means 1/3 of mixture is to be withdrawn to serve the
purpose.
So, answer is 33.33%.

430. Krishan has some hens and some cows. If the total number of animal
heads are 59 and the total number of feet are 190, how many cows does
Krishan have?
A. 36

B. 32
C. 23
D. 20
E. Can't be determined
Answer & Explanation :

Answer: Option A
Explanation :

Suppose he has x cows and y hens.


So x + y = 59 and 4x + 2y = 190.
Solving these two equations, we get x = 36.

So option A is the answer

431. The height of a room is 5 metres and its length is twice its breadth. If 240
metres of paper of breadth 50 cm are required for papering its walls, find the
area of the floor.
A. 128 m2
B. 64 m2

C. 320 m2
D. 32 m2

E. None of these
Answer & Explanation :
P a g e 239 | 580
Answer: Option D
Explanation :

H = 5, L = 2B, Area of 4 = Area of wall paper


2(LH) + 2(BH) = 240 × 50/100

10L + 10 B = 24 × 5, 10L + 5L= 24 × 5


15 L = 24 × 5, L =24×5/15 = 8
L= 8, B = 4,
∴area of floor = L × B

= 8 × 4 = 32
432. The respective ratio of salaries of A and B is 8:7, if the salary of B
increases by 20% and the salary of A increases by 21% the new ratio becomes
96 : 77 respectively. What is A's salary?
A. Rs. 22,560/-
B. Rs. 21,600/-
C. Rs. 20,640/-

D. Rs. 23.040/-
E. Cannot be determined
Answer & Explanation :
Answer: Option E
Explanation :
Only ratio of the salary is given, but salary is not given, so we cannot
determined the salary of A . Hence answer is option E.

433. (1/3)rd the diagonal of a square is 2. What is the measure of the side of
the concerned square?
A. 12 m
B. 9 m
C. 18 m
D. 6 m

P a g e 240 | 580
E. 7 m
Answer & Explanation :

Answer: Option B
Explanation :

Let a be the side of a square.


Therefore, diagonal (d) = √2a
We are given that
Hence the answer is option B.

434. (√4356 * √?) / √6084 = 11


A. 144
B. 196
C. 169
D. 81
E. 121
Answer & Explanation :

Answer: Option C

435. M, N, O and P divided Rs. 44,352/- among themselves M took (3/8)th of


the money, N took (1/6)th of the remaining amount and the rest was divided
among O and P in the ratio of 3:4 respectively. How much did O get as his
share?
A. Rs. 9,600/-

B. Rs. 20,600/-
C. Rs. 10,300/-
D. Rs. 8,700/-

E. Rs. 9,900/-
Answer & Explanation :

Answer: Option E

P a g e 241 | 580
Explanation :
Share of M = (3/8) * 44352 = 16632/-

Remaining amount = 44352 – 16632 = 27720/-


Share of N = (1/6) * 27720 = 4620/-

Remaining amount = 27720 – 4620 = 23100/-


Share of O = (3/7) * 23100 = 9900/-
Hence answer is option E.

436. 3 ? 14 55 274 1643

A. 11
B. 5
C. 6
D. 8
E. 7
Answer & Explanation :
Answer: Option B

Explanation :
>Here the given pattern is:
3 ? 14 55 274 1643

The pattern:
3 × 2 – 1 = 5, 5 × 3 – 1 = 14
14 × 4 – 1 = 55, 55 × 5 – 1 = 274
274 × 6 – 1 = 1643, So ? = 5

Hence answer is option B.

437. The perimeter of a rectangle whose length is 6m more than its breadth is
84m. What would be the area of a triangle whose base is equal to the diagonal
of the rectangle and whose height is equal to the length of the rectangle? (in
m2)

P a g e 242 | 580
A. 324
B. 372

C. 360
D. 364

E. 348
Answer & Explanation :
Answer: Option C
Explanation :
Let l, b and d be the length, breadth and diagonal of the rectangle

Therefore, l – b = 6 and 2(l + b) = 84 or l + b = 42


Solving the above equations, we get l = 24m and b = 18m
Therefore, d2 = ( l2 + b2) Putting the values, we get
d2 = 242 + 182 = 576 + 324 = 900 => d = 30m
Let b and h be the base and height of the triangle
We are given that b = d = 30m and h = l = 24m
Thus, area of triangle = (1/2)bh = (1/2) * 30 * 24 = 360 m2

438. The sum of the series (1 + 0.6 + 0.06 + 0.006 + 0.0006 + ....) is
A. 1 2/3
B. 1 1/3

C. 2 1/3
D. 2 2/3
Answer & Explanation :
Answer: Option A

Explanation :
1 + 0.6 + 0.06 + 0.006 + 0.0006 + … = 1.666 ….. = 1.6 = 1 6/9 = 1 2/3

439. A number, when divided by 114, leaves remainder 21. If the same number
is divided by 19, then the remainder will be

P a g e 243 | 580
A. 1
B. 2

C. 7
D. 17

Answer & Explanation :


Answer: Option B
Explanation :
If the first divisor is a multiple of second divisor, then the remainder in second
case = remainder obtained by dividing the first remainder by the second divisor.

Remainder = 21 ÷ 19 = 2

440. A shopkeeper earns a profit of 12% on selling a book at 10% discount on


the printed price. The ratio of the cost price and the printed price of the book is
A. 45 : 56
B. 45 : 51
C. 47 : 56

D. 47 : 51
Answer & Explanation :
Answer: Option A

Explanation :
Let the CP be Rs. 100

SP = Rs 112
If the marked price be Rs. x, then 90% of x = 112

=> x = 112*100/90 = Rs. 1120/9


= 900 : 1120 = 45 : 56

441. By selling a bicycle for Rs. 2,850, a shopkeeper gains 14%. If the profit is
reduced to 8%, then the selling price will be

A. Rs. 2,600

P a g e 244 | 580
B. Rs. 2,700
C. Rs. 2,800

D. Rs. 3,000
Answer & Explanation :

Answer: Option B
Explanation :
CP of bicycle
= 100/114 * 2850 = Rs. 2500
SP for a profit of 8%

= 108/100 * 2500 = Rs. 2700

442. If A's income is 50% less than that of B's, then B's income is what per cent
more than that of A?
A. 125
B. 100
C. 75

D. 50
Answer & Explanation :
Answer: Option B

Explanation :
Required percentage
= 50/100-50*100 = 100%

443. Simple interest on a certain sum is 16/25 of the sum. The rate percent if
the rate per cent and time (in years) are equal, is
A. 6%

B. 8%
C. 10%
D. 12%
P a g e 245 | 580
Answer & Explanation :
Answer: Option B

Explanation :
Interest/Principal = 16/25

Therefore Rate = S.I * 100/Principal*Time


=> x = 16/25*100/x
⇒ x2 16 × 4 = 64

⇒ x = √64 = 8% per annum

444. If the difference between S.I. and C.I. for 2 years on a sum of money lent
at 5% is Rs. 6, then the sum is
A. Rs. 2200
B. Rs. 2400

C. Rs. 2600
D. Rs. 2000
Answer & Explanation :
Answer: Option B
Explanation :

Difference = Pr2/10000
=> 6 = P*5*5/10000
=> P = 6 * 400 = Rs. 2400
445. At what percentage above the cost price must an article be marked so as
10 gain 33% after allowing customer a discount of 5% ?
A. 48%
B. 43%
C. 40%
D. 38%

Answer & Explanation :


Answer: Option C
P a g e 246 | 580
Explanation :
Condition

Let it be marked up by x%
A.T.C. (Successive % age females)

x-5-5x/100=33,
x = 40

446. Two persons start walking at a steady pace of 3 km/hour from a road
intersection along two roads that make an angle of 60° with each other. What
will be the shortest distance separating them at the end of 20 minutes?
A. 3kms
B. 2kms
C. 1.5kms
D. 1kms
Answer & Explanation :
Answer: Option D

447. X and Y work on a job together for four days and complete 60% of it. X
takes leave and then Y works for 8 more days to complete the job. How long X
will take to complete the entire job alone ?
A. 6 days
B. 8 days
C. 10 days

D. 11 days
Answer & Explanation :
Answer: Option C

448. A number when divided by 5 leaves remainder 3. What is the remainder


when the square of the same number is divided by 5?
A. 1
P a g e 247 | 580
B. 3
C. 0

D. 4
Answer & Explanation :

Answer: Option D
Explanation :
Remainder is a part of the number. So , whatever happens with the number,
happens with the remainder also. Square of remainder is 9.which will be divded
by 5 leaving remainder 4.

449. If 40% of the number exceeds the 25% of it by 54. Find the number
A. 360
B. 420
C. 560
D. 600
Answer & Explanation :

Answer: Option A
Explanation :
0.4x - 0.25x = 54. x = 360.

450. Among three numbers, the first is twice the second and thrice the third. If
the average of three numbers is 49.50, then what is the difference between the
first and the third number?

A. 28
B. 54
C. 39.50

D. 41.50
6x, 3x, 2x.

Answer & Explanation :

P a g e 248 | 580
Answer: Option B
Explanation :

Now average = 6x+3x+2x/9 = 39.5


On solving we get the difference in between first and the third number = 54.

451. Rita can knit a pair of socks in 3 days. Sita can knit the same in 6 days. If
they are knitting together, in how many days will they knit two pairs of socks.
A. 4
B. 1

C. 2
D. None of these
Answer & Explanation :
Answer: Option C
Explanation :
1 /3+ 1/6 = 1 /2 Together they will take, 1/ 1/2 = 2

452. The HCF and LCM of two numbers are 21 and 4641 respectively. If one of
number lies between 200 and 300, the two numbers are
A. 273, 357

B. 273, 361
C. 273, 359

D. 273, 363
Answer & Explanation :

Answer: Option A
Explanation :
Note that 273 is common in all the options

HCF × LCM Product of 2 numbers


21 × 4641 =273x, x = 357

P a g e 249 | 580
453. The fare on a commodity is diminished by 10% and its consumption
increases by 10%. The collection of the revenue derived from it is K% change.
The value of K is
A. 0

B. -1
C. 1
D. 2
Answer & Explanation :
Answer: Option B

454. In a Express Train passengers traveling in A.C. Sleeper Class, First Class
and Sleeper Class are in the ratio 1:2:3, and fare to each class in the ratio
5:4:2. If the income from this train is Rs. 57000, the income of A.C. Sleeper
Class is
A. Rs. 8000
B. Rs. 12000

C. Rs. 15000
D. Rs. 6000
Answer & Explanation :

Answer: Option C
Explanation :
Income is divided in the ratio 1×5:2×4:3×2= 5:8:6.
Now 5x + 8x + 6x = 19x = 57000.
∴ x =3000.

Therefore income from A.C. Sleeper Class = 3000 . 5=15000.


455. A farmer has decided to build a wire fence along one straight side of his
property. For this, he planned to place several fence-posts at 6 m intervals, with
posts fixed at both ends of the side. After he bought the posts and wire, he
found that the number of posts he had bought was 5 less than required.
However, he discovered that the number of posts he had bought would be just
sufficient if he spaced them 8 m apart. What is the length of the side of his
property and how many posts did he buy?
P a g e 250 | 580
A. 100 m, 15
B. 100 m, 16

C. 120 m, 15
D. 120 m, 16

Answer & Explanation :


Answer: Option D
Explanation :
Since it is given that length is sufficient for 6mt. and 8 mtr. gaps so it must be a
common multiple of 6 & 8 which out of choices can be 120 meters only. Now as
a concept to generate 15 gaps (= 120/8), you need 16 poles. So, answer is 4th
option.

456. The number of ways in which a committee of 3 ladies and 4 gentlemen can
be appointed from a meeting consisting of 8 ladies and 7 gentlemen, if Mrs. X
refuses to serve in a committee if Mr. Y is its member, is
A. 1960

B. 3240
C. 1540
D. None of these

Answer & Explanation :


Answer: Option C
Explanation :
Required no. of ways = No. of ways of selecting 3 ladies & 4 gentlemen - No. of
ways of selecting them when X & Y got selected
= 8C3 × 7C4 - 7C2 × 6C3 = 1960 - 420 = 1540.
Thus, option C is correct.

457. Ramesh takes twice as much time as Mahesh and thrice as much time as
Suresh to complete a job. If working together, they can complete the job in 4
days, then the time taken by each of them separately to complete the work is
A. 36, 24 and 16 day
P a g e 251 | 580
B. 20, 16 and 12 days
C. 24, 12 and 8 days

D. None of these
Answer & Explanation :

Answer: Option C
Explanation :
We are given that R = 2M = 3S. This means that Suresh is thrice efficient than
Ramesh and Mahesh is twice efficient than Ramesh.
So let us assume that Ramesh can do 1 unit work per day. so Mahesh will do 2
units per day and Suresh will do 3 units of the work per day.
So the total work done by all three in one day is 6 units. Now they together
take 4 days to complete the work.
So the total work is 6 x 4 = 24 units and individually Ramesh, Mahesh and
Suresh will take 24 days, 12 days and 8 days respectively.

458. Three sides of a triangular field are 20 meters, 21 meters and 29 meters
long, respectively. The area of the field is
A. 215 sq m
B. 230 sq m

C. 210 sq m
D. None of these
Answer & Explanation :
Answer: Option C
Explanation :
The given sides are 20,21,and 29 and 202+212=292
Since it is a right angled triangle, so area = 1/2 * 20 * 21 = 210 sq m.

459. Find the greatest number that will divide 964, 1238 and 1400 leaving
remainders 41, 31 and 51, respectively.
A. 71

P a g e 252 | 580
B. 81
C. 61

D. 73
Answer & Explanation :

Answer: Option A
Explanation :
We have 964 – 41 = 923, 1238 – 31 = 1207 and 1400 – 51 = 1349.
So the required number is the HCF of 923, 1207 and 1349 which is 71. Hence
1st option.

460. In a rhombus ABCD the diagonals AC and BD intersect at the point (3, 4).
If the point 'A' is (1, 2), the diagonal BD has the equation
A. x – y – l = 0
B. x – y + l = 0
C. x + y – l = 0
D. x + y – 7 = 0

Answer & Explanation :


Answer: Option D
Explanation :

Slope of AC = (2-4)/(1-3) = 1. As diagonals in a rhombus bisect each other at


90°, so slope of BD = -1.

Also diagonal BD has to lie on the point (3, 4), so equation of the line is y - 4 =
- 1(x - 3) ⇒ x + y – 7 = 0.

461. A circle has two parallel chords of lengths 6 cm and 8 cm. If the chords are
1 cm apart and the centre is on the same side of the chords, then the diameter
of the circle is of length:
A. 5 cm
B. 6 cm

C. 10 cm

P a g e 253 | 580
D. 12 cm
Answer & Explanation :

Answer: Option D
Explanation :

In the given figure, EB = 3 cm, FD = 4 cm, EF = 1 cm


Let OF = a cm
In triangle OFD, r2 =a2+42 in triangle OEB, r2 =(a+1)2+32

Solving the above two equations, we get a = 3cm


Putting the value of a in any of the above equation, we get r = 5 cm

Therefore, diameter = 10 cm.

462. An express train travelled at an average speed of 100 kmph, stopping for 3
minutes after every 75 km. A local train travelled at a speed of 50 kmph,
stopping for 1 minute after every 25 km. If the trains began travelling at the
same time, how many kilometres did the local train travel in the time it took the
express train to travel 600 km?
A. 900 km
B. 307.5 km
C. 1200 km

D. 100 km
Answer & Explanation :
Answer: Option B
Explanation :

P a g e 254 | 580
For 600 kms. The Express train needs 6 hrs. for the distance travelled @ 100
kmph. and it will stop for ( 600/75)=8.

So there must be 7 stoppages in between where it stops for 3 min. each


totalling upto 21 min. So total time will be 6 hrs.21 min.

Now the Local train travelling @ 50 kmph. will cover 300 kms. in 6 hrs. and in
rest 9 min. it will travel 7.5 kms.( 300/25=12 stoppages where it will stop for
12 min. So, 21- 12 = 9 ).
The total distance becomes 300 + 7.5 = 307.5 km.
ALTERNATE SOLUTION We can also think that since speed of express train is
double the local speed, so local train will tavel less than 600 km, Only option b
has such option and hence is the required answer

463. Two alloys contain zinc and copper in the ratio of 2 : 1 and 4 : 1. In what
ratio the two alloys should be added together to get as new alloy having zinc
and copper in the ratio of 3 : 1?
A. 7:5

B. 5:7
C. 3:5
D. None of the above
Answer & Explanation :
Answer: Option C

Explanation :

Required ratio = (4/5 – 3/4) : (3/4 - 2/3) = 3 : 5

P a g e 255 | 580
464. Swati and Rajani enter into a partnership with their capitals in the ratio 5 :
6. At the end of 8 months, Swati withdraws her capital. If they receive the profit
in the ratio of 5 : 9, find how long Rajani's capital was used?
A. 10 months

B. 12 months
C. 14 months
D. None of the above
Answer & Explanation :
Answer: Option B

Explanation :
Let the capital was invested for x months.
So (5 × 8)/(6 × x) = 5/9. Solving this, we get x = 12
465. The odds against a certain event are 5 : 2 and the odds in favour of
another independent event are 6 : 5. The probability that at least one of the
events will happen is:
A. 12/77

B. 25/77
C. 52/77
D. 65/77

E. None of these
Answer & Explanation :
Answer: Option C
Explanation :

Required probability = 1 – P (NO event will happen) = 1 – (5/7 × 5/11) =


52/77.

466. Number of times the hands of a clock are in a straight line (make 180°)
every day is
A. 44
B. 24

P a g e 256 | 580
C. 42
D. 22

Answer & Explanation :


Answer: Option D

Explanation :
In 12 hour, the hands of clock make 180° 11 times, thus total of 22 times in 24
hours.

467. A motor boat can travel at 10 km/h in still water. It traveled 91 km


downstream in river and then returned, taking altogether 20 hours. Find the
rate of flow of river.
A. 3
B. 5
C. 6

D. 7
E. None of these

Answer & Explanation :


Answer: Option A
Explanation :

Let the rate of flow be y kmph. The equation that will be formed is 91/(10+Y) +
91/(10-Y)=20
Solving we get y = 3.

468. If x^4 +(1/{x^4})=47 find the value of x^3 +(1/{x^3}) is


A. 18
B. 20

C. 22
D. 24

E. None of these

P a g e 257 | 580
Answer & Explanation :
Answer: Option A

469. A person invests Rs. 5508 in '4% stock at 102'. He afterwards sells out at
105 and reinvests in '5% stock at 126'. What is the change in his income?
A. 20
B. 7
C. 10
D. 9

E. None of these
Answer & Explanation :
Answer: Option D
Explanation :
Old income = 5508/102 × 4 = Rs. 216. No. of shares purchased = 5508/102 =
54. So sale proceeds = (54 × 105) = Rs. 5670.
Therefore, New income = (5670 × 5)/126 = Rs. 225.

So change in income = 225 – 216 = Rs. 9.

470. The ratio between the number of sides of two regular polygons is 1 : 2 and
the ratio between the measure of their interior angles is 2 : 3. The number of
sides of these polygons are respectively
A. 4,8
B. 5,10

C. 6,12
D. 8, 16
E. None of these

Answer & Explanation :


Answer: Option A

Explanation :

P a g e 258 | 580
Going by options, if the no of sides are 4 and 8, then the interior angles of the
polygons is 90deg and 135deg (int angles = 180°(N-2)/N)

Which are in the ratio 2 : 3. So the answer is 1st option.

471. How many two-digit odd numbers can be formed from the digits 1, 2, 3, 4,
5, if repetition of digit is allowed?
A. 5
B. 15
C. 35

D. 25
E. None of these
Answer & Explanation :
Answer: Option B
Explanation :
Here unit place can be filled in 3 ways (i.e. 1, 3, 5)
Ten's place can be filled in 5 ways. Required number of numbers = 5 x 3 = 15

472. Three equal glasses are filled with mixtures of sprit and water. The ratio of
the sprit to water is as follows: in the first glass as 3 : 4, in the second glass as
4 : 5 and in the third glass as 5 : 6.The contents of the three glasses are
emptied into a single vessel. What is the ratio of the sprit to water in the
mixture now?
A. 920:1159
B. 820:1149
C. 1120:1134
D. 1010 : 1122

E. None of these
Answer & Explanation :
Answer: Option A
Explanation :

P a g e 259 | 580
Required Ratio
{(3/7)+(4/9)+(5/11)}/{(4/7)+(5/9)+(6/11)} =920/1159

Hence answer is 1 option

473. Two pipes can fill a cistern in 14 and 16 hours respectively. The pipes are
opened simultaneously and it is found that due to leakage in the bottom, 32
minutes extra are taken for the cistern to be filled up. If the cistern is full, in
what time would the leak empty it?
A. 96

B. 102
C. 106
D. 112
E. None of The above
Answer & Explanation :

Answer: Option D
Explanation :

If both filling pipes are working together, time taken to fill the tank will be 14
X16/(14 + 16) = 112/15 hrs.
But due to leak it takes 32 min more. Hence total time taken = 112/15 + 32/60
= 8 hrs.
Let the leak takes x hours to empty the tank, so, 1/14 + 1/16 - 1/x = 1/8.
By solving we get the answer as 112 hours. So answer is 4th option.
474. In a race of 600 metres, A can beat B by 60 metres and in a race of 500
metres, B can beat C by 50 metres. By how many metres will A beat C in a race
of 400 metres?
A. 76 metres

B. 80 metres
C. 70 metres
D. 84 metres
Answer & Explanation :

P a g e 260 | 580
Answer: Option A
Explanation :

A runs B runs C runs


600 metres race 600m 540 m

500 metres race 500 m 450m


Combing ratio A runs B runs C runs
300metres - 2700meters - 2430metres
Unitary A runs B runs C runs
Method 400mtres - 360 metres - 324 metres

∴ A beats C by 400-324 = 76 metres.

475. If 20 men can build a wall 112 metres long in 6 days, what length of a
similar wall can be built by 25 men in 3 days?
A. 65mtr.
B. 52mtr.

C. 70mtr.
D. 78mtr.
Answer & Explanation :
Answer: Option C
Explanation :
20 men is 6 days can build 112 metres
25 men in 3 days can build = 112*(25/20)x(3/6)

= 70 meters

476. If the compound interest on a certain sum of money for 3 years at 10%
per annum be Rs. 993, what would be the simple interest?
A. Rs. 880

B. Rs. 890
C. Rs. 895
P a g e 261 | 580
D. Rs. 900
Answer & Explanation :

Answer: Option D
Explanation :

Let P = Principal
A - Amount
We have a = P(1 + R/100)3 and CI = A - P
ATQ 993 = P(1 + R/100)3 - P
∴ P = Rs 3000/-

Now SI @ 10% on Rs 3000/- for 3 yrs = (3000 x 10 x 3)/100


= Rs 900/-

477. What annual installment will discharge a debt of Rs. 4600 due in 4 years at
10% simple interest?
A. 1000

B. 1030
C. 1100
D. None of these
Answer & Explanation :
Answer: Option A
Explanation :
Let the annual instalment be Rs. 100. The first instalment will be paid one year
from now i.e. 3 years before it is actually due. The second instalment will be
paid two years from now i.e. 2 years before it is actually due.

The third instalment will be paid 1 year before it is actually due.


The fourth instalment will be paid on the day the amount is actually due.
On the first instalment the interest will be paid for 3 years, on the second for 2
years, on the third for 1 year, on the fourth for 0 year. In total an interest for 6
years will be paid (3 + 2 + 1 + 0) on Rs. 100 @ 10%. Interest = (100 × 6 ×
10)/100 = Rs. 60 and the principal is Rs 100 × 4 = Rs 400. The total loan that

P a g e 262 | 580
can be discharged is Rs. 400 + 60 = Rs. 460. Here the technique of Chain Rule
will be applied. I.e. for Rs. 460 the instalment required is Rs. 100, for Rs. 4600
the instalment required is 4600 × 100/460 = Rs. 1000.

478. A number whose fifth part increased by 5 is equal to its fourth part
diminished by 5, is
A. 160
B. 180
C. 200

D. 220
Answer & Explanation :
Answer: Option C
Explanation :
x/5 + 5 = x/4 - 5

⇒ x/5 - x/4 = 10

x/20 = 10
⇒ x = 200

479. Two numbers are such that the ratio between them is 3 : 5, but if each is
increased by 10, the ratio between them becomes 5:7. The numbers are
A. 3, 5
B. 7, 9
C. 13, 22
D. 15, 25
Answer & Explanation :
Answer: Option D
Explanation :

No's are in the ratio 3:5


Le the No's be 3x and 5x

P a g e 263 | 580
ATQ. (3x+10) : (5x+10) = 5:7
∴x=5

No's are (15,25)

480. A man rows downstream 30 km and upstream 18 km, taking 5 hours each
time. What is the velocity of the stream (current)?
A. 1.2 km/hr

B. 1.5 km/hr
C. 2.5 km/hr
D. 1.8 km/hr
Answer & Explanation :
Answer: Option A
Explanation :
LET X =SPEED OF BOAT AND Y = SPEED OF CURRENT.
⇒ 30/(X+Y)=18/(X-Y)=5 BY SOLVING Y = 1.2

481. A train 125 metre long is running at 50 km/hr. In what time will it pass a
man running at 5 km/hr in the same direction in which the train is going?

A. 25 sec
B. 10 sec
C. 20 sec
D. 15 sec
Answer & Explanation :
Answer: Option B
Explanation :
Distance = 125 metres Speed = 50-5 = 45kmph => 45 * (5/18) = 12.5 m/s
Time = 125 / 12.5 = 10 sec.

P a g e 264 | 580
482. A is twice as fast as B and B is thrice as fast as C is. The journey covered
by C in 42 minutes, will be covered by A in

A. 11 min
B. 14 min

C. 7 min
D. 17 min
Answer & Explanation :
Answer: Option C
Explanation :

B is thrice as fast as C
C covers in 42min
∴ B covers in 42/3 = 14 min
A is twice as fast as B
∴ A covers in 14 * (1/2) = 7 min

483. A worker is kept on a contract for 100 days to make some toys. On any of
these 100 days he does not make more than 20 toys. If on any day, he makes
more than 12 toys, then he makes at most 6 toys each on the next two days.
What is the maximum possible number of toys that he can make over the
period of 100 days?
A. 1109
B. 1208
C. 1100
D. 1076
E. None of these
Answer & Explanation :
Answer: Option B
Explanation :

If the worker makes more than 12 toys on any day then in three days period he
can make a maximum of 20 + 6 + 6 = 32 toys.

P a g e 265 | 580
On the other hand he could have made 36 toys over this span by making 12
toys each day.

So to achieve the maximum he must not make more than 12 toys on any day
except possibly the last day.

So, maximum number of toys he could have made = 99 × 12 + 20 = 1208

484. From a list of four comics, four friends discuss their favourite comics. At
least 2 friends vote for Mandrake, not more than 3 vote for Mammaji, 1 votes
for Alibaba and 2 vote for Tom & Jerry. If two friends have exactly voted for 2
different comics each, and 2 friends for exactly 3 different comics each, then
how many votes did Mandrake get?
A. 2
B. 3
C. 4
D. Can’t say
E. None of these

Answer & Explanation :


Answer: Option C
Explanation :

There are a total of 10 votes, of which exactly 3 go to Alibaba and Tom & Jerry
together.,
The remaining 7 are distributed between Mandrake and Mammaj
Maximum votes that Mandrake can get is 4 (since there are only 4 comics) and
maximum votes that Mammaji can get are 3.

485. From a bag containing 242 balls, one ball weighs 19.9 grams and all the
other weigh 19.5 grams each. Using a simple balance where balls can be kept
on either pan, what is the minimum weighs required to identify the defective
ball?
A. 3
B. 4

C. 5
P a g e 266 | 580
D. 7
E. none of the above

Answer & Explanation :


Answer: Option C

Explanation :
By dividing the total balls in three parts and putting two parts on the simple
balance we can find which of the three parts has ball of different weight. Again
divide this part into further three parts and put two parts on the simple balance.
From here again we can find the part having ball with different weight. Again
this part can be divided into three parts and so on. So 35 = 243 i.e up to 243
balls can be checked within 5 weighings.

DIRECTIONS for the question 4-5: Read the information given below
and answer the question that follows.

Five friends, viz. Shan, Monu, Jai, Karan and Bunty are living in five different
cities named Karanpur, Jaipur, Vizanagar, Barnala and Patiala, not necessarily
in that order.
Their salaries are 7 Lacs, 8 Lacs, 9 Lacs, 11 Lacs, 13 Lacs (INR per annum), in
no particular order. Further, the following information is given about them:
I. Karan, who does not live in Barnala, earns a salary that is a prime number
multiple of 1 Lac.
II. Monu made a call to one of his four mentioned friends who lives in Patiala
and who earns a perfect square multiple of 1 Lac in salary.
III. Jai's salary is 1 Lac more than the average salary of Karan and Shan
IV. Monu lives in the city, which has the shortest name amongst the above
cities.

486. If Karan lives in Vizanagar, then what is the average salary of the persons
living in Barnala and Karanpur?
A. 9

B. 10

P a g e 267 | 580
C. 12
D. Data insufficient

E. None of these
Answer & Explanation :

Answer: Option D
Explanation :
From the given information, we can summarize the data in the following table:
Karan Jai Shan Bunty Monu
Salaries:7 or 13 11 13 or 7 9 8

Cities: K/V B/K/V B/K/V Patiala Jaipur


Where ‘K’, ‘V’ and ‘B’ stands for ‘Karanpur’, ‘Vizanagar’ and ‘Barnala’
respectively.
If Karan, lives in Vizanagar, then Jai and Shan must be staying at Karanpur and
Barnala, not necessarily in that order. Their average salary in any case will be
Rs. 12 lakhs or 9 lakhs. So, the data is insufficient.

487. Who stays in Patiala?


A. Shan
B. Monu

C. Bunty
D. Jai

Answer & Explanation :


Answer: Option C

Explanation :
Monu called a friend, who gets Rs. 9 lakh as his salary is a perfect square
multiple of 100000 and stays in Patiala.

Bunty stays in Patiala.

488. If Monu and Jai live in cities with names starting with consecutive
alphabets, then who lives in Vizanagar?
P a g e 268 | 580
A. Shan
B. Monu

C. Bunty
D. Karan

Answer & Explanation :


Answer: Option D
Explanation :
Monu lives in Jaipur, so Jai must be living at Karanpur.
Since Karan is not staying at Barnala, he must be staying at Vizanagar.

489. A square and a regular hexagon have the same area. Find the ratio of the
perimeter of the square to the perimeter of the hexagon.
A. √3 : 2
B. 1 : 2
C. 4√3 : 4√4
D. 4√4 : 4√3

Answer & Explanation :


Answer: Option D

490. AnswIf x + y = 1, then what is the value of (x^3 + y^3 + 3xy)?


A. 1
B. 2
C. 9

D. 0
E. None of these
Answer & Explanation :

Answer: Option A

P a g e 269 | 580
491. Last Sunday, every customer who visited the CENTRA MALL was given a
gift coupon, on every purchase worth Rs. 1000, with a unique six-digit code
written on it. Each code was such that-
(i). The first digit was non-zero.

(ii). All the six digits were distinct.


(iii). The 1st and the 6th digits added up to 9 and so do the 2 nd and 5 th digits,
and also the 3rd and 4th digits.
A gift was given to a customer who had two coupons with codes such that the
numbers formed using the first three digits of each code were the reverse of
each other.
The number of coupons distributed could not have been more than

A. 504
B. 729
C. 432
D. 648
E. None of The above

Answer & Explanation :


Answer: Option C
The six-digit number on the coupon will look like

xyz9–z9–y9–x
Once we select the first, second and third digits of the number, the remaining
three digits get fixed. The first digit can be chosen out of 1, 2, 3, …9 in 9 ways.
Then, the second digit can be selected in (10 – 2) = 8 ways and the third digit
in (10 – 4) = 6 ways.
Hence, the maximum possible number of such six digit number is 9 × 8 × 6 =
432.

492. A basket contains 3 blue, 5 black and 3 red balls. If two balls are drawn at
random, what is the probability that none of them is blue?
A. 22/55
B. 3/55
C. 28/55

P a g e 270 | 580
D. 9/11
E. None of these

Answer & Explanation :


Answer: Option C

Explanation :
Here n(S) = 11C2
Now two balls can be drawn from 5(black)+3(red) balls = 8 balls in 8C2 ways.
Therefore n(E) = 8C2
Now, P(E) = (8C2/11C2) = (28/55)

Other method:
Required probability
= 1-(27/55) = (28/55)

493. A basket contains 3 blue, 5 black and 3 red balls. If 2 balls are drawn at
random, what is the probability that one is black and one is red?
A. 2/11

B. 8/11
C. 9/11
D. 3/11

E. None of these
Answer & Explanation :
Answer: Option B
Explanation :

Selecting 1 black ball out of 5 - 5C1 ways Selecting on red ball out of 3 - 3C1
ways
he required probability = (5C1 * 3C1) / (11C2) = (5*3) / 55 = 3/11

P a g e 271 | 580
494. Ramesh takes twice as much time as Mahesh and thrice as much time as
Suresh to complete a job. If working together, they can complete the job in 4
days, then the time taken by each of them separately to complete the work is
A. 36, 24 and 16 days

B. 20, 16 and 12 days


C. 24, 12 and 8 days
D. 20, 18 and 15 days
E. None of these
Answer & Explanation :

Answer: Option C
Explanation :
We are given that R = 2M = 3S. This means that Suresh is thrice efficient than
Ramesh and Mahesh is twice efficient than Ramesh. So let us assume that
Ramesh can do 1 unit work per day. so Mahesh will do 2 units per day and
Suresh will do 3 units of the work per day. So the total work done by all three in
one day is 6 units. Now they together take 4 days to complete the work. So the
total work is 6 x 4 = 24 units and individually Ramesh, Mahesh and Suresh will
take 24 days, 12 days and 8 days respectively.

495. A man buys a land and gives for it 20 times the annual rent Find the rate
of interest he gets for his money
A. 10%
B. 24%

C. 45%
D. 18%
E. 5%

Answer & Explanation :


Answer: Option E
Explanation :
let annual rent is 1 Rs. so buys the land at 20 Rs. So by investing Rs.20 he is
getting Rs.1 as interest. so on Rs.100 he gets Rs.5 . so rate%=5%.Hence
option E is the answer.
P a g e 272 | 580
496. What will be the compound interest on a sum of Rs. 7200/- at 5 p.c.p.a. in
2 years?
A. Rs. 841/-

B. Rs. 738/-
C. Rs. 793/-
D. Rs. 812/-
E. Rs.694/-
Answer & Explanation :

Answer: Option B

497. In a rhombus ABCD the diagonals AC and BD intersect at the point (3, 4).
If the point 'A' is (1, 2), the diagonal BD has the equation
A. x – y – 1 = 0
B. x – y + 1 = 0
C. x + y – 1 = 0

D. x + y – 7 = 0
E. x + y – 2 = 0
Answer & Explanation :

Answer: Option D
Explanation :

Slope of AC = (2-4)/(1-3) = 1. As diagonals in a rhombus bisect each other at


90°, so slope of BD = -1. Also diagonal BD has to lie on the point (3, 4), so
equation of the line is y - 4 = - 1(x - 3) => x + y – 7 = 0.

498. A circle has two parallel chords of lengths 6 cm and 8 cm. If the chords are
1 cm apart and the centre is on the same side of the chords, then the diameter
of the circle is of length:
A. 5cm
B. 6cm

P a g e 273 | 580
C. 10cm
D. 12cm

E. None of these
Answer & Explanation :

Answer: Option C
Explanation :

499. An express train travelled at an average speed of 100 kmph, stopping for 3
minutes after every 75 km. A local train travelled at a speed of 50 kmph,
stopping for 1 minute after every 25 km. If the trains began travelling at the
same time, how many kilometres did the local train travel in the time it took the
express train to travel 600 km?
A. 900km
B. 307.5km
C. 1200km
D. 100km

E. 300km
Answer & Explanation :

Answer: Option B
Explanation :
For 600 kms. The Express train needs 6 hrs. for the distance travelled @ 100
kmph. and it will stop for ( 600/75 )= 8. So there must be 7 stoppages in

P a g e 274 | 580
between where it stops for 3 min. each totalling upto 21 min. So total time will
be 6 hrs.21 min.

Now the Local train travelling @ 50 kmph. will cover 300 kms. in 6 hrs. and in
rest 9 min. it will travel 7.5 kms.( 300/25=12 stoppages where it will stop for
12 min. So, 21- 12 = 9 ). The total distance becomes 300 + 7.5 = 307.5 km.
ALTERNATE SOLUTION
We can also think that since speed of express train is double the local speed, so
local train will tavel less than 600 km, Only option b has such option and hence
is the required answer.

500. In how many different ways can the letters of the word 'PARTY' be
arranged?
A. 120
B. 2005
C. 2400
D. 720

E. None of these
Answer & Explanation :
Answer: Option A

Explanation :
Total no. of ways =5 x 4 x3 x 2 x 1 = 120

501. A & B together can complete a piece of work in 16 days, B alone can
complete the same work in 24 days. In how many days can A alone complete
the same work ?
A. 34 days

B. 50 days
C. 48 days
D. 42 days
E. None of these
Answer & Explanation :
P a g e 275 | 580
Answer: Option C
Explanation :

Let A alone can complete the work in x days and B alone can complete the work
in 24days.

Therefore, according to the given conditions,


1/x + 1/24 = 1/16,
1/x = 1/48
So, A's one day work is 1/48 which means that A will complete the entire work
in 48days.

Therefore, the answer is option C.


502. A and B are partners in a business. They invest in the ratio 5:6, at the end
of 8 months A withdraws. If they receive profits in the ratio of 5 : 9, find how
long B's investment was used?
A. 12 months

B. 10 months
C. 15 months

D. 14 months
E. 18 months
Answer & Explanation :

Answer: Option A
Explanation :
Ratio of profit is always distributed in the ratio of their investment and time.
5unit × 8 months:6 units × B’s months = 5:9

So B’s investment time = 12 months

503. There are 3 red balls, 4 blue balls and 5 white balls. 2 balls are chosen
randomly. Find probability that 1 is red and the other is white.
A. 5/22

B. 5/23
C. 7/22
P a g e 276 | 580
D. 4/9
E. None of These

Answer & Explanation :


Answer: Option E

Explanation :
There are 3 red balls, 4 blue balls and 5 white balls. 2 balls are chosen
randomly.
probability that 1 is red and the other is white = 3/12× 5/11 = 5/44

504. According to a new plan rolled out by H1SP Bank, the rate of simple
interest on a sum of money is 8% p.a. for the first two years, 10% p.a. for the
next three years and 6% p.a. for the period beyond the first five years. Simple
interest accrued on a sum for a period of eight years is Rs. 12,800. Find the
sum.
A. Rs. 24,000
B. Rs. 16,000

C. Rs. 15,000
D. Rs. 13,500
E. None of These

Answer & Explanation :


Answer: Option E
Explanation :
Let the sum of money be x.
So interest in first two years = 8×2 = 16% of x
Interest in next three years = 10×3 =30% of x
Interest in next five years = 6×3 = 18% of x

Total interest = 64% of x = Rs 12,800


We get x = Rs 20,000

P a g e 277 | 580
505. Three Science classes A, B and C take a Life Science test. The average
score of students of class A is 83. The average score of students class B is 76.
The average score of class C is 85. The average score of class A and 8 is 79 and
average score of class B and C is 81. Then the average score. Of classes A, B
and C is
A. 80
B. 80.5
C. 81
D. None of these

Answer & Explanation :


Answer: Option D
Explanation :

506. A hemispherical bowl of internal diameter 54 cm contains a liquid. The


liquid is to be filled in cylindrical bottles of radius 3 cm and height 9 cm. How
many bottles are required to empty the bowl?
A. 221
B. 343

C. 81
D. 243

E. None of these
Answer & Explanation :
Answer: Option E

507. Solve : (638 + 9709 - 216) ÷ 26 = ?


P a g e 278 | 580
A. 275
B. 365

C. 420
D. 300

E. 390
Answer & Explanation :
Answer: Option E
Explanation :
(638 + 9709 - 216) ÷ 26 = 10131/26 = 389.65

Approx 290
508. A dishonest dealer professes to sell his goods at the cost price but uses a
weight of 800gm instead of 1kg. Find his real gain percent.
A. 25%
B. 20%
C. 30%
D. None of these

Answer & Explanation :


Answer: Option A
Explanation :

200/800 ×100 = 25%

509. A sum of money lent out at simple interest amounts to Rs. 720 after 2
years and to Rs. 1,020 after a further period of 5 years. The sum and the rate
% are
A. Rs. 500, 5%
B. Rs. 400, 15%

C. Rs. 600, 10%


D. Rs. 700, 20%

Answer & Explanation :

P a g e 279 | 580
Answer: Option C
Explanation :

Amount after 2 years = Rs 720


Amount after 7 years = Rs 1020

Therefore, Interest for 5 years = Rs 300


Interest for 1 year = Rs 60
And Interest for 2 years = Rs 120
SO Principal = 720-120 = Rs 600
Also, 120 = (600*R*2)/100 = R = 10%

Amount after 2 years = Rs 720


Amount after 7 years = Rs 1020
Therefore, Interest for 5 years = Rs 300
Interest for 1 year = Rs 60
And Interest for 2 years = Rs 120
SO Principal = 720-120 = Rs 600
Also, 120 = (600*R*2)/100 = R = 10%

510. A train with 90 km/h crosses a bridge in 36 seconds. Another train 100
metres shorter crosses the same bridge at 45 km/h. What is the time taken by
the second train to cross the bridge ?
A. 61 seconds

B. 63 seconds
C. 62 seconds

D. 64 seconds
Answer & Explanation :
Answer: Option D

Explanation :
Train A, Speed = 90kmph
=90*(5/18)m/s = 25m/s = 25m/s, t=36s
P a g e 280 | 580
Let length, L = x+y = time*speed = 25*36 = 900m
=800m, Speed= 45*(5/18) = (25/2) m/s

t= (Distance/Speed) = (800/(25/2)) = (1600/25) = 64 seconds

511. Ramesh travels 760 km to his home, partly by train and partly by car He
takes 8 hours, if he travels 160 km by train and the rest by car. He takes 12
minutes more, if he travels 240 km by train and the rest by car. What are the
speeds of the train and of the car?
A. Speed of car = 90 km/h, speed of train = 60 km/h

B. Speed of car = 100 km/h, speed of train = 80 km/h


C. Speed of car = 80 km/h, speed of train = 70 km/h
D. Speed of car = 100 km/h, speed of train = 90 km/h
Answer & Explanation :
Answer: Option B

Explanation :
Let speeds be x and y for train and car respectively.

Then 8 = (160/8) + (600/y) .....(1)


And 8(1/5) = (240/x) + ((760-240)/y) .....(2)
Solving for x and y, we get 100 and 80 km/hr.

You can also use the option straightway for such Qs.

512. Some students planned a picnic. The budget for food was Rs. 500. But, 5
of them failed to go and thus the cost of food for each member increased by Rs.
5. How many students attended the picnic?
A. 15
B. 25

C. 20
D. 30

Answer & Explanation :


Answer: Option C
P a g e 281 | 580
Explanation :
By direction options,500/25=20 ,500/20=25

By mathematical method, the main steps are: xy = 500 …(1) and (x−5) (y+5)
= 500 …(2),

From eqn. 2, x−y = 5 or y = x−5 Put in eqn 1, x(x−5) = 500 or x2-5x-500=0 ,


i.e. x = 25 and attended ones = x − 5 = 20

513. After being set up, a company manufactured 6000 scooters in the third
year and 7000 scooters in the seventh year. Assuming that the production
increases uniformly by a fixed number every year, what is the production in the
tenth year?
A. 7850
B. 7650
C. 7750

D. 7950
Answer & Explanation :

Answer: Option C
Explanation :
You can use A.P.,Tn =a+(n-1)d ,6000=a+2d.....(1) and 7000 = a + 6d .....(2)

Eqn (2) – Eqn (1) ⇒ 1000=4d,


i.e. d = 250 and a = 6000 − 500 = 5500
T10 =5500 + 9 × 250 =7750

514. The average score of boys in an examination in a school is 71 and that of


the girls is 73. The average score of the school is 71.8. The ratio of the number
of boys to that of the girls that appeared in the examination is
A. 1 : 2
B. 3 : 2
C. 2 : 2

D. 4 : 2

P a g e 282 | 580
Answer & Explanation :
Answer: Option D

Explanation :
71.8 = (71x+73y)/(x+y)

71.8 (x+ y) = 71x + 73y


0.8x = 1.2y
x:y = 12:8 which is equals to 3:2

515. The mean monthly salary paid to 75 workers in a factory is Rs. 5,680. The
mean salary of 25 of them is Rs. 5,400 and that of 30 others is Rs. 5,700. The
mean salary of the remaining workers is
A. Rs. 5,000
B. Rs. 7,000
C. Rs. 6,000
D. Rs. 8,000
Answer & Explanation :

Answer: Option C
Explanation :
5680*75 = (5400*25+5700*30+x(75-25-30))/75

4,26,00 = 1,35,000 +1, 71,000 + 20x


X = 1,20,000/20, = 6,000

516. A sum of Rs. 25 was paid for a work which A can do in 32 days, B in 20
days, B and C in 12 days and D in 24 days. How much did C receive if all the
four work together?
A. Rs. 14/3

B. Rs. 16/3
C. Rs. 15/3

D. Rs. 17/3

P a g e 283 | 580
Answer & Explanation :
Answer: Option B

Explanation :
B+ C's 1 day's work = ½ and B's 1 day's work = 1/20

Therefore, C's 1 day's work = (1/12) – (1/20) = 4/120 = 1/30


Monet will be distributed according to the ratio of work done i.e A: B: C: D
= 1/32 : 1/20 : 1/30 : 1/24 = 15 :24:16:20
Therefore, C's Share = 16/(15+24+16+20) = Rs 16/3

517. A man sold two steel chairs for Rs. 500 each. On one, he gains 20% and
on other, he loses 12%. How much does he gain or lose in the whole
transaction?
A. 1.5% gain
B. 2% gain
C. 1.5% loss
D. 2% loss

Answer & Explanation :


Answer: Option A
Explanation :

CP/SP = 100/(100±x) , i.e. Total CP = 417 (500*100/200) +


568(500*100/88)≅ 985

Since CP<SP .Therefore, Profit = 100-985 = 15


P% ≅ 15/985 X 100 ≅ 1.5 %

DIRECTIONS for the questions 1 to 10: Solve the following question and mark
the best possible option

518. There are two motor cycles (A and B) of equal cost price. Motorcycle A was
sold at a profit of 14% and motorcycle B was sold for Rs. 4,290/- more than its
cost price. The net profit earned after selling both the motor cycles (A and B) is
20%. What is the cost price of each motorcycle?

P a g e 284 | 580
A. Rs. 16,500/-
B. Rs. 16,000/-

C. Rs. 15,500/-
D. Rs. 71,500/-

E. Rs. 17,000/-
Answer & Explanation :
Answer: Option D
Explanation :
Let the cost price of each motorcycle be Rs. ‘A’. So SP of A = 1.14A and SP of B
= A + 4290. Total CP = 2A. As net profit is given to be 20% on both the
motorcycles, so we can form the equation as (2.14A + 4290 - 2A)/2A = 20%.
Solving it further, we get (0.14A + 4290)5 = 2A. Solving this equation, we get
value of A as 16,500. Hence answer is option D.

519. A bag contains 3 white balls and 2 black balls. Another bag contains 2
white and 4 black balls. A bag and a ball are picked at random. What is the
probability that the ball drawn is white?
A. 7/11
B. 7/30

C. 5/11
D. 7/15
E. None of these
Answer & Explanation :
Answer: Option D
Explanation :
(3/5+2/6)×½, Solving, 7/15

520. A is thrice as efficient as B and takes 10 days less to do a piece of work


than B takes to do the same work. In how many days, B alone can finish the
whole work?
A. 15 days
P a g e 285 | 580
B. 10 days
C. 9 days

D. 8 days
E. 7 days

Answer & Explanation :


Answer: Option A
Explanation :
Efficiency A:B = 3:1
Therefore, no. of days A:B = 1:3

Given, 3x – x = 10
Solving, A = 5 days and B= 15 days.

521. The compound interest on a certain sum for 2 years at 10% per annum is
Rs. 525. The simple interest on the same sum for double the time at half the
rate percent per annum is ________
A. Rs. 400

B. Rs. 500
C. Rs. 600
D. Rs. 800

E. None of these
Answer & Explanation :

Answer: Option B
Explanation :

Let ‘P’ be the principle


P(1.1)2 – P = 525
P = 2500

S.I = (2500*4*5)/100 = 500

P a g e 286 | 580
522. There is a natural number which becomes equal to the square of a natural
number when 100 is added to it, and to the square of another natural number
when 168 is added to it. Find the number.
A. 189

B. 69
C. 156
D. 224
E. 255
Answer & Explanation :

Answer: Option B
Explanation :
Try by options. 3rd option is correct because 100 + 156 = 256 (square of 16)
and 168 + 156 = 324 (square of 18).
No other option is satisfying the 2nd condition.

523. In a rectangular auditorium, chairs are arranged in rows and columns. The
number of chairs in each column is more than the number of chairs in each row
by 5. If there are in all 300 chairs, find the number of chairs in each row and in
each column.

A. 25, 20
B. 30, 10
C. 23, 18
D. 20,15
E. None of these
Answer & Explanation :
Answer: Option D

Explanation :
Go by options. When there were 300 chairs, so product of chairs in rows and
chairs in columns should be 300. Difference of 5 should be there between the
seats in rows and columns. So no option out of 1st 3 is satisfying the conditions.
So answer is 4th option and correct values are 20 and 15.

P a g e 287 | 580
524. A started a business with an investment of Rs. 16,000. After 6 months
from the start of the business, B and C joined with Rs. 12,000 and Rs. 18,000
respectively and A invested an additional amount of Rs. 4000. If the difference
between A’s share and B’s share in the annual profit is Rs. 6000, what was the
annual profit received?
A. Rs. 17,600
B. Rs. 13,200
C. Rs. 14,300

D. Rs. 16,500
E. Rs. 11,000
Answer & Explanation :
Answer: Option D
Explanation :

Amount invested by A= 16000 for first 6 months, and 20000 for next 6 months
Amount invested by B= 12000 for 6 months

Amount invested by C= 18000 for 6 months


Now by compound partnership, IA:IB:IC = PA : PB : PC
16000*6 + 20000*6 : 12000*6 : 18000*6 = 6:2:3

Given 6x – 2x= 4x = 6000; x=1500


so total profit = 11x = 16500.

525. Shiva gives 20% of her monthly salary to his mother, 50% of the
remaining salary he invests in an insurance scheme and PPF in the respective
ratio of 5 : 3 and the remaining he keeps in his bank account. If the sum of the
amount he gives to his mother and that he invests in PPF is Rs. 12,600, how
much is Shiva’s monthly salary?
A. Rs. 36,000
B. Rs. 64,000
C. Rs. 42,000
D. Rs. 40,000
P a g e 288 | 580
E. None of these
Answer & Explanation :

Answer: Option A
Explanation :

Let the total amount be x.


0.2x = given to mother
0.25x= invested in insurance
0.15x= invested in ppf
0.4x= Bank account

Given, 0.2x+0.15x = 0.35x = 12600; x=36000

526. The respective ratio of radii of two right circular cylinders (A and B) is 4 :
7. The respective ratio of the heights of cylinders A and B is 2 : 1. What is the
respective ratio of volumes of cylinders A and B?
A. 25 :42
B. 23 : 42

C. 32 : 49
D. 30 : 49
E. 36 : 49

Answer & Explanation :


Answer: Option C

527. C is 20% more efficient than A. A and B together can finish a piece of work
in 16 days. B and C together can do it in 15 days. In how many days A alone
can finish the same piece of work?
A. 42

B. 48
C. 54

D. 36

P a g e 289 | 580
E. 45
Answer & Explanation :

Answer: Option C
Explanation :

Let total of 240 units of work to be done.


Now as given A+ B can do 240/16= 15 units/day
Also B + C can do 240/15= 16 units/day
And given, C= 1.2 A., Substituting and solving the equations we get, A= 5
units/day, ie.e A will complete work in 240/5= 48 days.

528. The odds against a certain event are 5 : 2 and the odds in favour of
another independent event are 6 : 5. The probability that at least one of the
events will happen is:
A. 12/77
B. 25/77

C. 52/77
D. 65/77

Answer & Explanation :


Answer: Option C
Explanation :

Required probability = 1 – P (NO event will happen) = 1 – (5/7 × 5/11) =


52/77.

529. Number of times the hands of a clock are in a straight line every day is:

A. 44
B. 24
C. 42

D. 22
Answer & Explanation :

Answer: Option A

P a g e 290 | 580
Explanation :
In 12 hour, the hands of clock coincide 11 times and they make 180° also 11
times, thus total of 22 times. So in 24 hours, the hands of clock will be forming
a straight line 22 + 22 = 44 times.

530. A motor boat can travel at 10 km/h in still water. It traveled 91 km


downstream in river and then returned, taking altogether 20 hours. Find the
rate of flow of river.
A. 3 kmph

B. 5 kmph
C. 6 kmph
D. 7 kmph
Answer & Explanation :
Answer: Option A

Explanation :
Let the rate of flow be y kmph. The equation that will be formed is 91/(10+y) +
91/(10-y) = 20
Solving we get y = 3.

531. A person invests Rs. 5508 in '4% stock at 102'. He afterwards sells out at
105 and reinvests in '5% stock at 126'. What is the change in his income?
A. Rs. 20
B. Rs. 7
C. Rs. 10
D. Rs. 9
Answer & Explanation :

Answer: Option D
Explanation :
Old income = 5508/102 × 4 = Rs. 216. No. of shares purchased = 5508/102 =
54. So sale proceeds = (54 × 105) = Rs. 5670.

P a g e 291 | 580
Therefore, New income = (5670 × 5)/126 = Rs. 225.
So change in income = 225 – 216 = Rs. 9.

532. The ratio between the number of sides of two regular polygons is 1 : 2 and
the ratio between the measure of their interior angles is 2 : 3. The number of
sides of these polygons are respectively:
A. 4, 8
B. 5, 10
C. 6, 12

D. 8, 16
Answer & Explanation :
Answer: Option A
Explanation :
Going by options, if the no of sides are 4 and 8, then the interior angles of the
polygons is 90° and 135° (int angles = 180°(N-2)/N)
Which are in the ratio 2 : 3. So the answer is 1st option.

533. How many two-digit odd numbers can be formed from the digits 1, 2, 3, 4,
5, if repetition of digit is allowed?

A. 5
B. 15
C. 35
D. 25

Answer & Explanation :


Answer: Option B
Explanation :

P a g e 292 | 580
Here unit place can be filled in 3 ways (i.e. 1, 3, 5)

Ten's place can be filled in 5 ways. Required number of numbers = 5×3 = 15

534. Three equal glasses are filled with mixtures of sprit and water. The ratio of
the sprit to water is as follows: in the first glass as 3 : 4, in the second glass as
4 : 5 and in the third glass as 5 : 6.The contents of the three glasses are
emptied into a single vessel. What is the ratio of the sprit to water in the
mixture now?
A. 920 : 1159
B. 820 : 1149
C. 1120 : 1134
D. 1010 : 1122
Answer & Explanation :

Answer: Option A
Explanation :
Required Ratio:
3/7 + 4/9 + 5/11 / 4/7 + 5/9 + 6/11 = 920 / 1159.
Hence, answer is A option.

535. Two pipes can fill a cistern in 14 and 16 hours respectively. The pipes are
opened simultaneously and it is found that due to leakage in the bottom, 32
minutes extra are taken for the cistern to be filled up. If the cistern is full, in
what time would the leak empty it?
A. 96 hours
B. 102 hours

P a g e 293 | 580
C. 106 hours
D. 112 hours

Answer & Explanation :


Answer: Option D

Explanation :
If both filling pipes are working together, time taken to fill the tank will be 14
X16/(14 + 16) = 112/15 hrs. But due to leak it takes 32 min more. Hence total
time taken = 112/15 + 32/60 = 8 hrs. Let the leak takes x hours to empty the
tank, so, 1/14 + 1/16 - 1/x = 1/8.

By solving we get the answer as 112 hours. So answer is 4th option.


536. If √1+x/961=32/31, then the value of x is
A. 63
B. 61
C. 65

D. 64
Answer & Explanation :

Answer: Option A

537. a and b are odd numbers, then which of the following is even?

A. a + b + ab
B. a + b-1

C. a + b + 1
D. a + b + 2ab

Answer & Explanation :


Answer: Option D
Explanation :

⇒ The sum of two odd number is even. The same is the case with their product.

∴ a + b + 2ab = Even number

P a g e 294 | 580
538. The single discount equal to three consecutive discounts of 10%, 12% and
5% is

A. 26.27%
B. 24.76%

C. 9%
D. 11%
Answer & Explanation :
Answer: Option B
Explanation :

Single equivalent discount for 10% and 12%.


⇒ (12 -10 -12*10 /100)% = 20.8%
Single equivalent discount for 20.8% and 5%.
⇒(20.8 -5 -20.8*5/100)% = 20.8%

= 24.76%

539. An alloy contains copper, zinc and nickel in the ratio of 5 : 3 : 2. The
quantity of nickel in kg that must be added to 100 kg of this alloy to have the
new ratio 5 : 3 : 3 is

A. 8
B. 10
C. 12
D. 15
Answer & Explanation :
Answer: Option B
Explanation :
Let x kg of nickel be mixed.
20+x/100+x= 3/11

⇒ 220 + 11x = 300 + 3x


⇒ 11x - 3x = 300-220

P a g e 295 | 580
⇒ 8x = 80 ⇒ x = 10 kg.

540. The ratio of the ages of Ram and Rahim 10 years ago was 1 : 3. The ratio
of their ages five years hence will be 2 : 3. Then the ratio of their present ages
is
A. 1:2
B. 3:5

C. 3:4
D. 2:5
Answer & Explanation :
Answer: Option B
Explanation :
Let the ages of Ram and Rahim 10 years ago be x and 3x years respectively.
After 5 years from now,
x+15/3x+15=2/3

⇒ 6x + 30 = 3x + 45
⇒ 3x = 45-30 = 15
⇒x=5

∴ Radio of their present ages


= (x+ 10) : (3x + 10)
= 15 : 25 = 3 : 5

541. The ratio between two numbers is 2 : 3. If each number is increased by 4,


the ratio between them becomes 5 : 7. The difference between the numbers is
A. 8
B. 6
C. 4

D. 2
Answer & Explanation :

P a g e 296 | 580
Answer: Option A
Explanation :

Let the numbers be 2x and 3x.


∴ (2x+4)/(3x+4) = 5/7

⇒ x = 28 – 20 = 8 = Required difference

542. Monthly incomes of A and B are in the ratio of 4 : 3 and their expenses
bear the ratio 3:2. Each of them saves Rs. 6,000 at the end of the month, then
the monthly income of A is
A. Rs. 12,000
B. Rs. 24,000
C. Rs. 30,000
D. Rs. 60,000

Answer & Explanation :


Answer: Option B
Explanation :
Let the monthly incomes of A and B be Rs. 4x and Rs. 3x respectively and their
expenditures be Rs. 3y and Rs. 2y respectively.

∴ 4x – 3y = 6000 and 3x – 2y = 6000


⇒ 4x – 3y = 3x – 2y ⇒ x = y ∴ 4x – 3y = 6000

⇒ x = 6000
⇒ A’s monthly income = 4x = Rs. 24000

543. The average of three consecutive odd numbers is 12 more than one third
of the first of these numbers. What is the last of the three numbers?

A. 15
B. 17
C. 19

D. Data inadequate

P a g e 297 | 580
Answer & Explanation :
Answer: Option C

Explanation :
If the smallest number be x, then

X/3+12=x+2=x+36=3x+6
⇒ 3x – x = 36 – 6 ⇒ 2x = 30 ⇒ x = 15
∴ Third number = 15 + 4 = 19

544. What will be the compound interest on Rs. 18,600/- for 2 years, the rate of
interest for first year being 8% and for the second year being 15%?
A. Rs. 4489.90
B. Rs. 4967.20
C. Rs. 4232.80
D. Rs. 4501.20

E. Rs. 3637.10
Answer & Explanation :
Answer: Option D
Explanation :
= 18600(1+8/100)(1+15/100)

⇒18600 (1.08) (1.15) = 23101.2


⇒CI = A – P = 23101.2 – 18600 = 4501.2
Hence, Option 4.

545. The sum of five numbers is 260. The average of the first two numbers is
30 and the average of the last two numbers is 70. What is the third numbers?

A. 33
B. 75
C. 60
D. Cannot be determined
E. None of these
P a g e 298 | 580
Answer & Explanation :
Answer: Option C

Explanation :
a+b+c+d+e=260

⇒a+b /2 =30
⇒ a+b=60

⇒ d+e /2 =70

⇒ d + e =140

⇒ 60+c+140=260

⇒ c = 60
Hence , Option 3.

546. A certain number of capsules were purchased for Rs.176/-. 6 more


capsules could have been purchased in the same amount if each capsule was
cheaper by Rs. 3/-.What was the number of capsules purchased?
A. 13
B. 16
C. 17

D. 8
E. 11
Answer & Explanation :

Answer: Option B
Explanation :

Let ‘n’ be the number of capsules purchased and ‘c’ be the cost of each capsule.
cn = 176

(n+ 6) (c – 3) = 176
Now check option
When n = 16, c = 11

Hence, Option 2.

P a g e 299 | 580
547. Ram was asked to find 7/8th of a fraction but made the error dividing the
fraction by 7/8. As a result of this, he was off the correct answer by 75/784.
What answer was Ram supposed to arrive at?

A. 13/32
B. 9/144
C. 5/16
D. 5/14
E. 9/16

Answer & Explanation :


Answer: Option C
Explanation :
ATQ (8/7)y - (7/8)y = 75/784 where y is the req. fraction.
y= 5/14.
Ram is supposed to arrive at (7/8)y = (7/8)*(5/14) = 5/16.
Hence, Option 3.

548. 36 workers can finish a piece of work in 14 days. If the work is to be


completed in 8 days, how many extra workers are required

A. 29
B. 33

C. 23
D. 31

E. 27
Answer & Explanation :
Answer: Option E

Explanation :
Let ‘x’ be the extra workers required

work workers Days

P a g e 300 | 580
w 36 14
w 36+x 8

⇒ 36*14 = (36 + x).8

⇒ on solving x = 27

Hence, Option 5.

549. A man can row 13 kmph downstream and 9 kmph upstream. What is the
speed of the man in still water? (in kmph)

A. 12
B. 10.5
C. 11
D. 10
E. 11.5

Answer & Explanation :


Answer: Option C
Explanation :
Speed of stream = s
Speed of man in still water = b

ATQ
b + s = 13
b–s=9
⇒ b = 11, s = 2

Hence, Option 3.

550. A is 60% more efficient than B. In how many days will ‘A’ and ‘B’ working
together complete a piece of work which ‘A’ alone takes 15 days to finish?

A. 124/13
B. 118/3
C. 56/3
P a g e 301 | 580
D. 131/3
E. 120/13

Answer & Explanation :


Answer: Option E

Explanation :
As A is 60% more efficient than B
⇒ We can suppose that if B does 5 units/days then A does 8 units/day.

Work done by A in 15 days = 120 units.


A and B working together do 13 units in a day.
⇒ A and B can do 120 units of work in

120/13 Days.
Hence, Option 5

551. The sum of the digits of a two-digit number is 12 and when the digits of
the two-digit number are interchanged, the new number is 36 more than the
original number. What is the original two digit number?
A. Cannot be determined
B. 93

C. 48
D. 39
E. 84
Answer & Explanation :
Answer: Option C
Explanation :
Let ‘ab’ be the Required Number
a+b=12….(i)
ATQ ba=ab+36

10b+a=10a+b+36
9(b-a)=36
P a g e 302 | 580
b-a=4….(ii)
From eqn. (i) and (ii)

b=8, a=4
⇒ Required Number = 48, Hence, Option (3).

552. A car covers the first 39 kms of its Journey in 40 minutes & covers the
remaining 25 kms in 35 minutes. What is the average speed of the car?

A. 40 km/hr
B. 164 km/hr
C. 49 km/hr
D. 48 km/hr
E. None of these
Answer & Explanation :
Answer: Option E
Explanation :

Average Speed = Total Distance Traveled / Total Time Taken


= (64*60)/75 = 51.2 km/hr.
Hence,Option 5.

553. A, B and C entered into a partnership by investing Rs. 64,000/-, Rs.


52,000/- and Rs. 36,000/- respectively. All of them invested for equal period of
time. If A got Rs. 35,584/- as his share of annual profit, what amount did C get
as his share of annual profit?
A. Rs. 20,632/-

B. Rs. 18,296/-
C. Rs. 21,084/-
D. Rs. 19,768/-
E. Rs. 20,016/-
Answer & Explanation :

P a g e 303 | 580
Answer: Option E
Explanation :

As time is same therefore

Profit of C= (36000/64000)*35584 =Rs.20,016/-.


Hence, Option 5.
554. The number missing in the series 2, 5, 10, 17, ?, 37, 50, 65 is?
A. 27
B. 22

C. 25
D. 26
Answer & Explanation :

Answer: Option D
Explanation :
The logic is that consecutive odd numbers starting from 3 are being added to
each number.
2, 2 + 3, 5 + 5, 10 + 7, 17 + 9 ( = 26 ), 26 + 11, ...and so on.

555. Which one of the following when divided by 19 gives the quotient 19 and
the remainder 9?
A. 370

B. 331
C. 281
D. 368

Answer & Explanation :


Answer: Option A

Explanation :
370 = 19 × 19 + 9.

P a g e 304 | 580
556. Ten men finish a piece of work in 10 days. How many men needed to finish
the work in 1 day?
A. 10

B. 25
C. 50
D. 100
Answer & Explanation :
Answer: Option D

Explanation :
10 Men can do it in 10 days
=> 100 Men can do it in 1 days.

557. Square root of 625 is


A. 12
B. 15

C. 25
D. 35
Answer & Explanation :

Answer: Option C
Explanation :
√625 = 25.

558. A watch hand reads 4:30. If the minute hand points East, in what direction
will the hour hand point?
A. North

B. North-West
C. North-East
D. South-East
P a g e 305 | 580
Answer & Explanation :
Answer: Option C

Explanation :
It is North East.

559. Anuj gets 3 marks for each correctly done question but loses 2 marks for
each wrongly done question. He attempts 30 questions and gets 40 marks. The
number of correctly attempted questions is?
A. 20

B. 25
C. 15
D. 21
Answer & Explanation :
Answer: Option A
Explanation :
For every wrong answer Anuj loses 5 Marks

(90 – x) × 5 = 40 => x = 10
10 wrong and 20 correct.

560. The price of the sugar having risen by 60%, by how much per cent must a
householder reduce his consumption of sugar so as not to increase his
expenditure?
A. 60%

B. 40%
C. 20%
D. 75/2%

Answer & Explanation :


Answer: Option D

Explanation :

P a g e 306 | 580
561. If Tuesday falls on the fourth of the month, what day will dawn three days
after the 24th?
A. Tuesday
B. Friday
C. Thursday

D. Monday
Answer & Explanation :
Answer: Option D
Explanation :
04th, 11th, 18th, 25th – Tuesday
=> 24th is Monday.

562. A clock is started at noon. By 10 minutes past 5, the hour hand has turned
through:
A. 145 degree

B. 150 degree
C. 155 degree
D. 160 degree
Answer & Explanation :

P a g e 307 | 580
Answer: Option C

563. Hour hand moves 30 degree in 1 hr


=> In 51⁄6How many bricks, each measuring 25 cm x 11.25 cm x 6 cm, will be
needed to build a wall of 8 m x 6 m x 22.5 cm?
A. 5600
B. 6400
C. 7000
D. 7200 it moves = 31/6 * 30 = 155 degree.

Answer & Explanation :


Answer: Option B
Explanation :
Required No. = (800 * 600 * 2.5)/(25 * 11.25 * 6) = 6400.
564. A number is such that when it is multiplied by ‘8’, it gives another number
which is as much more than 153 as the original number itself is less than 153.
What is 25% of the original number?

A. 8
B. 7.5
C. 10

D. 8.5
E. 6.5

Answer & Explanation :


Answer: Option D

Explanation :
Let the no. be x
Given 8x-153=153-x

Hence x=34
25% of x=8.5

P a g e 308 | 580
565. A and B can complete a piece of work in 80 days and 120 days
respectively. They started working together but A left after 20 days. After
another 12 days C joined B and they completed the work in 28 more days. In
how many days can C alone complete the work?

A. 110 days
B. 112 days
C. 114 days
D. 120 days
E. None of these

Answer & Explanation :


Answer: Option B
Explanation :

So efficiency of A and B are 3 units and 2 units respectively.


As they worked for 20 days together after that A left so total unit contribution in
20 days= (3+2) units * 20 days = 100 units
Remaining units = 240 – 100 = 140 units
After another 12 days C joined B and they completed the work in 28
more days, so total units contribution of B in 40 days = 2*40 = 80 units
Remaining units i:e = 140-80 = 60 units

Now 60 units is done by C in 28 days


So to do 240 units C require = 28/60 *240 = 112 days

566. The HCF and LCM of two numbers are 12 and 924 respectively. Then the
number of such pairs is
A. 0
B. 1

P a g e 309 | 580
C. 2
D. 3

E. 4
Answer & Explanation :

Answer: Option A
Explanation :
Let the numbers be 12x and 12y where x and y are prime to each other.
∴ LCM = 12xy

∴ 12xy = 924
=> xy = 77
∴ Possible pairs = (1,77) and (7,11)

567. What is the least number which, when divided by 5, 6, 7, 8 gives the
remainder 3 but is divisible by 9?
A. 1463
B. 1573
C. 1683
D. 1793

E. None of these
Answer & Explanation :
Answer: Option C
Explanation :
LCM of 5, 6, 7, 8 = 35 × 24 = 840
∴ Required number = 840 k + 3 which is exactly divisible by 9.

For k = 2, it is divisible by 9.
∴ Required number = 840k + 3
= 840 × 2 + 3 = 1683

P a g e 310 | 580
568. By walking at of his usual speed, a man reaches his office 20 minutes later
than his usual time. The usual time taken by him to reach his office is:

A. 75 minutes
B. 60 minutes

C. 40 minutes
D. 30 minutes
E. 20 minutes
Answer & Explanation :
Answer: Option B

Explanation :
4/3 of usual time = Usual time + 20 minutes
1/3 of usual time
= 20 minutes
Usual time = 20 × 3 = 60 minutes

569.
A. 1
B. 0.1

C. 0.01
D. 10

E. 0.001
Answer & Explanation :
Answer: Option A
Explanation :
4.41 x 0.16 / 2.1 x 1.6 x 0.21 =441 x 16 / 21 x 16 x 21 =1

P a g e 311 | 580
570. By selling a bicycle for Rs. 2,850, a shopkeeper gains 14%. If the profit is
reduced to 8%, then the selling price will be:

A. Rs. 2,600
B. Rs. 2,700

C. Rs. 2,800
D. Rs. 3,000
E. Rs.3,500
Answer & Explanation :
Answer: Option B

Explanation :
CP of bicycle = 100/114 x 2850 =Rs 2500
SP for a profit of 8% = 108/100 x 2500 = 2700

571. By selling an article, a man makes a profit of 25% of its selling price. His
profit per cent is:
A. 20

B. 25
C. 16
D. 15

E. 33 1/3
Answer & Explanation :

Answer: Option E
Explanation :

If the SP of article be Rs. x then its CP is x - x/4 =Rs. 3x/4


∴ Gain % = (x/4) / (3x/4) x 100 = 100/3 = 33 1/3 %

572. Monthly incomes of A and B are in the ratio of 4 : 3 and their expenses
bear the ratio 3:2. Each of them saves Rs. 6,000 at the end of the month, then
the monthly income of A is:

P a g e 312 | 580
A. Rs. 12,000
B. Rs. 24,000

C. Rs. 30,000
D. Rs. 60.000

E. Rs. 35,000
Answer & Explanation :
Answer: Option B
Explanation :
Let the monthly incomes of A and B be Rs. 4x and Rs. 3x respectively and their
expenditures be Rs. 3y and Rs. 2y respectively.
4x – 3y = 6000 and 3x – 2y = 6000
⇒ 4x – 3y = 3x – 2y ⇒ x = y = 4x – 3y = 6000
⇒ x = 6000

⇒ A’s monthly income = 4x = Rs. 24000

573. The average of three consecutive odd numbers is 12 more than one third
of the first of these numbers. What is the last of the three numbers?
A. 15

B. 17
C. 19
D. 21
E. Data inadequate
Answer & Explanation :

Answer: Option C
Explanation :
If the smallest number be x, then

x/3 + 12 = x + 2 ⇒ x + 36 = 3x + 6

⇒ 3x – x = 36 – 6 ⇒ 2x = 30 ⇒ x = 15
Third number = 15 + 4 = 19
P a g e 313 | 580
574. The total cost of 8 buckets and 5 mugs is Rs. 92 and the total cost of 5
buckets and 8 mugs is Rs. 77. Find the cost of 2 mugs and 3 buckets. A.

A. Rs. 35
B. Rs. 70

C. Rs. 30
D. Rs. 38
Answer & Explanation :
Answer: Option A
Explanation :

CP of 1 bucket = Rs. X
CP of 1 mug = Rs. Y
∴ 8x + 5y = 92………….. (i)
5x + 8y = 77 …………….(ii)
By equation (i) × 5 – equation (ii) × 8.
40x + 25y – 40x – 64y

= 460 – 616 ⇒ − 39y = - 156⇒ y = 4


From equation (i),
8x + 20 = 92 ⇒8x = 92 – 20 = 72 ⇒ x = 9

∴ CP of 2 mugs and 3 buckets


= 2 × 4 + 3 × 9 = 8 + 27 = Rs. 35

575. If a/(1-a) + b/(1-b) + c/(1-c) = 1 then the value of 1/(1-a) + 1/(1-b) +


1/(1-c) is:
A. 1

B. 3
C. 4
D. 0
Answer & Explanation :
Answer: Option C
P a g e 314 | 580
Explanation :

576. If 4x/3 + 2P = 12 for what value of P, x = 6?

A. 6
B. 4
C.
D. 1
Answer & Explanation :

Answer: Option C
Explanation :
When x = 6, (4 * 6)/3 + 2P = 12

⇒ 8 + 2P = 12
⇒ 2P = 12 – 8 = 4

⇒P=2

577. The value of (4+3√3)/(7+4√3) is:


A. 5√3 - 8
B. 5√3 + 8

C. 8√3 + 5

P a g e 315 | 580
D. 8√3 – 5
Answer & Explanation :

Answer: Option D
Explanation :

Expression = (4+3√3)/(7+4√3)
Rationalizing the denominator

578. A & B started a business together by investing Rs. 36000/- and Rs.
42000/- respectively. Both of them invested for one year whereas after 6
months from the start C joined them by investing a certain amount. If they
earned an annual profit of Rs. 39,200/- out of which C’s share is Rs. 7350/-.
What is the investment of C?
A. Rs. 24000/-
B. Rs. 36000/-
C. Rs. 27000/-

D. Rs. 33000/-
E. Rs. 30000/-
Answer & Explanation :
Answer: Option B
Explanation :
Let the investment of C=Rs x
Ratio of profits of A &B &C

=36000×12 : 42000×12 : x×6


=72000:84000: x

P a g e 316 | 580
Solving, x = 36000. B option.

579. What are the marks obtained by Harish in Maths ?


I. Average marks obtained by Harish in Maths and economies are 83.

II. Average marks obtained by Harish in Maths and statistics are 92.
A. The data in statement I alone are sufficient to answer the question, while the
data in statement II alone not sufficient to answer the question.
B. The data in both the statements I & II together are necessary to answer the
question.
C. The data either in statement I alone or in statement II are sufficient to
answer the question.

D. The data in both the statements I & II together are insufficient to answer the
question.
E. The data in statement II alone are sufficient to answer the question, while
the data in statement I alone are not sufficient to answer the question.
Answer & Explanation :
Answer: Option D
Explanation :
We cannot find the marks in maths alone even by using both the statements
together as average marks will not tell us exact marks in Maths.
Data is insufficient. Hence D option.

580. By what percent is Rajesh’s salary more/less than Mangesh’s ?


I. Salary of Rajesh is Rs. 12000/-.
II. Salary of Rajesh and Mangesh together is Rs 28000/-.
A. The data in statement I alone are sufficient to answer the question, while the
data in statement II alone not sufficient to answer the question.
B. The data in both the statements I & II together are necessary to answer the
question.
P a g e 317 | 580
C. The data either in statement I alone or in statement II are sufficient to
answer the question.

D. The data in both the statements I & II together are insufficient to answer the
question.

E. The data in statement II alone are sufficient to answer the question, while
the data in statement I alone are not sufficient to answer the question.
Answer & Explanation :
Answer: Option B
Explanation :

By using both the statements together we can find the salary of Mangesh .
Hence we can find the percentage change between the salaries of Rajesh and
Mangesh. B option.

581. What is the strength of MBA institute where students study only HR,
Marketing and Finance ?
I. Number of students studying HR, Marketing and Financial are in the ratio of 2
: 3: 5 respectively.
II. Number of students studying Marketing is more than those studying HR by
800. Answer: Choice No. C.
A. The data in statement I alone are sufficient to answer the question, while the
data in statement II alone are not sufficient to answer the question.
B. The data either in statement I alone or in statement II alone are sufficient to
answer the question.
C. The data in both the statements I & II together are necessary to answer the
question.
D. The data in statement II alone are sufficient to answer the question, while
the data in statement I alone not sufficient to answer the question.
E. The data in both the statements I & II together are insufficient to answer the
question.
Answer & Explanation :
Answer: Option C

Explanation :

P a g e 318 | 580
Using both statements together , Let no. of students of HR, marketing and
Finance be 2x,3x,5x and also 3x-2x=800

x=800
So total students =2x+3x+5x=10×800=8000. Hence C.

582. A shopkeeper purchased 96 identical shirts @ Rs. 220/- each. He spent Rs.
3800/- on transport and packing and fixed a marked price of Rs. 450/- each
shirt. However, he decided to give discount of 20% on the marked price of each
shirt. If he could sell all the shirts, what is the approximate percent profit
earned by him?
A. 44

B. 39
C. 33
D. 31
E. 30
Answer & Explanation :

Answer: Option B
Explanation :
Cost of 96 shirts=Rs96×220=Rs.21120

Cost of transportation and packaging =Rs3800


Total Cost price (CP)
=Rs 3800+21120 = Rs24920
Marked Price (MP) of 1 shirt =Rs.450
Discount=20% of Rs450
Net Selling Price (SP) of 1 shirt
=80% of Rs450 = Rs360

Total SP of 96 shirts =Rs 360×96=Rs.34560


Profit Percentage = (SP - CP)/CP * 100%

(34560 - 24920)/24920 * 100% = 38.68% = 39%. Hence, option B.

P a g e 319 | 580
583. Type A, 12 kg of rice worth Rs. 40/kg is mixed with Type B, rice worth Rs.
24/kg. What should be the quantity of Type B rice, if the mixture is sold at Rs.
45/kg with 25% profit added in it?
A. 18 Kg

B. 48 Kg
C. 4 Kg
D. can't say
Answer & Explanation :
Answer: Option C

Explanation :
C.P. of the mixture = 45/1.25 = Rs. 36/kg.
So, (40-36)/(36-24) = (N)/(12) ⇒ N = 4 Kg

584. A person spends 1/7th of his salary on travel,1/3rd of the remaining on


food, he then spends 1/4th of the remaining on rent. Finally he puts 1/6th of
the remaining as a monthly savings, after which he has 25000 left. What is his
salary (in Rs.)?
A. 70,000
B. 14,000

C. 84,000
D. 26,000
Answer & Explanation :

Answer: Option A
Explanation :

Let M be the total salary


Therefore, as per question, M × (6/7) × (2/3) × (3/4) × (5/6) = Rs 25000
M = Rs 70000

585. Point C(x,y) divides the distance AB with point A(8,12) and point B(16, 18)
in a ratio of 3:5, with AC being shorter than BC. What are the co-ordinates of C?

P a g e 320 | 580
A. (12,15)
B. (14.5, 12.5)

C. (14.5, 12.5)
D. (11,14.25)

Answer & Explanation :


Answer: Option C
Explanation :
x co-ordinate of C = [(5×8) + (3×16)] / (3+5) = 11
y co-ordinate of C = [(5×12) + (3×18)] / (3+5) = 14.25

Co-ordinates of C is ( 11, 14.25 )

586. How many terms of the sequence -12, -8, -4,…so on, to make a sum of
120?
A. 11
B. 12
C. 10

D. 13
Answer & Explanation :
Answer: Option B

Explanation :
The series is in A.P. where a = -12 and d = 4.
So 120 = (n/2)[2 × (-12) + (n -1) × 4].
On solving this, we get n = 12

587. The value of a machine depreciates from Rs 32,768 to Rs 21,952 in three


years. What is the rate % of depreciation?

A. 11%
B. 12.25%
C. 12.5%
P a g e 321 | 580
D. 33%
Answer & Explanation :

Answer: Option C
Explanation :

Let R be the rate of depriciation, therefore, 21952 = 32768[1-(R/100)]3 . On


solving this we get R = 12.5%

588. A TV set listed at Rs 3200 is sold to a retailer at a successive discount of


25% and 15%. The retailer desires a profit of 20%, after allowing a discount of
10% to the customer. At what price should he list the TV set (in Rs.)?
A. 2720
B. 2448
C. 2040
D. 2133

Answer & Explanation :


Answer: Option A

Explanation :
The retailers C.P. = 3200 × 0.75 × 0.85 = Rs 2040.
His expected S.P. = 2040 × 1.2 = Rs 2448.

But S.P. is 90% of the L.P., as there is a discount of 10%.


So L.P. = 2448/0.9 = Rs 2720

589. A student is to answer 10 out of 12 questions in an examination such that


he must choose at least 4 from the first five questions. The number of choices
available to him is
A. 140

B. 280
C. 196

D. 346
Answer & Explanation :
P a g e 322 | 580
Answer: Option A
Explanation :

4 questions can be chosen from the first 5 in 5 ways. Remaining 6 questions


can be selected from 8 questions in 8C6 ways. So required answer is 8C6 × 5 =
28 × 5 = 140.

590. A rectangular playground with the dimension of 50m X 30 m is surrounded


by a 5 m wide road all the sides. What is the area of the road?
A. 600 sq. m.

B. 500 sq. m.
C. 450 sq. m.
D. 900 sq. m.
Answer & Explanation :
Answer: Option D

Explanation :
AREA OF ROAD= area of bigger rectangle - area of smaller rectangle ⇒(60×40)-
(50×30)⇒2400-1500=900.

591. It takes 5 sec. for a clock to strike at 5’o clock. If the striking intervals are
uniform how much time will it take to strike 9’o clock (in sec.)?
A. 9
B. 10
C. 11
D. 12
Answer & Explanation :
Answer: Option B
Explanation :

There are 4 intervals in 5 strokes.


Time taken to strike 1 stroke will be 5/4 sec.
At 9, there will be 9 strokes and 8 intervals between two strokes.
P a g e 323 | 580
Thus time required = 5/4 × 8 = 10 sec

592. From the top of the Tower which is 240m high, if the angle of depression
of a point on the ground is 30°, then the distance of the point from the foot of
the Tower is
A. 40 √ 3
B. 80 √ 3
C. 120 √ 3
D. 240 √ 3

Answer & Explanation :


Answer: Option D
Explanation :

593. At the start of a seminar, the ratio of the number of male participants to
the number of female participants was 3 : 1. During the tea break, 16
participants left and 6 more female participants registered. The ratio of the
male to the female participants became 2:1. The total number of participants at
the start of the seminar was -

A. 64
B. 48
C. 54

D. Data Insufficient.

P a g e 324 | 580
Answer & Explanation :
Answer: Option D

Explanation :
In this question, as per the information given we cannot determine how many
are males and how many are females out of 16 participants who left. So, data is
insufficient to answer the given question.

594. A man can row 30 km upstream and 44 km downstream in 10 hours. Also,


he can row 40 km upstream and 55 km downstream in 13 hours. The rate of
the current is -
A. 3 km/hr
B. 3.5 km/hr
C. 4 km/hr
D. 4.5 km/hr

Answer & Explanation :


Answer: Option A

Explanation :
Let the speed of the man in still water be x km/hr and speed of the stream be y
km/h then

Solving (i) and (ii) x = 8 km/hr and y = 3 km/hr


So the rate of stream is 3km/hr.

595. There are two identical vessels, X and Y. Y is filled with water to the brim
and X is empty. There are two pails A and B, such that B can hold half as much
water as A. One operation is said to be executed when water is transferred from
Y to X using A once and water is transferred to Y from X using B once. If A can
hold a liter of water and it takes 40 operations to equate the water level in X
and Y, what is the total volume of water in the system

P a g e 325 | 580
A. 10 liters
B. 20 liters

C. 40 liters
D. 203⁄4 liters

Answer & Explanation :


Answer: Option C
Explanation :
In one complete operation, water transferred = 1 - (1/2) = 1/2 liters as 1 liter
goes from Y to X. So 1 liter is contained in X. But ½ liter goes from X to Y as
well. So net ½ liters remain in X. If it takes 40 operations to equate water level,
∴ 40 × (1/2) = 20 liters is contained in X and Y.
Hence total volume of water in the system is 40 litres. Hence, 3rd option.

596. A salesman's terms were changed from a flat commission of 5% on all his
sales to a fixed salary of Rs. 1,000 plus 2.5 % commission on all sales
exceeding Rs. 4,000. If his remuneration as per the new scheme was Rs. 600
more than by the first scheme, what were his sales worth
A. Rs. 11, 000
B. Rs. 17, 000

C. Rs. 16, 000


D. Rs. 12, 000
Answer & Explanation :

Answer: Option D
Explanation :

Let his sales were worth Rs. x.


So as per the question,

So answer is D option.

P a g e 326 | 580
597. In a class with a certain number of students if one student weighing 50 kg
is added then the average weight of the class increases by 1 kg. If one more
student weighing 50 kg is added then the average weight of the class increases
by 1.5 kg over the original average. What is the original average weight (in kg)
of the class?
A. 46
B. 4
C. 2
D. 47

Answer & Explanation :


Answer: Option D
Explanation :
Let x be the original average and n be the number of students. Let x1, x2, x3
,. . . .. xn be the weights of n students respectively. Therefore,
x1, x2, x3 ,. . . .. xn / n = x
x1, x2, x3 ,. . . .. xn = nx...............(1)

Now,according to 1st condition we have


x1, x2, x3 ,. . . .. xn + 50/ n+1 = x+1
x1, x2, x3 ,. . . .. xn = (n+1)(x+1) - 50..............(2)

Further,according to 2nd condition we have


x1, x2, x3 ,. . . .. xn + 50+50/ n+2 = x+1.5
x1, x2, x3 ,. . . .. xn = (n+2)(x+1.5) - 100..............(3)
Solving (1) and (2), we get
x + n = 49…………….. (4)
Solving (1) and (3), we get
4x + 3n = 194 ……………. (5)

Now, Solving (4) and (5), we get


n = 2, x = 47

So, the original average weight (in kg) of the class = 47


Alternate Solution :
P a g e 327 | 580
Let x be the original average and n be the number of students.
From the first increase in the average,

we get 50 – x = n + 1
From the second increase in the average,

we get 100 – 2x = 1.5 (n + 2)


Solving, we get the value of x = 47.

598. The average marks of a student in 8 subjects is 87. Of these, the highest
marks are 2 more than the one next in value. If these two subjects are
eliminated, the average marks of the remaining subjects are 85. What are the
highest marks obtained by him
A. 94
B. 91
C. 89

D. 96
Answer & Explanation :

Answer: Option A
Explanation :
Let highest marks be (x +2), So, next score = x Total of 8 subjects = 8 × 87 =
696
So, As per question, 696 - x = (x+2) / 6 =85
⇒ x = 92 So, highest marks = (x + 2) = 92 + 2 = 94.

599. If a bucket is 80% full, then it contains 2 liters more water than when it is
662/3% full. What is the capacity of the bucket?
A. 10 liters
B. 15 liters
C. 162/3 liters
D. 20 liters

Answer & Explanation :


P a g e 328 | 580
Answer: Option B
Explanation :

Let the capacity of bucket is x litre.


As per the question,

we have [(4x)/5 - (2x)/3] = 2


⇒ x = 15 litres.

600. The electricity bill of a certain establishment is partly fixed and partly
varies as the number of units of electricity consumed. When in a certain month
540 units are consumed, the bill is Rs. 1,800. In another month 620 units are
consumed and the bill is Rs. 2,040. In yet another month 500 units are
consumed. The bill for that month would be
A. Rs. 1,560
B. Rs. 1,680
C. Rs. 1,840

D. Rs. 1,950
Answer & Explanation :
Answer: Option B
Explanation :

Let V be the variable part of the bill and F be the fixed part. Given 540V + F =
1800 & 620 V + F = 2040. Solving these we get V = 3 and F = 180. So when
500 units are consumed, 500V + F = 500 × 3 + 180 = Rs. 1680.

601. Two cyclists start on a circular track from a given point but in opposite
directions with speeds of 7 m/sec and 8 m/sec respectively. If the
circumference of the circle is 300 metres, after what time will they meet at the
starting point for the first time
A. 20 sec
B. 100 sec

C. 300 sec
D. 200 sec

P a g e 329 | 580
Answer & Explanation :
Answer: Option C

Explanation :
S = D/T The times taken by the two cyclists to reach the starting point are
300/7 seconds and 300/8 seconds respectively. So, they will meet at the
starting point after LCM(300/7, 300/8) = 300 seconds.
602. The average weight of 8 persons increases by 2.5 kg when a new person
comes in place of one of them weighing 65 kg. What is the weight of the new
person?

A. 76.5 Kg
B. 78 Kg
C. 85 Kg
D. Inadequate Data
Answer & Explanation :

Answer: Option C
Explanation :

Increase of Average = 2.5 for 8 persons


Total Increase = 2.5 × 8 = 20 kg.
Weight of New Person = 65 kg + 20 kg = 85 kg

603. The difference between the length and breadth of a rectangle is 23 m. If


its perimeter is 206 m, then its area is:
A. 1520 sqm
B. 2520 sqm
C. 2480 sqm
D. 2720 sqm

Answer & Explanation :


Answer: Option B

Explanation :
P = 2 (L + B) L – B = 23
P a g e 330 | 580
=> 2 L + 2 B = 206
2 L – 2 B = 46

=> L = 63;
B = 40 Area = 63 × 40 = 2520 sqm

604. On 8th Feb, 2005 it was Tuesday. What was the day of the week on 8th
Feb, 2004?
A. Sunday
B. Monday

C. Tuesday
D. Thursday
Answer & Explanation :
Answer: Option A
Explanation :
Between 8th February 2004 & 8th Feb 2005 there are 366 days.
366 days => 52 weeks + 2 days

Hence, it was SUNDAY

605. In one hour, a boat goes 11 km/hour along the stream and 5 km/hour
against the stream. The speed of the boat in still water (in km/hour) is:
A. 5

B. 7
C. 8

D. 10
Answer & Explanation :
Answer: Option C

Explanation :
11/(B+S) = 1 hr
5/(B-S) = 1 hr
P a g e 331 | 580
=> B + S = 11 & B - S = 5
Therefore, on solving we get

= > Boat speed = 8km/hr


Stream Speed = 3km/hr

606. Ram purchased 20 dozens of toys at the rate of Rs.375 per dozen. He sold
each one of them at the rate of Rs.33. What was his percentage profit?
A. 3.6
B. 3.8

C. 4.1
D. 5.6
Answer & Explanation :
Answer: Option D
Explanation :
CP = 375 × 20 = 7500/-
SP = 33 × 12 × 20 = 7920/-

Therefore, % Profit = (7920 - 7500)/7500 × 100 = 5.6%

607. When the profit on a commodity triples then its selling price is doubled.
Find the profit percent.
A. 35

B. 50
C. 85

D. 100
Answer & Explanation :
Answer: Option D

Explanation :
Going by the options (d) satisfies
CP = 100/- Price = 100% SP = 200/-
P a g e 332 | 580
CP = 100/- Price = 300% SP = 400/-

608. In a 100 m race, A beats B by 10 m and C by 13 m. In a race of 180 m, B


will beat C by:

A. 4m
B. 6m
C. 8m
D. 9m
Answer & Explanation :

Answer: Option B
Explanation :
A -> runs 100m
B -> runs 90m
C -> runs 87m
=> B runs 180m
C runs 174 m => 6m

609. A dishonest milk seller professes to sell the milk at cost price but gain
20%. The proportion of milk and water is:

A. 5:3
B. 4:1

C. 6:5
D. 5:1

Answer & Explanation :


Answer: Option B
Explanation :

The amount of water is the Gain ( 20% )


=> Proportion is 4: 1

P a g e 333 | 580
610. It takes 5 hours for a boat to go down-stream and 10 hours to return to
the same place against the stream. If the stream is flowing at the rate of 40
km/hour, calculate the speed of the boat.
A. 150 km/hour

B. 170 km/hour
C. 120 km/hour
D. 100 km/hour
Answer & Explanation :
Answer: Option C

Explanation :
D/(B - 40) = 5 hr
D/(B + 40) = 10 hr
=> B - 40 = 5/D & B - 40 = 10/D
=> 2B - 80 = B + 40
Therefore, speed of boat = 120 km/hr

611. Cost price of 30 mangoes is equal to the selling price of 24 mangoes.


Calculate the percentage of profit.
A. 20

B. 25
C. 30

D. 32
Answer & Explanation :

Answer: Option B
Explanation :
CP of 1 Mango = 1/-

CP of 30 Mango = 30/-
SP of 24 Mangoes = 30/-

SP of 1 Mango = 1.25/-

P a g e 334 | 580
=> Profit = 25%
612. A cricketer played 80 innings and scored an average of 99 runs. His score
in the last inning was zero run. To have an average of 100 at the end, his score
in the last innings should have been

A. 60
B. 80
C. 10
D. 1
E. None of these

Answer & Explanation :


Answer: Option C
Explanation :
Let x be the score in the last inning to make the average of 100
Therefore, when average is 99, total score of 80 innings = 80 * 99 = 7920
And when the average is 100, total score of 80 innings = 80 * 100 = 8000
Therefore, x = 8000 - 7920 = 80

613. A man spends an average of Rs. 1,694.70 per month for the first 7 months
and Rs.1,810.50 per month for the next 5 months. His monthly salary if he
saves Rs. 3,084.60 during the whole year is
A. 1000
B. 2000
C. 2400

D. 3000
E. None of these
Answer & Explanation :

Answer: Option B
Explanation :

Monthly salary =
P a g e 335 | 580
614. A and B undertake to do a piece of work for Rs. 2,200. A alone can do it in
8 days, while B can do it in 6 days. With the help of C, they complete it in 3
days. Find C's share.

A. 150
B. 275
C. 245
D. 175
E. None of these

Answer & Explanation :


Answer: Option B
Explanation :
Let 1 be the total work
Therefore, C's one day work = (1/3)-[(1/8)+(1/6)]= 1/24
Ratio of share of A, B and C
i.e. A:B:C = 1/8:1/6:1/24=> [1/3] - {[1/8]+[1/6]} = (1/24) 3 : 4 : 1

So, C's share = {1/8} *2200=275-/

615. By selling an article at 80% of its marked price, a trader makes a loss of
10%. What will be the profit percentage if he sells it at 95% of its marked
price?
A. 5.9
B. 12.5

C. 6.9
D. 5
E. None of these

Answer & Explanation :


Answer: Option C

Explanation :

P a g e 336 | 580
Let us assume the MP to be 900/-
Therefore, SP = 80% of MP = 0.80 * 900 = 720/-

As loss is 10%, so CP = 720/90 *100=800


Now, if SP = 95% of MP = 0.95 *900 = 855/-

Profit% = 855-800 /800 =55/8=6.9

616. By selling an umbrella for Rs. 30, a shopkeeper gains 20%. During a
clearance sale, the shopkeeper allows a discount of 10% of the marked price.
His gain during the sale season is

A. 8
B. 9
C. 7
D. 7.5
E. None of these
Answer & Explanation :
Answer: Option A

Explanation :
Given that SP of umbrella = 30/- and profit% = 20. Therefore, CP of umbrella =
30/120 *100=25 SP of umbrella after 10% discount = 90/100 *30=27 Thus,
profit% = 27-25 /25 =8

617. From a vessel containing 100 ltr. of wine, 10 ltr. are drawn out and an
equal amount of water is added. From the mixture, 10 ltr. is again drawn out
and same quantity of water is added. What is the final ratio of wine and water?
A. 91:9
B. 81:19

C. 80:20
D. 90:10
E. None of these
Answer & Explanation :

P a g e 337 | 580
Answer: Option B
Explanation :

This is the question of removal and replacement If a vessel contains "x" litres of
liquid A and if "y" litres be withdrawn and replaced by liquid B, then if "y" litres
of the mixture be withdrawn and replaced by liquid B again, and the operation is
repeated "n" times in all, then:

So, in the given question, we have A as wine and B as water and 2 operations
have occurred.
Therefore, quantity of wine after 2 operations
Thus, in a mixture of 100 ltr, 81 is wine, so water will be 100 - 81 = 19 ltr

Required ratio of wine and water is 81 : 19

618. From each of two given numbers, half the smaller number is subtracted.
After such subtraction, the larger number is 4 times as large as the smaller
number. What is the ratio of the numbers?
A. 4:1
B. 4:2

C. 5:2
D. 1:4
E. None of these
Answer & Explanation :
Answer: Option C
Explanation :

P a g e 338 | 580
619. Men, women and children are employed to do a work in the proportion of 3
: 2 : 1 and their wages per person are in the proportion of 5 : 3 : 2. When 90
men are employed, total daily wages of all amounts to Rs. 10,350. Find the
daily wage of a man.

A. 115
B. 75
C. 45
D. 57.90
E. None of these

Answer & Explanation :


Answer: Option B
Explanation :
Let the daily wages be 5k, 3k and 2k of men, women and children respectively.
We are given that there are 90 men and total amount of wages is 10350/-

Thus, men, women and children are 90, 60 and 30 respectively.


Hence, total wages = 90 x 5k + 60 x 3k + 30 x 2k = 10350

=>k = Rs.15
Thus, daily wages of each man = 5k = Rs.15 x 5 = Rs. 75

620. The population of a town is 3,11,250. The ratio between women and men
is 43 : 40. If there are 24% literate among men and 8% literate among women,
the total number of literate persons in the town is
A. 56,800
B. 99,600
C. 41,800
D. 48,900

E. None of these
Answer & Explanation :

Answer: Option D
Explanation :
P a g e 339 | 580
Total number of literate people is 24 % of 40 % of men and 8 % of 43 % of
women.

Therefore, No of literate persons = [24/100 *40/83* 311250 ] + [ 8/100 *


43/83 * 311250] = 48900

621. In an examination, 52% of the candidates failed in English and 43% failed
in Mathematics. If 17% failed in both the subjects, then the percentage of
candidates, who passed in both the subjects, was
A. 25

B. 22
C. 23
D. 21
E. None of The above
Answer & Explanation :

Answer: Option B
Explanation :

% of students failed in Mathematics only = 43% – 17% = 26%


% of students failed in English only = 52% – 17% = 35%. So % of students
passed in both = 100 – (26 + 35 + 17) = 100 – 78 = 22% Alternate Solution:

% of students failed in Mathematics or English or both = 52% + 43% – 17% =


78%
So % of students passed in both = 100% – 78% = 22%. Hence, option B is
correct.
622. Suri gave 25% of her monthly salary to her mother. From the remaining
salary, she paid 15% towards rent and 25%, she kept aside for her monthly
expenses. The remaining amount she kept in bank account. The sum of the
amount she kept in bank and that she gave to her mother was Rs. 42000. What
was her monthly salary?
A. Rs. 50,000
B. Rs. 60,000
C. Rs. 65,000

D. Rs. 64,000
P a g e 340 | 580
E. Rs. 72,000
Answer & Explanation :

Answer: Option B
Explanation :

Let x be the total income


(75/100*60/100)x + 25/100x = 42000
Solving, x = 60000

623. Type A, 12 kg of rice worth Rs. 40/kg is mixed with Type B, rice worth Rs.
24/kg. What should be the quantity of Type B rice, if the mixture is sold at Rs.
45/kg with 25% profit added in it?
A. 18 kg
B. 48 kg
C. 4 kg
D. can't say
Answer & Explanation :

Answer: Option C
Explanation :
C.P. of the mixture = 45/1.25 = Rs. 36/kg.

So, (40-36)/(36-24) = (N)/(12) ⇒ N = 4 Kg

624. A person spends 1/7th of his salary on travel,1/3rd of the remaining on


food, he then spends 1/4th of the remaining on rent. Finally he puts 1/6th of
the remaining as a monthly savings, after which he has 25000 left. What is his
salary (in Rs.)?
A. 70,000
B. 14,000
C. 84,000
D. 26,000

Answer & Explanation :


P a g e 341 | 580
Answer: Option A
Explanation :

Let M be the total salary


Therefore, as per question, M × (6/7) × (2/3) × (3/4) × (5/6) = Rs 25000

M = Rs 70000

625. A TV set listed at Rs 3200 is sold to a retailer at a successive discount of


25% and 15%. The retailer desires a profit of 20%, after allowing a discount of
10% to the customer. At what price should he list the TV set (in Rs.)?

A. 2720
B. 2448
C. 2040
E. 2133
Answer & Explanation :
Answer: Option A
Explanation :

The retailers C.P. = 3200 × 0.75 × 0.85 = Rs 2040


. His expected S.P. = 2040 × 1.2 = Rs 2448.
But S.P. is 90% of the L.P., as there is a discount of 10%.

So L.P. = 2448/0.9 = Rs 2720

626. A student is to answer 10 out of 12 questions in an examination such that


he must choose at least 4 from the first five questions. The number of choices
available to him is
A. 140
B. 280

C. 196
D. 346

Answer & Explanation :

P a g e 342 | 580
Answer: Option A
Explanation :

4 questions can be chosen from the first 5 in 5 ways. Remaining 6 questions


can be selected from 8 questions in 8C6ways. So required answer is 8C6 × 5 =
28 × 5 = 140.

627. 6. Two years ago the average age of a family of 8 members was 18 years.
After the addition of a baby, the average age of family remains the same today.
What is the age of the baby ?

A. 1 year
B. 2 years
C. 4 years
D. 3.5 years
Answer & Explanation :

Answer: Option B

628. An article when sold at a gain of 5% yields Rs. 15 more than when sold at
a loss of 5%. The cost price of the article is:
A. Rs. 200

B. Rs. 150
C. Rs. 80
D. Rs. 64
Answer & Explanation :
Answer: Option B
Explanation :
10/100 × x = 15 ⇒ x=150

As the loss and profit both are earned on the cost price.

P a g e 343 | 580
629. The tax on a commodity is diminished by 10 % and its consumption
increased by 10 %. The effect on the revenue derived from it changes by K %.
Find the value of K.
A. 1

B. -2
C. -1
D. 2
Answer & Explanation :
Answer: Option C

Explanation :
Directly using the formula, when a value is increased by R% and then
decreased by R%, then net there is ( R^2)/100 decrease. Putting R = 10, we
get 1% decrease.

630. Ratio of Ashok's age to Pradeep's age is 4 : 3. Ashok will be 26 years old
after 6 years. How old is Pradeep now?

A. 18
B. 21
C. 15

D. 24
Answer & Explanation :
Answer: Option C
Explanation :
Given A/p= 4/3 Also A = 26 after 6 years, so his present age = 20years,
Substituting we get P = 15 years.

631. The incomes of Chanda and Kim are in the ratio 9 : 4 and their
expenditures are in the ratio 7 : 3. If each saves Rs. 2,000, then Chanda's
expenditure is
A. 60000
B. 80000
P a g e 344 | 580
C. 70000
D. None of these

Answer & Explanation :


Answer: Option C

Explanation :
Let the incomes of Chanda and Kim be 9x and expenditures be 7y and 3y
respectively. Since = Income – Expenditure, we get 9x – 7y = 2000 and 4x –
3y = 2000. Solving, we get, x = 8000 and y = 10000. So Chanda’s expenditure
= 7y = 7 × 10000 = Rs. 70,000.

632. 7 cannibals of XYZ island, decide to throw a party. As you may be aware,
cannibals are guys who eat human beings. The senior among them – Father
Cannibal decides that any 6 of them will eat up one cannibal, and then out of
the remaining six – five of them will eat up one cannibal and so on till one is
left. What is the time until one cannibal is left, if it takes one cannibal 3 hours to
eat up one cannibal independently?
A. 7 hrs 11 min

B. 6 hrs 12 min
C. 7 hrs 21 min
D. 18 hrs 16 min
Answer & Explanation :
Answer: Option C

Explanation :
At the beginning 6 cannibals will eat one, so time required will be 180/6 = 30
min.
Then out of the remaining six – five will devour one, so time required will be
180/5 = 36 min.

Thus the time until one cannibal is left will be = (180/6 + 180/5 + 180/4 +
180/3 + 180/2 + 180/1) min
= (30 + 36 + 45 + 60 + 90 + 180) min
= 441 min
= 7 hrs 21 min.

Hence option 3.
P a g e 345 | 580
633. Three articles are purchased for Rs. 1050, each with a different cost. The
first article was sold at a loss of 20%, the second at 1/3rd
A. 14.28% gain

B. 13% loss
C. 12% loss
D. 11.11% gain
Answer & Explanation :
Answer: Option A

Explanation :
Let us assume that their CPs are x, y & z respectively.
According to the given condition 0.8x = 1.33y = 1.6z
⇒ (80/100)x = 400y/(3 × 100) = (160/100)z

⇒x:y=5:3&y:z=6:5

Thus x : y : z = 10 : 6 : 5
Hence CPs of the articles are x = (10/21) × 1050 = 500,
y = (6/21) × 1050 = 300 &
z = (5/21) × 1050 = 250.

SP of the article with CP Rs. x is 0.8x = 0.8 × 500 = 400.


Since SPs are same, the total SP will be 400 × 3 = 1200.
Hence the gain % = (SP – CP)/CP × 100 = (1200 – 1050)/1050 × 100 =
14.28%.

634. In a game of tennis, A gives B 21 points and gives C 25 points. B gives C


10 points. How many points make the game?
A. 50
B. 45

C. 35
D. 30

P a g e 346 | 580
Answer & Explanation :
Answer: Option C

Explanation :
When B scored p -10 then C scored p - 25.

When B scores 1 then C scores (p-25)/(p-10)


So when B scores p points then C will score (p-25)/(p-10) × p
As per question (p-25)/(p-10) × p = p -10 . Solving this we get p = 35
A B C
p points (p-21)points (p-25)points

p points (p-10)points

635. Twice the speed of a boat downstream is equal to thrice the speed
upstream. The ratio of its speed in still water to the speed of current is
A. 1 : 5
B. 1 : 3
C. 5 : 1

E. 2 : 3
Answer & Explanation :
Answer: Option C

Explanation :
Let the boat speed in still water be b.
Let the stream speed be x.
2(b+ x) = 3(b-x)

5x=b
b/x=5/1

636. A person has a chemical of Rs. 25 per litre. In what ratio should water be
mixed with that chemical so that after selling the mixture at Rs. 20/litre he may
get a profit of 25%?

P a g e 347 | 580
A. 13 : 16
B. 12 : 15

C. 9 : 16
D. 19 : 22

Answer & Explanation :


Answer: Option C
Explanation :
This can be solved using allegation.
What is required at the end of mixing is a price of 20/1.25 = 16.

So the allegation would look like this –


Hence the ratio would be (25 – 16) : 16 = 9 : 16
Hence required ratio of Water : Chemical is 9:16.

637. The difference between the simple interest and compound interest on a
certain sum of money for 2 years at 15% p. a. is Rs. 45. Find the sum.
A. Rs. 2700

B. Rs. 2500
C. Rs. 2000
D. None of these

Answer & Explanation :


Answer: Option C
Explanation :
Since we know that the interest rate is 0.15, and knowing that the difference
between two years of compound interest is nothing but interest on interest, we
can find the first year’s interest as –
45/0.15 = 300.

Now if the interest is 300 at the end of one year, then the principal is 300 / 0.15
= 2,000.

P a g e 348 | 580
638. How many terms are there in an A.P. whose first and fifth terms are -14
and 2, respectively, and the sum of terms is 40?

A. 15
B. 10

C. 5
D. 20
Answer & Explanation :
Answer: Option B
Explanation :

Now the common difference of this AP is 16/4 = 4.


The sum of an AP is n/2 {2a + (n – 1)d}
Substituting we get, 40 = n/2 {2×-14 + (n – 1)4}
The best way to solve this is by plugging options. Put in n = 10 and get the RHS
as 40.
639. The difference of two numbers is 11 and one fifth of their sum is 9. The
numbers are :

A. 31, 20
B. 30, 19
C. 29, 18

D. 28, 17
Answer & Explanation :

Answer: Option D
Explanation :

x − y = 11, x + y = 5 × 9 x − y = 11, x + y = 45, y = 17, x = 28

640. How many numbers between 1 and 100 are divisible by 7?

A. 9
B. 11

C. 17

P a g e 349 | 580
D. 14
Answer & Explanation :

Answer: Option D
Explanation :

No. of divisible by 7 7, 14 --------- 98, n = a + (N - 1)d


98 = 7 + (N - 1) 7, 98 = 7 + 7N - 7
98/7= N = 14

641. What is the number which when multiplied by 13 is increased by 180?

A. 13
B. 15
C. 23
D. 35
Answer & Explanation :
Answer: Option B
Explanation :

13 × 15 = 195

642. In 24 minutes, the hour hand of a clock moves through an angle of:

A. 60°
B. 24°
C. 12°
D. 5°

Answer & Explanation :


Answer: Option C
Explanation :

12 hour = 360˚, 1 hr. = 360/12 = 30˚ 60 min = 30˚, 1 min 30/60 = .5˚ 24
min. = 1/2 ×24 = 12˚

P a g e 350 | 580
643. √0.0081 is equal to :

A. 0.09
B. 0.9

C. ±0.08
D. 0.81
Answer & Explanation :
Answer: Option A
Explanation :

√0.0081 = √0.0081/10000 = √81/10000 = 9/100 =0.09

644. A reduction of 20% in the price of mangoes enables a person to purchase


12 more for Rs. 15. What was the price of 16 mangoes before reduction of
price?
A. Rs. 6
B. Rs. 5

C. Rs. 7
D. Rs. 9
Answer & Explanation :

Answer: Option B
Explanation :

645. The average age of a man and his son is 28 years. The ratio of their ages
is 3 :1 respectively. What is the man's age?

A. 30 years
B. 38 years
P a g e 351 | 580
C. 44 years
D. 42 years

Answer & Explanation :


Answer: Option D

Explanation :
Total sum of man's age & his son's age =28 × 2 = 56 Now, the Ratio of their
ages is 3 : 1.Therefore, Man's age = (3/4) × 56 = 42
So, the correct answer is option D.

646. A cyclic quadrilateral ABCD is such that AB = BC, AD = DC, AC is


perpendicular to BD and ∠CAD = θ, then find the ∠ABC.
A. θ
B. θ/2
C. 2θ
D. 3θ

Answer & Explanation :


Answer: Option C
Explanation :
∠B + ∠D = 180° ∠A + ∠C = 180° ∠BAC = ∠BCA ∠DAC = ∠DCA ∴∠DAB = ∠DCB
= 90° ∠DAC = θ
∴∠ADE = 90° - θ = ∠CDE ∴ ∠ABC = 180° – 2(90° - θ) = 2θ

647. How many integers are there between 300 and 600 that are divisible by 9?
A. 33

B. 31
C. 28
D. 25
Answer & Explanation :
Answer: Option A

P a g e 352 | 580
Explanation :
The sequence is 306… 594

594=306+ (n-1)9⇒288= (n-1)9⇒n=33

648. What will be the ratio of petrol and kerosene in the final solution formed by
mixing petrol and kerosene that are present in three identical vessels in the
ratio 4:1,5:2 and 6:1 respectively?

A. 166 : 22
B. 83 : 22
C. 83 : 44
D. 78 : 55
E. None of these
Answer & Explanation :
Answer: Option B
Explanation :

Three identical vessels in the ratio 4:1, 5:2 and 6 :1 respectively.


Petrol : kerosene
(4 : 1 = 5)7
(5 : 2 = 7)5
(6 : 1 = 7)5
28 : 7 = 35
25 : 10 = 35
30 : 5 = 35
83 : 22

649. There are 6 consecutive odd numbers. The difference between the square
of the average of the first three numbers and the square of the average of the
last three numbers is 288. What is the last odd number?
A. 31
B. 27

P a g e 353 | 580
C. 29
D. 25

E. 33
Answer & Explanation :

Answer: Option C
Explanation :
Let the 6 consecutive odd no.’s are:
X, X+2, X+4, X+6, X+8, X+10
Avg. of 1st three no’s is X+2.

Avg. of Last three no’s is X+8.


X=19
Last Odd no. is X+10= 29.
Answer is 29.

650. In a bag there are 6 red balls and 9 green balls. Two balls are drawn at
random, what is the probability that at least one of the balls drawn is red?

A. 29/35
B. 7/15
C. 23/35

D. 2/5
E. 19/35
Answer & Explanation :
Answer: Option C

Explanation :
Probability of atleast one of the balls drawn is red= 1- (9/15) * (8/14)=23/35.
Answer is 23/35.

651. A started a business with an investment of Rs. 28,000. After 5 months


from the start of the business, B and C joined with Rs. 24,000and Rs. 32,000
P a g e 354 | 580
respectively and withdrew Rs. 8000 from the business. If the difference
between A’s share and B’s share in the annual profit is Rs. 2,400, what was the
annual profit received?
A. Rs. 15,600

B. Rs. 14,400
C. Rs. 14,040
D. Rs. 15,360
E. Rs. 13,440
Answer & Explanation :

Answer: Option B
Explanation :
Equivalent Contribution of A= 28000 * 5+20000 * 7= 280000
Equivalent Contribution of B= 24000 * 7= 168000
Equivalent Contribution of C= 32000 * 7= 224000
Let total profit be X.
Given that,

280000X/672000 – 168000X/672000=2400
112000/672000 * X=2400
or X=2400 * 672/112

X=14400

652. At present, Ami’s age is twice Dio’s age and Cami is two years older than
Ami. Two years ago, the respective ratio between Dio’s age at that time and
Cami’s age at that time was 4 : 9. What will be Ami’s age four years hence?
A. 40 years
B. 30 years

C. 42 years
D. 36 years

E. 48 years

P a g e 355 | 580
Answer & Explanation :
Answer: Option A

Explanation :
A = 2D; C = A+2 = 2D+2

Given, D-2/(2D+2)-2 = 4/9


Solving, D = 18 Years and A = 36+4 = 40 years.

653. Three Science classes A, B and C take a Life Science test. The average
score of students of class A is83. The average score of students class B is 76.
The average score of class C is 85. The average score of class A and 8 is 79 and
average score of class B and C is 81. Then the average score. Of classes A, B
and C is
A. 80
B. 80.5

C. 81
D. 81.5

E. None of these
Answer & Explanation :
Answer: Option D

Explanation :
T.I.=30,000, N.T.I.=18000
Taxable 12000, Income Tax = 25% of Taxable Income
= 1/4 ×: 12000 = RS.3000

P a g e 356 | 580
A:B:C = 3:4:5
Sum of A+B+C = 3*83+4*76+5*85 = 978

Average score = 978/12 = 81.5

654. A hemispherical bowl of internal diameter 54 cm contains a liquid. The


liquid is to be filled in cylindrical bottles of radius 3 cm and height 9 cm. How
many bottles are required to empty the bowl?
A. 221
B. 343

C. 81
D. 243
E. None of these
Answer & Explanation :
Answer: Option E
Explanation :
Area of a hemispherical bowl = 2/3 ×π ×r3

Area of a cylinder = πr2h


Area of a cylinder = n × Area of a hemispherical bowl
2/3 ×π ×273 = n ×π×32×9

we get n = 162

DIRECTIONS for the questions 655 & 657 : What approximate value will
come in place of question mark (?) in the given question?( You are not
expected to calculate the exact value.)

655. 26.00 - 154.001/6.995 = ?

A. 4
B. 18

C. 9

P a g e 357 | 580
D. 10
E. 14

Answer & Explanation :


Answer: Option A

Explanation :
26 – (154/7) = 4

656. 17.995/3.01 + 104.001/12.999 = ?


A. 11

B. 20
C. 23
D. 14
E. 17
Answer & Explanation :
Answer: Option D
Explanation :

18/3 + 104/13 = 14

657. 3/5 of 4/7 of 7/9 of 425 = ?

A. 121
B. 110
C. 118
D. 113

E. 124
Answer & Explanation :
Answer: Option D

Explanation :
3/5*4/7*7/9*425 = 113

P a g e 358 | 580
658. If 378 coins consist of rupee, 50 paise and 25 paise coins, whose values
are proportional to 13 :11 : 7, the number of 50 paise coins will be :
A. 128

B. 132
C. 133
D. 136
Answer & Explanation :
Answer: Option B

Explanation :
If values are proportional to 13 : 11 : 7, then the number of coins will be
proportional to 13/1 : 11/0.50 : 7/0.25
⇒ 13 : 22 : 28. Now from this the number of coins of 50 paise will be 378 ×
22/63 = 132.
659. How many words, with or without meaning can be formed with the letters
of the word MANAGEMENT?
A. 226800
B. 453600
C. 3907200

D. 1814400
E. 3628800
Answer & Explanation :

Answer: Option A
Explanation :

Reqd. number of ways = (10!/2!2!2!2!)=226800

660. The number of ways in which letters of the word PRAISE be arranged
A. 720
B. 610

P a g e 359 | 580
C. 360
D. 210

E. None of these
Answer & Explanation :

Answer: Option A
Explanation :
Reqd. number of ways = 6! = 720.

661. Jar A has 60 litres of mixture of milk and water in the respective ratio of 2:
1. Jar B which had 40 litres of mixture of milk and water was emptied into Jar
A, as a result in Jar A, the respective ratio of milk and water becomes 13 : 7.
What was the quantity of water in Jar B?
A. 8 litres
B. 15 litres

C. 22 litres
D. 7 litres

E. 1 litres
Answer & Explanation :
Answer: Option B

Explanation :
Jar A has 60 Litres.

Ratio between milk and water 2:1,


Quantity of Milk in Jar A = 2/3×60 = 40

Quantity of Water in Jar A = 1/3× 60 = 20


40 liters of Mixture B having milk and water is emptied into Jar A.
Therefore, Total Mixture = 60 +40 = 100

The respective Ratio of Milk and Water is 13:7


Quantity of Milk in Jar A = (13/20)×100 = 65

Quantity of Water in Jar A = (7/15)× 100 = 35

P a g e 360 | 580
Quantity of water in Jar B = 35 – 20 = 15 Litres.

662. The sum of a series of 5 consecutive odd numbers is 195. The second
lowest number of this series is one less than the second highest number of
another series of 5 consecutive even numbers. What is 40% of the second
lowest number of the series of consecutive even numbers?
A. 16.8
B. 14.8
C. 19.4

D. 17.6
E. 13.6
Answer & Explanation :
Answer: Option E
Explanation :

The sum of a series of 5 Consecutive odd numbers is 195


Let the Series of Consecutive odd numbers is X, X+2, X+4, X+6, X+8

X+ X+2+ X+4+ X+6+ X+8= 195


5X+20=195
X=35

Series of Consecutive odd numbers is 35, 37, 39, 41, 43


According to the question, the second lowest number of this series is 1 less than
the Second highest number of another series of 5 consecutive numbers.
Second lowest number is 37.

Second highest number of another series of 5 consecutive numbers = 37+1 =


38
Therefore, another series of 5 consecutive numbers 32, 34, 36, 38, 40.

40% of the lowest number of the series of consecutive numbers= 34×(4/100) =


13.6

P a g e 361 | 580
663. The sum of the dimensions of a room (i.e. length, breadth and height) is
18 metres and its length, breadth and height are in the ratio of 3 : 2 : 1
respectively. If the room is to be painted at the rate of Rs. 15 per m2, what
would be the total cost incurred on painting only the four walls of the room (in
Rs.)?
A. 3250
B. 2445
C. 1350
D. 2210

E. 2940
Answer & Explanation :
Answer: Option C
Explanation :
Ratio of length : breadth : height = 3:2:1

Sum of dimensions of room = 18


Length =(3/6) ×18 = 9

Breath =(2/6) ×18 = 6


Height =(1/6) ×18 = 3
Area of four walls = 2h×(l+b) = 2×3(9+6) = 90

Total cost of painting four walls = 90×15 = 1350

664. B is 4/3 times as efficient as A. If A can complete 5/8th of a given task in


15 days, what fraction of the same task would remain incomplete if B works on
it independently for 10 days only?
A. 3/4
B. 2/3

C. 5/8
D. 4/9
E. 2/3
Answer & Explanation :

P a g e 362 | 580
Answer: Option D
Explanation :

B is 4/3 times as efficient of A.


Ratio of time taken by A and B, A: B is 4:3.

A can complete 5/8th of a given task in 15 days


A can do alone his work in = 8/5×15= 24 days
Therefore, B can do this work= 18
B works independently for 10 days only, thus work done = 10/18 = 5/9
Remaining work (incomplete) = 1-(5/9) = 4/9

665. In a class, the average weight of boys is 64 kg and that of 75 girls is 70


kg. After a few days, 60% of the girls and 30% of the boys leave. What would
be the new average weight of the class (in kg)? Assume that the average
weight of the boys and the girls remain constant throughout.

A. 63
B. 66.5

C. 68.5
D. 65.5
E. Can’t be determined

Answer & Explanation :


Answer: Option E

Explanation :
In this question, number of boys is not mentioned .So, we can’t find new
average.

DIRECTIONS for question 666 to 668: What will come in place of


question mark (?) in the given number series?

666. 15 27 37 45 51 ?
A. 58
P a g e 363 | 580
B. 80
C. 65

D. 74
E. 55

Answer & Explanation :


Answer: Option E
Explanation :
Given series is:
15 27 37 45 51 ?

And the pattern is 27 – 15 = 12, 37 – 27 = 10, 45 – 37 = 8, 51 – 45 = 6.


Thus the next term is 51 + 4 = 55

667. 700 457 376 349 340 ?


A. 266
B. 329
C. 304

D. 337
E. 307
Answer & Explanation :

Answer: Option D
Explanation :
Given series is:
700 457 376 349 340 ?

Pattern is 700 – 457 = 243, 457 – 376 = 81, 376 – 349 = 27, 349 – 340 = 9
Thus, the next term is 340 – 3 = 337

668. 1 2 6 21 88 ?
A. 425

P a g e 364 | 580
B. 475
C. 295

D. 445
E. 395

Answer & Explanation :


Answer: Option D
Explanation :
Given series is:
1 2 6 21 88 ?

Pattern is 1×1+1= 2
2×2+2= 6
6×3+3=21
21×4+4= 88
88×5+5= 445
So the next term of the series is = 445
669. A 180-metre-long train crosses a platform of equal length in 18 seconds.
What is the speed of the train?
A. 22m/sec
B. 10m/sec

C. 15m/sec
D. 18m/sec
E. None of these
Answer & Explanation :

Answer: Option E
Explanation :
At the beginning 6 cannibals will eat one, so time required will be 180/6 = 30
min.
When the train crosses the platform , it covers its own lenth as well as lenth of
platform, Therefore total length becomes = 180+180 = 360m.

P a g e 365 | 580
Time taken is 18 seconds.
Therefore, required speed is

360/18= 20m/sec
Therefore, the correct answer is option E.

670. What should come in place of question mark (?) : 13 30 66 140 ? 592
A. 210
B. 290
C. 428

D. 430
E. None of these
Answer & Explanation :
Answer: Option B
Explanation :
13×2+4=30
30×2+6=66

66×2+8=140
290×2+12=592
Hence, the question mark should be replaced by 290

671. 8 men alone can complete a piece of work in 12 days. 4 women alone can
complete the same piece of work in 48 days and 10 children alone can complete
the piece of work in 24 days. In how many days can 10 men, 4 women and 10
children together complete the piece of work (in days)?
A. 5
B. 15

C. 28
D. 6

E. None of these

P a g e 366 | 580
Answer & Explanation :
Answer: Option D

Explanation :
From the given information we get:

Work done by (8×12) men = (4×48) women = (10×24) children i.e. work done
by 1 man = 2 women = 2.5 children.
Now, the required time to finish to work:
M1D1H1/W1 = M2D2H2/W2
(10×24)/((10×2.5)+4×(2.5/2)+10)

= 6 days

672. Subhash starts a business by investing Rs 25000. 6 months later Aditya


joins him by investing Rs 15,000. After another 6 months Aditya invests an
additional amount of Rs 15,000. At the end of 3 years they earn a profit of Rs
2,47,000. What is Aditya's share in the profit?
A. Rs 1,30,000

B. Rs 1,23,000
C. Rs 1,05,000
D. Rs 1,11,500

E. None of these
Answer & Explanation :
Answer: Option E
Explanation :

Since it is a compound partnership , Therefore profit is divided in the ratio of


I1T1 : I2T2
Where I is investment and T is time.

Subhash = 25000×36 = 900000/-


Aditya = (15000×6)+(30000×24) = 810000/-

Thus, ration of the profits of Subash and Aditya will be 900000:810000 i.e..
10:9

P a g e 367 | 580
Aditya's share = 9/(10+9) X 247000 = Rs. 117000

673. Ram’s age was square of a number last year and it will be cube of a
number next year. How long must he wait before his age is again the cube of a
number?
A. 39 years
B. 38 years
C. 10 years
D. 64 years

E. None of these
Answer & Explanation :
Answer: Option B
Explanation :
By a little guess work we can easily determine that the present age is 26 as
next year it will be 27 which is the cube of 3. One year ago it was 25 ,
which is square of 5. Next cube will be at 64 so he has to wait for 38 years.
Hence option B is the answer.

674. ‘A’ and ‘B’ can do a piece of work in 24 days and 32 days respectively.
They started working together, After how many days should B leave so that the
work is finished in 18 days.
A. 8 days
B. 4 days
C. 6 days
D. 2 days
E. None of these

Answer & Explanation :


Answer: Option A
Explanation :
Let total work is 96 units.

P a g e 368 | 580
So, A does 4 units per day.
B does 3 units per day.

As per the question, A remains for the whole time i.e. 18 days ,so out of 96
units he does 18x4=72 units.

So remaining 96 - 72 = 24 are done by B.


So B should leave after 24/3 = 8 days.

675. The difference between 38% of a number and 24% of the same number is
135.10. What is 40% of that number?

A. 394
B. 370
C. 378
D. 386
E. 390
Answer & Explanation :
Answer: Option D

Explanation :
Let the number be x,
Therefore, 0.38x – 0.24x = 135.10

We get x = 965
Thus, 0.40 × 965 = 386

676. The cost of 13 kg of sugar is Rs. 195/-. The cost of 17 kg. of rice is Rs.
544/- and the cost of 21 kg. of wheat is Rs. 336/-. What is the total cost of 21
kg of sugar, 26 kg of rice and 19 kg of wheat?
A. Rs. 1,451/-

B. Rs. 1,306/-
C. Rs. 1,500/-

D. Rs. 1,636/-

P a g e 369 | 580
E. None of these
Answer & Explanation :

Answer: Option A
Explanation :

Let the number be x,


13 kg sugar costs 195. So 1 kg costs 15.
17 kg rice costs 544, so 1 kg costs 32.
21 kg wheat costs 336, so 1 kg costs 16.
Hence 21 kg sugar + 26 kg rice + 19 kg wheat = (21 × 15) + (26 × 32) + (19
× 16) = 315 + 832 + 304 = Rs. 1451/-.

677. In an examination it is required to get 296 of the total maximum


aggregate marks to pass. A student gets 259 marks and is declared failed. The
difference of marks obtained by the student and that required to pass is 5%.
What are the maximum aggregate marks a student can get?
A. 680

B. 780
C. 740
D. 749

E. None of these
Answer & Explanation :
Answer: Option C
Explanation :

Let the aggregate marks be x.


Therefore, 296 - 259 = 37 will be 5% of the x.
Thus (5/100) of x = 37 => x = 740.

DIRECTIONS for the questions 678 to 682 : Study the given information
carefully to answer the given question:

P a g e 370 | 580
678. A boat takes a total time of twelve hours to travel 105 kms upstream and
the same distance downstream. The speed of the boat in still water is six times
of the speed of the current. What is the speed of the boat in still water? (In
km/hr)

A. 12
B. 30
C. 18
D. 24
E. 36

Answer & Explanation :


Answer: Option C
Explanation :
Let ‘x’ be the speed of Boat in still water, and ‘y’ be the speed of current.
Then, according to the question,

Speed of the boat in still water = 6 speed of current


x = 6y

Also given that ,


105/(x+y) +105/(x-y) =12
105/7y +105/5y =12

12y=36
y=3

Therefore, x= 6×3=18
Speed of the boat in still water= 18kmph

679. The respective ratio between numbers of bags available in store P in


August and that available in the same storein July was 5: 4. How many bags
were available in store P in August as compared to July?
A. 150

B. 90
C. 24
P a g e 371 | 580
D. 60
E. 45

Answer & Explanation :


Answer: Option A

Explanation :
Number of bags available in P store in July = 20% of 600 = 120.
Now, since we have
Number of bags available in store P in August : Number of bags available in P
store in July = 5:4

=> P : 120 = 5 : 4
Thus, number of bags available in Store P (In August) = (5/4)x120 = 150

680. In September, the total number of bags available in all the stores together
was 90 more than that available in July, What was the percent increase in the
total number of bags available in all the stores together from July to
September?

A. 10
B. 12
C. 20

D. 18
E. 15
Answer & Explanation :
Answer: Option E

Explanation :
According to the question, Total number of bags available in September =
600+90= 690

% change =690-600/600 ×100=90/6 = 15

P a g e 372 | 580
681. In July, 4/15 of the available bags in store Q remained unsold and 5/12 of
the available bags in store S remain unsold. How many bags were sold by
stores Q and S together in July?
A. 159

B. 146
C. 154
D. 168
E. 163
Answer & Explanation :

Answer: Option A
Explanation :
Bags sold by store Q = (1- 4/15) of 25% of 600 = 11/15 of 25% of 600 = 110
And bags sold by store S = (1- 5/12) of 14% of 600 = 7/12 of 14% of 600 = 49
Thus, total bags sold by stores Q and S together in July = 159

682. Two years ago, the respective ratio between A's age at that time and B's
age at that time was 5:9. A's age three years ago was 13 years less than B's
age six years ago. What is B's present age?
A. 38 years

B. 30 years
C. 34 years
D. 32 years
E. 36 years

Answer & Explanation :


Answer: Option A
Explanation :

Two year ago, ratio between A’s age and B’s age at that time = 5:9
It means A’s age at that time = 5X, B’s age at that time = 9X

A’s age 3 year ago = 5X - 1, B’s age 6 years ago = 9X - 4

P a g e 373 | 580
As given in question, 9X – 4 – (5X - 1) = 13
Solving, we get X = 4

Thus, B’s Present age = 9X+2 = 9×4+2 = 38 years

683. A box contains 2 white, 3 black and 4 red balls. In how many ways can
three balls be drawn from the box, if at least one black ball is to be included in
the draw?
A. 64
B. 32

C. 48
D. 96
E. None of these
Answer & Explanation :
Answer: Option A
Explanation :
Reqd. number of ways =( Total of 3 from 9)-( Total of 3 from 6 non
black)=9C3–6C3 = 84 – 20 = 64

684. There are 3 vacancies in a firm and 15 applicants. Find the total number of
ways of filling these vacancies.
A. 3375
B. 2730
C. 560

D. 600
E. None of these
Answer & Explanation :

Answer: Option B
Explanation :

Reqd. number of ways = 15 × 14 × 13 = 2730

P a g e 374 | 580
685. There is a natural number which becomes equal to the square of a natural
number when 100 is added to it, and to the square of another natural number
when 168 is added to it. Find the number.

A. 189
B. 69
C. 156
D. 224
E. None of these

Answer & Explanation :


Answer: Option C
Explanation :
Try by options. 3rd option is correct because 100 + 156 = 256 (square of 16)
and 168 + 156 =

324 (square of 18).


No other option is satisfying the 2nd condition.

686. In a rectangular auditorium, chairs are arranged in rows and columns. The
number of chairs in each column is more than the number of chairs in each row
by 5. If there are in all 300 chairs, find the number of chairs in each row and in
each column.
A. 25,20
B. 30,10
C. 23,18
D. None of these
Answer & Explanation :

Answer: Option D
Explanation :
Go by options. When there were 300 chairs, so product of chairs in rows and
chairs in columns

P a g e 375 | 580
should be 300.
Difference of 5 should be there between the seats in rows and columns.

So no option out of 1st 3


is satisfying the conditions. So answer is 4th option and correct values are 20
and 15.

687. 629 ÷ 9.02 – 139.996 ÷ 7.06=?


A. 75
B. 35

C. 50
D. 65
E. 25
Answer & Explanation :
Answer: Option C
Explanation :
630 ÷ 9 – 140 ÷ 7

= 70 – 20 = 50
688. On a particular day, sweets were to be equally distributed among 960
students of a school. However, on that particular day 360 students remained
absent, Hence each student present on that day got three sweets extra. Had all
960 students remained present that day, how many sweets would each, student
have got?
A. 3
B. 5
C. 7
D. 8

Answer & Explanation :


Answer: Option B

Explanation :
Let number of sweets per student = X
P a g e 376 | 580
According to the given conditions, we have
960X = 600(X+3)

X=5
So, the correct answer is option B.

689. What should come in place of the question mark (?) in the following
number series ?
9 10 24 81 340?
A. 1376

B. 1780
C. 1570
D. 1725
E. None of these
Answer & Explanation :
Answer: Option D
Explanation :

Here, the pattern follows


9 × 1 + 1 =10
10 × 2 + 4 =24

24 × 3 +9 = 81
81 × 4 + 16 = 340
Similarly, 340 × 5 +25 = 1725
So, the correct answer is option D.

690. What would be the compound interest accrued on an amount of Rs9,000/-


at the rate of 11 p.c.p.a. in two years ?

A. Rs.2088.90
B. Rs.2140.90
C. Rs.2068.50
P a g e 377 | 580
D. Rs. 2085
E. None of these

Answer & Explanation :


Answer: Option A

Explanation :
CI = A-P
CI = 9000 x(1+11/100)2- 9000=2088.90
So, the correct answer is option A.

691. What is the least number to be added to 2530 to make it a perfect square
?
A. 50
B. 65
C. 75
D. 80
E. None of these

Answer & Explanation :


Answer: Option E
Explanation :

Check for squares of the numbers which gives value around 2530, So
Square of 50 = 2500
Square of 51 = 2601
2601 - 2530 = 71 should be added.

So, the correct answer is option E(none of these).

692. The difference between 20% of a number and 45% of the same number is
125. What is 40% of that number ?
A. 186
B. 200
P a g e 378 | 580
C. 196
D. 465

E. None of these
Answer & Explanation :

Answer: Option B
Explanation :
Let no. be x, then
45% of x – 20% of x =125
So, x = 500

Therefore, 40% of 500 = 200


The correct answer is option B.

693. A train running at the speed of 108 kmph, crosses a 365 meter long
platform in 21 secs. What is the length of the train?
A. 260 meters
B. 275 meters

C. 265 meters
D. 285 meters
E. None of these

Answer & Explanation :


Answer: Option C
Explanation :
Let the length of train = X meters

Speed of train = 108kmph = 108 × 5/18 = 30m/s


Distance = Speed × time
365 + X = 30 × 21

X = 265
So, the correct answer is option C.

P a g e 379 | 580
694. In order to pass in an examination, a student is required to get 280 marks
out of the aggregate marks. Hema got 264 marks and was declared fail by 2
percent. What is the minimum passing percentage of the examination ?

A. 33%
B. 35%
C. 40%
D. 44%
E. None of these

Answer & Explanation :


Answer: Option B
Explanation :
Let maximum marks = X
Then, 2% of X = 280 - 264 = 16
=> X = 800
Then minimum passing %age = (280 /800) × 100 = 35

Therefore, the correct answer is option B.

695. he cost of 8 fans and 14 Ovens is Rs 36,520/-.What is the cost of 12 fans


and 21 Ovens?
A. Rs. 56,800/-

B. Rs. 54,780/-
C. Rs. 57,950/-

D. Can't be determined
E. None of these
Answer & Explanation :

Answer: Option B
Explanation :

Let the cost of one fan be Rs.F and the Cost of Oven be Rs.O.

P a g e 380 | 580
Then, According to the question,
8F+14O = 36520

Dividing by 2, we get, 4F + 7O = 18260


Multiply by 3, we will get, 12F + 21O = 54780.

So, the correct answer is option B

696. Radius of a circular garden is 7 meter more than length of a rectangle


whose perimeter is 364 meter and breadth is 84 meter. What will be cost of
fencing the garden (only at the circumference), if the cost of fencing is Rs 8 per
meter?
A. Rs. 5,456/-
B. Rs. 6,144/-
C. Rs. 5,296/-
D. Rs. 5,280/-

E. None of these
Answer & Explanation :

Answer: Option D
Explanation :
If the Length of rectangle = L meters

Then, according to the question,


2(L +84) = 364

=> L = 98
Radius of circle = 98 + 7 = 105 meters

Circumference of the circle = 2πr = 660


Cost of fencing = 660 × 8 = Rs 5280
Hence, the correct answer is option D.

697. If 43x + 43y = 4816, what is the average of x and y ?

A. 56

P a g e 381 | 580
B. 112
C. 62

D. 124
E. None of these

Answer & Explanation :


Answer: Option D
Explanation :
Divide the given equation by 43, we get, x + y = 112
Average = (x + y)/2 = 112/2 = 56

Hence, the correct answer is option A


698. A cricketer played 80 innings and scored an average of 99 runs. His score
in the last inning was zero run. To have an average of 100 at the end, his score
in the last innings should have been
A. 60 runs
B. 80 runs
C. 10 runs

D. 1 run
Answer & Explanation :
Answer: Option B

Explanation :
Let x be the score in the last inning to make the average of 100

Therefore, when average is 99, total score of 80 innings = 80 × 99 = 7920


And when the average is 100, total score of 80 innings = 80 × 100 = 8000

Therefore, x = 8000 – 7920 = 80

699. A man spends an average of Rs. 1,694.70 per month for the first 7 months
and Rs.1,810.50 per month for the next 5 months. His monthly salary if he
saves Rs. 3,084.60 during the whole year is

A. Rs. 1000

P a g e 382 | 580
B. Rs. 2000
C. Rs. 2400

D. Rs. 3000
Answer & Explanation :

Answer: Option B
Explanation :
Monthly Salary = [(1694.70*7)+(1810.5*5)+3084.60] / 12 = 24000/12 =
2000/-

700. A and B undertake to do a piece of work for Rs. 2,200. A alone can do it in
8 days, while B can do it in 6 days. With the help of C, they complete it in 3
days. Find C's share.
A. Rs. 150
B. Rs. 275

C. Rs. 245
D. Rs. 175

Answer & Explanation :


Answer: Option B
Explanation :

Let 1 be the total work


Therefore, C’s one day work = [1/3]-[1/8 + 1/6]= 1/24

Ratio of share of A, B and C


i.e. A:B:C = 1/8 : 1/6 :1/24 => 3 : 4 : 1

So, C’s share = 1/8*2200=275

701. By selling an article at 80% of its marked price, a trader makes a loss of
10%. What will be the profit percentage if he sells it at 95% of its marked
price?

A. 5.9
B. 12.5
P a g e 383 | 580
C. 6.9
D. 5

Answer & Explanation :


Answer: Option C

Explanation :
Let us assume the MP to be 900/-
Therefore, SP = 80% of MP = 0.80 × 900 = 720/-
As loss is 10%, so CP = 720/90 * 100=800
Now, if SP = 95% of MP = 0.95 × 900 = 855/-

Profit% = [{855-800}/800 ]*100 =6.87

702. By selling an umbrella for Rs. 30, a shopkeeper gains 20%. During a
clearance sale, the shopkeeper allows a discount of 10% of the marked price.
His gain during the sale season is
A. 8
B. 9

C. 7
D. 7.5
Answer & Explanation :

Answer: Option A
Explanation :

Given that SP of umbrella = 30/- and profit% = 20.


Therefore, CP of umbrella = 30/120 *100=25

SP of umbrella after 10% discount = 90/100 *30= 27


Thus, profit% = {(27-25) /25 }*100=8

703. The Arithmetic mean of 5, 10, 12, 18, 20 is:


A. 5
B. 65
P a g e 384 | 580
C. 13
D. 12

Answer & Explanation :


Answer: Option C

Explanation :
Arithmetic mean= 5+10+12+18+20/5=65/5=13

704. From each of two given numbers, half the smaller number is subtracted.
After such subtraction, the larger number is 4 times as large as the smaller
number. What is the ratio of the numbers?
A. 4:1
B. 4:5
C. 5:2
D. 1:4
Answer & Explanation :
Answer: Option C

Explanation :
Let x and y be the two numbers, with x > y
Now are given that x -y/2=4[y - y/2]

Solving the above equation, we get x/y = 5:2

705. What is the remainder if 281 is divided by 5?


A. 4

B. 1
C. 2
D. 3

Answer & Explanation :


Answer: Option C
Explanation :
P a g e 385 | 580
706. What is the largest number which divides 150, 180 and 144 leaving the
same remainder in each case?
A. 12

B. 3
C. 6
D. 8
Answer & Explanation :
Answer: Option C

Explanation :
HCF (180-150,180-144,150-144)

HCF(30,36,6)=6

707. In an examination, 52% of the candidates failed in English and 43% failed
in Mathematics. If 17% failed in both the subjects, then the percentage of
candidates, who passed in both the subjects, was

A. 25
B. 22
C. 23

D. 21
Answer & Explanation :

Answer: Option B
Explanation :

% of students failed in Mathematics only = 43% – 17% = 26%


% of students failed in English only = 52% – 17% = 35%.
So % of students passed in both = 100 – (26 + 35 + 17)

= 100 – 78 = 22%

P a g e 386 | 580
Alternate Solution:
% of students failed in Mathematics or English or both = 52% + 43% – 17% =
78%
So % of students passed in both = 100% – 78% = 22%. Hence, option B is
correct.
708. What would be the compound interest drawn on an amount of Rs 18,400
@ 12 p.c.p.a. at the end of 3 years?
A. Rs. 4680.96
B. Rs. 7450.6752

C. Rs. 6235.2143
D. Rs. 8042.16
D. None of these
Answer & Explanation :
Answer: Option B

Explanation :
The formula for compound interest is:

CI = P(1+R/100)t - P
The required interest :
= 18400×[ (112/100)×(112/100)×(112/100) ] - 18400

= Rs. 7450.6752

709. What should come in place of question mark (?) : 3, 5, 15, ?, 1125,
84375,

A. 75
B. 20
C. 45

D. 80
E. None of these

Answer & Explanation :

P a g e 387 | 580
Answer: Option A
Explanation :

3 × 5 = 15
5 × 15 = 75

15 × 75 = 112
75 × 1125 = 84375
Hence 75 is the answer.

710. If the digits of a two-digit number are interchanged, the number so


obtained is greater than the original number by 27. If the sum of the two digits
of the number is 11, what is the original number?
A. 47
B. 38
C. 74
D. Can't be determined
E. None of these

Answer & Explanation :


Answer: Option A
Explanation :

Let the original number be 10x + y


New number = 10y + x

Now, 10y + x - (10x + y) = 27


=> 9y - 9x = 27

We have, different of the two digits = 27 / 9 = 3


Sum of the two digits is = 11
Now, the two digits are (11+3 / 4) and (11-3 / 2) i.e. 7 and 4

Thus, the number is 47 because 47 < 74.


You can check it: 74 - 47 = 27
So the answer is A.
P a g e 388 | 580
711. 59.99% of 255.012 + 22.98% of 182.005 = ?

A. 162
B. 146

C. 195
D. 225
E. 178
Answer & Explanation :
Answer: Option C

Explanation :
59.99% of 255.012 + 22.98% of 182.005 can be written as :
~= 60% of 255 + 23% of 182
~= 153 + 41.86
~= 194.86

712. 12 × 958 ÷ 17 = ?

A. 532
B. 675
C. 765

D. 483
E. 806
Answer & Explanation :
Answer: Option B

Explanation :
12 × 958 ÷ 17 = 676

713. Wages for 40 women amount to Rs. 15000 in 60 days. How many men
must work for 15 days to receive Rs. 6000? The daily wage of a man is double
that of a woman.
P a g e 389 | 580
A. 28 men
B. 30 men

C. 32 men
D. 35 men

E. None of these
Answer & Explanation :
Answer: Option C
Explanation :
Daily wage of woman = 15000 / (60*40) = 6.25

Therefore, daily wageof man = 2 × 6.25 = 12.5


Number of men = 6000 / (15×12.5) = 32

714. The simple interest on a certain sum of money for 2 years at 8% p.a. is
Rs. 600. What is the compound interest at the same rate and for the same
time?
A. Rs. 620

B. Rs. 624
C. Rs. 625
D. Rs. 630

E. None of these
Answer & Explanation :

Answer: Option B
Explanation :

SI for 1 year will be 600 / 2 = Rs.300.


As in 1st year SI and CI are same.
So 1st year CI will also be Rs.300.

In CI, in second year 300 will also be added in principal.


So second year CI = 300 + 8% of 300 = 324.
So total CI after 2 years = 300 + 324 = Rs. 624.
P a g e 390 | 580
715. The cost of 13 kg of sugar is Rs. 195/-. The cost of 17 kg. of rice is Rs.
544/- and the cost of 21 kg. of wheat is Rs. 336/-. What is the total cost of 21
kg of sugar, 26 kg of rice and 19 kg of wheat?

A. Rs. 1,451/-
B. Rs. 1,306/-
C. Rs. 1,500/-
D. Rs. 1,636/-
E. None of these

Answer & Explanation :


Answer: Option A
Explanation :
13 kg sugar costs 195. So 1 kg costs 15.
17 kg rice costs 544, so 1 kg costs 32.
21 kg wheat costs 336, so 1 kg costs 16.
Hence 21 kg sugar + 26 kg rice + 19 kg wheat = (21 × 15) + (26 × 32) + (19
× 16)
= 315 + 832 + 304 = Rs. 1451.

716. The difference between 38% of a number and 24% of the same number is
135.10. What is 40% of that number?
A. 394
B. 370

C. 378
D. 386
E. None of these

Answer & Explanation :


Answer: Option D

Explanation :

P a g e 391 | 580
Let the number be x
Therefore, 0.38x – 0.24x = 135.10

We get x = 965
Thus, 0.40 × 965 = 386

717. In an examination it is required to get 296 of the total maximum


aggregate marks to pass. A student gets 259 marks and is declared failed. The
difference of marks obtained by the student and that required to pass is 5%.
What are the maximum aggregate marks a student can get?

A. 690
B. 780
C. 740
D. 749
E. None of these

Answer & Explanation :


Answer: Option C

Explanation :
Let the aggregate marks be x.
Therefore, 296 - 259 = 37 will be 5% of the x.

Thus (5 / 100) of x = 37 => x = 740.


718. When n is divided by 6, the remainder is 4. When 2n is divided by 6, the
remainder is
A. 2

B. 0
C. 4
D. 1

Answer & Explanation :


Answer: Option A

Explanation :

P a g e 392 | 580
When n is divided by 6, the remainder is 4. Hence,the number can be written as
n = 6k +4

hence, 2n = 12k + 8 and when 2n is divided by 6 : remainder will be ( 12 k + 8


)/6=2

So,12k gives remainder 0 with 6 and 8 gives remainder 2 with 6. So, remainder
is 0 + 2 = 2.Hence, Ans is option A.

719. The value of is 1/20 + 1/30 + 1/42 + 1/56 + 1/72 + 1/90 is


A. 1/10

B. 3/5
C. 3/20
D. 7/20
Answer & Explanation :
Answer: Option C

Explanation :
1/(4×5) + 1/(5×6) + 1/(6×7) + 1/(7×8) + 1/(8×9) + 1/(9×10)

= (1/4 - 1/5) + (1/5 - 1/6) + (1/6 - 1/7) + (1/7 - 1/8) + (1/8 - 1/9) + (1/9 -
1/10)
= 1/4 -1/10 = 3/20

So answer is option C.

720. Let a = 1/(2 - √3) + 1/(3 - √8) + 1/(4 - √15). Then we have
A. a > 18

B. a >= 18
C. a = 18
D. a = 9

Answer & Explanation :


Answer: Option A

P a g e 393 | 580
721. If part of a journey takes 10 minutes, then to complete ( 3 /
5 )th of that journey, it will take
A. 40 minutes
B. 45 minutes

C. 48 minutes
D. 36 minutes

Answer & Explanation :


Answer: Option C

Explanation :

part of journey means = 4 - 155/40 = 5/40th of the journey will take 10 mins
Hence, for the full journey it will take 80 mins
Therfore for 3/5 th of the journey,it will take = ( 3/5 ) * 80 = 48 mins
Hence, Ans is option C.

722. A reduction of 20% in the price of rice enables a customer to purchase


12.5 kg more for Rs. 800. The original price of rice per kg is
A. Rs. 14
B. Rs. 16/-
C. Rs. 12/-

D. Rs. 15/-
Answer & Explanation :
Answer: Option B
P a g e 394 | 580
Explanation :
Let the price of rice is Rs x / kg

As per the question


800/ 0.8x - 800 / x = 12.5

160/0.8x = 12.5
on solving x= 16

723. In two alloys A and B, the ratio of zinc to tin is 5 : 2 and 3 : 4 respectively.
Seven kg of the alloy A and 21 kg of the alloy B are mixed together to form a
new alloy. What will be the ratio of zinc and tin in the new alloy?
A. 2:1
B. 1:2
C. 2:3
D. 1:1
Answer & Explanation :
Answer: Option D

Explanation :
Alloy A : which weighted 7 kg has zinc & tin in ratio = 5 : 2 i.e Zinc in alloy A =
5 kg & tin in alloy A = 2kg

Alloy B : which weighted 21 kg has zinc & tin in ratio = 3 : 4 i.e Zinc in alloy A
= 9 kg & tin in alloy A = 12kg
Hence, total Zinc : total tin = 14 : 14 = 1 : 1 So, answer is option D.

724. If A : B = 3 : 4 and B : C = 6 : 5, then C : A is


A. 10;9
B. 9:10

C. 8:9
D. 9:8

Answer & Explanation :

P a g e 395 | 580
Answer: Option A
Explanation :

A : B = 3 : 4 and B : C = 6 : 5,
Then C : A will be = A : B : C = 9 : 12 : 10

Hence, ratio of C : A = 10: 9.


Hence, answer is option A.
DIRECTIONS for the questions 1 to 10: Solve the following question and mark
the best possible option.

725. If a population of yeast cells grows from 10 to 320 in a period of five


hours, what is the rate of growth?
A. It doubles its numbers every hour.
B. It triples its numbers every hour
C. It doubles its numbers every two hours.
D. It triples, its numbers every two hours
Answer & Explanation :

Answer: Option A
Explanation :
Cells grow from 10-320, so let the growth % be R per annum.

So it doubles itself in every hour.

726. The number of red blood corpuscles in one cubic millimeter is about
5,000,000, and the number of white blood corpuscles in one cubic millimeter is
about 8,000. What, then, is the ratio of white blood corpuscles to red blood
corpuscles?
A. 1:625

P a g e 396 | 580
B. 1:40
C. 4:10

D. 5:1250
Answer & Explanation :

Answer: Option A
Explanation :
8000 : 5,000,000 = 8 : 5000 =1:625.

727. Which of the following numbers can be divided evenly by 19?

A. 54
B. 63
C. 76
D. 82
Answer & Explanation :
Answer: Option C
Explanation :

76 comes in table of 19.

728. A rectangular tract of land measures 860 feet by 560 feet. Approximately
how many acres is this? (one acre = 43,560 square feet)
A. 12.8 acres
B. 11.06 acres
C. 10.5 acres

D. 8.06 acres
Answer & Explanation :
Answer: Option B

Explanation :
Area of the rectangular tract = Length x Breadth = 860 x 560 Sq.ft. =
{860x560}/43560 acres = 11.06 acres.
P a g e 397 | 580
729. On a particular morning the temperature went up 1 ° every two hours. If
the temperature was 53° at 5 A.M., at what time was it 57°?
A. 7 a.m.

B. 8 a.m
C. 12 p.m
D. 1 p.m
Answer & Explanation :
Answer: Option D

Explanation :
Difference in temperature= {57-53}= 4 degrees. 1 degrees rise happens in 2
hours , so 4 degrees rise would happen in 4 x 2 = 8 hrs, so it will be 8 hrs after
5 A.M. i.e. 1 P.M.

730. For health reasons, Amir wants to drink eight glasses of water a day. He
has already had six glasses. What fraction is Amir left with?

A. 1/8
B. 1/6
C. 1/4

D. 1/3
Answer & Explanation :
Answer: Option C
Explanation :

The fraction of water he drank = 6/8= ¾, so fraction left = 1-3/4 = 1/4.

731. A movie is scheduled for two hours. The theatre advertisements are 3.8
minutes long. There are two previews; one is 4.6 minutes long, and the other
is.2.9 minutes long. The rest of time is devoted to the feature film. How long is
the feature film?
A. 108.7 minutes

P a g e 398 | 580
B. 97.5 minutes
C. 118.98 minutes

D. 94.321 minutes
Answer & Explanation :

Answer: Option D
Explanation :
Total time apart from the feature film = { 3.8 + 4.6 + 2.9 }=11.3 minutes,
therefore time for feature film = { 120-11.3}min= 108.7 min.

732. Twelve is 20% of what number?


A. 5
B. 20
C. 60
D. 240
Answer & Explanation :
Answer: Option C

Explanation :
20% of a no. is 12, so no.= 12/20%= 60.

733. The product of two and four more than three times a number is 20.What is
the number?

A. 2
B. 16

C. 44
D. 87
Answer & Explanation :

Answer: Option A
Explanation :
The equation is 2 X {3x+4} = 20, x=2.
P a g e 399 | 580
734. Find three consecutive odd integers whose sum is 117.

A. 31, 33, 35
B. 37, 39, 41

C. 38, 39, 40
D. 39, 41, 43
Answer & Explanation :
Answer: Option B
Explanation :

Let the three integers be 2x + 1, 2x + 3 , 2x + 5. Therefore, 6x + 9 = 117, x =


18 The three integers are 37, 39, 41.
735. The average age of a man and his son is 28 years. The ratio of their ages
is 3 :1 respectively. What is the man's age?
A. 30 years
B. 38 years
C. 40 years

D. 42 years
E. None of these
Answer & Explanation :

Answer: Option D
Explanation :

Total sum of man's age & his son's age =28 × 2 = 56 Now, the Ratio of their
ages is 3 : 1.Therefore, Man's age = (3/4) × 56 = 42 So, the correct answer
is option D.
So, the correct answer is option D.

736. A basket contains 3 blue, 5 black and 3 red balls. If 3 balls are drawn at
random what is the probability that all are black?

A. 2/11
B. 1/11
P a g e 400 | 580
C. 3/11
D. 8/33

E. None of these
Answer & Explanation :

Answer: Option E
Explanation :
Ways of selecting 3 black balls out of 5 - 5C3 Total ways of selecting 3 balls -
11C3 The required probability = (5C3 / 11C3 ) = (10/165) = (2/33)

737. The ratio of the capacity to do work of A and B is 3 : 2. If they together


can complete a work in 18 days, then how long does A take to complete the
work alone?
A. 45
B. 30

C. 24
D. 40

E. None of these
Answer & Explanation :
Answer: Option A

Explanation :
Let A and B take 3x and 2x days to complete the work

1/3x+1/2x=1/18 ;x=15
So A will take 45 days.

738. How many integers are there between 300 and 600 that are divisible by 9?
A. 33

B. 31
C. 28

D. 25

P a g e 401 | 580
E. None of these
Answer & Explanation :

Answer: Option A
Explanation :

The sequence is 306,… 594 594=306+(n-1)9⇒288=(n-1)9⇒n=33

739. The sum of two numbers is 14. If the sum of their reciprocals is 7/24, find
the numbers:
A. 4,8

B. 8,12
C. 4,10
D. 6,8
E. None of these
Answer & Explanation :
Answer: Option C
Explanation :

Going by option , option number 3 and 4 will give sum = 14 but 4th option will
give l.c.m 24 and third option gives LCM as 20. It has to be the fourth option.

740. A, B, C and D are four consecutive odd numbers and their average is 42.
What is the product of B and D?

A. 1860
B. 1890

C. 1845
D. 1677
E. None of these

Answer & Explanation :


Answer: Option C

Explanation :

P a g e 402 | 580
As there are As diff. Is same so average should lie between B and C so B is 41 &
C is 43 so D must be 45 as we have to find the product of B and D so it would
be 1845.

741. The product of two alternative multiples of 4 is eight more than five times
their sum. What are the two numbers?
A. 16,32
B. 12,20
C. 8,16

D. 16,24
E. None of these
Answer & Explanation :
Answer: Option C
Explanation :

Though options, 3rd option satisfies all the conditions.

742. Find the three terms in arithmetic series such that their sum is 30 and
product is 910
A. 8,10,12

B. 7,10,13
C. 6,10,14
D. 5,13,14
E. None of these

Answer & Explanation :


Answer: Option B
Explanation :

Option 2nd gives product 910.

743. A car covers the first 30 km of its journey in 45 minutes and the remaining
25 km in 30 minutes. What is the average speed of the car?
P a g e 403 | 580
A. 40km/hr
B. 64km/hr

C. 49km/hr
D. 48km/hr

E. None of these
Answer & Explanation :
Answer: Option E
Explanation :

Average speed = = 44 km/hr So option E is the


answer.

744. What is 25% of 30% of 2/5 of 2000 ?:


A. 36
B. 40
C. 56
D. 60
E. none of these
Answer & Explanation :
Answer: Option D
Explanation :
25% of 30% of 2/5 of 2000 1/4 A— 3/10 A— 2/5 A— 2000 =60
DIRECTIONS for the questions 1 to 5: Solve the following question and mark
the best possible option.

745. A boat takes a total time of twelve hours to travel 105 kms upstream and
the same distance downstream. The speed of the boat in still water is six times

P a g e 404 | 580
of the speed of the current. What is the speed of the boat in still water? (In
km/hr)?

A. 12
B. 30

C. 18
D. 24
E. 36
Answer & Explanation :
Answer: Option C

Explanation :
Let ‘x’ be the speed of Boat in still water, and ‘y’ be the speed of current.
Then, according to the question,
Speed of the boat in still water = 6 speed of current
x = 6y
Also given that ,
105/(x+y) +105/(x-y) =12

105/7y +105/5y =12


12y=36
y=3

Therefore, x= 6×3=18
Speed of the boat in still water= 18kmph

746. At 60% of its usual speed, a train of length L metres crosses platform 240
metres long in 15 seconds. At its usual speed, the train crosses a pole in 6
seconds. What is the value of L (in metres)?
A. 440

B. 425
C. 220

D. 480

P a g e 405 | 580
E. 240
Answer & Explanation :

Answer: Option D
Explanation :

Let usual speed = S m/s.


According to the question,
60% of S= (L+240)/15
9S= L+240………… (i)
Also given that, S = L/6

L = 6S …. (ii)
From (i) & (ii)
9S= 6S+240
3S= 240
S=80 m/s
From (ii)
L= 6×80 =480 m

747. P, Q and R have a certain amount of money with themselves. Q has 50%
more than what P has, and R has 1/3rd of what Q has. If P, Q and R together
have Rs. 240 then how much money does P alone have? (in Rs.)
A. 75

B. 60
C. 120

D. 80
E. 90
Answer & Explanation :

Answer: Option D
Explanation :
Assume that P has 2x amount
P a g e 406 | 580
Therefore, Q = 3x amount (50% more than P)
And R = 3x × 1/3 = x

Ratio of P : Q : R = 2x : 3x : x = 2 : 3 : 1
Thus, P alone have = (2/6) × 240 = Rs. 80

748. A and B both start a small business with an investment of Rs. 3500 and
Rs. 5600 respectively. At the end of few months from the start of the business,
A withdrew from the business completely. If the annual profit was divided
between A and B in the respective ratio of 5 : 12, then after how many months
from the start of the business, did A leave the business?
A. Eight
B. Nine
C. Ten
D. Five

E. Four
Answer & Explanation :

Answer: Option A
Explanation :
Let A invested his money for ‘X’ months.

A:B
3500×X : 5600×12 35X : 672
As A and B have profit ratio as 5:12
Thus,35x/672 =5/12

X=8
Hence, A has invested money for 8 months

749. The difference between S.I and C.I on certain of money for 3 years at 10%
per annum is Rs. 248. Find the sum?

A. 2000
B. 8000
P a g e 407 | 580
C. 1600
D. 4000

E. None of these
Answer & Explanation :

Answer: Option B
Explanation :
Let us consider sum of money is Rs. 1000.
S.I. = 1000*10*3/100 =300
C.I. = 1000(1+10/100)*(1+10/100)*(1+10/100)-1000 =331

Difference between SI and CI = 331 – 300 = 31


Rs. 31 is difference, when sum = Rs. 1000
Rs. 248 is Difference, then sum = 1000*248/31 =8000.

DIRECTIONS for the questions 6 to 8: Solve the following question and


mark the best possible option.
X≤Y

X≥Y
X>Y
X<Y

X = Y or No relationship between X and Y

750.
i. 2x2 + 7x + 5 = 0

ii. 3y2 + 5y + 2 = 0
Answer & Explanation :
Answer: Option A

Explanation :
I. 2x2 +7x+5 = 0

P a g e 408 | 580
=> x = - 5/2, -1
II. 3y2+5y+2=0

=> y = -3/2, -1
Thus, X ≤ Y

751.
i. 2x2 -13x + 21 = 0
ii. 3y2 -14y + 15 = 0
Answer & Explanation :

Answer: Option B
Explanation :
I. 2x2 -13x+21=0
=> x = 3, 7/2
II. 3y2-14y+15=0
=> y = 3, 5/3
Hence, X ≥ Y

752.
i. 2x2 - 13x + 18 = 0

ii. y2 – 7y + 12 = 0
Answer & Explanation :
Answer: Option A
Explanation :

I. 2x2 -13x+18=0
=> x = 9/2, 2
II. y2-7y+12=0

=> y = 4,3
Thus, no relationship between X and Y

P a g e 409 | 580
753. A can do a piece of work in 20 days. He worked for 5days. After this B did
the remaining work in 3 days. How many days A and B will together take to
finish the whole work?

A. 10 days
B. 10/3 days
C. 5/3 days
D. 12 days
E. None of these

Answer & Explanation :


Answer: Option D
Explanation :
A’s one day work = 1/20 ; A’s = five day work = 5/20 = 1/4 , Remaining work
= 1- 1/4 =3/4 , B can do 3/4 of work in 3 days, B can do 1 work in 3*4/3=4
days, B’s one day work = 1/4 , A’s one day work = 1/20, (A+B)’s one day work
1/4 +1/20 = 5+1/20 = 6/20 =3/10 . So, A and B together can do the work in
10/3 days.

754. A and B can do a piece of work in 10 days, B and C in 12 days and C and A
in 15 days. If B alone works for 15 days and then joined by A and C, in how
many days will the work be finished?

A. 20 days
B. 15 days

C. 16 days
D. 18 days
E. 12 days

Answer & Explanation :


Answer: Option D
Explanation :
A + B)’s one day work = 1/10; (B + C)’s one day work =1/12 ;

P a g e 410 | 580
(C+A)’s one day work = 1/15 . 2(A + B + C)’s one day work = 1/10 + 1/12
+1/15 = 6+5+4/60 = 15/60 = 1/4 ;

A + B + C one day work =1/8 ; B’s 1 day work = (A + B + C)’s 1 days work –
(C + A)’s 1day work = 1/8 -1/15 7= 15-8/120 = 7/120; B’s 15 day work =
7/120 *15 = 7/8 Remaining work = 1-7/8 = 1/8.
This remaining is to be done by A + B + C , A + B + C does 1 work in 8 days. A
+ B + C does 1/8 of work in 8 *1/8 = 1 day.
Total time taken to finish the work = 15 + 1 = 16 days.
755. A is able to do a piece of work in 15 days and B can do the same work in
20 days. If they can work together for 4 days, what is the fraction of work left?
A. 8/15

B. 7/15
C. 11/15
D. 2/11
E. Other than those given as options
Answer & Explanation :

Answer: Option A
Explanation :
Total work done by A + B in 1 day = 1/15 + 1/(20 ) = 7/60

Work done in 4 days = 7/60 × 4 = 7/15


Therefore, fraction of work left = 1 - 7/15 = 8/15

756. Cost price of each of the articles A and B is Rs. 'X'. Article A was sold at a
profit of 10% and article B was sold at a profit of 30%. If the overall profit
earned after selling both the articles is Rs. 136/-, what is the value of 'X'?
A. Rs.340/-

B. Rs.300/-
C. Rs.360/-
D. Rs.380/-
E. Rs.320/-

P a g e 411 | 580
Answer & Explanation :
Answer: Option A

Explanation :
Given : 0.1x + 0.3x = 136 => x = 340

757. Population of a village increased by 5% from 2007 to 2008 and by 25%


from 2005 to 2009. If the population of the village was 480 in 2007, what was
its population in 2009?
A. 640

B. 610
C. 630
D. 620
E. 650
Answer & Explanation :
Answer: Option C
Explanation :

Population in 2007 = 480


In 2008 = 1.05 × 480 = 504
In 2009 = 1.25 × 504 = 630

758. 24% of 150 × 3/4= ?

A. 64
B. 40

C. 72
D. 24
E. 48

Answer & Explanation :


Answer: Option E
Explanation :
P a g e 412 | 580
24% of 150 × 3/4= x = 36 × 4/3 = 48

759. In the month of March, Hiten spent 45% of his monthly salary on paying
bill and rent. Out of the remaining salary, he invested 60% in PPF and the
remaining he deposited in bank. He deposited Rs. 15,400 in bank. If in April, he
got an increment of 10%, what was his salary in April?
A. Rs. 84,000
B. Rs. 77,000
C. Rs. 1,10,000

D. Rs. 59,000
E. Rs. 68,000
Answer & Explanation :
Answer: Option B
Explanation :

Let the initial salary of Hiten be ‘S’. Then:


(0.55 × 0.4) S = 15400 => S = 70000

After increment, Hiten’s salary = 1.1 × 70000 = 77000

760. A person covers a certain distance by travelling at a uniform speed of 120


km/h for 90 minutes. At what speed will he have to travel in order to cover the
same distance in 1 hour 20 minutes? (in km/h)
A. 135
B. 125
C. 140
D. 130
E. 145

Answer & Explanation :


Answer: Option A

Explanation :

P a g e 413 | 580
Distance = 120 × 90/60 = 180 km Speed required to cover 180 km in 1 hr 20
mins ( 4/3 hrs) = 180 × ¾ = 135 km/hr

761. In Jar A, 120 litres milk was mixed with 24 litre water. 12 litre of this
mixture was taken out and 3 litre water was added. If 27 litre of newly formed
mixture is taken out, what will be the resultant quantity of water in the jar? (In
litre)
A. 20
B. 10

C. 15
D. 30
E. 25
Answer & Explanation :
Answer: Option A

Explanation :
Ratio of milk : water in Jar A = 120 : 24 = 5 : 1

12 lts of this mixture is taken out => milk = 5/6 × 12 = 10 lts and water = 2
lts taken out
3 lts of water added = 24 – 2 + 3 = 25 lts => new ratio of milk : water

= 110 : 25 = 22 : 5
Now 27 lts of this mixture is again taken out =>
water taken out = 5/27 × 27 = 5 lts
water left = 25 – 5 = 20 lts

762. A boat, whose speed in 15 km/hr in still water goes 30 km downstream


and comes back in a total of 4 hours 30 minutes. What is the speed of the
stream? (in km/hr)
A. 10
B. 6
C. 5

P a g e 414 | 580
D. 4
E. 15

Answer & Explanation :


Answer: Option C

Explanation :
Speed of boat = 15 k/h
Let the speed of stream be ‘S’
Given : 30/(15+S) + 30/(15-S) = 9/2
Solving we get, S = 5 km/h

763. Six years ago, the ratio of the ages of Kunal and Sagar was 6 : 5. Four
years hence, the ratio of their ages will be 11:10. What is Sagar's present age?
A. 16 years
B. 18 years
C. 20 years
D. 22 years

E. 25 years
Answer & Explanation :
Answer: Option A

Explanation :
Let the ages of Kunal and Sagar be K and S respectively.
Given : (K-6)/(S-6) = 6/5 => 5K – 6S = -6 ……(i)
And: (K+4)/(S+4) = 11/10 => 10K – 11 S = 4 …..(ii)

Solving (i) & (ii) we get: S = 16 years.


764. In an election there were only two candidates. One of the candidates
secured 40% of votes and is defeated by the other candidate by 298 votes. The
total number of votes polled is
A. 1490

B. 1500

P a g e 415 | 580
C. 745
D. 1460

Answer & Explanation :


Answer: Option A

Explanation :
Let x be the total no of votes
Therefore, 40% × x + 298 = 60% × x => 20% × x = 298 => x = 1490
Alternatively we can also say if one candidate got 40 % votes then other got 60
% votes , Therefore difference is 20 % votes

20 % of votes = 298
Therefore 100 % = 1490.

765. If |2x + 3| > 8, then which of the following is true?


A. -5.5 > x > 2.5
B. 2.5 > x > -5.5
C. 5.5 > x > -2.5

D. -5.5 > x > -2.5


Answer & Explanation :
Answer: Option A

Explanation :
Either ( 2x + 3 ) > 8 or –( 2x + 3 ) > 8, so we get 2x > 5 or -2x > 11 i.e. x >
2.5 or x < -5.5 - 5.5 > x > 2.5

766. In a container there is 90 ltrs of mixture of milk and water in the ratio
11:4. What part of the mixture should be replaced with same quantity of water,
so that the ratio after replacement becomes 11:7?

A. 5/6
B. 3/11

C. 1/6

P a g e 416 | 580
D. 1/3
Answer & Explanation :

Answer: Option C
Explanation :

In 90 ltrs., the ratio of milk and water is 11:4 i.e. M = 66 ltr & W = 24 ltr
After the replacement, the ratio becomes 11:7, i.e. M = 55 ltr & W = 35 ltr
Since qty. of milk is reduced to1/6th , therefore the overall removal is also 1/6 th

767. Ram can do a certain work in 16 days and Shyam can do the same work in
10 days. If they start the work together, then how many days early can Shyam
quit, if the work is to be done in 8 days?
A. 3 days
B. 5 days
C. 6 days
D. 2 days
Answer & Explanation :

Answer: Option A
Explanation :
Ram does (1/16) × 8 = 1/2 work in 8 days, so the other half has to be done by
Shyam, for which he will need 10/2 = 5 days. So he can quit ( 8 - 5 ) = 3 days
early.

768. A jar contains 10 red marbles and 30 green ones. How many red marbles
must be added to the jar so that 60% of the marbles will be red?
A. 25
B. 30

C. 35
D. 40

Answer & Explanation :


Answer: Option C
P a g e 417 | 580
Explanation :
Let 'x' red marbles will be added to the jar Then, according to the conditions,

( 10 + x ) = 60 / 100 × ( 40 + x ) ==> x = 35

769. If a number multiplied by 25% of itself gives a number which is 200%


more than the number, then the number is
A. 12
B. 16
C. 20

D. 24
Answer & Explanation :
Answer: Option A
Explanation :
Let the no. be x.
Then x * ( 25/100 * x ) = x + 200/100 * x
==> x = 12

770. The square root of ( 7 + √35 ) ( 7 - √35 ) is


A. √14

B. √17
C. 14
D. 3√5
Answer & Explanation :

Answer: Option A

771. If √5 = 2.236, then the value of ( √5 / 2 ) - ( 10 / √5 ) + √125 is equal to

A. 5.59
B. 7.826

P a g e 418 | 580
C. 8.944
D. 10.062

Answer & Explanation :


Answer: Option B

Explanation :

772. A number X is 150 more than a second number, Y. If the sum of X and Y is
5 times Y, what is the value of Y ?
A. 50
B. 40
C. 80
D. 60
E. 70
Answer & Explanation :

Answer: Option B
Explanation :
X = 150 + Y
X + Y = 5Y

150 + 2Y = 5Y
150 = 3Y => Y = 50

P a g e 419 | 580
773. A square field has an area of 50625 m2. Find the cost of fencing around it
at Rs. 15 per metre, (in Rs.)
A. 12,500

B. 6750
C. 17,500
D. 13500
E. 16250
Answer & Explanation :

Answer: Option D
Explanation :

Area of square field = 50625 => side of the field = = 225

cost of fencing @ Rs. 15/ meter = 4 × 225 × 15 = Rs. 13500

774. The average weight of 8 person's increases by 2.5 kg when a new person
comes in place of one of them weighing 65 kg. What will be the weight of the
new person ?

A. 76
B. 76.5
C. 85
D. 80
E. 90
Answer & Explanation :
Answer: Option C

Explanation :
Increased wt. of 8 persons = 8 × 2.5 = 20 kg
=>wt. of new person = 65 + 20 = 85 kg

P a g e 420 | 580
775. 50% of a number is 18 less than two-third of that number. Find the
number ?

A. 123
B. 115

C. 119
D. 108
E. 101
Answer & Explanation :
Answer: Option D

Explanation :
Let the no. be x
Given : 50x/100 = 2x/3 – 18
½ x – 2/3 x = - 18
=> x = 108

776. Running at the same constant rate, 6 identical machines can produce a
total of 270 bottles per minute. At this rate, how many bottles could 10 such
machines produce in 4 minutes?
A. 648

B. 1800
C. 2700

D. 2000
E. 3080

Answer & Explanation :


Answer: Option B
Explanation :

Let the no. of bottles be ‘B’. Using chain rule:

P a g e 421 | 580
777. 652.84 + 482.26 + ? = 1200

A. 62.16
B. 54.18

C. 56.1
D. 64.9
E. 66.1
Answer & Explanation :
Answer: Option D

Explanation :
652.84 + 482.26 + x = 1200,=> 1135.1 + x = 1200 => x = 64.9

778. 60% of 250 -? = 75


A. 25
B. 45
C. 60

D. 75
E. 100
Answer & Explanation :

Answer: Option D
Explanation :
=>60% of 250 - x = 75
= 150 – x = 75 => x = 75

779. 681 + ? * 40 = 1161


A. 14

B. 12
C. 24

P a g e 422 | 580
D. 16
E. 8

Answer & Explanation :


Answer: Option B

Explanation :
681 + x × 40 = 1161
= 681 + 40x = 1161 => 40x = 1161 – 681 = 480
=> x = 480/40 = 12

780. Different words are formed with the help of the letters of word RELATION.
Find the number of ways in which vowels always occupy even places.
A. 292
B. 654
C. 356
D. 576
E. 8!

Answer & Explanation :


Answer: Option D
Explanation :

Vowels i.e. A, E, I, O can be arranged = 4C4 ×4! = 4! ways.

Remaining letters R, L, T, N can be arranged in = 4! ways. So the total number


of ways = 4! ×4! = 576 ways.

781. The number of ways in which 8 persons can be seated at a round table if 2
particular persons must always sit together

A. 288

P a g e 423 | 580
B. 720
C. 1440

D. 2880
E. None of these

Answer & Explanation :


Answer: Option C
Explanation :
Reqd. number of ways = 2! × (7 – 1)! = 2! × 6! = 1440
782. Simple interest on Rs. 1200 @ 13 p.c.p.a. for 'X' years is Rs. 624/-. What
is the amount on Rs. 'X+1000' at the same rate of interest for 3 years?
A. Rs. 1872/-
B. Rs. 1384/-
C. Rs. 936/-
D. Other than those given as options
E. Rs. 1404/-
Answer & Explanation :

Answer: Option D
Explanation :
624 = (1200×13×x)/100 => x = 4

Now, P = x + 1000 = 4 + 1000 = 1004


I = (1004×13×3)/100 = 392
=> amount = 1004 + 392 = 1396/-

783. A circle and rectangle have the same perimeter. The sides of the rectangle
are 18 cm and 26 cm. What will be the area of the circle? (in cm2)
A. 88

B. 1250
C. 154
D. 128
P a g e 424 | 580
E. Other than those given as options
Answer & Explanation :

Answer: Option E
Explanation :

Perimeter of rectangle = perimeter of circle = 2 (18 + 26) = 88


2Πr = 88 => r = 14cm
Area of circle = 22/7 ×14 × 14 = 616 cm2

784. 101 + 1001 + 2003 + 30005 + 9056 =?

A. 42616
B. 42166
C. 41266
D. 42156
E. 42661
Answer & Explanation :
Answer: Option B

Explanation :
=>101 + 1001 + 2003 + 30005 + 9056 = 42166.

785. 5/6th of 348 -1/8th of 232 = ?


A. 267
B. 258
C. 257

D. 261
E. 263
Answer & Explanation :

Answer: Option D
Explanation :

P a g e 425 | 580
=>5/6th of 348 -1/8th of 232 = x
=> x = 290 – 29 = 261

786. 3060 -2460 = ? × 30

A. 30
B. 50
C. 20
D. 60
E. 43

Answer & Explanation :


Answer: Option C
Explanation :
=>3060 -2460 = x * 30
=> 600 = 30x => x = 20

787. The product of the digits of a two-digit number is twice as large as the sum
of its digits. If we subtract 27 from the required number, we get a number
consisting of the same digits written in the reverse order. Find the number.
A. 36

B. 27
C. 63

D. 46
E. None of these

Answer & Explanation :


Answer: Option C
Explanation :

Go by options. 3rd option is the answer because 63 Þ product of digits = 6*3 =


18. Sum of digits = 6 + 3 = 9.

P a g e 426 | 580
Hence product of digits is twice as the sum of the digits. Also 63 – 27 = 36. So
digits are reversed.

788. The product of the digits of a two-digit number is one-third of that


number. If we add 18 to the required number we get a number consisting of the
same digits written in reverse order. Find the number.
A. 42
B. 24
C. 72

D. 27
E. None of these
Answer & Explanation :
Answer: Option B
Explanation :

Go by options 2nd option correct because 24 = 3*2*4 . Number is thrice the


product of its digits

24+18 = 42. Hense digits are reversed.

789. Jar A has 60 litres of mixture of milk and water in the respective ratio of 2:
1. Jar B which had 40 litres of mixture of milk and water was emptied into Jar
A, as a result in Jar A, the respective ratio of milk and water becomes 13 : 7.
What was the quantity of water in Jar B?
A. 8 litres
B. 15 litres
C. 22 litres
D. 7 litres

E. 1 litres
Answer & Explanation :
Answer: Option B
Explanation :

P a g e 427 | 580
Jar A has 60 Liters.
Radio between milk and water 2:1;

Quantityof milk in Jar A=2/3*60 = 40;


Quantityof waterin Jar A=1/3*60 = 20;

40 Liters of Mixture B having Milk and Water is empty;


Therefore ,total Mixture = 60 + 40 =100;
The respective Ratio of Milk and Water is 13:7;
Quantity of Milk in Jar A = 13 /20 *100 = 65;
Quantity of Waterin Jar A = 7/20 *100 = 35;

Quantity of Waterin Jar B = 35 -20 = 15 liters.

790. The sum of a series of 5 consecutive odd numbers is 195. The second
lowest number of this series is one less than the second highest number of
another series of 5 consecutive even numbers. What is 40% of the second
lowest number of the series of consecutive even numbers?
A. 16.8

B. 14.8
C. 19.4
D. 17.6

E. 13.6
Answer & Explanation :

Answer: Option E
Explanation :

The sum of the series of 5 Consecutive odd numbers is 195;


Let the Series of Consecutive odd numbers is X, X+2, X+4, X+6, X+8 ;
X+X+2+X+4+X+6+X+8 = 195 ;

5X+20 = 195;
X=35.

Series of consecutive odd numbers is 35, 37 , 39 , 41 ,43 ;

P a g e 428 | 580
According to the question ,the second lowest number of this series is less than
the ;

Second highest number of another series of 5 consecutive numbers;


Second lowest number is 37;

Second highest number of another series of 5 consecutive numbers = 37 +1 =


38 ;
Therefore , another series of 5 consecutive no 32 , 34 ,36 ,38 , 40 ;
40% of the lowest of the series of consecutive numbers = 34 *40 / 100 = 13.6.

791. The sum of the dimensions of a room (i.e. length, breadth and height) is
18 metres and its length, breadth and height are in the ratio of 3 : 2 : 1
respectively. If the room is to be painted at the rate of Rs. 15 per m2, what
would be the total cost incurred on painting only the four walls of the room (in
Rs.)?
A. 3250
B. 2445

C. 1350
D. 2210
E. 2940

Answer & Explanation :


Answer: Option C
Explanation :
Ratio of length : breadth : height = 3 : 2 : 1
Sum of dimensions of room = 18
Length = 3/6×18 = 9
Breath = 2/6×18 = 6

Height = 1/6×18 = 3
Area of four walls = 2h×(l+b) = 2×3(9+6) = 90
Total cost of painting four walls = 90×15 = 1350

P a g e 429 | 580
792. There are 3 vacancies in a firm and 15 applicants. Find the total number of
ways of filling these vacancies

A. 3375
B. 2730

C. 560
D. 600
E. None of these
Answer & Explanation :
Answer: Option B

Explanation :
Reqd. number of ways = 15 × 14 × 13 = 2730

793. The number of ways of selecting four numbers from 1 to 30 so as to


exclude every selection of four consecutive numbers is
A. 27378
B. 29465

C. 26368
D. 11448
E. None of these

Answer & Explanation :


Answer: Option A

Explanation :
Reqd. number of ways = 30C4 – 27= 27378. Here 27 are the number of ways
in which we can have 4 consecutive numbers

794. A train does a journey without stopping in 8 hours. If it had traveled 5 km


an hour faster, it would have done the journey in 6 hours 40 min. What is its
slower speed?

A. 35 km/hr
B. 25 km/hr
P a g e 430 | 580
C. 40 km/hr
D. 20 km/hr

E. 30 km/hr
Answer & Explanation :

Answer: Option B
Explanation :

795. A person sets to cover a distance of 12 km in 45 minutes. If he covers 3/4


of the distance in 2/3 of time, then what is the speed in the remaining time?
A. 16 km/hr
B. 8 km/hr
C. 12 km/hr
D. 55 km/hr
E. 10 km/hr

Answer & Explanation :


Answer: Option C
Explanation :

796. If 8 men or 12 women can do a piece of work in 25days, in how many


days, can the work be done by 6 men and 11 women working together?
A. 12 days

B. 15 days

P a g e 431 | 580
C. 9 days
D. 18 days

E. 10 days
Answer & Explanation :

Answer: Option B
Explanation :

797. Some men promised to do a job in 18 days, but 6 of them became absent
and remaining men did the job in 20 days. What is the original number of men?
A. 50 men
B. 60 men

C. 65 men
D. 70 men
E. 55 men
Answer & Explanation :
Answer: Option B

Explanation :
Let the number of men originally = M. According to the given condition M * 18
= ( M – 6) * 20 =>M = 60

798. The product of the digits of a two-digit number is one-third of that


number. If we add 18 to the required number we get a number consisting of the
same digits written in reverse order. Find the number
A. 42
B. 24
C. 72

D. 27

P a g e 432 | 580
E. None of these
Answer & Explanation :

Answer: Option A
Explanation :

Go by options. 2nd option is correct because 24 = 3 *2 *4. Number is thrice the


product of its digits.
Also 24 + 18 = 42. Hence digits are reversed.

799. Bill, Simon, and John are brothers, given Simon is the eldest. Bill is as
many years younger than one brother as he is older than the other. Simon is 7
years younger than twice the age of John. John is 5 years older than half the
age of Bill. What is the sum of the ages of Bill, Simon and John?
A. 12
B. 24

C. 48
D. 42

E. Can’t say
Answer & Explanation :
Answer: Option C

Explanation :
S - B = B - J; J = B/2 + 5;
S = 2J - 7; S = B + 10 - 7 = B-3;
J= B/2 + 5 ; 2S =B + 10;

B/2 + 5 + B + 3 = 2B;
B/2 = 8 ; B = 16 ; S = 19 ; J = 13;
So B - S - J =16 - 19 - 13 = 48;

800. Cost price of each of the articles A and B is Rs. 'X'. Article A was sold at a
profit of 10% and article B was sold at a profit of 30%. If the overall profit
earned after selling both the articles is Rs. 136/-, what is the value of 'X'?

P a g e 433 | 580
A. 2
B. 16

C. 44
D. 87

Answer & Explanation :


Answer: Option A
Explanation :
Given : 0.1x + 0.3x = 136 => x = 340

801. Population of a village increased by 5% from 2007 to 2008 and by 25%


from 2005 to 2009. If the population of the village was 480 in 2007, what was
its population in 2009?
A. 640
B. 610
C. 630
D. 620

E. 650
Answer & Explanation :
Answer: Option C

Explanation :
Population in 2007 = 480

In 2008 = 1.05 × 480 = 504


In 2009 = 1.25 × 504 = 630

802. The fourth root of 24010000 is


A. 7
B. 49

C. 490
D. 70
Answer & Explanation :
P a g e 434 | 580
Answer: Option D
Explanation :

√24010000 = 4900
Again, √4900 = 70

∴ 4√√24010000 = 70

803. The greatest 4 digit member which is a perfect square, is

A. 9999
B. 9909
C. 9801
D. 9081
Answer & Explanation :
Answer: Option C
Explanation :
The greatest four -digit perfect square will be the square of the greatest two
digit number, hence it will be 99 × 99 = 9801.

804. A piece of work can be done by Ram and Shyam in 12 days, by Shyam and
Hari in 15 days and by Hari and Ram in 20 days. Ram alone will complete the
work in
A. 30 days
B. 32 days
C. 36 days
D. 42 days

Answer & Explanation :


Answer: Option A
Explanation :

(Ram’s + Shyam’s) 1 day’s work = 1/12


(Shyam’s + Hari’s) 1 day’s work = 1/15

P a g e 435 | 580
(Hari’s + Ram’s) 1 day’s work = 1/20
Adding all three,

2 (Ram’s + Shyam’s + Hari’s 1 day’s work = 1/12 + 1/15 + 1/20


= (5+4+3)/60 = 1/5

∴ (Ram’s + Shyam’s + Hari’s) 1 day’s work = 1/10


∴Ram’s 1 day’s work

= 1/10 - 1/15 = (3-2)/30 = 1/30


∴ Ram alone will do the work in 30 days.

805. 3 men or 5 women can do a work in 12 days. How long will 6 men and 5
women take to finish the work?

A. 4 days
B. 5 days
C. 6 days
D. 7 days
Answer & Explanation :
Answer: Option A
Explanation :

3 men = 5 women
6 men + 5 women = 15 women
∴ By M1 D1 = M2 D2
5 × 12 = 15 × D2

D2 = (5 * 12)/15 = 4 days.

806. Pipe A can fill a tank in 30 mins and Pipe B can fill it in 45 mins. If both are
opened together, then after how much time must B be turned off so that the
tank gets filled in 24 mins.
A. 9 mins
B. 12 mins

P a g e 436 | 580
C. 6 mins
D. 18 mins

Answer & Explanation :


Answer: Option A

Explanation :
Work done by pipe A is 24 mins = 24/30 = 4/5
This means work left for B is 1/5. B can fill the entire tank in 45 mins. So now it
has to fill only 1/5 of it which should take 45/5 = 9 mins.

807. The difference of perimeter and diameter of a circle is X unit. The diameter
of the circle is
A. X/(π - 1) unit
B. X/(π + 1) unit
C. X/π unit
D. (X/π - 1) unit
Answer & Explanation :

Answer: Option A
Explanation :
If the diameter of the circle be d units, then

πd–d=X
⇒ d(π - 1) = X

=> d = X/(π - 1) units

808. A sphere of diameter 6 cm is dropped in a right circular cylindrical vessel


party filled with water. The diameter of the cylindrical vessel is 12 cm. If the
sphere is just completely submerged in water, then the rise of water level in the
cylindrical vessel is
A. 2 cm
B. 1 cm

C. 3 cm
P a g e 437 | 580
D. 4 cm
Answer & Explanation :

Answer: Option B

809. A shopkeeper marks his goods 20% above his cost price and gives 15%
discount on the marked price. His gain percent is
A. 5%
B. 7%
C. 2%

D. 1%
Answer & Explanation :
Answer: Option C
Explanation :
If the CP of goods be Rs. 100, then
Marked Price = Rs. 120
Hence, S.P. = (120 x 85)/100 = Rs. 102

Therefore, Gain Percent = (102 - 100)*100/100


=> Gain Percent = 2%
Hence, option C is the answer.

810. A shopkeeper earns a profit of 12% on selling a book at 10% discount on


printed price. The ratio of the cost price to printed price of the book is
A. 45 : 56

B. 50 : 61
C. 90 : 97
D. 99 : 125

Answer & Explanation :


Answer: Option A
Explanation :
P a g e 438 | 580
CP of the book = Rs. X
Printed price = Rs. Y

Therefore, (y * 90)/100 = x * 112/100 => x/y = 90/112 = 45/56.


811. A can do a work in 60 days and B can do the same work in 40 days. They
work together for 12 days and then 'A' goes away. In how many days will 'B'
finish the remaining work?
A. 16 days
B. 20 days
C. 25 days

D. 28 days
E. 24 days
Answer & Explanation :
Answer: Option B
Explanation :
Work done by A and B in 12 days is
= 12 * 5/120 = 1/2

Therefore Remaining work = 1- 1/2 = 1/2work


B does 1/40 work in one day
Therefore B does 1/2 work in 40*1/2 =20days

812. 32 49 83 151 287 559?

A. 1118
B. 979

C. 1103
D. 1120
E. None of these

Answer & Explanation :


Answer: Option C
Explanation :
P a g e 439 | 580
32 49 83 151 287 559 ?
+17 +34 +68 +136 +272 +544

+17 +34 +68 +136 +272


So answer is 559 + 544 = 1103.

813. 462 552 650 756 870 992?


A. 1040
B. 1122
C. 1132

D. 1050
E. None of the Above
Answer & Explanation :
Answer: Option B
Explanation :
462 552 650 756 870 992
+90 +98 +106 +114 +122 +130

+8 +8 +8 +8 +8
So next number is 992 + 130 = 1122.

814. The average age of a woman and her daughter is 42 years. The ratio of
their ages is 2 : 1 respectively. What is the daughter's age?
A. 28 years
B. 48 years

C. 52 years
D. 31 years
E. None of these

Answer & Explanation :


Answer: Option B

P a g e 440 | 580
Explanation :
Let the age of mother be M and that of her daughter be D Therefore, [M+D]/2
= 42 [M/D]=2/1 and Solving the abvoe equations we get D = 28 yrs.

815. The price of sugar is increased by 25%.Find by how much percent the
consumption of sugar be decreased so as not to increase the expenditure?
A. 25%
B. 40%
C. 20%

D. 30%
E. None of these
Answer & Explanation :
Answer: Option C
Explanation :
Using the formula to calculate % decrease as [R/(100+R)]x100 where R =
percentage increase in price, we get

Required % decrease in consumption = 25/125 MULTIPLIED 100 = 20%.

816. A car travels a distance of 45 km at the speed of 15 km/hr. It covers the


next 50 km of its journey at the speed of 25km/hr and the last 25 km of its
journey at the speed of 15 km/hr. What is the average speed of the car?
A. 40 km/hr
B. 24 km/hr
C. 15 km/hr
D. 18 km/hr
E. None of these

Answer & Explanation :


Answer: Option D

Explanation :

P a g e 441 | 580
We know, Average speed = Total distance travelled / Total time taken Average
= [45+50+25]/[3+2+{25/15}] = 18 kmph

817. A car travels a distance of 170 km in 2 hours partly at a speed of 100


km/h and partly at 50 km/h. The distance travelled at a speed of 50 km/h is?
A. 50 km
B. 40 km
C. 30 km
D. 60 km

E. None of these
Answer & Explanation :
Answer: Option C
Explanation :
Suppose he covers x km at 100 kmph
So he covers 170-x at 50 kmph
So {X/100}+{170-X}/50=2

Solving this equation, we get x = 140.


So he covers 30km at 50 kmph.

818. Even after reducing the marked price of a transistor by Rs. 32, a
shopkeeper makes a profit of 15%. If the cost price be Rs. 320, what
percentage of profit would he have made if he had sold the transistor at the
marked price?
A. 25%
B. 20%
C. 10%

D. 15%
E. None of these

Answer & Explanation :


Answer: Option A
P a g e 442 | 580
Explanation :
.Let x be the marked price,

So x - 32 = 320 X 1.15
x = 400.

So required value is
400 = 320 (1 + profit/100),
So profit is 25%

819. The ratio of the present ages of A and B 9 : 5. Five years earlier the ratio
of their was 2 : 1. What is the average of their present ages?
A. 20 years
B. 25 years
C. 35 years
D. 40 years
E. None of these
Answer & Explanation :

Answer: Option C
Explanation :
A/b=9/5; i.e. 5a = 9B ..........(i)

A-5/B-5 = 2/1; i.e. A-5 = 2B .........10


∴A - 2B = -5 ...................(ii)
From equation (i) and (ii), A = 45, B = 25

∴ Average =45+25/2 = 70/2 = 35.

820. 20 boys and 32 girls form a group for social work. During their
membership drive same no. of boys and girls joined the group. How many
members does the group have now, if the ratio of boys to girls is 3:4
respectively?
A. 75

P a g e 443 | 580
B. 86
C. 68

D. 82
E. None of The above

Answer & Explanation :


Answer: Option E
Explanation :
Let x be the new boys as well as girls , Therefore
[20+X]/[32+X]=3/4

Solving this we get x = 16


So total will be 36 + 48 = 84.
1. (364 + 514 - ?) ¸ 4 = 200
A. 62
B. 82
C. 78
D. 68

E. 72
Answer & Explanation :
Answer: Option C

Explanation :
(364 + 514 - x) ¸ 4 = 200
= 878 – x = 200 × 4 => x = 878 – 800 = 78.

821. In the month of March, Hiten spent 45% of his monthly salary on paying
bill and rent. Out of the remaining salary, he invested 60% in PPF and the
remaining he deposited in bank. He deposited Rs. 15,400 in bank. If in April, he
got an increment of 10%, what was his salary in April?
A. Rs. 84,000

B. Rs. 77,000

P a g e 444 | 580
C. Rs. 1,10,000
D. Rs. 59,000

E. Rs. 68,000
Answer & Explanation :

Answer: Option B
Explanation :
Let the initial salary of Hiten be ‘S’. Then:
(0.55 × 0.4) S = 15400 => S = 70000
After increment, Hiten’s salary = 1.1 × 70000 = 77000.

822. A person covers a certain distance by travelling at a uniform speed of 120


km/h for 90 minutes. At what speed will he have to travel in order to cover the
same distance in 1 hour 20 minutes? (in km/h)
A. 135
B. 125
C. 140

D. 130
E. 145
Answer & Explanation :

Answer: Option A
Explanation :

Distance = 120 × 90/60 = 180 km


Speed required to cover 180 km in 1 hr 20 mins ( 4/3 hrs) =

180 × ¾ = 135 km/hr.

823. The average weight of 8 person's increases by 2.5 kg when a new person
comes in place of one of them weighing 65 kg. What will be the weight of the
new person?

A. 76

P a g e 445 | 580
B. 76.5
C. 85

D. 80
E. 90

Answer & Explanation :


Answer: Option C
Explanation :
Increased wt. of 8 persons = 8 × 2.5 = 20 kg
=>wt. of new person = 65 + 20 = 85 kg.

824. 50% of a number is 18 less than two-third of that number. Find the
number?
A. 123
B. 115
C. 119
D. 108

E. 101
Answer & Explanation :
Answer: Option D

Explanation :
Let the no. be x
Given : 50x/100 = 2x/3 – 18
½ x – 2/3 x = - 18

=> x = 108.

825. Running at the same constant rate, 6 identical machines can produce a
total of 270 bottles per minute. At this rate, how many bottles could 10 such
machines produce in 4 minutes?

A. 648

P a g e 446 | 580
B. 1800
C. 2700

D. 2000
E. 3080

Answer & Explanation :


Answer: Option B
Explanation :
let the no of bottles be 'B'.Using chain rule:
(6*1)/270 =(10*4)/B => B = 1800.

826. A boat, whose speed in 15 km/hr in still water goes 30 km downstream


and comes back in a total of 4 hours 30 minutes. What is the speed of the
stream? (in km/hr)
A. 10
B. 6
C. 5

D. 4
E. 15
Answer & Explanation :

Answer: Option C
Explanation :

Speed of boat = 15 km /hrs .


let the speed of stream be 's'

Given : (30/ 15+s )+ (30 /15- s) = 9/2


solving we get ,s = 5 km/hrs

827. Six years ago, the ratio of the ages of Kunal and Sagar was 6 : 5. Four
years hence, the ratio of their ages will be 11:10. What is Sagar's present age?

A. 16 years

P a g e 447 | 580
B. 18 years
C. 20 years

D. 22 years
E. 25 years

Answer & Explanation :


Answer: Option A
Explanation :
let the age of kunal and sagar be k and s respectively
Given : k - 6 / s -6 =6/5 => 5k -6s = -6..........(1)

and : k+4 /s +4 = 11/10 =>10k - 11s =4..........(2)


Solving (1)&(2) we get s =16 years.

828. A number X is 150 more than a second number, Y. If the sum of X and Y is
5 times Y, what is the value of Y.
A. 50
B. 40

C. 80
D. 60
E. 70

Answer & Explanation :


Answer: Option A
Explanation :
X = 150 + Y

X + Y = 5Y
150 + 2Y = 5Y
150 = 3Y => Y = 50.

DIRECTIONS for the question 834 : Solve the following question


approximately and mark the best possible answer. You are not
expected to find the exact value.

P a g e 448 | 580
829. 39% of 129 - 43% of 97 = ?

A. 14
B. 15

C. 17
D. 9
Answer & Explanation :
Answer: Option D
Explanation :

129×39/100 = 50.3
97×43/100 = 41.7
? = 50.3-41.7 ≈ 9

830. 39% of 129 - 43% of 97 = ?


A. 14
B. 15

C. 17
D. 9
Answer & Explanation :

Answer: Option D
Explanation :
129×39/100 = 50.3
97×43/100 = 41.7

? = 50.3-41.7 ≈ 9

831. 84.99 × 18.98 ÷ 7.2 + 62.7 = ?

A. 390
B. 340

P a g e 449 | 580
C. 287
D. 240

Answer & Explanation :


Answer: Option C

Explanation :
? = 85×19 ÷ 7.2 + 62.7
= 85 × 2.64 + 62.7
= 224.3 + 62.7 = 287

832. Find the value of Square root of ( √36.1 + √28.9 ).


A. 3.4
B. 4.2
C. 4.5
D. 2.8
Answer & Explanation :
Answer: Option A

Explanation :
√36.1 = 6
√28.9 = 5.4

6 + 5.4 = 11.4
√11.4 = 3.4

833. 56.5% of 286 + 45.8% of 435 = ?

A. 85
B. 75
C. 361

D. 125
Answer & Explanation :

P a g e 450 | 580
Answer: Option C
Explanation :

286 × 56.5/100 = 161.6


435 × 45.8/100 = 199.2

? = 161.6 + 199.2 ≈ 361

834. 9.71 × 52.8 ÷ 50.8 + 83.4 = ?


A. 93.58
B. 103.1

C. 53.7
D. 108.4
Answer & Explanation :
Answer: Option A
Explanation :
? = 9.71 × 52.8 ÷ 50.8 + 83.4
= 10.1 + 83.4 = 93.5

835. A man crosses a stationary bus in 8 seconds. The same bus crosses a pole
in 4 seconds. What is the respective ratio between the speed of the bus and the
speed of the man ?

A. 9 : 2
B. 9 : 4

C. 18 : 5
D. Cannot be determined

E. None of these
Answer & Explanation :
Answer: Option E

Explanation :
Length of bus / Speed of Man = 8, Length of bus / Speed of Bus = 4
Dividing both we get Speed of bus / Speed of Man = 8/4 = 2 : 1.
P a g e 451 | 580
As this is not given in any of the options, the answer will be none of these.

836. An intelligence agency decides on a code of 2 digits selected from 0, 1, 2,


…. , 9. But the slip on which the code is hand-written allows confusion between
top and bottom, because these are indistinguishable. Thus, for example, the
code 91 could be confused with 16. How many codes are there such that there
is no possibility of any confusion?
A. 25
B. 75

C. 80
D. 95
E. None of these
Answer & Explanation :
Answer: Option C

Explanation :
Total number of two digit codes that can be formed is 10 x 10 = 100.

Out of them 0, 1, 6, 8, 9 can create confusion.


Using these five digits total number of two digit numbers that can be made is 5
x 5 = 25.

But out of these 25 numbers 00,11,88,69 and 96 will not make any confusion.
Hence answer is 100 - 25 + 5 =80

837. A nine digit number is made by using the digits 1,2,3,4,5,6,7,8,9 such that
digits can be repeated. Find the probability that this number is divisible by 4?
A. 2/91
B. 81/921

C. 2/9
D. 1/81
E. None of these
Answer & Explanation :

P a g e 452 | 580
Answer: Option C
Explanation :

Favourable cases = 18 (12, 16, 24, 28, 32, 36, 44, 48, 52, 56,
64, 68, 72, 76, 84, 88, 92, 93)

Required probability = (18 x 97)/99 =18/81 = 2/9

838. A report consists of 20 sheets each of 55 lines and each such line consist
of 65 characters. This report is retyped into sheets each of 65 lines such that
each line consists of 70 characters. The percentage reduction in number of
sheets is closest to
A. 20
B. 5
C. 30
D. 35

E. None of these
Answer & Explanation :

Answer: Option A
Explanation :
The total volume of the matter = (20 x 55 x 65).

Let the number of pages in the new format be n.


Hence the volume will be (65 x 70 x n).

Since the volume remains same we can equate the two and hence n = 15.71 =
16 sheets.

Hence percentage reduction in sheet = 4/20 = 20%.

839. Three distinct numbers are selected from 1st 100 natural numbers. The
probability that all the three numbers are divisible by both 2 and 3 is…
A. 2/1155

B. 4/1155
C. 6/47
P a g e 453 | 580
D. 56/1155
E. None of these

Answer & Explanation :


Answer: Option B

Explanation :
A number is divisible by 2 and 3 if it is divisible by 6.
There are 16 multiple of 6 till 100.
⇒ Required probability = 16C3/100C2=4/1155

840. A bag contains 7 blue balls and 5 yellow balls. If two balls are selected at
random, what is the probability that none is yellow?
A. 5/33
B. 5/22
C. 7/22
D. 7/33

E. 7/66
Answer & Explanation :
Answer: Option C
Explanation :
Probability of getting 1st blue ball = 7/12
Probability of getting 2nd blue ball = 6/11
So, probability of getting no yellow ball = 7/12 × 6/11 = 7/22

841. A die is thrown twice. What is the probability of getting a sum 7 from both
the throws?
A. 5/18
B. 1/18

C. 1/9
D. 1/6
P a g e 454 | 580
E. 5/26
Answer & Explanation :

Answer: Option D
Explanation :

Cases of getting a sum of 7 = (1,6), (2,5), (3,4), (6,1), (5,2), (4,3) = 6


Total cases = 36
Required probability = 6/36 = 1/6

DIRECTIONS for the question 842 to 844: Read the information given
below and answer the question that follows.
In a team there are 240 members (males and females). Two-third of them are
males. Fifteen percent of males are graduates. Remaining males are non-
graduates. Three-fourth of the females are graduates. Remaining females are
non-graduates.

842. What is the difference between the number of females who are non-
graduates and the number of males who are graduates?
A. 2
B. 24

C. 4
D. 116
E. 136
Answer & Explanation :
Answer: Option C
Explanation :
Males ⇒ 160

Females ⇒ 80

Males graduate ⇒ 24

Males Non-graduate ⇒ 136

Female graduate ⇒ 60
P a g e 455 | 580
Females non-graduate ⇒ 20

Difference between the number of females who are non-graduates and the
number of males who are graduates = 24 – 20 = 4.

843. What is the sum of the number of females who are graduates and the
number of males who are non-graduates?
A. 184

B. 96
C. 156
D. 84
E. 196
Answer & Explanation :
Answer: Option E
Explanation :
Sum of the number of females who are graduates and the number of males who
are non-graduates = 60 + 136 = 196

844. What is the ratio between the total number of males and the number of
females who are non-graduates?

A. 6 : 1
B. 8 : 1
C. 8 : 3

D. 5 : 2
E. 7 : 2

Answer & Explanation :


Answer: Option B
Explanation :
Total number of males = 160
Number of females who are non-graduates = 20

P a g e 456 | 580
Required ratio = 160 : 20 = 8 : 1.
Hence, second option is the answer.

845. The product of a two-digit number and a number consisting of the same
digits written in the reverse order is equal to 2,430. Find the numbers.

A. 45, 54
B. 65, 56
C. 52, 25
D. None of these
Answer & Explanation :

Answer: Option A
Explanation :
1st is the correct answer because 45 54 = 2430.

846. There is a natural number which becomes equal to the square of a natural
number when 100 is added to it, and to the square of another natural number
when 168 is added to it. Find the number.

A. 189
B. 69
C. 156

D. 224
Answer & Explanation :

Answer: Option C
Explanation :

3rd option is correct because 100 + 156 = 256 (square of 16)


and 168 + 156 = 324 (square of 18).

847. A and B are two stations 1000 km. A train starts from A at 70 km/h and
another train starts from B at same time at the rate of 30 km/h. How far from A
will they cross each other?
A. 700 km.
P a g e 457 | 580
B. 300 km
C. 70 km

D. 200 km
E. None of these

Answer & Explanation :


Answer: Option A
Explanation :
Total distance = 1000 km, Relative speed = 70 + 30 = 100 km/h.
Time after which the train will meet.1000/100 -10hours.

In 10 hour train A will cover distance = 70 * 10 = 700 km.

848. Two cyclist A and B starts from same place at the same time, one going
towards east at a rate of 20 km /h and another towards west at a rate of 10
km/h what time will they take to be 75 km apart.
A. 3 hour
B. 2.5 Hours

C. 2 Hours
D. 3.25 Hours
E. None of these

Answer & Explanation :


Answer: Option B

Explanation :
Relative speed = 20+10 = 30 km/h, Distance to be covered = 75 km Time
taken to cover that distance = 75/30 = 5/2 hour = 2.5 hour.

849. A person covers half of his journey at 30 km/hr and the remaining half at
20 km/hr. The average speed for the whole journey is
A. 18 km/hr

B. 28 km/hr

P a g e 458 | 580
C. 32 km/hr
D. 20 km/ hr

E. 24 km/hr
Answer & Explanation :

Answer: Option E

850. A person covers a certain distance by travelling at a uniform speed of 120


km/h for 90 minutes. At what speed will he have to travel in order to cover the
same distance in 1 hour 20 minutes? (in km/h)

A. 135
B. 125
C. 140
D. 130
E. 145
Answer & Explanation :
Answer: Option A

Explanation :
Distance = 120 × 90/60 = 180 km
Speed required to cover 180 km in 1 hr 20 mins

( 4/3 hrs) = 180 × ¾ = 135 km/hr

851. In Jar A, 120 litres milk was mixed with 24 litre water. 12 litre of this
mixture was taken out and 3 litre water was added. If 27 litre of newly formed
mixture is taken out, what will be the resultant quantity of water in the jar? (In
litre)
A. 20

B. 10
C. 15

D. 30
E. 25
P a g e 459 | 580
Answer & Explanation :
Answer: Option A

Explanation :
Ratio of milk : water in Jar A = 120 : 24 = 5 : 1

12 lts of this mixture is taken out => milk = 5/6 × 12 = 10


lts and water = 2 lts taken out
3 lts of water added = 24 – 2 + 3 = 25
lts => new ratio of milk : water = 110 : 25 = 22 : 5
Now 27 lts of this mixture is again taken out

=> water taken out = 5/27 × 27 = 5 lts water left = 25 – 5 = 20 lts

852. A boat, whose speed in 15 km/hr in still water goes 30 km downstream


and comes back in a total of 4 hours 30 minutes. What is the speed of the
stream? (in km/hr)
A. 10
B. 6

C. 5
D. 4
E. 15

Answer & Explanation :


Answer: Option C

Explanation :
Speed of boat = 15 k/h

Let the speed of stream be ‘S’


Given : 30/(15+S) + 30/(15-S) = 9/2
Solving we get, S = 5 km/h

853. 39% of 129 - 43% of 97 = ?


A. 14
P a g e 460 | 580
B. 15
C. 17

D. 9
Answer & Explanation :

Answer: Option D
Explanation :
129×39/100 = 50.3
97×43/100 = 41.7
? = 50.3-41.7 ≈ 9

854. 84.99 × 18.98 ÷ 7.2 + 62.7 = ?


A. 390
B. 340
C. 287
D. 240
Answer & Explanation :

Answer: Option C
Explanation :
? = 85×19 ÷ 7.2 + 62.7

= 85 × 2.64 + 62.7
= 224.3 + 62.7 = 287
855. Cost price of each of the articles A and B is Rs. X. Article A was sold at a
profit of 10% and article B was sold at a profit of 30%. If the overall profit
earned after selling both the articles is Rs. 136/-, what is the value of 'X'?
A. Rs 340
B. Rs 300

C. Rs 360
D. Rs 380
E. Rs 320
P a g e 461 | 580
Answer & Explanation :
Answer: Option A

Explanation :
Given : 0.1x + 0.3x = 136 => x = 340

856. Population of a village increased by 5% from 2007 to 2008 and by 25%


from 2005 to 2009. If the population of the village was 480 in 2007, what was
its population in 2009?
A. 640

B. 610
C. 630
D. 620
E. 650
Answer & Explanation :
Answer: Option C
Explanation :

Population in 2007 = 480


In 2008 = 1.05 × 480 = 504
In 2009 = 1.25 × 504 = 630

857. 400 + 206 × 12 = x

A. 2800
B. 6666

C. 4666
D. 2400
E. 2872

Answer & Explanation :


Answer: Option E
Explanation :
P a g e 462 | 580
400 + 206 × 12 = x
=> x = 400 + 2472 = 2872

858. 430% of 25 + 75% of 430 = ?

A. 340
B. 860
C. 516
D. 86
E. 630

Answer & Explanation :


Answer: Option A
Explanation :
430% of 25 + 75% of 430 = x 430/100 ( 25 + 75 ) => x = 430

859. There is a natural number which becomes equal to the square of a natural
number when 100 is added to it, and to the square of another natural number
when 168 is added to it. Find the number.
A. 189
B. 69

C. 156
D. 224

E. None of these
Answer & Explanation :

Answer: Option C
Explanation :
Try by options. 3rd option is correct because 100 + 156 = 256 (square of 16)
and 168 + 156 = 324 (square of 18).
No other option is satisfying the 2nd condition.

P a g e 463 | 580
860. In a rectangular auditorium, chairs are arranged in rows and columns. The
number of chairs in each column is more than the number of chairs in each row
by 5. If there are in all 300 chairs, find the number of chairs in each row and in
each column.

A. 25,20
B. 30,10
C. 23,18
D. 25,12
E. None of these

Answer & Explanation :


Answer: Option D
Explanation :
Go by options. When there were 300 chairs, so product of chairs in rows and
chairs in columns should be 300. Difference of 5 should be there between the
seats in rows and columns. So no option out of 1st 3 is satisfying the conditions.
So answer is 4th option and correct values are 20 and 15.

861. A person sets to cover a distance of 12 km in 45 minutes. If he covers of


the distance in of time, then what is the speed in the remaining time?
A. 16 km/hr
B. 8 km/hr
C. 12 km/hr
D. 55 km/hr
E. 10 km/hr
Answer & Explanation :
Answer: Option C

Explanation :
Distance already covered = = 9 km, Time spent = min = 30 min Distance left =
(12 – 9) km = 3 km, Time left = (45 – 30) min = 15 min ∴ Required speed =
3/(5/16) km/hr = 12 km/hr.

P a g e 464 | 580
862. If a train 110 metres in length passes a man walking at the rate of 6
km/hr against it in 6 seconds, It will pass another man walking at the same
speed in the same direction in time of
A. 9 1/3sec

B. 10 2/3sec
C. 8 sec
D. 6 sec
E. 7 1/3 sec
Answer & Explanation :

Answer: Option E
Explanation :
Let the speed of the train = x km/hr. Relative speed = (x + 6) km/hr = (x +
6)* 5/18 m/sec. ∴ (x + 6) * 5/18 * 6 = 110 ∴x = 60. ∴ Speed of train = 60
km/hr for 2nd person, Relative speed = (60 – 6) km/hr = 54 * 5/18 m/sec = 15
m/hr.∴Time taken to cross 2nd person = 110/15 = 22/3 = 7 1/3 sec.

863. The number of ways in which letters of the word PRAISE be arranged
A. 720
B. 610
C. 360
D. 210
E. None of these
Answer & Explanation :
Answer: Option A
Explanation :
Reqd. number of ways = 6! = 720.
864. Product of two co-prime numbers is 117. Then their L.C.M. is

A. Rs 3
B. Rs 39
C. Rs 117
P a g e 465 | 580
D. Rs 9
Answer & Explanation :

Answer: Option C
Explanation :

HCF of two prime numbers =1


∴ Product of numbers = HCF * LCM
LCM = 117

865. The diameters of two circles are the side of a square and the diagonal of
the square respectively. The ratio of the areas of the smaller circle and the
larger circle is
A. √2 : √3
B. 1 : √3
C. 1 : 2
D. 1 : 4

Answer & Explanation :


Answer: Option C
Explanation :
Side of square = x units
Diagonal of square =√2x units
Radius of smaller circle =x/2 units
Radius of larger circle =√2x/2=x/√2 units
⇒ Required ratio of areas = π(x2/4).π(x2/2)

=2:4=1:2

866. The total surface area of a sphere is 8π square unit. The volume of the
sphere is

A. (8√3)π cubic unit


B. (8√3/5)π cubic unit

P a g e 466 | 580
C. (8√2/3)π cubic unit
D. (8/3) π cubic unit

Answer & Explanation :


Answer: Option C

867. A and B together can complete; a piece of work in 12 days, B and C can do
it in 20 days and C and A can do it in 15 days. A, B and C together can
complete it in
A. 8 days

B. 10 days
C. 12 days
D. 6 days
Answer & Explanation :
Answer: Option B
Explanation :
(A+B)’s 1 day’s work = 1/12

(B+C)’s 1 day’s work = 1/20


(C+A)’s 1 day’s work = 1/15
On adding all there, 2(A+B+C)’s day’s work = (1/12)+(1/20)+(1/15)=1/5

⇒(A+B+C)’s 1 day’s work = 1/10


Hence, the work will be finished in 10 days.

868. A and B together can complete a work in 3 days. They start together. But,
after 2 days, B left the work. If the work is completed after 2 more days. B
alone could do the work in
A. 6 days
B. 8 days
C. 10 days
D. 4 days

P a g e 467 | 580
Answer & Explanation :
Answer: Option A

Explanation :
(A+B)’s 2 day’s work = 2/3

Remaining work = 1- (2/3)= 1/3


Time taken by A in 1/3 doing work = 2 days
Time taken by A in completing the work = 6 days
B’s 1 day’s work = (1/3)-(1/6)= 1/6
B alone will complete the work in 6 days

869. A does 20% less work than B. If A can complete a piece of work in 15/2
hours, then B can do it in
A. 6 hours
B. 8 hours
C. 10 hours
D. 4 hours

Answer & Explanation :


Answer: Option A
Explanation :

Efficiency of A and B = 4 : 5
Ratio of respective time = 5 : 4
⇒Time taken by B = (4/5)*(15/2)= 6 hours

870. A rational number between 3/4 and 3/8 is

A. 16/9
B. 9/16
C. 12/7

D. 7/3

P a g e 468 | 580
Answer & Explanation :
Answer: Option B

Explanation :
3/4 = (3 * 4)/(4 * 4) = 12/16 3/8 = 6/16 ⇒
6/16,7/16,8/16,9/16,10/16,11/16,12/16 ⇒ Required rational number = 9/16.

8. The number of ways in which letters of the word PRAISE be arranged


A. 500

B. 1200
C. 200
D. 700
Answer & Explanation :
Answer: Option A

Explanation :
Let the prices of school bag and shoes is 7x and 5x.
Given that 7x - 5x = 200 ⇒ 2x = 200 ⇒ x = 100.
So the price of a pair of shoes = 5x = 5 * 100 = 500

871. A sum of Rs. 300 is divided among P, Q and R in such a way that Q gets
Rs. 30 more than P and R gets Rs. 60 more than Q. The ratio of their share is
A. 3 : 2 : 5
B. 2 : 5 : 3
C. 5 : 3 : 2

D. 2 : 3 : 5
Answer & Explanation :
Answer: Option D

Explanation :
Q = P + 30 ⇒ Q – P = 30 and R – Q = 60 = 2 × 30
⇒Required ratio = 2 : 3 : 5
P a g e 469 | 580
Look : 3 – 2 = 1, 5 – 3 = 2
872. The average of nine numbers is 50. The average of first five numbers is 54
and the average of last three numbers is 52. What will be the value of sixth
number?

A. 24
B. 44
C. 30
D. 34
Answer & Explanation :

Answer: Option A
Explanation :
The sixth number = 9 * 50 – 5 * 54 – 3 * 52 = 450 – 270 – 156 = 24.

873. The average of the first nine integral multiples of 3 is


A. 15
B. 18

C. 21
D. 12
Answer & Explanation :

Answer: Option A
Explanation :

Required average = 3(1+2+3+...+9)/9 = (9 * 10)/(2*3) = 15

874. An article is sold for Rs. 300 at a profit of 20%. Had it been sold for Rs.
235, the loss percentage would have been
A. 5

B. 6
C. 16

D. 3

P a g e 470 | 580
Answer & Explanation :
Answer: Option B

Explanation :
C.P. of the article = (100/120)*130 = Rs. 250 On selling at Rs. 235, Loss
percent = (15/250)*100 = 6%

875. A box has 100 blue balls, 50 red balls, 50 black balls. 25% of blue balls
and 50% of red balls are taken away. % of black balls at present is
A. (100/3)%

B. 40%
C. 50%
D. 25%
Answer & Explanation :
Answer: Option A
Explanation :
After taking away respective balls, the number of balls in the box = 75 + 25 +
50 = 150. Number of black balls = 50. Now the percentage of black balls =
100*50/150 = 33.33%, the first option.

876. A dozen pairs of socks quoted at Rs. 180 are available at discount of 20%.
How many pairs of socks can be bought for Rs. 48?
A. 2 pairs
B. 5 pairs
C. 3 pairs
D. 4 pairs
Answer & Explanation :

Answer: Option D
Explanation :

⇒ S.P. of a dozen pairs of socks = (180* 80)/100 = Rs.144

⇒ S.P. of 1 pair of socks = 144/12 = 12


P a g e 471 | 580
⇒ No. of pairs available for Rs. 48 = 48/12 = 4.

877. The marked price of a table is Rs. 12,000. If it was sold for Rs. 10,500
after allowing a certain discount, then the rate of discount is
A. 112.5%
B. 15%
C. 17.5%

D. 10%
Answer & Explanation :
Answer: Option A
Explanation :
Discount = 12000 - 10500 = 1500.
Now the percentage discount is = 100 *1500/12000 = 12.5%
Hence first option is the answer.

878. The marked price of a radio set is Rs. 480. The shopkeeper allows a
discount of 10% and gains 8%. If no discount is allowed, his gain percent would
be
A. 18.5%
B. 20%
C. 25%
D. 18%
Answer & Explanation :
Answer: Option B

Explanation :
If the C.P. of ratio be SR. X then

If no discount is allowed,

P a g e 472 | 580
Gain = 480 – 400 = Rs. 80 Gain percent =

879. N is the foot of the perpendicular from a point P of a circle with radius 7
cm, on a diameter AB of the circle. If the length of the chord PB is 12 cm, the
distance of the point N from the point B is
A. 26/7 cm
B. 72/7 cm
C. 47/7 cm
D. 86/7 cm

Answer & Explanation :


Answer: Option B

880. Two cars are moving with speeds v1, v2 towards a crossing along two
roads. If their distances from the crossing be 40 meters and 50 metres at an
instant of time then they do not collide if their speeds are such that
A. v1 : v2 ≠ 5 : 4

B. v1 : v2 = 25 : 16
C. v1 : v2 = 16 : 25
D. v1 : v2 ≠ 4 : 5

Answer & Explanation :


Answer: Option D

Explanation :

If 40/V1= 50/V2 ,then they will collide i.e. cars will reach at the same time

P a g e 473 | 580
881. A certain distance is covered at a certain speed. If half of this distance is
covered in double the time, the ratio of the two speeds is:
A. 1:4

B. 2:1
C. 1:2
D. 4:1
Answer & Explanation :
Answer: Option D

Explanation :
If the original speed be S1 units and time = 11 units and distance = D, then
(D/2)/2t1= S2
⇒ S2 = D/4t1

and S1= D/t1

882. The sum of the dimensions of a room (i.e. length, breadth and height) is
18 metres and its length, breadth and height are in the ratio of 3 : 2 : 1
respectively. If the room is to be painted at the rate of Rs. 15 per m2, what
would be the total cost incurred on painting only the four walls of the room (in
Rs.)?
A. 3250
B. 2445
C. 1350
D. 2210

E. 2940
Answer & Explanation :
Answer: Option C
Explanation :
Ratio of length : breadth : height = 3 : 2 : 1
P a g e 474 | 580
Breath = 2/6×18 = 6
Height = 1/6×18 = 3

Area of four walls = 2h×(l+b) = 2×3(9+6) = 90


Total cost of painting four walls = 90×15 = 1350

883. B is 4/3 times as efficient as A. If A can complete 5/8th of a given task in


15 days, what fraction of the same task would remain incomplete if B works on
it independently for 10 days only?
A. 3/4

B. 2/3
C. 5/8
D. 4/9
E. 2/3
Answer & Explanation :
Answer: Option D
Explanation :

B is 4/3 times as efficient of A.


Ratio of time taken by A and B, A: B is 4:3.
A can complete 5/8th of a given task in 15 days

A can do alone his work in = 8/5× 15= 24 days


Therefore, B can do this work= 18

B works independently for 10 days only, thus work done = 10/18 = 5/9
Remaining work (incomplete) = 1-5/9 = 4/9

884. In a class, the average weight of boys is 64 kg and that of 75 girls is 70


kg. After a few days, 60% of the girls and 30% of the boys leave. What would
be the new average weight of the class (in kg)? Assume that the average
weight of the boys and the girls remain constant throughout.

A. 63
B. 66.5
P a g e 475 | 580
C. 68.5
D. 65.5

E. Can't be determined
Answer & Explanation :

Answer: Option E
Explanation :
In this question, number of boys is not mentioned .So, we can’t find new
average.

885. 382 380 374 356 302 ?


A. 212
B. 240
C. 140
D. 201
E. 158
Answer & Explanation :

Answer: Option C
Explanation :
The pattern is as follows:

382 – 2 = 380
380 – 6 = 374
374 – 18 = 356
356 – 54 = 302

302 – 162 = 140

886. 3 9 45 315 ? 31185

A. 2465
B. 2685

P a g e 476 | 580
C. 2955
D. 2835

E. 2785
Answer & Explanation :

Answer: Option D
Explanation :
The pattern is as follows:
3*3=9
9 * 5 = 45

45 * 7 = 315
315 * 9 = 2835

887. 12 14 18 26 42 ?
A. 106
B. 74
C. 92

D. 68
E. 84
Answer & Explanation :

Answer: Option B
Explanation :
The pattern is as follows:
12 + 2 = 14

14 + 4 = 18
18 + 8 = 26
26 + 16 = 42

42 + 32 = 74

P a g e 477 | 580
888. In how many ways can 3 integers be selected from the set {1, 2, 3, ……..,
37} such that sum of the three integers is an odd number?

A. 3876
B. 7638

C. 6378
D. 1938
E. 969
Answer & Explanation :
Answer: Option A

Explanation :
There are 18 even and 19 odd numbers in the given set. For sum to be odd
either all 3 numbers should be odd or 2 of them even and one odd. This is
possible in 19C3 + (18C2 × 19C1) = 3876 ways

889. There are 13 married couples, 5 single men and 7 single women in a party.
Every man shakes hand with every woman once, but no one shakes hand with
his wife. How many handshakes took place in the party?
A. 247
B. 347

C. 360
D. 191
E. 100
Answer & Explanation :
Answer: Option B
Explanation :
Any single man will have = 13 + 7 = 20 options.

Total number of handshakes by single men = 20 × 5= 100. Any married man


will have 12 + 7 = 19 options.
Total number of handshakes by married men = 19 × 13 = 247. Total number =
247 + 100 = 347.

P a g e 478 | 580
890. A and B are two towns. A car goes from A to B at a speed of 64 km/hr and
return to A at a slower speed. If its average speed for the whole journey is 56
km/hr, it returned with speed

A. 152.54 km/hr
B. 47.52 km/hr
C. 49.78 km/hr
D. 53 km/hr
E. 57 km/hr

Answer & Explanation :


Answer: Option A
Explanation :

Let the speed on the return journey be x km/hr. Then, 56 = = 7(64


+ x) = 16x or 9x = 448 ∴ x = 448/9 = 49.78 km/hr.

891. A train travels 225 km in 3.5 hours and 370 km in 5 hours. Find the
average speed of train.
A. 80 km/hr
B. 60 km/hr

C. 70 km/hr
D. 63 km/hr
E. 50 km/hr

Answer & Explanation :


Answer: Option C

Explanation :
Here, x1 = 225, x2 = 370, T1 = 3.5 and T2 = 5.
∴ Average speed of train = (X1 + X2)/(T1 + T2) = (225+370)/(3.5+5) = 70
km/hr.

P a g e 479 | 580
892. The diagonals AC and BD of a cyclic quadrilateral ABCD intersect each
other at the point P. Then, it is always true that

A. AP x BP = CP x DP
B. AP x CP = AB x CP

C. BP x AB = CD x CP
D. AP x CP = BP x DP
Answer & Explanation :
Answer: Option D
Explanation :

Here, AC and BD are chords of the circle so AP x PC = BP x PD.

893. If tan Θ = 3/4 and Θ is acute, then cosec Θ is


A. 5/4

B. 4/3
C. 4/5
D. 5/3
Answer & Explanation :

Answer: Option D
Explanation :
tan Θ=3/4

cot Θ = 4/3
cosec2Θ - cot2Θ = 1

P a g e 480 | 580
cosec Θ= √ 1+cot2Θ = √ 1+(4/3)2 = √1+16/9=√25/9=5/3

894. If O be the circum centre of a triangle PQR and ∠QOR = 110 °, ∠OPR = 25
°, then the measure of ∠PRQ is

A. 55 °
B. 60 °

C. 65 °
D. 50 °

Answer & Explanation :


Answer: Option B
Explanation :

∠QOR = 110 ° ∠OPR = 250 ° ∠QPR = 110 ° / 2 = 55 ° OR = OP ∠OPR = ∠PRO


= 25° ∠OQR = ∠ORQ = 70/2 = 35 ° PRQ = 25 ° + 35 ° = 60 °

895. A vertical stick 12 cm long casts a shadow 8 cm long on the ground. At the
same time, a tower casts a shadow 40m long on the ground. The height of the
tower is
A. 65m
B. 70m
C. 72m
D. 60m

P a g e 481 | 580
Answer & Explanation :
Answer: Option D

Explanation :
height of Tower/Length Of Stick

=Length of shadow of Tower/Length of shadow of Stick


h/12=40/8
h=40*12/8 = 60 metre

896. A, B, C, D are four points on a circle. AC and BD intersect at a point E such


that ∠BEC = 1300 and ∠ECD = 200. ∠BAC is
A. 100 °
B. 110 °
C. 120 °
D. 90 °
Answer & Explanation :

Answer: Option B
Explanation :

∠BEC = 130 °

∠DEC = 180 ° - 130 ° = 50 °

∠EDC = 180 °-50 ° -20 ° = 110 °

∠BAC = ∠EDC = 110 °

(Angle on the same arc)

P a g e 482 | 580
897. In a triangle, if three altitudes are equal, then the triangle is

A. Right
B. Isosceles

C. Obtuse
D. Equilateral
Answer & Explanation :
Answer: Option D
Explanation :

The only possible case is when the Triangle is equilateral.

898. The tops of two poles of height 24m and 36 m are connected by a wire. If
the wire makes an angle of 60 ° with the horizontal, then the length of the wire
is
A. 8 m
B. 6 √3 m

C. 6 m
D. 8√3m
Answer & Explanation :

Answer: Option D
Explanation :

DE = 36-24 = 12 metre
P a g e 483 | 580
From △ADE, Sin 60 ° = DE/AD

√3/2=12/AD

AD = 12 x 2/√3
=8√3 metre

899. The value of 1/cosecΘ-cotΘ-1/sinΘ is


A. Cosec Θ

B. tan Θ
C. 1
D. cot Θ
Answer & Explanation :
Answer: Option D
900. The sum of the squares of the digits constituting a two-digit number is 10,
and the product of the required number by the number consisting of the same
digits written in the reverse order is 403. Find the number.

A. 13
B. 31
C. 41
D. Both 1 & 2
E. None of these
Answer & Explanation :
Answer: Option D

901. The product of a two-digit number and a number consisting of the same
digits written in the reverse order is equal to 2,430. Find the numbers.
A. 45,54
B. 65,56

C. 52,25
D. 35,53

P a g e 484 | 580
E. None of these
Answer & Explanation :

Answer: Option A
Explanation :

Try by options. 1st is the correct answer because 45*54 = 2430.

902. A garrison of 750 men has provisions for 20 weeks. If at the end of 4
weeks, they are re-inforced by 450 men, how long will the provision last?
A. 8 weeks

B. 12 weeks
C. 14 weeks
D. 15 weeks
E. 10 weeks
Answer & Explanation :
Answer: Option E
Explanation :

750*20 = 750*4+1200*W =>W = 10 weeks

903. If 8 men can reap 80 hectares in 24 days, how many hectares can 36 men
reap the same field in 36 days?
A. 540 hectares

B. 450 hectares
C. 420 hectares

D. 489 hectares
E. None of these
Answer & Explanation :

Answer: Option A
Explanation :
H = 80*36/8*36/24= 540 hectares
P a g e 485 | 580
904. A person covers half of his journey at 30 km/hr and the remaining half at
20 km/hr. The average speed for the whole journey is
A. 18 km/hr

B. 28 km/hr
C. 32 km/hr
D. 20 km/ hr
E. 24 km/hr
Answer & Explanation :

Answer: Option E
Explanation :

Here, s1 = 30 and s2 = 20.therefore Average speed = =2*30*20/30+20


= 24 km/hr.

905. Rajesh covers a certain distance by bus at 16 km/hr and return back at the
starting point on a cycle at 9 km/hr. His average speed for the whole journey is
A. 13.54 km/hr
B. 11 km/hr

C. 15.52 km/hr
D. 11.52 km/hr

E. None of these
Answer & Explanation :

Answer: Option D
Explanation :

Here, s1 = 16 and s2 = 9.therefore Average speed = = 2*16*9/16*9=


11.52 km/hr.

P a g e 486 | 580
906. 430% of 25 + 75% of 430 = ?
A. 430

B. 860
C. 516

D. 86
E. 630
Answer & Explanation :
Answer: Option A
Explanation :

430% of 25 + 75% of 430 = x


430/100 ( 25 + 75 ) => x = 430

907. Approximate value of Square root of 2412 – 1592= ?


A. 320
B. 75
C. 150

D. 180
E. 210
Answer & Explanation :

Answer: Option C

908. (364 + 514 - ?)/4=200


A. 162

B. 82
C. 78
D. 68

E. 72
Answer & Explanation :

P a g e 487 | 580
Answer: Option C
Explanation :

(364 + 514 - x)/4 = 200


= 878 – x = 200 × 4 => x = 878 – 800 = 78

909. The difference between S.I and C.I on certain of money for 3 years at 10%
per annum is Rs. 248. Find the sum?
A. 2000
B. 8000

C. 16000
D. 4000
E. None of these
Answer & Explanation :
Answer: Option B
Explanation :
Let us consider sum of money is Rs. 1000. S.I. =1000*10*3/100= 300 C.I. =
1000(1+10/100)3 – 1000 = 331 Difference between SI and CI = 331 – 300 =
31 Rs. 31 is difference, when sum = Rs. 1000 Rs. 248 is Difference, then sum
=1000*248/31= 8000

910. The average score of a cricketer for 13 matches is 42 runs. If his average
score for the first 5 matches is 54, then what was his average score (in runs)
for last 8 matches?
A. 37
B. 39
C. 34.5
D. 33.5

Answer & Explanation :


Answer: Option C
Explanation :
Total Score = Average * Number of matches

P a g e 488 | 580
Total score of 13 matches = 13 × 42 = 546
Total score of first 5 matches = 5 × 54 = 270

Therefore, total score of last 8 matches = 546 - 270 = 276


Average = 276/8 = 34.5

Hence the answer is option C

911. If the graphs of the equations x + y = 0 and 5y + 7x = 24 intersect at (m,


n), then the value of m +n is
A. 2

B. 1
C. 0
D. -1
Answer & Explanation :
Answer: Option C
Explanation :
By solving the given two equations, we get the intersection point (12, - 12).

So, m = 12, n = -12


Hence, m + n = 12 – 12 = 0 So, ans. is option C.

912. A function f(x) is defined as f(x) = f(x – 2) - x(x + 2) for all the integer
values of x and f(1) + f(4) = 0. What is the value of f(1) + f(2) + f(3) + f(4) +
f(5) + f(6)?
A. 0

B. 89
C. -89
D. None of these

Answer & Explanation :


Answer: Option C

Explanation :

P a g e 489 | 580
Let S = f(1) + f(2) + f(3) + f(4) + f(5) + f(6)
As f(1) + f(4) = 0, therefore S = f(2) + f(3) + f(5) + f(6) ------ (1)

f(2) = f(0) - 8
f(3) = f(1) - 15

f(4) = f(2) - 24 = f(0) - 32


f(5) = f(3) - 35 = f(1) - 50
f(6) = f(4) - 48 = f(0) - 80
Put the above values in equation (1), we get
S = f(0) - 8 + f(1) - 15 + f(1) - 50 + f(0) - 80

S = 2(f(0) + f(1)) - 153 ------ (2)


As we already know f(1) + f(4) = 0 ⇒f(1) + f(0) - 32 = 0 ⇒f(1) + f(0) = 32
Putting this value in equation 2, we get S = 2(32) - 153 = -89
So, Ans is option C.

913. What annual payment will discharge a debt of Rs. 6,450 due in 4 years at
5% per annum simple interest?
A. 1400
B. 1500
C. 1550
D. 1600
E. None of these
Answer & Explanation :

Answer: Option B
Explanation :

Let the annual installment be rs. x


therefore (x + x*3*5/100)+(x + x*2*5/100)
+(x + x*1*5/100)+x =6450

=>115x/100+110x/100+105x/100+x=6450

P a g e 490 | 580
=>115x+110x+105x+100x=6450*100
=>430x=6450*100

x=6450*100/430=rs.1500

914. The average of the first 100 positive integers is


A. 100
B. 51
C. 50.5
D. 49.5

E. 49
Answer & Explanation :
Answer: Option C
Explanation :
n(n+1)/2
1+2+3+....+n
therefore average of these numbers=n+1/2

therefore required average


100+1/2=50.5

915. In a family, the average age of a father and a mother is 35 years. The
average age of the father, mother and their only son is 27 years. What is the
age of the son?
A. 12 years

B. 11 years
C. 10.5 years
D. 10 years

E. None of these
Answer & Explanation :
Answer: Option B
P a g e 491 | 580
Explanation :
Father+Mother=2*35=70 years

Father+Mother+Son=27*3=81 years
therefore Son's age=81-70=11 years

916. The length and breadth of a rectangle are increased by 20% and 40%
respectively. What is the percentage increase in its area?
A. 60%
B. 68%

C. 78%
D. Data inadequate
Answer & Explanation :
Answer: Option B
Explanation :
Apply the percentage formula. The percentage increase in the area will be P + Q
+ PQ / 100. So we get the answer as 20 +

40 + 20 × 40/100 = 68%. i.e. 2nd option.

917. The diameter of a circle is 21 metres. It will take how many revolutions to
cover a distance of 6.6 km?
A. 80

B. 50
C. 200

D. 100*
Answer & Explanation :
Answer: Option D

Explanation :
No. of revolutions = Distance/circumference.

Distance = 6.6 × 1000 = 6600 metres.

P a g e 492 | 580
Circumference = 2 × 22/7 × 21/2 = 66 metres.
No. of revolutions = 6600/66 = 100 revolutions. i.e. 4thoption

918. There was one mess for 30 boarders in a certain hostel. If the number of
boarders is increased by 10, the expenses of the mess were increased by `
4,000 per month, while the average expenditure per head diminished by ` 200.
Find the original monthly expenses.
A. Rs. 36,000
B. Rs. 41,000
C. Rs. 39,000

D. Rs. 48,000
Answer & Explanation :
Answer: Option A
Explanation :
Average Expenditure for 40 boarders = (x + 4000)/40. The difference = x / 30
+ (x + 4000)/40 = 200. Solving x = 36000

919. In two alloys, copper and zinc are related in the ratios of 4 : 1 and 1 : 3.
10 kg of 1st alloy, 16 kg of 2nd alloy and some of pure copper are melted
together. An alloy was obtained in which the ratio of copper to zinc was 3 : 2.
Find the weight of the new alloy.
A. 45 kg
B. 40 kg
C. 35 kg
D. 50 kg
Answer & Explanation :
Answer: Option C

Explanation :
In First alloy, Ratio of copper and zinc is 4:1.
So the ratio will be 8:2 for 10 kg.
In Second alloy, Ratio of copper and zinc is 1:3

P a g e 493 | 580
So the ratio will be 4:12 for 16 kg.
They are mixed together to get new copper and zinc ratio 12:14

Now, because of adding pure copper, resultant ratio is 3:2 and we have 12:14
which means 2x = 14. So 3x = 21.

So, 9 kgs of pure copper should get added to get 3:2 ratio.
So total addition of mixture is 12 + 14 + 9 = 35 kg.

920. An iron cube of side 10 cm is hammered into a rectangular sheet of


thickness 0-5 cm. If the sides of the sheet be in the ratio 1:5, then the sides are

A. 40 cm, 200 cm
B. 20 cm, 100 cm
C. 10 cm, 50 cm
D. None of these
Answer & Explanation :
Answer: Option B
Explanation :

10 cm has been hammered in 0.5 cm then


10 cm / 0.5 cm = 20.
So the smaller side will be 20 and the ratio of 1:5 becomes 20:100.

921. A train overtakes two persons walking along a railway track. The first one
walks at 4.5 km/h. The other one walks at 5.4 km/h. The train needs 8.4 and
8.5 sec respectively to overtake them. What is the speed of the train if both the
persons are walking in the same direction as the train?
A. 78 km/h
B. 72 km/h

C. 66 km/h
D. 81 km/h

Answer & Explanation :


Answer: Option D
P a g e 494 | 580
Explanation :
If the length (in km) and speed (in km) of the train is L and St resp. We have

L / (St – 4.5) = 8.4/3600 and


L/(St-5.4) = 8.5/3600

Thus we get two equations,


3600 L = 8.4 St – 54 and
3600 L = 8.5 St – 45.9
On Equating, we get 0.1St = 8.1St
= Speed of train = 81

922. A train 300 m long is running at a speed of 90 km/h. How many seconds
will it take to cross a 200 m long train running in the opposite direction at a
speed of 60 km/h?
A. 12
B. 36/5
C. 60

D. 20
Answer & Explanation :
Answer: Option A

Explanation :
90 km/h = 90 x 5/18 m/s = 25 m/s.

60 km/h = 60 x 5/18 m/s = 50/3 m/s


T = D / S = (300+200) / (25 + 50/3) = 12 m/s.

923. A monthly fee of a student consists of a constant part and a part which
varies according to the number of activity clubs he wishes to join. The fee for all
activity clubs is the same. A student has to pay Rs.1,075 per month, if he
enrolls in three activity clubs and Rs.950 per month, if he enrolls in two activity
clubs. The total monthly bill of three students who are enrolled in four activity
clubs each is

P a g e 495 | 580
A. Rs.2800
B. Rs.3600

C. Rs.3300
D. Rs.4800

Answer & Explanation :


Answer: Option B
Explanation :
For enrolling in three activities = Rs.1075
For enrolling in two activities = Rs.950

So increase of Rs.125 for 1 more activity.


Thus, for enrolling in four activities = Rs.1200
So for three students enrolling in four activities
= 3 x Rs 1200 = Rs 3600

924. In an election, a total of 9801 votes were polled. 126 votes were invalid.
The successful candidate got 5 votes for every 4 votes his opponent had. At
what margin did the successful candidate win his election if there were only 2
candidates?
A. 1205

B. 949
C. 1136

D. 1075
Answer & Explanation :

Answer: Option D
Explanation :
Total votes = 9801

But 126 votes were invalid.


Effective votes are 9675

So total votes are distributed as 5x + 4x = 9675

P a g e 496 | 580
9x = 9675 => x = 1075.

925. Two water taps together can fill a tank in 75/8 hours. The tap of the longer
diameter takes 10 hours less than the smaller one to fill the tank separately.
The time in which the smaller tap can fill the tank separately is
A. 25 hours
B. 10 hours
C. 15 hours
D. 15/4 hours

Answer & Explanation :


Answer: Option A
Explanation :
Two water taps together can fill a tank in 75/3 hrs.
1/(x -10) + 1/x = 8/75 => x =15/4 or 100/4
Smaller tap can fill the tank separately in 25 hours.

926. Meena builds a circular swimming pool of radius 5 m inside a circular


garden of radius 12 m. In order to compensate the area covered due to
construction of pool, she extends the radius by 'r' metres keeping the garden
still circular. What is the value of V?
A. 1/2 m
B. 2 m
C. 1 m
D. 4 m
Answer & Explanation :
Answer: Option C

927. How many kg of sugar costing Rs. 57.5 per kg should be mixed with 75 kg
of cheaper sugar costing Rs. 45 per kg so that the mixture is worth Rs. 55 per
kg?

P a g e 497 | 580
A. 350kg
B. 300kg

C. 50kg
D. 325kg

Answer & Explanation :


Answer: Option B
Explanation :
(X x 57.5 + 75 x 45) / (X + 75) = 55
x = 300 kg.

928. A metallic sphere of radius 10.5 cm is melted and then recast into small
cones each of radius 3.5 cm and height 3 cm. The number of cones thus formed
is
A. 140
B. 132
C. 112
D. 126

Answer & Explanation :


Answer: Option D
Explanation :

929. If the ratio of the diameters of two right circular cones of equal height be
3: 4, then the ratio of their volumes will be
A. 3:4
B. 9:16
P a g e 498 | 580
C. 16:9
D. 27:64

Answer & Explanation :


Answer: Option B

Explanation :

930. The sum of two numbers is equal to 15 and their arithmetic mean is 25 per
cent greater than their geometric mean. Find the numbers.
A. 5 & 10
B. 3 & 12
C. 1 & 14
D. 6 & 9
E. None of these
Answer & Explanation :
Answer: Option B
Explanation :
AM of 2 numbers is a+b/2.GM of 2 numbers is √a+b.When sum of 2
numbers is 15, their AM is 7.5. AM = 1.25 (GM)
=> GM =7.5/1.25 = 6. Hence 36 = ab. So product of 2 numbers is 36. Try by

options.2nd is the correct answer.

931. The product of the digits of a two-digit number is twice as large as the sum
of its digits. If we subtract 27 from the required number, we get a number
consisting of the same digits written in the reverse order. Find the number.

P a g e 499 | 580
A. 36
B. 27

C. 63
D. 88

E. None of these
Answer & Explanation :
Answer: Option C
Explanation :
Go by options. 3rd option is the answer because 63 => product of digits = 6*3
= 18.
Sum of digits = 6 + 3 = 9. Hence product of digits is twice as the sum of the
digits. Also 63 – 27 = 36.
So digits are reversed.

932. The average weight of 8 person's increases by 2.5 kg when a new person
comes in place of one of them weighing 65 kg. What will be the weight of the
new person?
A. 76
B. 76.5

C. 85
D. 80
E. 90
Answer & Explanation :

Answer: Option C
Explanation :
Increased wt. of 8 persons = 8 × 2.5 = 20 kg

wt. of new person = 65 + 20 = 85 kg

933. 50% of a number is 18 less than two-third of that number. Find the
number.
P a g e 500 | 580
A. 123
B. 115

C. 119
D. 108

E. 101
Answer & Explanation :
Answer: Option D
Explanation :
Let the no. be x

Given : 50x/100 = 2x/3 – 18


½ x – 2/3 x = - 18
=> x = 108

934. Running at the same constant rate, 6 identical machines can produce a
total of 270 bottles per minute. At this rate, how many bottles could 10 such
machines produce in 4 minutes?

A. 648
B. 1800
C. 2700

D. 2000
E. 3080

Answer & Explanation :


Answer: Option B

Explanation :
Let the no. of bottles be ‘B’. Using chain rule:
(6×1)/270 = (10×4)/B => B = 1800

P a g e 501 | 580
935. At 60% of its usual speed, a train of length L meters crosses platform 240
meters long in 15 seconds. At its usual speed, the train crosses a pole in 6
seconds. What is the value of L (in meters)?
A. 140

B. 225
C. 220
D. 480
E. 240
Answer & Explanation :

Answer: Option D
Explanation :
Let usual speed = S m/s.
According to the question,
60% of S=
9S= L+240………… (i)
Also given that, S = L/6

L = 6S …. (ii)
From (i) & (ii)
=> 9S= 6S+240

=> 3S= 240


S=80 m/s

From (ii)
L= 6×80 =480 m

936. Shiva gives 20% of her monthly salary to his mother, 50% of the
remaining salary he invests in an insurance scheme and PPF in the respective
ratio of 5 : 3 and the remaining he keeps in his bank account. If the sum of the
amount he gives to his mother and that he invests in PPF is Rs. 12,600, how
much is Shiva’s monthly salary?
A. Rs. 36,000
B. Rs. 64,000

P a g e 502 | 580
C. Rs. 42,000
D. Rs. 40,000

E. None of these
Answer & Explanation :

Answer: Option A
Explanation :
Let the total amount be x.
0.2x = given to mother.
0.25x= invested in insurance

0.15x= invested in ppf


0.4x= Bank account
Given, 0.2x+0.15x = 0.35x = 12600; x=36000.

937. The respective ratio of radii of two right circular cylinders (A & B) is 4 : 7.
The respective ratio of the heights of cylinders A and B is 2 : 1. What is the
respective ratio of volumes of cylinders A and B?

A. 25 :42
B. 23 : 42
C. 32 : 49

D. 30 : 49
E. 36 : 49

Answer & Explanation :


Answer: Option C

938. At present, Aanshi is five years younger to Binny. Binny’s age twenty-
years hence will be equal to twice of Aanshi’s age five years ago. What will be
Binny’s age eight year hence?
A. 42 years

B. 35 years

P a g e 503 | 580
C. 30 years
D. 48 years

E. None of these
Answer & Explanation :

Answer: Option D
Explanation :
Let age of Aanshi be A, Bunny be B.
According to equation, A= B-5
Also, B+20 = 2(B-10)

Solving these we get, B= 40. So 8 years hence his age will be 48 years.

939. A number is such that when it is multiplied by 6, it gives another number


which is s more than 168 as the original number itself is less than 168. What is
15% of the original number?
A. 8.4
B. 7.8

C. 6.6
D. 8.8
E. 7.2

Answer & Explanation :


Answer: Option E

Explanation :
Let the no. be x.

Now as given, 6x - 168= 168 – x


7x = 336; x= 48
=> 0.15x = 7.2

940. Dharma invested Rs. P for 3 years in scheme A which offered 12% p.a.
simple interest. She also invested Rs. P + 400 in scheme B which offered 10%

P a g e 504 | 580
compound interest (compounded annually), for 2 years. If the amount received
from scheme A was less than that received from scheme B, by Rs. 304, what is
the value of P?
A. Rs. 1400

B. Rs. 1000
C. Rs. 1500
D. Rs. 900
E. Rs. 1200
Answer & Explanation :

Answer: Option E

941. Percent profit earned when an article is sold for Rs. 558/- is double the
percent profit earned when the same article is sold for Rs. 504/-. If the marked
price of the article is 30% above the cost price, what is the marked price of the
article?
A. Rs. 585/-

B. Rs. 595/-
C. Rs. 624/-
D. Rs. 590/-

E. Rs. 546/-
Answer & Explanation :
Answer: Option A
Explanation :
Let Percent profit be P
558 = (1+ 2P/100)CP
504 = (1+P/100)CP

Solving these equations we get, CP=Rs.450, p=12%


Now, MP=1.3*450 = Rs.585/-.

P a g e 505 | 580
942. Dhruva gave 35% of her monthly salary to her mother. From the
remaining salary, she paid 18% towards rent and 42% she kept aside for her
monthly expenses. The remaining amount she kept in bank account. The sum of
the amount she kept in bank and that she gave to her mother was Rs. 43,920.
What was her monthly salary?
A. Rs. 80,000
B. Rs. 75,000
C. Rs. 64,000
D. Rs. 76,000

E. Rs. 72,000
Answer & Explanation :
Answer: Option E
Explanation :
Let ‘x’ be the monthly salary, then

(65/100 × 40/100)x + 35/100x = 43920


Solving, X= 72000

943. 18 litres of pure water was added to a vessel containing 80 litres of pure
milk. 49 litres of the resultant mixture was then sold and some more quantity of
pure milk and pure water was added to the vessel in the respective ratio of 2 :
1. If the resultant respective ratio of milk and water in the vessel was 4 : 1,
what was the quantity of pure milk added in the vessel? (in litres)
A. 4

B. 8
C. 10
D. 12

E. 2
Answer & Explanation :
Answer: Option E
Explanation :
80(M) + 18(W) = 98
P a g e 506 | 580
49 liters sold => 49 is left
40(M) + 9(W)

Let x be the quantity of pure milk added


Given, (40 + 2x)/(9 + x) = 4/1

Solving, x = 2

944. There are two motor cycles (A & B) of equal cost price. Motorcycle A was
sold at a profit of 14% and motorcycle B was sold for Rs. 4,290/- more than its
cost price. The net profit earned after selling both the motor cycles (A & B) is
20%. What is the cost price of each motorcycle?
A. Rs. 16,500/-
B. Rs. 16,000/-
C. Rs. 15,500/-
D. Rs. 71,500/-

E. Rs. 17,000/-
Answer & Explanation :

Answer: Option A
Explanation :
Let the cost price of each motorcycle be Rs. ‘A’. So SP of A = 1.14A and SP of B
= A + 4290. Total CP = 2A. As net profit is given to be 20% on both the
motorcycles, so we can form the equation as (2.14A + 4290 - 2A)/2A = 20%.
Solving it further, we get (0.14A + 4290)5 = 2A. Solving this equation, we get
value of A as 16,500

945. A bag contains 3 white balls and 2 black balls. Another bag contains 2
white and 4 black balls. A bag and a ball are picked at random. What is the
probability that the ball drawn is white?
A. 7/11
B. 7/30
C. 5/11
D. 7/15

P a g e 507 | 580
E. None of them
Answer & Explanation :

Answer: Option D
Explanation :

(3/5+2/6)×½,Solving, 7/15
946. The average age of a man and his son is 28 years. The ratio of their ages
is 3 : 1 respectively. What is the man's age?
A. 30 years
B. 38 years

C. 44 years
D. 42 years
E. None of these
Answer & Explanation :
Answer: Option D
Explanation :
Total sum of man's age & his son's age =28 A— 2 = 56

Now, the Ratio of their ages is 3 : 1.


Therefore, Man's age = (3/4) A— 56 = 42
So, the correct answer is option D.

947. A car manufacturing plant manufactures 96 dozen cars in eight days. How
many dozen cars will the plant manufacture in 17 day ?
A. 210

B. 224
C. 204
D. 209

E. None of these
Answer & Explanation :
Answer: Option C
P a g e 508 | 580
Explanation :
The plant manufactures 96 dozen cars in 8 days

that means, It manufactures 96/8 = 12 dozen cars/day


So, In 17 days, it will manufacture 17 A— 12 = 204 dozen cars

Therefore, the answer is option C.

948. A & B together can complete a piece of work in 16 days, B alone can
complete the same work in 24 days. In how many days can A alone complete
the same work ?

A. 34 days
B. 50 days
C. 48 days
D. 42 days
E. None of these
Answer & Explanation :
Answer: Option C

Explanation :
Let A alone can complete the work in x days and B alone can complete the work
in 24days.

Therefore, according to the given conditions,


1/x + 1/24 = 1/16

1/x = 1/48
So, A's one day work is 1/48 which means that A will complete the entire work
in 48days.
Therefore, the answer is option C.

949. Find the average of the following set of scores


142, 93, 102, 206, 115, 98

A. 122

P a g e 509 | 580
B. 106
C. 138

D. 117
E. None of these

Answer & Explanation :


Answer: Option E
Explanation :
Adding all, we get 756
So, average = 756/6 = 126

So, the correct answer is option E (none of these).

950. The average of four consecutive odd numbers P, Q, R and S respectively


(in increasing order) is 104. What is the sum of P & S?
A. 204
B. 208
C. 206

D. 212
E. None of these
Answer & Explanation :

Answer: Option B
Explanation :
Let the four consecutive odd numbers P, Q, R and S be X, X+2, X+4 and X+6
respectively.

So, average = X+3 =104


=> X = 101
So P = 101 & S = 107

And Sum of S and P will be 208.


Hence, the answer is option B.

P a g e 510 | 580
951. If cos Θ + sin Θ = √2cos Θ, then cos Θ - sin Θ is
A. -√2 sinΘ

B. √2 sinΘ
C. √2 tanΘ

D. -√2 cosΘ
Answer & Explanation :
Answer: Option B
Explanation :
cosΘ + sinΘ = √2 cosΘ on squaring both sides,

cos2Θ + sin2Θ + 2 cosΘ. sinΘ =2cos2Θ


=> cos2Θ - sin2Θ = 2sinΘ, cosΘ
=> (cosΘ + sinΘ) (cosΘ - sinΘ)
= 2 sinΘ, cosΘ
=> (2 sinΘcosΘ)/ √2 cosΘ = √2 sinΘ

952. Number of students institute A & B were in the ration of 7: 15 respectively


in 2012. In 2013 the no. of students in institute A increased by 25% and the no.
of institute B institute B increased by 26%, then what was the respective ratio
between no. of students in institutes A & B respectively in 2013?

A. 25:56
B. 24:55

C. 24:53
D. 25:53

E. 25:54
Answer & Explanation :
Answer: Option E

Explanation :
Let the students in institute A and B 7x,15x

In 2013

P a g e 511 | 580
Students in A = 5/4(7x)=35x/4
Students in B = 126/100(15x)

Required. Ratio = (35x)100/4(126)(15x) = 25/54


Hence , Option 5

953. An employer pays Rs.26/- for each day a worker works and forfeits Rs. 7/-
for each day he is idle. All the end of 56 days. If the worker got Rs. 829/- for
how many days did the worker remains idle?
A. 21

B. 15
C. 19
D. 13
E. 17
Answer & Explanation :
Answer: Option C
Explanation :

Say he works for x days


26x-7 (56-x)=829
33x=1221

x =37
As he works for 37 days so he is idle for 19 days

Hence, Option 3
954. A right pyramid stands on a square base of side10 cm. If the height of the
pyramid is 12 cm, the area (in cm2) of its slant surface is
A. 520
B. 420

C. 360
D. 260

Answer & Explanation :

P a g e 512 | 580
Answer: Option D

955. If each interior angle of a regular polygon is 150°, the number of sides of
the polygon is

A. 8
B. 10
C. 15
D. None of these
Answer & Explanation :

Answer: Option D
Explanation :
Each interior angle in a regular polygon = (n-2) x 180/n
where, n is the no of sides.
Here, (n-2) x 180/n = 150
=> 180n - 360 = 150n
=> 30n = 360

=> n = 12
So, Ans. is option D.

956. If the Altitude of a right prism is 10 cm and its base is an equilateral


triangle of side 12 cm, then its total surface area ( in cm2 ) is

A. 5 + 3√3
B. 36√3

C. 360
D. 72( 5 + √3 )
Answer & Explanation :

Answer: Option D

957. The value of tan 10° tan 15° tan 75° tan 80° is
P a g e 513 | 580
A. 0
B. 1

C. -1
D. 2

Answer & Explanation :


Answer: Option B
Explanation :
tan10 tan15 tan75 tan80
we know that tan10 = tan(90 - 80) = cot 80o

and tan15 = tan( 90 - 75) = cot 75o


Therefore,tan10 tan15 tan75 tan80
= cot80 cot75 tan75 tan80
=1/tan80 x 1/tan75 x tan80 =1 =so,Answer is option B

958. If sin 7x = cos11x, then the value of tan 9x + cot 9x is


A. 1

B. 2
C. 3
D. 4

Answer & Explanation :


Answer: Option B
Explanation :
sin 7x = cos 4x

=>sin (9x-2x) = cos(9x-2x)


=>sin 9x.cos 2x - cos9x.sin2x =cos9x.cos2x -sin 9x.sin 2x
=>sin 9x.cos 2x + sin 9x.sin 2x =cos9x.cos2x +cos9x.sin2x

=>sin 9x.(cos 2x + sin 2x) = cos 9x.(cos 2x +sin 2x)


=>sin 9x/cos 9x = 1

P a g e 514 | 580
=>tan 9x = 1
Now,tan 9x+ cot 9x

= tan 9x + 1 /tan 9x
= 1 + 1/1 = 2

959. A person sets to cover a distance of 12 km in 45 minutes. If he covers of


the distance in of time, then what is the speed in the remaining time?
A. 16 km/hr
B. 8 km/hr

C. 12 km/hr
D. 55 km/hr
E. 10 km/hr
Answer & Explanation :
Answer: Option C
Explanation :
Distance already covered = 3/2*12 = 9 km, Time spent = 2/3*45 min = 30 min

Distance left = (12 – 9) km = 3 km, Time left = (45 – 30) min = 15 min
Therfore, Required speed = 3/ 15/60 km/hr = 12 km/hr.

960. If a train 110 metres in length passes a man walking at the rate of 6
km/hr against it in 6 seconds, it will pass another man walking at the same
speed in the same direction in time of
A. 91/3sec

B. 101/3sec
C. 8 sec
D. 6 sec

E. 7 1/3sec
Answer & Explanation :

Answer: Option E

P a g e 515 | 580
Explanation :
Let the speed of the train = x km/hr. Relative speed = (x + 6) km/hr = (x + 6)
X 15/8 m/sec
(x + 6)x 5/18x6 = 110, x = 60. Speed of train = 60 km/hr for 2nd person,

Relative speed = (60 – 6) km/hr = 54x5/18 m/sec = 15 m/hr. Time taken to


cross 2nd person =110/15=22/3= 7 1/3 sec.
961. The difference of two numbers is 11 and one fifth of their sum is 9. The
numbers are :
A. 31,20

B. 30,19
C. 29,18
D. 28,17
Answer & Explanation :
Answer: Option D

Explanation :
x − y = 11, x + y = 5 × 9 x − y = 11, x + y = 45, y = 17, x = 28

962. How many numbers between 1 and 100 are divisible by 7 ?


A. 9

B. 11
C. 17

D. 14
Answer & Explanation :

Answer: Option D
Explanation :
No. of divisible by 7 7, 14 --------- 98, n = a + (N - 1)d

98 = 7 + (N - 1) 7, 98 = 7 + 7N - 7
98/7= N = 14

P a g e 516 | 580
963. Three cubes of edges 6 cms, 8 cms and 10 cms are meted without loss of
metal into a single cube. The edge of the new cube will be:

A. 8 cms
B. 12 cms

C. 14 cms
D. 16 cms
Answer & Explanation :
Answer: Option B

964. If 378 coins consist of rupee, 50 paise and 25 paise coins, whose values
are proportional to 13 :11 : 7, the number of 50 paise coins will be :
A. 128
B. 132
C. 133
D. 136
Answer & Explanation :

Answer: Option B
Explanation :
If values are proportional to 13 : 11 : 7, then the number of coins will be
proportional to 13/1 : 11/0.50 : 7/0.25 ⇒ 13 : 22 : 28. Now from this the
number of coins of 50 paise will be 378 × 22/63 = 132.

965. In how many ways can 3 integers be selected from the set {1, 2, 3, ……..,
37} such that sum of the three integers is an odd number?

A. 3876
B. 7638
C. 6378
D. 1938
E. 969

Answer & Explanation :


P a g e 517 | 580
Answer: Option A
Explanation :

There are 18 even and 19 odd numbers in the given set. For sum to be odd
either all 3 numbers should be odd or 2 of them even and one odd. This is
possible in 19C3 + (18C2 × 19C1) = 3876 ways

966. The mean daily profit made by a shopkeeper in a month of 30 days was
Rs. 350. If the mean profit for the first fifteen days was Rs. 275, then the mean
profit for the last 15 days would be

A. Rs. 200
B. Rs. 350
C. Rs. 275
D. Rs. 425
Answer & Explanation :

Answer: Option D
Explanation :

Average would be : 350 = (275 + x)/2 On solving, x = 425.

967. There were 35 students in a hostel. If the number of students increases by


7, the expenses of the mess increase by Rs. 42 per day while the average
expenditure per head diminishes by Re 1. Find the original expenditure of the
mess.
A. Rs. 480
B. Rs. 520
C. Rs. 420
D. Rs. 460

Answer & Explanation :


Answer: Option C
Explanation :
Let d be the average daily expenditure

P a g e 518 | 580
Original expenditure = 35 × d
New expenditure = 35 × d + 42

New average expenditure will be :


(35 × d + 42)/42 = d - 1

On solving, we get d = 12
Therefore original expenditure = 35 × 12 = 420

968. Six years ago, the ratio of the ages of Kunal and Sagar was 6 : 5. Four
years hence, the ratio of their ages will be 11:10. What is Sagar's present age?

A. 16 years
B. 18 years
C. 20 years
D. 22 years
E. 25 years
Answer & Explanation :
Answer: Option A

Explanation :
Let the ages of Kunal and Sagar be K and S respectively.
Given : (K-6)/(S-6) = 6/5 => 5K – 6S = -6 ……(i)

And: (K+4)/(S+4) = 11/10 => 10K – 11 S = 4 …..(ii)


Solving (i) & (ii) we get: S = 16 years.

969. A number X is 150 more than a second number, Y. If the sum of X and Y is
5 times Y, what is the value of Y?
A. 50
B. 40

C. 80
D. 60
E. 70
P a g e 519 | 580
Answer & Explanation :
Answer: Option A

Explanation :
X = 150 + Y

X + Y = 5Y
150 + 2Y = 5Y
150 = 3Y => Y = 50
970. 4/9 of 3/16 of 6/13 of ? = 155
A. 4420

B. 4240
C. 4320
D. 4030
E. 4120
Answer & Explanation :
Answer: Option D
Explanation :

971. 156.25 × 12.4 + 1.8 × 52.5 = ? – 175.85


A. 2124.5
B. 2212.6

P a g e 520 | 580
C. 2207.85
D. 2684.8

E. 2624.4
Answer & Explanation :

Answer: Option C
Explanation :
156.25×12.4+1.8×52.5=?-175.85
1937.5+94.5=?-175.85
2032=?-175.85

?=2207.85
Hence, Option C.

972.
A. 35 1/5
B. 35 5/8
C. 31 3/8

D. 32 2/5
E. 33 1/8
Answer & Explanation :
Answer: Option C
Explanation :

P a g e 521 | 580
DIRECTIONS for the questions 973 to 978 : Solve the following question
and mark the best possible option.

973. S mixed 36 kg of sugar @ Rs. 45/- per kg and 24 kg of sugar @ Rs. 48/-
per kg and sold the mixture as to earn 20% profit. At what rate per kg must he
sell the sugar.
A. Rs. 52.56
B. Rs. 52.42

C. Rs. 52.36
D. Rs. 55.44
E. Rs. 54.25
Answer & Explanation :
Answer: Option D

Explanation :
Total cost price=36×45+24×48=2772/-

Average cost price per kg = 2772/60 = Rs. 46.2


S.P=Rs 55.44 per kg( @20% profit)

974. The length of a rectangle exceeds its breadth by 6m. Length of the
rectangle is equal to the side of a square whose area is 729 sq. m. What is the
perimeter of the rectangle? (in m)
A. 96
B. 108
C. 92
D. 88

E. 84
Answer & Explanation :

Answer: Option A
Explanation :
P a g e 522 | 580
Let ‘l’ be the length, ‘b’ be the breadth of the rectangle.
Length of the rectangle =Side of the square of area 729m2

∴ Length of the rectangle =27m. A.T.Q. l-b=6

⇒ b = l - 6 = 21m ⇒ l = 27m

Perimeter of the rectangle = 2 (l + b) = 2 (27 + 21) = 96m


Hence, Option A.

975. Raghuvir purchased 10 calculators and 16 watches for Rs. 56,100/- and
sold them so as to earn an overall profit of 20%. At what total price should he
sell 15 calculators and 24 watches together so as to earn the same percent
profit?
A. Rs. 1,00,980/-
B. Rs. 1,16,176/-

C. Rs. 1,21,176/-
D. Rs. 1,00,660/-
E. Rs. 1,24,132/-
Answer & Explanation :
Answer: Option A

Explanation :
CP of 10 calculators , 16 watches =Rs.56100/-
CP of 15 calculators, 24 watches=1.5×56100=84150/-
Profit=20%
∴S.P. of 15 calculators, 24 watches =1.2×84150 = Rs. 100,980/-

Hence, Option A.

976. Some chocolates were distributed among 4 friends A, B, C and D such that
respective ratio of chocolates received by A to chocolates received by C was
7:9. B received 29 more chocolates than A and D received 33 more chocolates
than C. If B received 15 more chocolates than C, how many chocolates did D
receive?

P a g e 523 | 580
A. 84
B. 96

C. 72
D. 99

E. 87
Answer & Explanation :
Answer: Option B
Explanation :
A=7x, C=9x, B=7x+29, D=9x+33

A.T.Q. 29 + 7x =15+9x
⇒x=7
∴ D received 96 chocolates
Hence, Option B.

977. Abhijit invested an amount with company X for two years @ simple
interest rate 15 p.c.p.a. The entire amount obtained from company X after two
years he invested with company Y @ compound interest rate 12 p.c.p.a for two
years. If the amount finally received by him was Rs. 81,536. Find the money
invested by abhijit in company X.
A. Rs. 65,000/-
B. Rs. 60,000/-
C. Rs. 56,000/-
D. Rs. 50,000/-
E. Rs. 45,000/-
Answer & Explanation :
Answer: Option D
Explanation :

Let ‘y’ be the invested amount


A.T.Q.

P a g e 524 | 580
1.3y(1.12)2=81536
On Solving y=Rs.50,000/-

∴Hence, Option D.

978. Average weight of 40 students in a class is 55 kg. Six of them whose


average weight is 52 kg left the class and another set of six students, whose
average weight is 42kg joined the class. What is the next average weight of the
class (in kg?)
A. 54.25
B. 52.75
C. 53.5
D. 54
E. 53
Answer & Explanation :
Answer: Option C

Explanation :
New Average weight = (40 * 55 - 60)/40 = 53.5 kg
Hence, Option C.

978. The ratio of roses and lilies in a garden is 3: 2 respectively. The average
number of roses & lilies is 180. What is the number of lilies in the garden?
A. 144

B. 360
C. 182

D. 216
E. None
Answer & Explanation :
Answer: Option A
Explanation :

P a g e 525 | 580
Let the number of so be ‘3x’ and the number of lilies be 2x
Average number of roses and lilies = (3x + 2x)/2 = 2.5x

A.T.Q
2.5x = 180, x = 72

Number of lilies = 2x = 144. Hence, Option A


979. Salaries of A, B and C are in the ratio 4 : 5 : 9. If their salaries are
increased by 25%, 10% and 50% respectively, what will be the new ratio of
their salaries?
A. 6:5:13

B. 10:11:27
C. 5:6:15
D. 11:20:27
E. None of these
Answer & Explanation :
Answer: Option B
Explanation :

Let their original salaries be 4k, 5k and 9k.


Now, new salaries are 1.25 4k : 1.15k : 1.59k = 5k : 5.5k : 13.5k = 10:11:27

980. The ratio of the present ages of A and B 9 : 5. Five years earlier the ratio
of their was 2 : 1. What is the average of their present age

A. 20 years
B. 25 years

C. 35 years
D. 40 years
E. None of these

Answer & Explanation :


Answer: Option C

Explanation :

P a g e 526 | 580
let present age of A=9x
So present age of B=5x

According to question
9x-5/5x-5=2/1

solving this we get,


x=5 so average is 45+25/2=35 years.

981. A Shopkeeper wants to earn 12% profit on an item after giving 20%
discount to the customer. By what percentage should he increase his marked
price to arrive at the label price?
A. 24%
B. 32%
C. 40%
D. 16%
Answer & Explanation :
Answer: Option C

Explanation :
Let the current price be Rs. 100.
For getting 12% profit he should sell it at Rs. 112.

Let the label price be x.

i.e. he need to increase 40% on label price

P a g e 527 | 580
982. A can do a work in 60 days and B can do the same work in 40 days. They
work together for 12 days and then A goes away. In how many days will B
finish the remaining work?
A. 16 days

B. 20 days
C. 25 days
D. 28 days
E. 24 days
Answer & Explanation :

Answer: Option B
Explanation :
Work done by A and B in 12 days is
=12(1/60+1/40)=12 * 5/120=0.5
Remaining work = 1- [0.5]=0.5 work
B does 1/40 work in one day
so B does 0.5 work in 40 * 0.5 days = 20 days

983. A train crosses platforms of length 160 meters and 220 metres in 16
seconds and 20 seconds respectively. What is the length of the train?

A. 60 metres
B. 175 metres

C. 80 metres
D. 100 metres

E. 120 metres
Answer & Explanation :
Answer: Option C

Explanation :
Let the length of the train 'x' and speed of train be 'V' ms-1.

{160+x}/16={220+x}/20=V

P a g e 528 | 580
{160+x}/4={220+x}/5
800+5x=880+4x;

5x-4x=880-800
x=80 metres

984. 8423 + 3120 + 6543) / (1536 + 377 + 189) =?


A. 5.8
B. 14.6
C. 8.6

D. 18.3
E. 17.2
Answer & Explanation :
Answer: Option C
Explanation :
(8423+3120+6543) is nearly equal to 18000
& (1536+377+189) is nearly equal to 2100.

So required answer is (18000) /2100 = 8.6.

985. (13% of 7439) * (3.23 % of 537) =?

A. 16243
B. 14002
C. 18674
D. 19874

E. 16774
Answer & Explanation :
Answer: Option E

Explanation :
first we calculate 13% of 7439

P a g e 529 | 580
10% of 7439 = 744 & its 3% is nearly 224 therefore 13% of 7439 = 744 + 224
= 968.

Similarly 3.23 % of 537 = 16.2 + 1 = 17.2. So we get answer as 17.2 x 968 =


16660 (approx).

Hence option E is the answer.

986. (15.28 x 3.56) /3.15=?


A. 12
B. 9

C. 68
D. 17
E. 42
Answer & Explanation :
Answer: Option D
Explanation :
3.56/3.15=1 app.

So answer will be little above than 15 so answer is 17


987. 93 + 26 * 3 - 51 = ?
A. 201

B. 102
C. 120

D. 210
E. None of these

Answer & Explanation :


Answer: Option C
Explanation :

93 + 26 * 3 - 51 = 93 + 78 - 51 = 171 - 51 = 120

988. 63 * 9 * 14 /? = 98
P a g e 530 | 580
A. 83
B. 86

C. 88
D. 91

E. None of these
Answer & Explanation :
Answer: Option E
Explanation :
Let the missing number be x Then 63 * 9 * 14 /x = 98 63 * 9 * 14=98 x = 63 *
9 * 14/98=81 So, the correct answer is option E.

989. 7[1/3]+ 5[4/9]-4[4/9]=?


A. 8[7/9]
B. 8[1/3]
C. 8[2/3]
D. 8[5/9]

E. None of these
Answer & Explanation :
Answer: Option B

Explanation :
7[1/3]+5[4/9]-4[4/9]=7+5-4+[1/3+4/9-4/9]=8[1/3]

990. 6[3/8]+3[2/7]+9[7/8]=?

A. 19[13/28]
B. 15[19/28]
C. 19[15/28]

D. 15[13/28]
E. None of these

P a g e 531 | 580
Answer & Explanation :
Answer: Option C

Explanation :
(6+3+9)+(3/8+2/7+7/8)=19[15/28]

991. If the area of a rectangle is 1248 square meters and its breadth is 32
meters how much is its perimeter?
A. 142 meters
B. 128 meters

C. 148 meters
D. 124 meters
E. None of these
Answer & Explanation :
Answer: Option A
Explanation :
Length of rectangle = area /breadth So, length will be 1248/32 = 39 Perimeter
will be 2 (length + breadth) = 2 (39 + 32) = 142 meters.

992. What will be the compound interest on a sum of Rs. 7200/- at 5 p.c.p.a. in
2 years?
A. Rs. 841/-

B. Rs. 738/-
C. Rs.793/-

D. Rs. 812/-
E. Rs. 694/-
Answer & Explanation :

Answer: Option B
Explanation :

P a g e 532 | 580
993. In how many different ways can the letters of the word 'PARTY' be
arranged?
A. 120
B. 2005

C. 2400
D. 720
E. None of these
Answer & Explanation :

Answer: Option A
Explanation :
Total no. of ways =5 * 4 *3 *2 *1 = 120

994. Cost of 68 pens and 96 pencils is Rs. 788/-. What is the cost of 17 pens
and 24 pencils?
A. Rs.193/-
B. Rs.189/-
C. Rs 197/-
D. Rs. 183/-

E. None of these
Answer & Explanation :
Answer: Option C

Explanation :
Cost of 68 pens and 96 pencils = 788/-

P a g e 533 | 580
Cost of 17 pens and 24 pencils = 788/4 = 197/-

995. 5 10 25 70 205 ?
A. 650

B. 670
C. 550
D. 610
E. None of these
Answer & Explanation :

Answer: Option D
Explanation :
5 10 25 70 205 ?
+5 +15 +45 +135 +405
*3 *3 *3 *3
So, the missing term is 205 + 405 = 610
996. Water is flowing at the rate of 3 km/hr through a circular pipe of 20 cm
internal diameter into a circular cistern of diameter 10m and depth 2m. In how
much time will the cistern be filled?
A. l hour

B. 1 hour 40 minutes
C. 1 hour 20 minutes

D. 2 hours 40 minutes
Answer & Explanation :

Answer: Option B
Explanation :
Water flowed in 1 hour through the pipe

=22/7 * 10*10*3000/10000 meter3


=660/7 meter3
Volume of circular/cylindrical cistern
P a g e 534 | 580
=660/7 * 5*5*2 = 1100/7 meter3
= Required Time = 1100/7/660/7 = 5/3 hours

= 1 hour 2/3 * 60 minutes


= 1 hour 40 minutes

997. A bicycle wheel makes 5000 revolutions in moving 11 km. The diameter of
the wheel, in cm, is
A. 35
B. 55

C. 65
D. 70
Answer & Explanation :
Answer: Option D
Explanation :
Distance covered by wheel in one revolution = 2Πr
∴ 5000 × 2Πr = 11 × 1000

⇒ 5000 × 2 × 22/7 × r = 11000


⇒ r =11000*7 / 5000*2*22 = 0.35 metre

= 35 cm
∴ Diameter
= 2 × radius = 2 × 35 = 70 cm

998. At each corner of a triangular field of sides 26 m, 28 m and 30 m, a cow is


tethered by a rope of, length 7 m. The area (in m2) ungrazed by the cows is

A. 336
B. 259
C. 154
D. 77
Answer & Explanation :
P a g e 535 | 580
Answer: Option C

999. A shopkeeper allows 23% commission on his advertised price and still
makes a profit of 10%. If he gains Rs. 56 on one item, his advertised price of
the item, in Rs., is
A. 820
B. 780
C. 790
D. 800

Answer & Explanation :


Answer: Option D
Explanation :
Let the advertised price be Rs. x. ∴ S.P. = Rs. 77x/100

∴ C.P. = Rs(77x/100 - 56)

⇒ 77x -5600/100 =(100/110 * 77x/100)


⇒77x -5600/100 = 7x/10
⇒ 7x -5600 = 70x ⇒ 7x = 5600 ⇒ x = Rs. 800

1000. The average of 25 observations is 13. It was later found that an


observation 73 was wrongly entered as 48. The new average is
A. 12.6
B. 14

C. 15
D. 13.8
Answer & Explanation :
Answer: Option B
Explanation :

Difference of two observations = 73 - 48 = 25


∴ New average = 13 + 25/25 = 14

P a g e 536 | 580
1001. If the cost price of 10 articles is equal to the selling price of 8 ar¬ticles,
then gain per cent is
A. 10%

B. 8%
C. 50%
D. 25%
Answer & Explanation :
Answer: Option D

Explanation :
Profit percent= 10-8/8 × 100 = 25%

1002. An article is marked 40% above the cost price and a discount of 30% is
allowed. What is the gain or loss percentage?
A. 10% gain
B. 5% gain

C. 2% loss
D. 12% loss
Answer & Explanation :

Answer: Option D
Explanation :

Let the C.P. of article be Rs. 100. ∴ Marked price = Rs. 140 S.P. = 70% of 140
= Rs. 98

8. A man bought oranges at the rate of 8 for Rs. 34 and sold them at the rate of
12 for Rs. 57. How many oranges should be sold to earn a net profit of Rs. 45?
A. 90
B. 100
C. 135

P a g e 537 | 580
D. 150
Answer & Explanation :

Answer: Option A
Explanation :

Let the man buy 24 (LCM of 8 and 12) oranges. ∴ C.P. of 24 oranges = 34/8 ×
24 = Rs. 102 S.P. of 24 oranges = 57/12 × 24 = Rs. 114 Gain= 114- 102 = Rs.
12 ∵ Rs. 12≡ 24 oranges ∴ Rs. 45 ≡ 24/12 × 45 = 90 oranges

1003. The number 0.121212.... in the form p/q is equal to


A. 4/11
B. 2/11
C. 4/33
D. 2/33

Answer & Explanation :


Answer: Option C
Explanation :
0.121212 …. = 0.12 = 12/99 =4/33
1004. Simple interest on Rs. 1200 @ 13 p.c.p.a. for 'X' years is Rs. 624/-. What
is the amount on Rs. 'X+1000' at the same rate of interest for 3 years?
A. Rs. 1872/-
B. Rs. 1384/-
C. Rs. 936/-
D. Other than those given as options
E. Rs. 1404/-
Answer & Explanation :
Answer: Option D
Explanation :

624 = (1200×13×x)/100 => x = 4 Now, P = x + 1000 = 4 + 1000 = 1004 I =


(1004×13×3)/100 = 392 => amount = 1004 + 392 = 1396/-

P a g e 538 | 580
1005. P, Q and R have a certain amount of money with themselves. Q has 50%
more than what P has, and R has 1/3rd of what Q has. If P, Q and R together
have Rs. 240 then how much money does P alone have? (in Rs.)

A. 75
B. 60
C. 120
D. 80
E. 90

Answer & Explanation :


Answer: Option D
Explanation :
Assume that P has 2x amount. Therefore, Q = 3x amount (50% more than P)
And R = 3x ×1/3= x Ratio of P : Q : R = 2x : 3x : x = 2 : 3 : 1 Thus, P alone
have = (2/6) × 240 = Rs. 80

1006. Bill, Simon, and John are brothers, given Simon is the eldest. Bill is as
many years younger than one brother as he is older than the other. Simon is 7
years younger than twice the age of John. John is 5 years older than half the
age of Bill. What is the sum of the ages of Bill, Simon and John?
A. 12
B. 24
C. 48
D. Can’t say
E. None of these
Answer & Explanation :

Answer: Option C
Explanation :
S–B=B–J
J = B/2 + 5.

P a g e 539 | 580
S = 2J – 7.
S = B + 10 – 7 = B + 3.

J = B/2 + 5. 2S = B + 10.B/2 + 5 + B + 3 = 2B. B/2 = 8. B = 16, S = 19, J =


13. So B + S + J = 16 + 19 + 13 = 48.

1007. The sum of the squares of the digits constituting a two-digit number is
10, and the product of the required number by the number consisting of the
same digits written in the reverse order is 403. Find the number.
A. 13

B. 31
C. 41
D. Both 1 & 2
E. None of these
Answer & Explanation :

Answer: Option D
Explanation :

1st condition is satisfied by 1st and 2nd options. 2nd condition is also satisfied
by both these options because (1)2 + (3)2 = 10 and (3)2 + (1)2 = 10. Also 31
* 13 = 403 and 13*31 = 403. S the answer is 4th option.

1008. A circle and rectangle have the same perimeter. The sides of the
rectangle are 18 cm and 26 cm. What will be the area of the circle? (in cm2)
A. 88
B. 1250
C. 154
D. 128

E. Other than those given as options


Answer & Explanation :
Answer: Option E
Explanation :

P a g e 540 | 580
Perimeter of rectangle = perimeter of circle = 2 (18 + 26) = 88 2πr = 88 => r
= 14cm Area of circle = 22/7 ×14 × 14 = 616 cm2

1009. Some men promised to do a job in 18 days, but 6 of them became absent
and remaining men did the job in 20 days. What is the original number of men?
A. 50 men
B. 60 men
C. 65 men
D. 70 men

E. 55 men
Answer & Explanation :
Answer: Option B
Explanation :
Let the number of men originally = M. According to the given condition M => 18
= ( M – 6) => 20 => M = 60

1010. The number of ways in which 8 persons can be seated at a round table if
2 particular persons must always sit together
A. 288

B. 720
C. 1440
D. 2880
E. None of these

Answer & Explanation :


Answer: Option C
Explanation :

Reqd. number of ways = 2! × (7 – 1)! = 2! × 6! = 1440.

1011. The number of ways in which 7 boys and 8 girls can be seated in a row so
that they are alternate
P a g e 541 | 580
A. 203121800
B. 29030400

C. 3628800
D. 203212800

E. 3628800
Answer & Explanation :
Answer: Option D
Explanation :
Reqd. number of ways = 7! × 8! = 203212800.

1012. 50% of a number is 18 less than two-third of that number. Find the
number.
A. 123
B. 115
C. 119
D. 108

E. 101
Answer & Explanation :
Answer: Option D

Explanation :
Let the no. be x
Given : 50x/100 = 2x/3 – 18
½ x – 2/3 x = - 18

=> x = 108

1013. Running at the same constant rate, 6 identical machines can produce a
total of 270 bottles per minute. At this rate, how many bottles could 10 such
machines produce in 4 minutes?

A. 648

P a g e 542 | 580
B. 1800
C. 2700

D. 2000
E. 3080

Answer & Explanation :


Answer: Option B
Explanation :
Let the no. of bottles be ‘B’. Using chain rule:
(6×1)/270 = (10×4)/B => B = 1800

1014. In a 200 meters race, A beats B by 20 meters, while in a 100 meters


race, B beats C by 5 meters. A beats C in a kilometer race by
A. 105 meters
B. 225 meters
C. 205 meters
D. 145 meters
Answer & Explanation :

Answer: Option D
Explanation :
When A = 200, B = 180.

When B = 100, C = 95.


Make B = 900. A will be 1000. C will be 855.

1015. A person purchases tomatoes from each of the 4 places at the rate of 1
kg, 2 kg, 3 kg, 4 kg per rupee respectively. On an average he has purchased x
kg of tomatoes per rupee. Then the value of x is
A. 2

B. 2.5
C. 1.92

D. None of these

P a g e 543 | 580
Answer & Explanation :
Answer: Option B

Explanation :
x = 10/4 = 2.5

1016. A sum of money doubles itself in 7 years. In how many years it becomes
four fold?
A. 35 years
B. 28 years

C. 14 years
D. 21 years
Answer & Explanation :
Answer: Option D
Explanation :
Double in seven years.
So 4 times will be in = 7 × (4 − 1) = 21 years.

1017. A fast train takes 3 hours less than a slow train for a journey of 600 km,
If the speed of the slow train is 10 km/hr less than that of the fast train, the
speeds of the two trains are
A. 60 km/hr and 70 km/hr

B. 50 km/hr and 60 km/hr


C. 40 km/hr and 50 km/hr

D. 30 km/hr and 40 km/hr


Answer & Explanation :
Answer: Option C

Explanation :
(600/x)-(600/x+10)=(3/1)

Solving this, we get x = 40

P a g e 544 | 580
1018. When 75% of a number is added to 75, the result is the same number.
The number is
A. 150

B. 300
C. 100
D. 450
Answer & Explanation :
Answer: Option B

Explanation :
Let the number be equal to x.
3x/4 +75 = x
Hence x=300

1019. If 5 spiders can catch five flies in five minutes, how many flies can
hundred spiders catch in 100 minutes?

A. 100
B. 1000
C. 500

D. 2000
Answer & Explanation :

Answer: Option D
Explanation :

5 × 5 × x = 100 × 100 × 5 ⇒ x = 2000.

1020. A car travels 25km an hour faster than a bus for a journey of 500km. If
the bus takes 10 hours more than the car, then the speeds of the bus and the
car are
A. 25km/hr and 40km/hr respectively

P a g e 545 | 580
B. 25km/hr and 60 km/hr respectively
C. 25km/hr and 50km/hr respectively

D. None of these
Answer & Explanation :

Answer: Option C
Explanation :
Let the speeds of bus and car be x and y. Here, y = x +25
As per the question, (500/x)-(500/x+25)=10
⇒ 500(x + 25) − 500(x) =10x(x+25) ⇒ x = 25, y = 50.

For such questions, you can also use direct substitution from the given choices

1021. An article is bought for Rs. 600 and sold for Rs. 750. The gain percent is:
A. 20%
B. 25%
C. 30%

D. None
Answer & Explanation :

Answer: Option B

1022. If P : Q = 3 : 4 and Q : R = 5 : 6, then P : Q : R is:


A. 3 : 4 : 6
B. 3 : 5 : 6

C. 15 : 20 : 24
D. 5 : 6 : 3

Answer & Explanation :


Answer: Option C
Explanation :

P:Q=3:4, Q:R=5:6, P:Q:R=3×5:5×4:6×4

P a g e 546 | 580
⇒ 15:20:24

1023. Two years ago the average age of a family of 8 members was 18 years.
After the addition of a baby, the average age of family remains the same today.
What is the age of the baby ?
A. 1 year
B. 2 years
C. 4 years

D. 3.5 years
Answer & Explanation :
Answer: Option B

1024. An article when sold at a gain of 5% yields Rs. 15 more than when sold at
a loss of 5%. The cost price of the article is:
A. Rs. 200
B. Rs. 150

C. Rs. 80
D. Rs. 64
Answer & Explanation :
Answer: Option B
Explanation :
10/100 × x = 15 ⇒ x=150

As the loss and profit both are earned on the cost price.

1025. If X gets 25% more than Y and Y gets 20% more than Z, the share of Z
out of Rs. 740 will be:
A. Rs. 300
B. Rs. 200

C. Rs. 240
D. Rs. 350

P a g e 547 | 580
Answer & Explanation :
Answer: Option B

Explanation :
Z share = Z,

Y = 1.2Z
X = 1.25×1.2Z,
X+Y+Z =740
(1.25× 1.2+1.2+1)Z=740
3.7Z=740, Z=200

1026. In a mixture of 35 litres the ratio of milk and water is 4 : 1. If 7 litres of


water is added to the mixture, the ratio of milk to water in the resultant mixture
will become :
A. 2 : 3
B. 3 : 2
C. 2 : 1

D. 1 : 2
Answer & Explanation :
Answer: Option C

Explanation :
Mixture = 35L, Milk = 4/5 × 35 =28L

Water = 1/5 × 35 =7L


New 28/7+7 = M/W

⇒ 28/14 = M/W =2/1

1027. What will be the income tax payable by a person getting a taxable income
of Rs. 30,000 during the year if the first Rs. 18000 is not taxable and the tax
rates are 25% of the next Rs. 12000 ?
A. Rs. 3000

P a g e 548 | 580
B. Rs. 3250
C. Rs. 3500

D. Rs. 3750
Answer & Explanation :

Answer: Option A
Explanation :
T.I.=30,000, N.T.I.=18000
Taxable 12000, Income Tax = 25% of Taxable Income
= 1/4 ×: 12000 = RS.3000

1028. 10 men can finish a piece of work in 10 days whereas it takes 12 women
to finish it in 10 days. If 15 men and 6 women undertake to complete the work,
how many days will they take to complete the work?
A. 2
B. 4
C. 5

D. 11
Answer & Explanation :
Answer: Option C

Explanation :
10 men = 10 days, 1 men 1 day = 1/100 work

12 women = 12 days, 1 women 1 day = 1/120


15 men + 6 women, 15 × 1/100 + 6 × 1/120

3/20 +1/20= 4/20 =1/5 work= 5 days

1029. A merchant marks his wares 40% more than the real price and allows
20% discount. His profit is:
A. 20%

B. 18%

P a g e 549 | 580
C. 16%
D. 12%

Answer & Explanation :


Answer: Option D

Explanation :
Let the CP = 100 Rs.
Mark Price = 140
Discount = 20%
Selling price 80/100 × 140 = 112

Hence profit = 12%

1030. A train 280 metres long is moving at 60 kms per hour. The time taken by
the train to cross a platform 220 metres long is:
A. 20 seconds
B. 25 seconds
C. 30 seconds

D. 35 seconds
Answer & Explanation :
Answer: Option C

1031. A person spends 40% of his salary on food, 25% on house rent, 15% on
entertainment and 5% on conveyance. If his savings at the end of the month is
Rs. 1200, then his salary per month in rupees is:

A. 4000
B. 6000
C. 8000

D. 10000
Answer & Explanation :

Answer: Option C

P a g e 550 | 580
Explanation :
Total expenditure = 40 + 25 + 15 + 5 = 85%

Saving = (100 - 85) = 15%


15/100 × Salary = 1200, Salary = 8000 Rs.

1032. 11.7 * 4.1 - 5.97 = ?


A. 48
B. 42
C. 46
D. 39

E. None of these
Answer & Explanation :
Answer: Option B
Explanation :
11.7 * 4.1 - 5.97 = 47.97 - 5.97 = 42

1033. The value of 1/cosecΘ-cotΘ-1/sinΘ is

A. Cosec Θ
B. tan Θ
C. 1

D. cot Θ
Answer & Explanation :
Answer: Option D
Explanation :

Expression = 1/cosec Θ - cot Θ - 1/cosec Θ cosec2Θ-cot2Θ/cosecΘ-cotΘ-cosecΘ


=cotΘ (cosec2Θ - cot2Θ-1; 1/sinΘ = cosecΘ)

1034. In a triangle, if three altitudes are equal, then the triangle is


A. Right

P a g e 551 | 580
B. Isosceles
C. Obtuse

D. Equilateral
Answer & Explanation :

Answer: Option D
Explanation :
The only possible case is when the Triangle is equilateral.

1035. A sum was put at simple interest at a certain rate for 2 years. Had it been
put at 3% higher rate, it would have fetched Rs 300 more. The sum is
A. 5300
B. 5500
C. 5000
D. None of these
Answer & Explanation :
Answer: Option C

Explanation :
Increase of 3% fetched Rs.300 more. It is for 2 years. For 1 year Increase of
3% will fetch Rs.150. So 1 % will fetch Rs.50 100% = 5000.

1036. A sum of Rs 1,550 was lent partly at 5% and partly at 8% per annum
simple interest. The total interest received after 3 years was Rs 300. The ratio
of money lent at 5% to that lent at 8% is

A. 5:8
B. 8:5
C. 31:6

D. 16:15
Answer & Explanation :

Answer: Option D

P a g e 552 | 580
Explanation :
Partly division of 1550 is x and x - 1550 5 % of x + 8 % of (1550 – x) = 0.05x
+ 0.08(1550 – x) For 3 years, the total interest is 300. 3[0.05x + 0.08(1550 –
x)] = 300 x = 800 and 1550 – x = 750 The ratio of money = 16:15.

1037. The cost price of item B is Rs. 150/- more than the cost price of item A,
Item A was sold at a profit of 10% and Item B was sold at a loss of 20%. If the
respective ratio of selling price of items A and B is 11:12, what is the cost price
of item B?

A. Rs. 450/-
B. Rs. 420/-

C. Rs. 400/-
D. Rs. 350/-
E. Rs. 480/-
Answer & Explanation :
Answer: Option A

Explanation :
Let us assume cost price of A= X
So that Cost price of B= X+150.

SP of A= X*1.1
SP of B=(X+150)*0.8
Given that
SPA: SPB
11:12
So that 1.1X/(X+150)*0.8= 11/12
X=300

CP of B= 300+150

1038. A vessel contains a mixture of milk and water in the respective ratio of 10
: 3. Twenty-six litres of this mixture was taken out and replaced with 8 litres of

P a g e 553 | 580
water. If the resultant respective ratio of milk and water in the mixture was 5 :
2, what was the initial quantity of mixture in the vessel? (in litres)

A. 143
B. 182

C. 169
D. 156
E. 130
Answer & Explanation :
Answer: Option E

Explanation :
26 L mixture is taken out.
Quantity of Milk is taken out= 26*(10/13)=20
Quantity of Water is taken out=26*(3/13)=6
As we know that,
(10X-20)/(3X-6+8)=5/2
X=10,

Initial quantity of mixture in the vessel is= 13*X=13*10=130L.

1039. There are 6 consecutive odd numbers. The difference between the square
of the average of the first three numbers and the square of the average of the
last three numbers is 288. What is the last odd number?

A. 31
B. 27

C. 29
D. 25
E. 33

Answer & Explanation :


Answer: Option C

Explanation :

P a g e 554 | 580
Let the 6 consecutive odd no.’s are :
X, X+2, X+4,X+6, X+8, X+10

Avg. of 1st three no’s is X+2.


Avg. of Last three no’s is X+8.

Given that (X+8)2-(X+2)2=288


X=19
Last odd no. is X+10= 29.
Answer is 29.

1040. In a bag there are 6 red balls and 9 green balls. Two balls are drawn at
random, what is the probability that at least one of the balls drawn is red?
A. 29/35
B. 7/15
C. 23/35
D. 2/5
E. 19/35

Answer & Explanation :


Answer: Option C
Explanation :

Probability of atleast one of the balls drawn is red= 1- (9/15)*(8/14)=23/35.


Answer is 23/35.

1041. 81% of 4915 = ?


A. 3819.15

B. 3871.15
C. 3981.15
D. 3918.15

E. None of these
Answer & Explanation :
Answer: Option C
P a g e 555 | 580
Explanation :
81/100*4915=3981.15

1042. 1682 /58 * ? = 377

A. 13
B. 15
C. 16
D. 14
E. None of these

Answer & Explanation :


Answer: Option A
Explanation :
Let the missing number =X
Then 1682/58*x=377
x=377*58/1682=13

1043. A started a business by investing Rs. 25000. At the end of 4th month
from the stat of the business B joined with Rs. 15000 and at the end of 6th
month from the start of the business, C joined with Rs. 20000. If the A’s share
in profit at the end of year was Rs. 7,750, what was the total profit received?
A. Rs. 13,950

B. Rs. 13,810
C. Rs. 13,920

D. Rs. 12,780
E. Rs. 14,040
Answer & Explanation :

Answer: Option A
Explanation :

A = 25000*12 , B = 15000*8, C = 20000*6

P a g e 556 | 580
Ratio = A:B:C = 5:2:2. Let TP be the total profit.
Given, 5/9 * TP = 7750 => TP = 13950.

1044. The respective ratio of radii of two right circular cylinders (A & B) is 3 : 2.
The respective ratio of volumes of cylinders A and B is 9 : 7, then what are the
heights of cylinders A and B?
A. 8 : 5
B. 4 : 7
C. 7 : 6

D. 5 : 4
E. 6 : 5
Answer & Explanation :
Answer: Option B

1045. (512/?) *(39/16)*328=128


A. 3199

B. 3168
C. 3189
D. 3188

E. None of these
Answer & Explanation :

Answer: Option E
Explanation :

Let missing number be x then (518/x)*(39/16)*328=128


X= (512*39*328)/(16*128)=3198 So, the correct answer is option E.

1046. The cost price of an article is Rs. 7950/-. If it is to be sold at a profit of


18%, how much would be its selling price ?

A. Rs. 9431/-

P a g e 557 | 580
B. Rs. 9183/-
C. Rs. 9218/-

D. Rs. 9381/-
E. None of these

Answer & Explanation :


Answer: Option D
Explanation :
SP={(100+profit%)/100}*Cost price
SP={(100+18%)/100}*7950=9381

1047. The ratio of the ages of A and B is 4:3 respectively. The ratio of their
ages eight years from now will be 6:5 respectively. How old was A, when B was
7 years old?
A. 16 years
B. 11 years
C. 9 years

D. 12 years
E. None of these
Answer & Explanation :

Answer: Option B
Explanation :

Let the ages of A and B be 4x and 3x respectively.


now after 8 years their ages will be 4x+8 and 3x+8

then according to question (4x+8)/(3x+8)=6/5 Then x will be 4.So present age


of B is 12 and present age of A is 16.
So when B was 7 years old, A was 11 years old.

1048. The sum of three consecutive even numbers is 1434. What is the largest
number ?

P a g e 558 | 580
A. 438
B. 484

C. 476
D. 472

E. None of these
Answer & Explanation :
Answer: Option E
Explanation :
The sum of three consecutive even numbers is 1434

So, their average will be 1434/3 = 478, which is also the middle no.
So, maximum no. will be 478 + 2 = 480.
Therefore, the correct answer is option E.

1049. ? 7800 1560 390 130 65


A. 47120
B. 49900

C. 46800
D. 48350
E. None of these

Answer & Explanation :


Answer: Option C
Explanation :
The series pattern is: 65×2= 130,

130 ×3= 390,


390×4= 1560,
1560 ×5= 7800,

7800×6= 46800.
therefore, 46800 is the answer.

P a g e 559 | 580
1050. 7 35 210 ? 6300 31500

A. 1040
B. 1060

C. 1080
D. 1030
E. None of these
Answer & Explanation :
Answer: Option E

Explanation :
Pattern is *5, *6, *5, *6, *5.
Following the same, the answer should be 210 * 5 = 1050.
So, the correct answer is option E.
1051. A boat takes a total time of twelve hours to travel 105 kms upstream and
the same distance downstream. The speed of the boat in still water is six times
of the speed of the current. What is the speed of the boat in still water? (In
km/hr)
A. 12
B. 30

C. 18
D. 24

E. 36
Answer & Explanation :

Answer: Option C
Explanation :
Let ‘x’ be the speed of Boat in still water, and ‘y’ be the speed of current.

Then, according to the question,


Speed of the boat in still water = 6 speed of current

x = 6y

P a g e 560 | 580
Also given that ,
105/(x+y) +105/(x-y) =12

105/7y +105/5y =12


12y=36

y=3
Therefore, x= 6×3=18
Speed of the boat in still water= 18kmph

1052. At 60% of its usual speed, a train of length L meters crosses platform
240 meters long in 15 seconds. At its usual speed, the train crosses a pole in 6
seconds. What is the value of L (in meters)?
A. 440
B. 425
C. 220
D. 480
E. 240

Answer & Explanation :


Answer: Option D
Explanation :

Let usual speed = S m/s.


According to the question,

60% of S= (L+240)/15
9S= L+240………… (i)

Also given that, S = L/6


L = 6S …. (ii)
From (i) & (ii)

9S= 6S+240
3S= 240
S=80 m/s
P a g e 561 | 580
From (ii)
L= 6×80 =480 m

1053. P, Q and R have a certain amount of money with themselves. Q has 50%
more than what P has, and R has 1/3rd of
A. 75
B. 60
C. 120
D. 80

E. 90
Answer & Explanation :
Answer: Option D
Explanation :
Assume that P has 2x amount.
Therefore, Q = 3x amount (50% more than P)
And R = 3x × 1/3 = x

Ratio of P : Q : R = 2x : 3x : x = 2 : 3 : 1
Thus, P alone have = (2/6) × 240 = Rs. 80

1054. Suri gave 25% of her monthly salary to her mother. From the remaining
salary, she paid 15% towards rent and 25%, she kept aside for her monthly
expenses. The remaining amount she kept in bank account. The sum of the
amount she kept in bank and that she gave to her mother was Rs. 42000. What
was her monthly salary?
A. Rs. 50,000
B. Rs. 60,000

C. Rs. 65,000
D. Rs. 64,000

E. Rs. 72,000
Answer & Explanation :
P a g e 562 | 580
Answer: Option B
Explanation :

Let x be the total income


(75/100*60/100)x + 25/100x = 42000

Solving, x = 60000

1055. At present, Ami’s age is twice Dio’s age and Cami is two years older than
Ami. Two years ago, the respective ratio between Dio’s age at that time and
Cami’s age at that time was 4: 9. What will be Ami’s age four years hence?

A. 40 years
B. 30 years
C. 42 years
D. 36 years
E. 48 years
Answer & Explanation :
Answer: Option A

Explanation :
D; A = 2D; C = A+2 = 2D+2
Given, D-2/(2D+2)-2 = 4/9

Solving, D = 18 Years and A = 36+4 = 40 years.

1056. If tan Θ = 3/4 and Θ is acute, then cosec Θ is


A. 5/4

B. 4/3
C. 4/5
D. 5/3

Answer & Explanation :


Answer: Option D
Explanation :
P a g e 563 | 580
tan Θ=3/4
cot Θ = 4/3

cosec2Θ - cot2Θ = 1
cosec Θ= √ 1+cot2Θ = √ 1+(4/3)2 = √ 1+16/9 = √ 25/9 =5/3

1057. A vertical stick 12 cm long casts a shadow 8 cm long on the ground. At


the same time, a tower casts a shadow 40m long on the ground. The height of
the tower is
A. 65m

B. 70m
C. 72m
D. 60m
Answer & Explanation :
Answer: Option D
Explanation :
height of Tower/Length Of Stick

=Length of shadow of Tower/Length of shadow of Stick


h/12=40/8
h=40*12/8 = 60 metre

1058. 40.1% of 360.2 + 58.98% of ? = 150

A. 10
B. 20

C. 30
D. 40
E. 50

Answer & Explanation :


Answer: Option A
Explanation :
P a g e 564 | 580
40*360+59%of X=150
144+59%of X=150

X=10(approx)

1059. 96.894 + 33.002 + 15.02 * 7.99 = ?


A. 180
B. 250
C. 140
D. 269

E. 170
Answer & Explanation :
Answer: Option B
Explanation :
97+33+15*8=250
Answer 250.

1060. (42.11×5.006)-√17×15.08=?
A. 250
B. 150

C. 45
D. 200
E. 125
Answer & Explanation :

Answer: Option B
Explanation :
(42*5)-4*15=150

Answer 150.

P a g e 565 | 580
1061. The sum of a number and the two numbers preceding it is equal to 30.
Find the number:

A. 10
B. 11

C. 9
D. 8
Answer & Explanation :
Answer: Option C
Explanation :

Number is 9,
As 9 +10+11=30

1062. Sanjay invested an amount of Rs 16,000 for two years on compound


interest and received an amount of RS 17,640 on maturity. What is the rate of
interest per annum?
A. 4%

B. 5%
C. 8%
D. Data Inadequate

Answer & Explanation :


Answer: Option C

1063. Six pipes are fitted to a water tank. Some of these are inlet pipes and the
others outlet pipes. Each inlet pipe can fill the tank in 9 hours and each outlet
pipe can empty the tank in 6 hours. On opening all the pipes, an empty tank is
filled in 9 hours. How many inlet pipes are there?

A. 2
B. 4
C. 3
D. 5

P a g e 566 | 580
Answer & Explanation :
Answer: Option B

Explanation :
x / 9 – y / 6 = 1/9;

2x – 3y = 2
We can compute that x = 4 and y = 2.
Thus, Inlet pipe = x = 4.

1064. A reservoir is provided by two pipes A and B. A can fill the reservoir 5
hours faster than B. If both together fill the reservoir in 6 hours, the reservoir
will be filled by A alone in
A. 12 hours
B. 8 hours
C. 10 hours
D. 11 hours
Answer & Explanation :

Answer: Option C
Explanation :
If x is the speed then speed of A= x + 5 and B = x

Time taken by A and B will be x and x + 5 resp.


1/x + 1/x + 5 = 1/6 ; x2 – 7x - 30 = 0 x = -3 or x = 10.

Since time can’t be negative, x =10.

1065. A dealer buys dry fruits at the rate of ` 100, ` 80 and ` 60 per kg. He
bought them in the ratio 12 : 15 : 20 by weight. He in total gets 20% profit by
selling the first two and at last he finds he has no gain or no loss in selling the
whole quantity which he had. What was the percentage loss he suffered for the
third quantity?
A. 30%
B. 40%

P a g e 567 | 580
C. 20%
D. 50%

Answer & Explanation :


Answer: Option B

Explanation :
Total quantity rate =
(12 * 100 + 15 * 80 + 20 * 60) = 3600
For first 2 quantity, (12 * 100) + (15 * 80) = 2400
But he gets 20% profit = 2400 * 1.2 = 2880

So the third quantity = 3600 – 2880 = 720


Actual third quantity rate = 20 * 60 = 1200
Loss suffered = (1200 - 720) / 1200
= 480/1200 = 40 %

1066. How many kgs of flour worth Rs 25 per kg must be blended with 30 kgs
of flour worth Rs 30 per kg so that by selling the blended variety at Rs 30 per
kg there should be a gain of 10%?
A. 32 kg
B. 40 kg

C. 36 kg
D. 42 kg

Answer & Explanation :


Answer: Option C

Explanation :
(25X+30×30 )/(X+30)=300/11
X=36

1067. A boat takes 19 hours for travelling downstream from point A to point B
and coming back to a point C midway between A and B. If the velocity of the

P a g e 568 | 580
stream is 4 km/h and the speed of the boat in still water is 14 km/h, what is the
distance between A and B?

A. 200 km
B. 180 km

C. 160 km
D. 220 km
Answer & Explanation :
Answer: Option B
Explanation :

Speed of boat for downstream = 14 + 4 = 18 km/hr


Speed of boat for upstream = 14 – 4 = 10 km/hr
Distance = x
x / 18 + (x /2)/10 = 19
x = 180 km

1068. The speed of a boat in still water is 4 km/h and the speed of current is 2
km/h. If the time taken to reach a certain distance upstream is 9 hours, the
time it will take to go the same distance downstream is
A. 3.5 hours

B. 2.5 hours
C. 2 hours

D. 3 hours
Answer & Explanation :

Answer: Option D
Explanation :
Upstream speed of a boat = 4 – 2 = 2 km/hr

Downstream speed of a boat = 4 + 2 = 6 km/hr


Suppose time taken = x

Then for Upstream case: 9 = x / 2 So, x = 18 km

P a g e 569 | 580
Now for downstream case :Time = 18/6 = 3 hrs

1069. In a stream running at 2 km/h, a motor boat goes 10 km upstream and


back again to the starting point in 55 min. The speed of the motorboat in still
water is
A. 22 km/h
B. 21 km/h
C. 20 km/h
D. 24 km/h

Answer & Explanation :


Answer: Option A
Explanation :
Let "x" be the speed of Motor boat in still water
Distance = 10 km; Time = 55/60 hrs
For upstream, the speed of motorboat = x - 2
For downstream, the speed of motorboat = x + 2

55 / 60 = 10/ (x – 2) + 10/(x + 2) So, x = 22.

1070. A man can row 4.5 km/h in still water and he finds that it takes him twice
as long to row up as to row down the river. The speed of the stream is
A. 2.5 km/h

B. 1.5 km/h
C. 2 km/h

D. 1.75 km/h
Answer & Explanation :
Answer: Option B

Explanation :
Let speed of stream be S and x be upstream speed.

Then for downstream, 4.5 + S = D

P a g e 570 | 580
And for upstream , 4.5 – S = U
So we get, D + U = 9

But, it takes him twice as long to row up as to row down the river.
3U = 9 ; U = 3. Thus, 4.5 – S = 3

S=1.5 kmph
1071. The average score of a cricketer for 13 matches is 42 runs. If his average
score for the first 5 matches is 54, then what was his average score (in runs)
for last 8 matches?
A. 37

B. 39
C. 34.5
D. 33.5
E. 37.5
Answer & Explanation :
Answer: Option C
Explanation :

Total Score = Average * Number of matches


Total score of 13 matches = 13 × 42 = 546
Total score of first 5 matches = 5 × 54 = 270

Therefore, total score of last 8 matches = 546 – 270 = 276


Average = 276/8 = 34.5

Hence the answer is option C

1072. Simple interest on a sum of money for 4 years at 7 p.c.p.a is Rs. 3584/-.
What would be the compound interest (compounded annually) on the same
amount of money for 2 years at 4p.c.p.a?

A. Rs. 1,162.32
B. Rs. 1,098.72

C. Rs. 992.38

P a g e 571 | 580
D. Rs. 1,231.76
E. Rs. 1,044.48

Answer & Explanation :


Answer: Option E

Explanation :
Let P be the sum
Therefore,
3584 = (P * 7 * 4) / 100 => P = 12800/-
CI = 12800(1 + (4/100))2 - 12800

=> 13844.48 - 12800 = 1044.48/-


Hence the answer is option E

1073. 6*136/8+132 / 628/16-26.25=?


A. 15
B. 24
C. 18

D. 12
E. 28
Answer & Explanation :

Answer: Option C
Explanation :
6*136/8+132 / 628/16-26.25
⇒ 234/13=18 Hence the answer is option C.

1074. (1097.63 + 2197.36 - 2607.24) ÷ 3.5 = ?


A. 211.5
B. 196.5

C. 209.5

P a g e 572 | 580
D. 192.5
E. 189.5

Answer & Explanation :


Answer: Option B

Explanation :
? = (1097.63 + 2197.36 - 2607.24) ÷ 3.5 = 687.75 ÷ 3.5
= 196.5
Hence answer is option B

1075. A certain number of capsules were purchased for Rs. 216/-. 15 more
capsules could have been purchased in the same amount if each capsule was
cheaper by Rs. 10/-. What was the number of capsules purchased?
A. 6
B. 14
C. 8
D. 12

E. 9
Answer & Explanation :
Answer: Option D

Explanation :
Let x be the cost of each capsule and y be the no of capsules

Therefore, x = 216/y and x - 10 = 216/(y+15)


Putting the value of x from eq. 1 into eq. 2, we get

216/y - 216/(y+15) = 10
y2 +15y - 324 = 0
Solving for y, we get y = 12

Hence answer is option D.

1076. What will come in place of question mark (?)in the given question?
P a g e 573 | 580
4 6 34 ? 504 1234
A. 194

B. 160
C. 186

D. 156
E. 172
Answer & Explanation :
Answer: Option B

1077. A and B are two numbers. 6 times square of B is 540 more than the
square of A. if the respective ratio between A and B is 3:2. What is the value of
B?
A. 10
B. 12
C. 16
D. 8

E. 14
Answer & Explanation :
Answer: Option B

1078. A and B promise to do a work for Rs. 75. A alone can do it in 20 days and
B in 30 days, with the help of C they are able to finish it in 8 days. How will A, B
and C respectively distributes the wages?
A. Rs. 20, Rs. 30, Rs. 25

B. Rs. 25, Rs. 20, Rs. 30


C. Rs. 30, Rs. 25, Rs. 20
D. Rs. 30, Rs. 20, Rs. 25

E. None of these
Answer & Explanation :

Answer: Option D

P a g e 574 | 580
Explanation :
Everybody will get the wages according to their labour. A’s one day work = , A’s
8 days work = = ; B’s one day work = ;
B’s 8 days work = = , Remaining work = 1- - = ; C did the of work in 8 days.

wages are divided in the ratio of 2/5 : 4/15 :1/3 = 6 : 4 : 5 , A’s share = 6/15*
75= 30 Rs.
B’s share = 4/15* 75 = 20 Rs. ,C’s share = 5/15* 75 = 25 Rs.

1079. If 8 men or 12 women can do a piece of work in 25days, in how many


days, can the work be done by 6 men and 11 women working together?
A. 12 days
B. 15 days
C. 9 days
D. 18 days

E. 10 days
Answer & Explanation :

Answer: Option B
Explanation :
8M = 12W s => 1M = W => ( 6M + 11W ) => D = 12W => 25 => ( 6 => W +
11W ) => D = 12W => 25 => 20 => D = 12 => 25 => D = 15 days

1080. A started a business by investing Rs. 25000. At the end of 4th month
from the stat of the business B joined with Rs. 15000 and at the end of6th
month from the start of the business, C joined with Rs. 20000. If the A’s share
in profit at the end of year was Rs. 7,750, what was the total profit received?
A. Rs. 13,950

B. Rs. 13,810
C. Rs. 13,920
D. Rs. 12,780
E. Rs. 14,040

P a g e 575 | 580
Answer & Explanation :
Answer: Option A

Explanation :
A = 25000*12 , B = 15000*8, C = 20000*6

Ratio = A:B:C = 5:2:2. Let TP be the total profit.


Given, 5/9 * TP = 7750 => TP = 13950

1081. The respective ratio of radii of two right circular cylinders (A & B) is 3 : 2.
The respective ratio of volumes of cylinders A and B is 9 : 7, then what are the
heights of cylinders A and B?
A. 8 : 5
B. 4 : 7
C. 7 : 6
D. 5 : 4
E. 6 : 5
Answer & Explanation :

Answer: Option B

1082. Suri gave 25% of her monthly salary to her mother. From the remaining
salary, she paid 15% towards rent and 25%, she kept aside for her monthly
expenses. The remaining amount she kept in bank account. The sum of the
amount she kept in bank and that she gave to her mother was Rs. 42000. What
was her monthly salary?
A. Rs. 50,000
B. Rs. 60,000
C. Rs. 65,000

D. Rs. 64,000
E. Rs. 72,000
Answer & Explanation :
Answer: Option B

P a g e 576 | 580
Explanation :
Let x be the total income

(75/100*60/100)x + 25/100x = 42000


Solving, x = 60000

1083. Nine students of a class contribute a certain sum. Seven of them give Rs.
5 each. The remaining two give Rs. 5 and Rs. 9 more than the average
contribution of all the 9 students respectively. The average contribution of the
class of 9 students is -

A. Rs. 10
B. Rs. 14
C. Rs. 7
D. Rs. 12
E. Rs 16

Answer & Explanation :


Answer: Option C

Explanation :
Let the average contribution of class = x (7*5) + (x + 5) + (x + 9) = 9x => 35
+ 2x + 14 = 9x => 49 = 7x => x = 7.

1084. One-fourth of my marks in English is equal to one third of my marks in


Hindi, The total number of marks secured by me in both the subjects is 140.
The marks secured by me in English are...
A. 60
B. 80
C. 75

D. 85
E. None of these
Answer & Explanation :
Answer: Option B

P a g e 577 | 580
Explanation :
Let marks in English = x & Hindi = y.x/4 = y/3 Also x + y = 140. Solving the 2
equations we get x = 80.

1085. A boat takes a total time of twelve hours to travel 105 kms upstream and
the same distance downstream. The speed of the boat in still water is six times
of the speed of the current. What is the speed of the boat in still water? (In
km/hr)
A. 12

B. 30
C. 18
D. 24
E. 36
Answer & Explanation :

Answer: Option C
Explanation :

Let ‘x’ be the speed of Boat in still water, and ‘y’ be the speed of current.
Then, according to the question,
Speed of the boat in still water = 6 speed of current

x = 6y
Also given that ,
105/(x+y) +105/(x-y) =12
105/7y +105/5y =12

12y=36
y=3
Therefore, x= 6×3=18

Speed of the boat in still water= 18kmph

P a g e 578 | 580
1086. At 60% of its usual speed, a train of length L metres crosses platform
240 metres long in 15 seconds. At its usual speed, the train crosses a pole in 6
seconds. What is the value of L (in metres)?
A. 440

B. 425
C. 220
D. 480
E. 240
Answer & Explanation :

Answer: Option D
Explanation :
Let usual speed = S m/s.
According to the question,
60% of S= (L+240)/15
9S= L+240………… (i)
Also given that, S = L/6

L = 6S …. (ii)
From (i) & (ii)
9S= 6S+240

3S= 240
S=80 m/s

From (ii)
L= 6×80 =480 m

1087. P, Q and R have a certain amount of money with themselves. Q has 50%
more than what P has, and R has 1/3rd of what Q has. If P, Q and R together
have Rs. 240 then how much money does P alone have? (in Rs.)
A. 75

B. 60

P a g e 579 | 580
C. 120
D. 80

E. 90
Answer & Explanation :

Answer: Option D
Explanation :
Assume that P has 2x amount.
Therefore, Q = 3x amount (50% more than P)
And R = 3x × 1/3 = x

Ratio of P : Q : R = 2x : 3x : x = 2 : 3 : 1
Thus, P alone have = (2/6) × 240 = Rs. 80

P a g e 580 | 580

You might also like